Skip to main content

ORTHOPEDIC MCQS 20 OB TRAUMA 1C

190 views
622 min read

ORTHOPEDIC MCQS ONLINE 20 OB TRAUMA 1C

 

Figure A

  1. Buttress plating is most appropriate in which of the following clinical situations?

 

 

 

 

 

 

 

 

 

 

 

  1. Figure A

  2. Figure B

  3. Figure C

  4. Figure D

  5. Figure E

 

Corrent answer: 3

 

There are 4 main types of plating techniques: 1. Bridging 2. Neutralization 3. Dynamic Compression 4. Buttress plating. Plates can utilize locking or non-locking screws.

 

Buttress plating is appropriate for a Shatzker Type I (see illustration C), as it can prevent collapse and axial deformity from shear or bending forces.

 

Karunakar et al showed that there was no significant difference between split depression tibial plateau fractures (Shatzker II) fixed with either a buttress plate with rafting screws versus a periarticular plate with built in rafting screw hole options, similar to the commonly used pre-contoured periarticular locking plate.

 

 

 

OrthoCash 2020

 

  1. Percutaneous placement of a lateral proximal tibial locking plate that extends down to the distal third of the leg is associated with postoperative decreased sensation of which of the following distributions?

    1. Medial hindfoot

    2. Lateral hindfoot

    3. First dorsal webspace

    4. Dorsal midfoot

    5. Plantar foot

     

    Corrent answer: 4

     

    Placement of long lateral tibial plates have been shown to have a risk of iatrogenic injury to the superficial peroneal nerve, which has a sensory distribution to the dorsal foot. This risk is seen especially with percutaneous approaches, such as those used with the LISS plate.

     

    The first reference by Deangelis et al found a risk of superficial peroneal injury with percutaneous screw placement of holes 11-13 in the LISS plate.

     

    The second referenced article by Roberts et al noted a slightly increased distance to the neurovascular bundle when interlocking tibial nails in a lateral to medial direction (compared to medial to lateral locking) and slightly increased biomechanical strength when locking in a medial to lateral direction.

     

    The third referenced article by Wolinsky et al notes a risk of iatrogenic injury to the deep peroneal nerve and anterior tibial artery with an anterolateral approach to the distal tibia, but notes the superficial peroneal nerve is safe with an appropriate exposure.

     

     

     

    OrthoCash 2020

     

  2. A 64-year-old female sustains a nondisplaced distal radius fracture and undergoes closed treatment using a cast. Three months after the fracture she reports an acute loss of her ability to extend her thumb. What is the most likely etiology of her new loss of function?

    1. Posterior interosseous nerve entrapment

    2. Extensor pollicis longus rupture

    3. Extensor pollicis longus entrapment

    4. Distal radius malunion

    5. Intersection syndrome

     

    Corrent answer: 2

     

    According to the referenced article by Jupiter and Fernandez, the most common scenario of extensor pollicis longus rupture after a distal radius fracture is when the fracture is non or minimally displaced. The hypothesis is that the rupture happens at an area of relative hypovascularity and healing callus can aggravate this area, leading to a degenerative tear.

    Hove et al reported an incidence of delayed tendon rupture after distal radius fracture of 0.3 percent. In their series of 18 extensor pollicis longus tendon ruptures, 15 were treated with tendon transfers. They reported good results: nearly 100% patient satisfaction, all patients were able to elevate the thumb to the level of the palm, and full independent index finger movements.

     

     

     

    OrthoCash 2020

     

  3. Presence of diabetes-induced peripheral neuropathy has been shown to be an independent risk factor for postoperative complications of which of the following injuries?

    1. Distal radius fractures

    2. Patella fractures

    3. Metatarsal fractures

    4. Ankle fractures

    5. Distal femoral fractures Corrent answer: 4

    Presence of peripheral neuropathy has important implications in treating ankle fractures in diabetic patients. Increased immobilization periods, attention to tight glucose control, and adjunct/alternative operative techniques may be necessary for an optimal outcome.

     

    The first referenced article by Chaudry et al is an excellent review of diabetic ankle fractures.

     

    The second reference by Costigan et al noted that peripheral neuropathy is the most significant risk factor for postoperative complications, followed closely by lack of pedal pulses preoperatively.

     

    The last referenced article by Jones et al noted a significantly higher complication rate in diabetics with operative ankle fractures, and reported that neuroarthropathy is a significant risk factor for postoperative complications.

     

     

     

    OrthoCash 2020

     

  4. A 45-year-old male falls off his motorcycle and injures his arm. AP and lateral radiographs reveal a proximal ulnar shaft fracture, 30 degrees apex anterior, and a radial head dislocation. Which direction is the radial head most likely dislocated?

    1. Lateral

    2. Posterior

    3. Posterolateral

    4. Anterior

    5. Anteromedial

     

    Corrent answer: 4

     

    A Monteggia fracture with apex anterior ulnar shaft fracture is associated with an anterior radial head dislocation (See Illustration A).

     

    Bado initially described and classified these injuries. The most common injury pattern in children is an extension (type 1) with anterior radial head dislocation and apex anterior ulnar shaft fracture, whereas the type II variety is most common in adults. The apex of the ulna fracture determines the direction of the radial head subluxation or dislocation. Adults typically require ORIF of the ulna.

     

    These fractures in children are often treated non-operatively with closed reduction if the ulna fracture is transverse and stable as illustrated by Fowles' case series, but may also require pinning if the fracture is oblique and unstable.

     

     

     

     

     

     

    OrthoCash 2020

     

  5. A 34-year-old male sustains a traumatic injury to his foot following a motorcycle accident. The patient's neurovascular status necessitates the amputation demonstrated in figures A through C. One year following the amputation, the patient complains of difficulty with gait and deformity of the ankle. Which of the following statements best describes the forces resulting in this deformity?

     

     

     

     

     

     

    1. Tight posterior capsule tissues of the ankle

    2. Neuropraxia of the deep peroneal nerve

    3. Unopposed pull of gastrocnemius-soleus only

    4. Unopposed pull of gastrocnemius-soleus, posterior tibialis, and peroneus

      brevis

    5. Unopposed pull of gastrocnemius-soleus and posterior tibialis Corrent answer: 5

    The clinical photograph and radiograph demonstrate a modified Lisfranc amputation. The loss of the peroneus longus, peroneus brevis, EHL, EDL, and peroneus tertius insertions result in an equinovarus deformity due to the pull of the gastroc-soleus complex, posterior tibialis, and anterior tibialis.

     

    Several surgical techniques have been described to address or prevent equinovarus deformities after Lisfranc amputation. Open or percutaneous achilles tendon lengthening, open gastrocnemius recession, or endoscopic gastrocnemius recession have shown to address the equinus deformity. Split tibialis anterior tendon transfer (STATT), 4th and 5th digit flexor-to-extensor tendons tenodesis, as well as peroneus brevis (PB) to peroneus longus (PL) tendon transfers have shown to address the varus deformity.

     

    Early et al. state the Lisfranc amputation should be considered when there is inadequate soft tissue coverage for transmetatarsal amputation or instability at the LisFranc joint. In addition, they comment that the deforming forces, the gastroc-soleus complex and posterior tibialis, are primarily innervated by the S1 nerve root.

     

    Figures A through C show clinical and radiographic images of a modified Lisfranc amputation. The classic Lisfranc amputation describes transaction of the first, third, fourth, and fifth tarsometatarsal joints. The second TMT is usually left in place, as it provides stability for the middle cuneiform. These radiographs show a midfoot amputation carried out more transversely across the bones of the midfoot.

     

    Incorrect Answers:

    Answers 1-4: The equinus deformity occurs after the elimination of extensor digitorum longus and extensor hallucis longus muscles, which cause an imbalance between the posterior compartment and the anterior compartment muscles that across the ankle. The varus deformity occurs when tibialis anterior and posterior compartment muscles overpower the eversion strength of the peroneus brevis muscle.

     

     

     

    OrthoCash 2020

  6. A 32-year-old female sustains the injury shown in Figure A. What is the most reliable method to evaluate the competence of the deltoid ligament?

     

     

     

     

    1. Medial ankle tenderness

    2. Medial ankle ecchymosis

    3. Squeeze test

    4. Stress radiography of the ankle

    5. Canale view radiograph Corrent answer: 4

    Figure A shows a lateral malleolar fracture from a rotational-type injury. Evaluation of the medial structures (deltoid ligament) is important for therapeutic reasons, as medial sided instability will portend a poor prognosis if treated nonoperatively.

     

    The referenced study by McConnell et al showed that physical exam is a poor indicator of medial ankle injury and that stress radiography is needed for proper medial ankle evaluation. All ankles found to be stable via stress examination healed with an intact mortise.

     

    The study by Gill et al showed that gravity stress radiographs are equivalent to manual stress radiographs of the ankle when evaluating rotational ankle fractures. This was true when both pronation-external rotation and supination-external rotation ankle fractures were examined.

     

    Illustration A shows the proper way to obtain a gravity stress radiograph.

     

     

     

     

     

    OrthoCash 2020

     

  7. A 34-year-old male is involved in a motor vehicle collision and sustains several orthopaedic injuries. Figure A shows a red line representating a fracture of the proximal femur. This fracture orientation is most often present when found concomitantly with which of the following orthopaedic injuries?

     

     

     

     

    1. Ipsilateral acetabular fracture

    2. lumbar spine burst fracture

    3. Ipsilateral femoral shaft fracture

    4. Anterior-posterior compression pelvic injury

    5. Ipsilateral calcaneus fracture Corrent answer: 3

    Femoral neck fractures are seen less than 10% of the time with femoral shaft fractures, but they are frequently missed on initial evaluation. When present, the pattern is typically nondisplaced, vertical, and basicervical.

     

    The review article by Peljovich and Patterson note that the femoral shaft component of the combined injury is typically in the middle third and is often comminuted.

     

    The article by Tornetta et al reports that they reduced the delay in diagnosis of concomitant femoral neck fractures by 91% by instituting a protocol that included: dedicated AP internal rotation plain radiograph, a fine (2-mm) cut CT scan through the femoral neck, an intraoperative fluoroscopic lateral radiograph prior to fixation, as well as postoperative AP and lateral radiographs of the hip in the operating room prior to awakening the patient.

     

    The article by Wiss et al noted that 18% of their ipsilateral femoral neck/shaft cohort developed a symptomatic varus nonunion requiring a valgus osteotomy. It is significant to note that in this study, the shaft fractures were fixed prior to definitive neck stabilization, and the review article by Peljovich emphasizes that the neck fracture should be treated first and the shaft fracture second.

     

     

     

    OrthoCash 2020

     

  8. A 74-year-old female falls from a standing height and sustains the fracture shown in Figure A. The occurrence of this injury most increases her risk of sustaining which of the following fractures?

     

     

     

     

    1. Sacral fracture

    2. Hip fracture

    3. Distal radius fracture

    4. Distal fibula fracture

    5. Distal humerus fracture Corrent answer: 2

    Clinton et al performed a Level 2 study of 8049 women that demonstrated that proximal humeral fractures independently increased the risk of a subsequent hip fracture. The risk was more than five times in the first year after the humeral fracture but was not associated with a significant increase in the hip fracture risk in subsequent years.

     

    Johnell et al performed a Level 4 review that found that men and women had an increased risk of hip, forearm and spine fractures following a prior spine, hip or shoulder fracture.

     

    Schousboe et al performed a Level 2 investigation of 9516 community-dwelling elderly women and found that 521 hip fractures occurred after 10 years of follow-up. They found that prior non-spine fractures, non-hip fractures, and prevalent moderate to severe radiographic vertebral fractures were modestly associated with incident hip fracture.

     

     

     

    OrthoCash 2020

     

  9. A 47-year-old male sustained a comminuted calcaneus fracture in a motorcyle accident. He subsequently develops the post-traumatic condition shown in Figure A. All of the following would be indications for a subtalar distraction arthrodesis using a bone graft instead of an in-situ subtalar arthrodesis EXCEPT:

     

     

     

    1. Decreased calcaneus height

    2. Decreased talocalcaneal angle

    3. Decreased talar declination angle

    4. Presence of a collapsed subtalar joint from AVN

    5. Presence of full ankle dorsiflexion with no tibiotalar impingement Corrent answer: 5

    Subtalar distraction arthrodesis using a bone graft is not indicated in a patient with full ankle dorsiflexion and no tibiotalar impingement.

     

    The radiograph demonstrates post-traumatic subtalar arthritis with deformity. Subtalar arthrodesis in situ would not correct the deformity. Substantial loss of heel height may lead to symptomatic anterior tibiotalar impingement and is considered the most appropriate indication for subtalar distraction arthrodesis. Measurement of the talocalcaneal angle is made using a line representing the long axis of the talus and its intersection with the longitudinal axis of the calcaneus. It is demonstrated in Illustration A. The talar declination angle is formed by the axis of the talus to the plane of support and is demonstrated as angle H in Illustration B. The talocalcaneal height is measured from the dome of the talus to the base of the calcaneus displayed as Line K in Illustration C. Each of these measurements along with longitudinal arch are often decreased in patients with post-traumatic subtalar arthritis and are discussed as deformities that can be corrected with bone block subtalar distraction arthrodesis. Chandler et al and Trnka et al each performed Level 4 studies that concluded that distraction arthrodesis was most appropriate in the setting anterior ankle impingement.

     

     

     

     

     

     

     

     

     

     

     

    OrthoCash 2020

     

  10. A 30-year-old male sustains a right shoulder injury with initial radiographs shown in Figures A and B. What single piece of additional information would best assist in determining this patient's functional outcome?

     

     

     

     

     

     

    1. Lower extremity injury

    2. Neurological deficit

    3. Contralateral upper extremity injury

    4. Proximal humerus fracture

    5. Worker's compensation

     

    Corrent answer: 2

     

    Figures A and B show a scapulothoracic dissociation, with significant lateralization of the scapula and widening of the acromioclavicular joint to over 4 cm (Figure A). This can be thought of as an internal disarticulation of the scapulothoracic association and acromioclavicular joints.

     

    The referenced article by Riess et al revealed that functional outcomes are

    worse with brachial plexus injuries and concomitant scapulothoracic dissociation than with isolated brachial plexus injuries. In fact, at 2 year followup, only 57% of the dissociation patients could carry anything over 5 lb with the injured side.

     

    The other referenced article by Zelle et al found that complete brachial plexus injuries portended the worst outcome for scapulothoracic dissociation injuries.

     

     

     

    OrthoCash 2020

     

  11. During a dual incision fasciotomy of the leg, the soleus is elevated from the tibia to allow access to which of the following compartments?

    1. Superficial posterior

    2. Deep posterior

    3. Lateral

    4. Anterior

    5. Mobile wad

     

    Corrent answer: 2

     

    The soleus is elevated/released from the posterior tibia during the medial approach to allow access to the deep posterior compartment. Release of this compartment cannot be done without proper elevation of the soleus. The superficial posterior compartment mass is primarily located in the proximal half of the leg, while the deep posterior musculature is located in the distal 2/3 of the leg.

     

    Illustration A depicts the musculature and septums compartmentalizing the lower leg. The transverse intermuscular septum separates the deep from the superficial posterior compartments.

     

     

     

     

     

    OrthoCash 2020

     

  12. Longitudinal radioulnar dissociation, including Essex Lopresti fractures, requires disruption of the interosseous membrane (IOM). The interosseous membrane (IOM) consists of all of the following ligaments EXCEPT?

    1. Central band ligament

    2. Accessory band ligament

    3. Annular ligament

    4. Dorsal oblique accessory cord ligament

    5. Distal oblique bundle ligament Corrent answer: 3

    The IOM includes 5 types of ligaments: central band, accessory band, distal oblique bundle, proximal oblique cord, and dorsal oblique accessory cord. The annular ligament is not a part of the IOM.

     

    The IOM bridges the radius and ulna and acts as a hinge for rotation of radius about ulna. The central portion is thickened, and forms the central band which is the most important ligament for IOM load distribution characteristics.

     

    Noda et al, in a cadaver study, identified the precise anatomical insertions and attachment points of each of the 5 IOM components. They found the most distal and proximal ends of the radial origin of the central band were 53% and 64% of total radial length from the tip of the radial styloid, whereas those of the ulnar insertion were 29% and 44% of total ulnar length from the ulnar head.

    Pfaeffle et al also performed a cadaveric biomechanical study applying compressive loads to specimens with IOMs that are intact, cut, or cut/reconstructed with flexor carpi radialis allografts. They found that reconstruction of the IOM can restore the normal load transfer chararcteristics and that the central band of the IOM is the most important portion of the IOM to be reconstructed.

     

    Illustration A shows the ligaments of IOM membrane: CB = central band, AB = accessory band, distal oblique bundle, proximal oblique cord, and dorsal oblique accessory cord

     

    Incorrect Answers:

    The annular ligament is a strong band of fibers, which encircles the head of the radius and keeps the radius in contact with the radial notch of the ulna, but is not a part of the IOM.

     

     

     

     

     

     

    OrthoCash 2020

     

  13. During surgical treatment of an olecranon fracture with a tension band construct as seen in Figure A, what nerve is at risk with over penetration of the proximal anterior cortex of the ulna with the Kirchner wire?

     

     

     

     

    1. Median nerve

    2. Anterior interosseous nerve

    3. Posterior interosseous nerve

    4. Ulnar nerve

    5. Radial nerve

     

    Corrent answer: 2

     

    An inability to flex the thumb interphalangeal joint or the index finger distal interphalangeal joint is indicative of an anterior interosseous nerve palsy/injury.

     

    This has been reported with tension band fixation of olecranon fractures, especially with overpenetration of the anterior cortex of the proximal ulna by the Kirschner wire. Anterior interosseous nerve palsy can also be possible by overpenetration of drill bits or screws through the anterior cortex. Initial treatment of this nerve injury is observation.

     

    The referenced article by Parker et al reports a case of anterior interosseous nerve palsy after such tension band fixation. They note that this was a direct injury from the Kirchner wires.

     

    The referenced article by Adams et al reviews nerve injuries about the elbow. They note that an appreciation of the complex anatomy of the region and an understanding of treatment options are necessary for surgeons who treat elbow injuries.

    Figure A is a lateral radiograph of a olecranon fracture fixed with a tension band construct.

     

    Incorrect Answers: The other nerves are not associated with overpenetration of the far cortex in an olecranon tension band fixation.

     

     

     

    OrthoCash 2020

     

  14. In an isolated ankle syndesmotic injury, the fibula is unstable in the incisura fibularis of the tibia. In what direction is the fibula most unstable?

    1. Anterior-posterior

    2. Medial-lateral

    3. Proximal-distal

    4. Varus-valgus

    5. Equivalent instability in all axes Corrent answer: 1

    In an ankle syndesmosis injury, the fibula is most unstable in an anterior and posterior direction. This is whether or not there is an accompanying ankle fracture. Most commonly, the fibula will subluxate in an anterior-posterior direction in an ankle fracture model.

     

    The first referenced article by Xenos et al found that stress lateral radiographs have more interobserver reliability than stress AP/mortise radiographs and that two syndesmotic screws are stronger than one.

     

    The referenced article by Candal-Couto et al is a biomechanical study that found more anterior-posterior instability in a syndesmosis injury model, and more ankle instability is noted with syndesmosis injury and a concomitant deltoid injury.

     

    The referenced article by Zalavras et al is an excellent review article on ankle syndesmosis injuries.

     

     

     

    OrthoCash 2020

     

  15. A 27-year-old male sustains closed injuries to his right foot in a motor vehicle collision. He is a nonsmoker. A radiograph and computed tomography scan are provided in Figures A and B. All of the following

    are prognostic of a superior outcome with operative treatment EXCEPT:

     

     

     

     

     

     

     

    1. Male

    2. Works as an attorney

    3. Nonsmoker

    4. Twenty-seven years old

    5. He was injured while off his job Corrent answer: 1

    The clinical presentation, radiographs, and CT are consistent with a displaced and severely depressed intra-articular calcaneus fracture.

     

    Rammelt et al reviews the operative treatment of calcaneal fractures. Notably

    poorer outcomes are associated with open fractures, delayed reduction after more than 14 days and individual risk factors such as high body mass index and smoking.

     

    The two studies by Buckley et al are both data taken from a multicenter, prospective, randomized study of calcaneal fractures. Women who were treated operatively achieved superior clinical outcomes compared to those women treated nonoperatively. Patients who were not on workers compensation and less than 29 years old and those that had a moderate Bohler's angle (0-14 degrees), less fracture comminution, more sedentary jobs, and an anatomic reduction reported superior clinical outcomes than those treated nonoperatively.

     

    Figure A is a lateral radiograph demonstrating an intra-articular joint depression calcaneus fracture. Figure B is a coronal CT scan demonstrating intra-articular displacement of the posterior facet.

     

    Answers 2-5 are all incorrect because these are predictive of superior outcome with operative treatment.

     

     

     

    OrthoCash 2020

     

  16. A 25-year-old male involved in a motorcyle accident sustains the injury seen in Figures A and B. After initial adequate debridement of nonviable tissue, which of the following irrigation methods and devices should be used?

     

     

     

     

     

     

    1. Antibiotic solution applied by low pressure gravity flow device

    2. Antibiotic solution applied by high pressure pulsatile flow device

    3. Saline solution applied by low pressure gravity flow device

    4. Saline solution applied by high pressure pulsatile flow device

    5. Antibiotic solution applied by high pressure pulsatile flow device followed by low pressure gravity flow device

    Corrent answer: 3

     

    Initial wound treatment is critical in the treatment of open fractures and contaminated wounds. Figures A and B reveal a Type IIIB open tibia fracture with obvious gross contamination and devitalized tissue. Systemic antibiotics and tetanus prophylaxis should be administered immediately upon arrival, and

    the lower extremity should be splinted while the remaining trauma workup takes place. Once cleared, this injury should be adequately debrided of all devitalized tissue and subsequently irrigated with a saline solution to reduce the bacterial count. Some evidence suggests that high-pressure pulsatile lavage damages bone structure and disrupts soft tissue.

     

    In an animal model, Hassinger et al showed that high-pressure pulsatile lavage caused deeper penetration of bacteria and results in greater bacterial retention in soft tissue when compared with low-pressure lavage.

     

    Owens et al, in a sheep model of contaminated soft tissue, compared low and high pressure lavage with normal saline solution, bacitracin solution, castile soap, and benzalkonium chloride. At 48 hours the group treated by low pressure lavage and saline showed the lowest rebound in bacterial counts.

     

     

     

    OrthoCash 2020

     

  17. A patient presenting with scapulothoracic dissocation and ipsilateral extremity neurologic injury is most likely to have which of the following outcomes?

    1. Glenohumeral arthritis

    2. Return of 3/5 motor strength in distal extremity

    3. Full return of extremity sensory function only

    4. Flail extremity

    5. Death

     

    Corrent answer: 4

     

    Scapulothoracic dissociation is a high-energy injury resulting from massive traction injury to the shoulder girdle with disruption of the scapulothoracic articulation. The most common long term result from this injury is complete loss of motor and sensory function of the extremity (flail limb), with death in the acute or semi-acute period also common.

     

    The referenced study by Althausen et al found that outcomes from this injury were: a flail extremity in 52%, early amputation in 21%, and death in 10%.

     

    The other referenced study by Ebraheim et al found that 12/15 patients had a complete brachial plexus injury and that none recovered any function (the other 3 patients died in the acute period).

     

     

     

    OrthoCash 2020

  18. Vascular complications are most commonly seen with which of the following fractures about the knee?

     

     

     

     

     

     

     

     

     

     

     

     

     

     

    1. Figure A

    2. Figure B

    3. Figure C

    4. Figure D

    5. Figure E

     

    Corrent answer: 2

     

    Figure B represents a medial plateau fracture/dislocation pattern (Schatzker IV). This fracture typically requires more energy to occur than the corresponding lateral plateau injury, which is due to the more dense bone on the medial side. A fracture-dislocation of the knee must be suspected with these injuries, as the femur will sometimes follow the displaced medial tibial condyle. Along with a proper vascular exam, ankle brachial indices (ABI) must be immediately taken and if abnormal further vascular testing is warranted.

    Furthermore, these injuries which are a hybrid of a dislocation and a fracture will often have a benign appearance on radiographs, but a high rate of vascular complications.

     

    Berkson et al present a review article and they stress the importance of safeguarding tissue vascularity and while emphasizing joint reduction and restoration of the mechanical axis of the limb.

     

    Ottolenghi et al in their review article showed a vascular injury rate of 2% for

    tibial plateau fractures.

     

    Stark et al in their review showed a very high incidence of acute compartment syndrome in Schatzker IV and VI injuries. In their series, 18% of Schatzker VI and 53% of Shatzker IV fractures developed compartment syndrome.

     

     

     

    OrthoCash 2020

     

  19. A 26-year-old male wrestler suffers the elbow injury shown in Figure A. On physical exam he is neurologically intact and has a palpable radial pulse. He is treated with closed reduction in the emergency room. In order to optimize his clinical outcomes, which of the following treatment and rehabilitation protocols should be avoided?

     

     

     

     

    1. Immediate active and active-assist range of motion through a stable arc

    2. Initial splinting and immobilization for 4 weeks followed by physical therapy

    3. Initial splinting in 90 degrees of flexion with neutral forearm rotation

    4. A range of motion protocol that limits full extension in the early phases of rehab

    5. Light duty use of the affected arm immediately following immobilization Corrent answer: 2

    Prolonged splinting following simple elbow dislocations of greater than 2 weeks after reduction can lead to chronic stiffness and poor outcomes and should be avoided. All of the remaining answers are appropriate in the rehabilitation of these injuries.

     

    Simple elbow dislocations are second only to the shoulder in rates of joint dislocation. Closed reduction with early rehabilitation has proven the most effective treatment for these injuries when the elbow is stable. Rehabilitation for a simple elbow dislocations should include initial splinting in 90 degrees of flexion with neutral forearm rotation and immediate active and active-assist range of motion through a stable arc. Full extension should be avoided initially

    in rehab. The patient should be encouraged to use the affected arm for light duty immediately following immobilization.

     

    Mehloff et al. determined that elbows splinted longer than 2 weeks had decreased ROM at 1 year. They further describe the benefits of using the affected arm for light duty after 2 weeks of rest.

     

    Smith et al. describe the effectiveness of immediate active and active-assist ROM exercises after reduction.

     

    O'Driscoll et al. describe how to test an elbow for posterolateral instability, which can occur after an elbow dislocation. They emphasize the elbow is most unstable in extension immediately after injury. Therefore, the elbow should be splinted in 90 degrees of flexion with neutral forearm rotation until therapy begins.

     

    Figure A is a lateral radiograph of the elbow which shows a simple posterior elbow dislocation. The anatomic description is based on the anatomic location of olecranon relative to humerus.

     

     

     

    OrthoCash 2020

     

  20. Increasing the oxygen gradient for diffusion is the mechanism of action for which of the following methods of treatment of lower extremity trauma?

    1. Open fasciotomy

    2. Percutaneous fasciotomy

    3. High-dose anti-inflammatories

    4. Hyperbaric oxygen therapy

    5. Negative pressure wound therapy Corrent answer: 4

    Hyperbaric oxygen therapy (HBO) allows patients to breathe 100% oxygen in a chamber under conditions of increased barometric pressure.

     

    This tremendous partial pressure of oxygen supports gas diffusion for a much greater distance than under normal conditions, thus delivering oxygen to relatively ischemic and hypoxic tissues. Trauma-related indications for HBO therapy include carbon monoxide intoxication, gas gangrene, crush injury, compartment syndrome, necrotizing fasciitis, treatment of chronic osteomyelitis, support of grafts and flaps, and burns. Contraindications relate to issues of gas exchange, oxygen sensitivity, and technical safety.

    The cited reference is a useful review article of hyperbaric oxygen therapy in extremity trauma.

     

     

     

    OrthoCash 2020

     

  21. All of the following indicators of resuscitation may be within normal limits for a trauma patient that is in "compensated" shock EXCEPT:

    1. Systolic blood pressure

    2. Urine output

    3. Heart rate

    4. Serum lactate

    5. Mean arterial pressure Corrent answer: 4

      Historically, normal blood pressure, heart rate, and urine output have been endpoints to signal complete resuscitation in the polytrauma patient.

       

      The review article by Porter et al states that there is a high incidence of patients (as much as 85%) in "compensated" shock despite normal vital signs and urine output parameters. Compensated shock is secondary to a maldistribution of blood flow and tissue oxygenation as splanchnic organs have less distribution of the cardiac output compared to the heart and the brain.

       

      The article by Elliott is also a review, and it states that serum lactate is the best indicator of peripheral organ perfusion and tissue oxygenation. It also states that base deficit and gastric mucosal pH are appropriate end points to determine the complete resuscitation of trauma patients.

       

       

       

      OrthoCash 2020

       

  22. A 25-year-old man sustains the fracture seen in Figure A and is seen in pre-op holding prior to surgery. What position of his lower extremity would result in the lowest intracapsular hip pressure?

     

     

     

    1. Neutral rotation and flexion

    2. External rotation and flexion

    3. Internal rotation and extension

    4. External rotation and extension

    5. Internal rotation and flexion Corrent answer: 2

    The femoral neck fracture seen in Figure A is considered by most to be a surgical emergency in a 25-year old due to the at risk blood supply of the femoral head. Due to the intra-capsular hematoma and hemarthrosis that occurs, the involved extremity is often found to be in external rotation and flexion as the intra-capsular volume is the highest in this position. Debate exists of the significance of the pressure caused by the fracture hemarthrosis as it is believed by some to cause a local compartment syndrome adding further insult to the already tenuous blood supply. Advocates of early fixation have proposed that reduction maneuvers without capsulotomy can compromise the circulation of the femoral head by increasing the hip joint pressure.

    Maruenda et al showed in their study of 34 consecutive patients with femoral neck fractures that the mean intracapsular pressure was the highest with the hip in extension and internal rotation.

     

    Bonnaire et al in their prospective study of 55 patients with intracapsular femoral neck fractures found the lowest pressure to be at 70 degrees of

    flexion. Both Maruenda and Bonnaire's studies showed no significant intracapsular pressure difference based on fracture displacement.

     

    Corollary to adult fracture patients, it is also noted that pediatric patients with a septic hip hold their hip in a flexed and externally rotated position to maximize intracapsular volume as shown in Illustration A.

     

     

     

     

     

     

    OrthoCash 2020

     

  23. A 15-year-old male falls from his bicycle and lands directly on his left hip. He is unable to bear weight and presents to the emergency department with the AP pelvis radiograph seen in Figure A. Which of the following radiographic views could aid in classifying this patient's fracture pattern?

     

     

     

     

    1. Iliac oblique (Judet) view

    2. Traction AP of the hip with the leg in neutral rotation

    3. Outlet view of the pelvis

    4. Traction AP of the hip with the leg internally rotated 15°

    5. Traction AP of the hip with the leg externally rotated 15° Corrent answer: 4

    The ability to define the location of a femoral neck fracture aids in determining the optimal treatment. The addition of a "traction-internal rotation" view (as described in Answer 4) to standard hip radiographs may assist with classification of femoral neck fractures by accounting for the anteversion of the femoral neck.

     

    Koval et al performed a study in which proximal femur fractures were classified with standard radiographs (AP pelvis, AP hip, cross-table lateral), followed by the addition of a physician-assisted view with traction and 15 degrees of internal rotation. The inclusion of the traction-internal rotation view led to increased agreement in classification between the authors, and was especially helpful for differentiating displaced femoral neck fractures versus stable intertrochanteric fractures.

     

     

     

    OrthoCash 2020

     

  24. A 25-year-old female sustains the isolated fracture seen in Figure

    A. The patient elects to have nonoperative management. When compared to operative treatment, which of the following is true of the clinical outcome following nonoperative management?

     

     

     

    1. Long arm cast immobilization is necessary with nonoperative management

    2. Twenty degree loss of forearm rotation is expected with nonoperative management

    3. Loss of wrist motion is expected with nonoperative management

    4. Loss of elbow motion is expected with nonoperative management

    5. Equivalent clinical outcomes Corrent answer: 5

    The clinical presentation is consistent for an isolated ulnar fracture, or "night stick" fracture. For minimally displaced and angulated fractures, nonoperative management has equivalent clinical outcomes to surgical treatment.

     

    Isolated ulnar shaft fractures with greater than 10 degrees of angulation or 50% displacement can result in loss of forearm rotation if treated nonoperatively. For minimally displaced fracture, nonoperative treatment results in union with good function. Time to union has been found to be greater in nonoperative groups, but clinical outcomes equivalent.

     

    Pollock et al. compared two patient groups treated with short term immobilization or long term cast treatment and found no significant difference between wrist and elbow motion between the two groups. The short term immobilization group in their study had a shorter time to union and lower non union rate as well.

     

     

     

    OrthoCash 2020

     

  25. A comminuted proximal humerus fracture is treated with a shoulder hemiarthroplasty as shown in Figure A. The superior border of the pectoralis major tendon can be used to determine accurate restoration of which of the following?

     

     

     

    1. Humeral prosthesis height and retroversion

    2. Humeral prosthesis offset and retroversion

    3. Humeral prosthesis head-neck angle and height

    4. Humeral prosthesis stem width and height

    5. Humeral prosthesis stem length and retroversion Corrent answer: 1

    The height of the prosthesis is best determined by the superior border of the pectoralis major insertion (PMI), which has been shown in several clinical and cadaveric studies to be the most reliable instrument to assess humeral prosthesis height. In addition, the PMI can be used to position the implant in regards to retroversion, as the distance and angle of the PMI to the posterior fin of the implant has been investigated and reported (see second reference below). No evidence exists to differentiate this against using the forearm as a landmark for retroversion, however.

     

    The referenced study by Murchavosky et al found that in cadaveric dissection, pectoralis major tendons inserted 5.6 cm distal to the superior aspect of the humeral head.

     

    The referenced study by Torrens et al also found that the insertion point was

    5.6 cm distal to the superior aspect of the humeral head and very reproducible regarding its relationship to retroversion. The mean distance of the PMI to the

    posterior fin of the prosthesis was 1.06 cm. The mean angle between the PMI and the posterior fin of the prosthesis was 24.65 degrees.

     

    The referenced study by Greiner et al found that clinical results from utilizing this reference in fracture cases improved radiographic and clinical outcomes; they also reported that clinical outcomes primarily depended on tuberosity healing.

     

    Figure A shows a shoulder hemiarthroplasty.

     

     

     

    OrthoCash 2020

     

  26. A 35-year-old male involved in a high-speed motor vehicle collision presents to the trauma bay hypotensive and with a clinically unstable pelvis. A pelvis radiograph is shown in Figure A. He is placed in a pelvic binder, and his blood pressure normalizes temporarily. An abdominal CT demonstrates free fluid and air in the intraperitoneal cavity, and a laparotomy is indicated. What is the most appropriate next step in orthopaedic management?

     

     

     

     

    1. Percutaneous SI screw placement

    2. External fixation placement

    3. Pubic symphysis plating

    4. Posterior pelvic plating

    5. Anterior sacroiliac plating Corrent answer: 2

    In the management of patients with multiple injuries, controversy often arises as to the appropriate method of initial pelvic stabilization. It is generally agreed upon that applying an external frame is appropriate in the setting of an unstable patient with intraperitoneal fluid and labile blood pressure. Ex-fix placement can support hemodynamic stabilization and assist the general surgeons with their laparotomy procedure. Plate or screw fixation of the pelvis should be delayed because the laparotomy takes precedence in a patient who is hemodynamically unstable, and internal fixation in the presence of bowel contamination can result in increased rates of infection.

     

    Furthermore, Tile noted increased septic complications with intrapelvic hardware fixation in the setting of intraperitoneal soft tissue damage and bleeding. Angiography and embolization may help with the pelvic bleeding, but will not stabilize the pelvis during the laparotomy. The review article by Tile et al discusses the assessment of the patient with a pelvic injury, and summarizes the various methods of temporary and definitive pelvic fixation.

     

     

     

    OrthoCash 2020

     

  27. A 32-year-old female sustains an isolated midshaft clavicle fracture, as shown in Figure A. Her clinical exam does not reveal skin tenting or neurovascular injury, but shortening is measured at 2.6 cm. Which of the following treatment methods has been shown to have the lowest rate of nonunion and symptomatic malunion?

     

     

     

     

    1. Open reduction and internal fixation with plating

    2. Open reduction and percutaneous pinning

    3. Closed reduction and percutaneous pinning

    4. Closed reduction and external fixation

    5. Nonoperative treatment with a sling and early range of motion Corrent answer: 1

    Figure A shows a left clavicle fracture with significant shortening. Open reduction and internal fixation with plate and screw constructs of displaced, shortened clavicle fractures has been shown to lead to the best patient reported functional outcomes as well as have the least incidence of nonunion and symptomatic malunion. Factors associated with poor functional outcome as well as nonunion in these injuries include fracture shortening >2cm, fracture comminution, fracture displacement > 100%, female gender, and advancing age.

     

    The referenced article by Khan et al is an excellent review of the indications, treatment methods and outcomes of clavicle fractures.

     

     

     

    OrthoCash 2020

     

  28. A 78-year-old female falls and sustains the fracture seen in Figure A. Surgical treatment is pursued with open reduction internal fixation with a lateral locking plate. Postoperative radiographs are provided in Figure B. What is the most common complication with this mode of fixation?

     

     

     

    1. Infection

    2. Osteonecrosis

    3. Axillary artery injury

    4. Screw cut-out

    5. Axillary nerve injury Corrent answer: 4

    Screw cut-out is the most common complication following open reduction and internal fixation of 3 and 4 part proximal humerus fractures. A radiograph of this phenomenon in this patient at 5 months is shown in illustration A. This complication ultimately results from fracture settling and axial compression allowing the screws to penetrate the joint.

     

    Owsley and Gorczyca evaluated a single surgeon's experience treating 53 patients with displaced proximal humeral fracture with a standard post-op regime. They found that screw cut-out was the most common complication following fixation of 3 and 4 part fractures with locking plate technology.

     

    Frankhauser et al. describe their results with a "new" proximal humeral locking plate (AO/ASIF technology) in 28 patient with 29 fractures. They describe good to excellent clinical/radiographic results by following radiographs and patient related Constant scores.

     

     

     

     

     

     

    OrthoCash 2020

     

  29. A 72-year-old female presents to your office with a 12-month old painful nonunion of a 2-part (surgical neck) fracture of the proximal humerus. Prior to her injury, she denied shoulder pain and an excellent range of motion. Since the injury, she has persistent debilitating pain and dysfunction with motion above the shoulder level. X-Rays show good bone stock, no significant shoulder arthritis or avascular

    necrosis, and well-positioned tuberosities. Which of the following is the most optimal next step in management?

    1. Closed reduction and percutaneous pinning

    2. Shoulder hemiarthroplasty

    3. Superior capsular reconstruction (SCR)

    4. Open reduction and internal fixation (ORIF) with or without bone grafting

    5. Reverse total shoulder replacement with lattismus dorsi transfer to assist with internal rotation

    Corrent answer: 4

     

    Treatment of 2 part proximal humerus malunion in patient with good bone stock with evidence of arthritis or osteonecrosis can be effectively treated with ORIF with locked plating with or without bone grafting.

     

    Treatment of a chronic nonunion of the proximal humerus in the elderly should be treated with fixation when possible. Critical attention should be paid to correct all deformities: tuberosity positioning, articular surface realignment, soft tissue balancing, rotator cuff repair (when needed), and treatment of soft tissue contractures. Attempts at arthroplasty are generally recommended only when the fracture has eroded enough to prevent successful fixation, if the tuberosities have resorbed, if the rotator cuff has a pre-existing tear, or other findings are present that would limit the success rate of fixation.

     

    Quadlbauer et al. evaluated the results of ORIF with locked plating without bone grafting for 9 patients with proximal humerus non-unions with a mean 31-month followup. They noted that all 9 patients went on achieve bony union with improvement in ROM in all planes except fo adduction. They concluded that ORIF with locked plating without bone grafting is a reasonable and safe option for treating proximal humerus nonunion with high union rates and minimal risk of complications

     

    Cadet et al. in a JAAOS review article discussed the various management options for proximal humerus non-unions. The authors noted that several recent series did demonstrate union in >90% of patients treated with reconstruction using locking plates and autogenous bone graft. They discuss that while reverse shoulder arthroplasty (rTSA) has also showed excellent clinical results, its should be reserve for patients with nonviable humeral heads, severe humeral osteoporosis

     

    Incorrect Answers:

    Answer 1: There is no role for closed reduction in the setting of a non-union Answer 2: While rTSA has shown great results with, similar results have not

    recently been shown for shoulder hemiarthroplasty

    Answer 3: SCR is reserved for a patient with irreparable superior rotator cuffs who are considered too young to undergo arthroplasty

    Answer 5: While rTSA is an effective treatment option here, the addition of a lattismus dorsi transfer would assist with EXTERNAL rotation not INTERNAL rotation.

     

     

     

    OrthoCash 2020

     

  30. There is a risk of impaired forearm rotation after tension band fixation of an olecranon fracture with which of the following?

    1. Ipsilateral proximal humerus fracture

    2. Protrusion of Kirschner wire fixation through the volar cortex of the proximal ulna

    3. Use of ulnar intramedullary Kirschner wire fixation

    4. Olecranon fracture comminution

    5. Lack of triceps tendon repair Corrent answer: 2

    Impaired pronation/supination can be seen if the K-wire is advanced either too radial or too far through the volar (anterior) cortex of the proximal ulna. The anterior interosseous nerve is also at risk with overpenetration. Conversely, migration and loosening of the K-wire is reduced with involvement of the anterior cortex.

     

    The referenced study by Candal-Couto et al is a cadaveric study that found that K-wire insertion in less than 30 degrees in an ulnar direction led to impingement of the K-wire on the radial head/neck, biceps or supinator. They recommend placing these wires away from this danger zone in order to minimize rotation blocks.

     

    The referenced study by Matthews et al is a case series of two patients who had limited forearm rotation after K-wire fixation. The etiology of limited rotation was found to be from direct overpenetration of the K-wire, which led to a mechanical block.

     

     

     

    OrthoCash 2020

     

  31. When performing a saline load test to evaluate for a traumatic arthrotomy of the knee, a mininum of how much saline should be utilized?

    1. 30 mL

    2. 50 mL

    3. 75 mL

    4. 100 mL

    5. 155 mL

     

    Corrent answer: 5

     

    A saline load test is commonly utilized to evaluate for intraarticular penetration of superficial lacerations (traumatic arthrotomies). In this technique, a large gauge needle is used (18 ga) and saline is injected into the knee with passive ROM provided. Saline egress from the soft tissue injury is a positive test (traumatic arthrotomy present).

     

    The study by Nord et al found that the volumes of saline that were needed in order to effectively diagnose 75%, 90%, 95%, and 99% of the knee

    arthrotomies were 110, 145, 155, and 175 mL, respectively.

     

    The other referenced study by Keese et al found that 50 mL of saline successfully identified only 46% of known knee arthrotomies and that 194 mL was needed to reach a 95% identification rate.

     

     

     

    OrthoCash 2020

     

  32. Regarding the role of the orthopaedic surgeon in addressing domestic and family violence, all of the following statements are true EXCEPT:

    1. Report all cases of child abuse, as this is required by all states

    2. Report all cases of adult spousal or intimate partner abuse, as this is required by all states

    3. Hospitalize elderly victims who are in immediate danger and help develop a plan to ensure their safety

    4. Advocate for appropriate legislation and public policy on violence and abuse related to health care

    5. Orthopedic surgeons are responsible for knowing the reporting laws and procedures for suspected abuse

      Corrent answer: 2

       

      Reporting requirements for adult spousal or intimate partner abuse is not standardized among states and it is the responsibility of the orthopaedic surgeon to understand the laws of his or her state. The AAOS Advisory statement gives information to assist in meeting the ethical and legal

      obligations on Domestic and Family Violence and Abuse.

       

      Domestic and family violence affects over 10% of the US population (approximately 32 million Americans). Child abuse and neglect contributed to 1,400 fatalities in 2002 and there was 565,747 reports of suspected elder abuse.

       

      Reporting of suspected child abuse is required in all states. The orthopaedic surgeon should hospitalize elderly victims who are in immediate danger and help develop a plan to insure their safety.

       

       

       

      OrthoCash 2020

       

  33. A 42-year-old female sustains the injury shown in Figure A. What other anatomic structure is most commonly injured with this fracture?

     

     

     

     

    1. Volar long radiolunate ligament

    2. Radioscaphocapitate ligament

    3. Dorsal radioulnar ligaments

    4. Ligament of Testut and Kuentz

    5. Scapholunate ligament

     

    Corrent answer: 3

    Figure A shows a fracture of the distal 1/3 of the radius. This fracture is often associated with dislocation of the distal radioulnar joint (DRUJ). When the two injuries are present together, it is commonly referred to as a Galeazzi fracture.

     

    DRUJ instability is a result of injury to the volar and dorsal radioulnar ligaments which are the primary stabilizers of this joint. Imaging often shows widening of the DRUJ on AP view, dorsal or volar displacement on lateral view and radial shortening. Treatment of a Galeazzi injury is operative, consisting of open reduction and internal fixation of the radius with a plate and screw construct, followed by intraoperative assessment of DRUJ alignment.

     

    The referenced study by Giannoulis et al is an excellent review of Galeazzi fractures and treatment methods.

     

    Ward et al found in a cadaver study that the most significant increases in translation of the wrist occurred after sectioning the dorsal radioulnar ligament in pronation and after sectioning the palmar radioulnar ligament in supination.

     

    This patient's AP radiograph is shown in illustration A, and this shows an obvious DRUJ dislocation.

     

    Incorrect answers

    Choice 1,2,4 and 5 are not associated with injury of the DRUJ

     

     

     

     

     

    OrthoCash 2020

     

  34. A 34-year-old man sustains a twisting injury to his left ankle playing soccer. Radiographs from the ER are provided in figures A and

    B. Four hours later, he undergoes open reduction internal fixation. An intraoperative fluoroscopy image is provided in figure C. Which of the following is the best method to assess the integrity of the syndesmosis?

     

     

     

     

     

     

    1. Measurement of medial clear space widening

    2. Measurement of the tibiofibular overlap

    3. Anterior drawer test with comparison to the contralateral ankle

    4. External rotation stress radiograph

    5. Evaluation of the syndesmosis on preoperative CT scan

    Corrent answer: 4

     

    The radiograph demonstrates a Weber B ankle fracture. A dynamic external rotation stress test with the ankle dorsiflexed to 90 degrees is the most accurate way to evaluate the integrity of the syndesmosis.

     

    Nielson et al evaluated 70 ankle fractures with radiographs and MRI. Neither measurements of the tibiofibular clear space nor the tibiofibular overlap correlated with syndesmotic injury on MRI. Medial clear space widening of more than 4 mm occurred with MRI evidence of disruption of the deltoid and the tibiofibular ligaments.

     

    Nielson et al used this same cohort of ankle fractures in a separate study to evaluate whether the level of the fibular fracture correlated with syndesmotic incompetence. They found no correlation. The level of fracture on the fibula cannot be used to accurately predict disruption of the syndesmosis.

     

    Ebraheim et al reviewed a series of Weber B fibula fractures with deltoid injury and syndesmotic disruption. Their findings concluded that the surgeon's dynamic assessment of syndesmotic stability was more predictive of syndesmotic stability than any imaging parameters including radiographs and CT scans.

     

     

     

    OrthoCash 2020

     

  35. A 45-year-old male sustains a Gustilo and Anderson Type II open transverse humeral shaft fracture. He undergoes the treatment shown in Figures A and B. What is the advantage of this treatment choice as compared to antegrade intramedullary nailing?

     

     

     

     

     

     

    1. Decreased risk of post-operative elbow pain

    2. Decreased risk of radial nerve injury

    3. Decreased risk of reoperation

    4. Decreased risk of infection

    5. Decreased risk of blood loss Corrent answer: 3

    Humeral shaft fractures treated with antegrade intramedullary nailing (IMN) have a higher risk of receiving an additional operation compared to those treated with plate fixation as seen in Figures A and B. Additionally, there is a higher incidence of shoulder impingement following IMN compared to plate fixation. There has been no difference found between the incidence of infection, elbow pain or radial nerve injury when comparing the two treatment options. Blood loss is not lower in an open case compared to nailing.

     

    In a meta-analysis of 3 randomized studies comparing compression plate fixation to IMN fixation of humeral shaft fractures, Bhandari et al, found a significant relative risk reduction of 74% to reoperation with the use of plates and screws versus IMN. They also found a relative risk reduction of 90% in shoulder impingement with the use of plates and screws versus IMN. No significant difference was found between the two regarding rates of infection and/or radial nerve palsies.

     

    In a prospective randomized study comparing IMN fixation to plate fixation in humeral shaft fractures, McCormack et al, also showed a significantly higher rate or reoperation in patients treated with IMN fixation. They also showed no difference in functional outcome or pain scores between the two groups. There were more nonunions in the IMN group in these studies, but the numbers were small and it did not reach statistical significance. Previous studies have shown higher nonunion rates in the IMN patients.

     

     

     

    OrthoCash 2020

     

  36. A 27-year-old man sustains a Gustilo and Anderson type II open tibia fracture during a motorcycle accident. He had a full course of tetanus vaccination as an infant and child. He also had a tetanus booster vaccination 12 months ago when he began a new job. In addition to intravenous antibiotics, what tetanus prophylaxis should be administered?

    1. No prophylaxis required

    2. Tetanus vaccine

    3. Tetanus immune globulin

    4. Tetanus vaccine and tetanus immune globulin

    5. Tetanus vaccine and tetanus immune globulin with a booster vaccine required 6 months from now

    Corrent answer: 1

     

    Any time an open wound is encountered, the tetanus prophylaxis protocol should be initiated in the emergency room. The correct treatment depends upon the severity of the wound and the patient's tetanus vaccine status.

    Treatment may entail no further action, vaccination, or vaccination and administration of the tetanus immune globulin. The tetanus vaccine, booster, and immune globulin are used to enhance the immune response to clostridium tetani, a gram positive bacillus found in soil. In this case, the patient's tetanus had been updated within the past 5 years so he does not need an update of the vaccination or immune globulin.

     

    Illustration A is a concise table that can be used as an algorithm to provide appropriate tetanus prophylaxis. Of note, this may vary from nation to nation.

     

     

     

     

     

     

    OrthoCash 2020

     

  37. A 67-year-old woman slips on the ice while retrieving her mail and lands on her outstretched left hand. She complains of wrist pain and deformity. On physical exam she has no sensation of the volar thumb, index, and middle fingers. Radiographs are provided in Figure

    A. Two hours following closed reduction, the deformity is corrected, but the numbness and wrist pain is worsening. Which of the following interventions should be taken?

     

     

     

    1. Evaluation of volar compartment pressures with a needle monitor

    2. Icing and elevation of the arm with follow-up evaluation in 8 hours

    3. Immediate EMG evaluation of the left upper extremity

    4. Closed reduction, carpal tunnel release, and sugar tong splinting

    5. Emergent open reduction internal fixation with carpal tunnel release Corrent answer: 5

    The radiograph demonstrates a displaced distal radius fracture along with a scaphoid fracture, and the patient is presenting with neurologic deficits of acute carpal tunnel syndrome. This is a surgical emergency requiring release of the carpal tunnel to prevent permanent dysfunction.

     

    Dyer et al performed a retrospective cohort comparison of patients with distal radius fractures. Fracture translation was the most significant risk factor for development of acute carpal tunnel syndrome. Ipsilateral extremity fracture and multi-extremity trauma were trending, but not statistically significant risk factors.

     

    In a review article, Schnetzler describes the pathogenesis, evaluation, and treatment of acute carpal tunnel syndrome.

     

     

     

    OrthoCash 2020

  38. Which of the following techniques has not been shown to prevent valgus angulation during intramedullary nailing of proximal one-third tibia fractures?

    1. Use of a blocking screw lateral to midline in the proximal segment

    2. Use of femoral distractor

    3. Use of a lateral tibial nail starting point

    4. Use of supplementary plate and screw fixation

    5. Use of a suprapatellar nailing portal Corrent answer: 5

    Proximal tibial shaft fractures treated with intramedullary nails are most commonly malreduced with apex anterior and valgus deformities. Several techniques are available to overcome this malalignment: proximal and lateral nail starting point, usage of a femoral distractor or temporary plating, suprapatellar nailing, and lateral parapatellar approaches. Suprapatellar nailing portals have not been shown to affect coronal angulation - the concept is to affect the apex anterior deformity.

     

    A final technical trick is the usage of blocking (Poller) screws - the referenced article by Ricci et al had 100% correction and maintenance of reduction with usage of blocking screws without other adjunct techniques. These should be placed in the lateral aspect of the proximal and distal fragments when needed. Remember, the blocking screws should go in the concavity of the deformity, or where you don't want the nail to go.

     

    The referenced study by Krettek et al is a biomechanical evaluation of blocking screws in a tibial model that showed significantly increased strength when they were utilized.

     

    Illustration A demonstrates proper AP screws placed lateral to the nail (white large arrows) to prevent valgus deformation, and ML screws placed posterior to the nail (white small arrows) to prevent procurvatum.

     

     

     

     

     

    OrthoCash 2020

     

  39. A 25-year-old female falls from her horse and injures her left wrist. There are no open wounds and the hand is neurovascularly intact. Radiographs are provided in Figures A-C. Which of the following will best achieve anatomic reduction, restore function, and prevent future degenerative changes of the wrist?

     

     

     

     

     

     

     

     

    1. Long arm cast above the elbow for 6 weeks

    2. Long arm cast for 3 weeks followed by a short arm cast for 3 additional weeks

    3. Closed reduction and external fixation

    4. Closed reduction and percutaneous pinning

    5. Open reduction and internal fixation Corrent answer: 5

    The radiographs demonstrate an intra-articular distal radius fracture with dorsal angulation. Most importantly, there is a large dorsal articular fragment. Open reduction of this fragment is best acheived through an open approach.

    The choice of dorsal vs. volar approach is debatable, but most importantly the articular surface, radial height, and volar tilt must be reduced anatomically.

     

    Yu et al reviewed 104 distal radius fractures treated with either dorsal or volar plating. There was no difference in the rate of tendon irritation or rupture between the groups. Volar plating was associated with significantly more neuropathic pain complications.

     

    Illustrations A and B demonstrate the open reduction and internal fixation of this fracture with a dorsal plate.

     

     

     

     

     

     

     

     

    OrthoCash 2020

     

  40. Of all the pelvic ring injury types, anteroposterior compression type III pelvic ring injuries have the highest rate of which of the following?

    1. Head injury

    2. Pulmonary injury

    3. Traumatic amputation

    4. Need for transfusion

    5. Upper extremity fractures Corrent answer: 4

    Of the pelvic ring injuries, APC type III have the highest rate of mortality, blood loss, and need for transfusion. They also have a high rate of urogenital injury and abdominal organ injury. Lateral compression injuries (especially type III) have the highest rate of head injury. Vertical shear and combined injuries also have significant rates of concomitant injuries. The referenced article by Dalal et al is a review of Shock Trauma's pelvic ring injuries; they found significant increases in associated injuries as the grade of pelvic ring injury increased, regardless of mechanism/pattern. The aforementioned information was also found to be true with their patient review.

     

     

     

    OrthoCash 2020

     

  41. A 34-year-old male is an inpatient at a rehabilitation hospital after sustaining severe lower extremity injuries in a motor vehicle collision. As a result, his energy expenditure while ambulating is 40% above baseline after being fitted with an appropriate prosthetic prescription. What is this patient's most likely lower extremity amputation level?

    1. Unilateral long transtibial

    2. Unilateral average transtibial

    3. Bilateral transtibial

    4. Unilateral transfemoral

    5. Bilateral transfemoral

     

    Corrent answer: 3

     

    Bilateral transtibial amputee patients have an energy expenditure that is 40% above baseline levels.

     

    The comparative study by Huang et al measured energy consumption by indirect calorimetry and found that mean oxygen consumption was 9% higher for a unilateral transtibial amputee, 49% higher for a unilateral transfemoral amputee (Miller's Review states 65%), and 280% higher for a bilateral transfemoral amputee.

     

    The study by Traugh et al measured oxygen consumption in 9 transfemoral amputees. They found that crutch walking and prosthetic ambulation require the same amount of energy expenditure. Miller's Review also states that a long

    transtibial amputation expends only 10% above baseline compared to an average length transtibial amputation which expends energy 25% above baseline.

     

     

     

    OrthoCash 2020

     

  42. A 34-year-old female presents to the trauma bay with hemodynamic instability following a motor vehicle collision. A chest radiograph shows a left-sided hemothorax and her pelvis radiograph is shown in Figure A. Which of the following is the next most appropriate step in managment?

     

     

     

     

    1. Circumferential pelvic sheeting

    2. Retrograde urethrogram to evaluate for associated urologic injury

    3. Emergent transport to OR for pelvic anterior external fixator placement

    4. CT scan to assess for occult femoral neck fracture

    5. Bedside posterior pelvic C-clamp application Corrent answer: 1

    Unstable anteroposterior compression (APC) pelvic fractures are most appropriately managed with a pelvic binder or circumferential pelvic sheeting as described by Routt et al in the emergency room prior to definitive treatment. Illustration A demonstrates the utility of circumferential wrapping for the case shown in Figure A. Rapid, temporary fixation of unstable pelvic fracture patients with hemodynamic instability can be performed in the trauma bay. Pelvic binders can remain in place during further diagnostic tests such as

    pelvic vessel angiography.

     

    Bottlang et al performed a cadaveric study in JBJS of Young-Burgess type-II and III anteroposterior compression fractures and found that a pelvic binder reduced rotation instability by 61%.

     

    The study by Krieg et al followed 16 patients treated with pelvic binders and found that the binder reduced the pelvic fracture displacement by 9% which closely approximated the reduction achieved with definitive fixation.

     

    The Bottlang article published in JOT is a cadaveric study which determined that 180 +/- 50 Newtons of circumferential compression is needed to stabilize an unstable pubis symphysis diastasis. Tile Type A pelvic fractures are stable and include avulsion, iliac-wing, anterior-arch fracture due to a direct blow, or transverse sacrococcygeal fractures.

     

     

     

     

     

     

    OrthoCash 2020

     

  43. Excision of the 1 centimeter talar fragment shown in Figure A would lead to complete incompetence of which of the following structures?

     

     

     

    1. Bifurcate ligament

    2. Inferior peroneal retinaculum

    3. Lateral talocalcaneal ligament

    4. Arcuate ligament

    5. Posterior talofibular ligament Corrent answer: 3

    Figure A shows a lateral process talar fracture. Excision of a fragment of approximately 1 cubic centimeter is reported for acute injury as well as painful nonunion/fibrous union. Excision of this piece is shown to sacrifice the entire lateral talocalcaneal ligament as well as 10% of the anterior and posterior talofibular ligaments.

     

    Langer et al performed a cadaveric biomechanical analysis of such an excision and found that ankle and subtalar instability were not created with simple excision of this fragment.

     

    DiGiovanni et al found that only three ligaments attach to the lateral process of the talus: lateral talocalcaneal, anterior talofibular, and posterior talofibular, with the lateral talocalcaneal ligament inserting closest to the tip/apex of the lateral process.

     

    Illustration A shows an anatomical drawing of the ligaments of the lateral ankle. Illustration B shows a cadaveric image of the ligaments of the lateral ankle. The numbers in Illustration B correspond to the following structures: 1-calcaneofibular ligament, 2-lateral talocalcaneal ligament, 3-anterior talofibular ligament, 4-peroneal tubercle.

     

     

     

     

     

     

     

     

    OrthoCash 2020

     

  44. Isolated exchange reamed interlocking nailing is most likely indicated as the next step in treatment for which of the following clinical scenarios:

    1. Tibial shaft nonunion with a 4cm bone defect

    2. Infected tibial shaft nonunion

    3. Hypertrophic diaphyseal tibial nonunion

    4. Atrophic tibial shaft nonunion

    5. Hypertrophic metadiaphyseal distal tibia nonunion Corrent answer: 3

      If a hypertrophic nonunion is present, it is most likley a mechanical issue. Tibial diaphyseal hypertrophic nonunions (Illustration A) have approximately an 85-90% incidence of union with exchange reamed nailing. A nonunion that has bone loss or appears atrophic (Illustration B) will usually require improved mechanical stability as well as biological stimulation in the form of either autograft or an osteoinductive substance like BMP. A bone defect of up to 5-6cm in length can usually achieve union with bone grafting. In the presence of an infected nonunion, the infectious process needs to be addressed prior to the introduction of any revision hardware. If a patient does not show radiographic signs of tibial fracture union for 9 months and does not have progression toward healing for 3 consecutive months, then revision surgery would be indicated.

       

      Tempelman et al looked at 71 tibial shaft fractures treated with nonlocked or dynamically locked IM nails and found a loss of alignment in 11% of the fractures that were not transverse in nature. They concluded that these nailing techniques should not be used in the treatment of spiral or oblique tibial shaft fractures.

       

      Incorrect Answers:

      1-A 4cm bone defect could not be corrected with exchanged nailing alone, and would either need extensive grafting or bone transport

      2-An infected tibial shaft nonunion would require infection clearance prior to exchanged nailing

      4-Atrophic nonunions typically need biologic stimulation in the form of acute grafting or insertion of a BMP type substance

      5-Hypertrophic metadiaphyseal distal tibial nonunions can be treated with isolated exchanged nailing, however this does not have the same success as diaphyseal injuries. It is difficult to acheive appropriate stability to allow for fracture healing in the metadiaphyseal region, and other modalities such as plating need to be considered.

       

       

       

       

       

      OrthoCash 2020

       

  45. A 58-year-old man injures his knee in a high-speed motor vehicle collision. Radiographs and CT are shown in Figures A thru C. What is the most appropriate surgical plan based on the images provided?

     

     

     

     

     

     

    1. ORIF with medial and lateral plating with grafting of metaphyseal defect

    2. ORIF with lateral plating with grafting of metaphyseal defect

    3. ORIF with medial plating

    4. ORIF with lateral plating

    5. Percutaneous articular fragment reduction and screw fixation Corrent answer: 2

    The injury is a Schatzker type 2 with a significant split and depressed lateral tibial plateau fracture. The anteromedial cortex and medial plateau remain intact and connected to the tibial shaft making this a partial articular fracture (AO type B) and obviating the need for medial plate fixation. The best fixation strategy includes reduction of the articular surface with metaphyseal support with bone graft or bone substitute and a lateral plate for buttress support and subchondral screw support of the articular fragment.

    Marsh et al revisited the OTA and AO fracture and dislocation classifications to unify coding of these fractures. Illustration A shows the Schatzker classification of tibial plateau fractures.

     

     

     

     

     

     

    OrthoCash 2020

     

  46. Anterior penetration of an iliosacral screw through the sacral ala would most likely lead to weakness of which of the following movements?

    1. Hip flexion

    2. Hip adduction

    3. Knee extension

    4. Ankle plantarflexion

    5. Great toe dorsiflexion Corrent answer: 5

    Penetration of an iliosacral screw through the sacral ala would injure the ipsilateral L5 nerve root (great toe dorsiflexion). This can be avoided with proper understanding of the sacral anatomy as well as iliosacral screw starting points. The three required views for placement of this screw are: lateral sacral, pelvic inlet, and pelvic outlet.

     

    The referenced study by Ziran et al is an excellent review of fluoroscopic evaluation of screw placement. They reported that the anterior border of the S1 body is best seen with overlap of the S1 and S2 anterior cortex while the superior aspect of the S1 foramen is best seen with overlap of the S2 foramen on the superior pubic ramus.

     

    The referenced study by Routt et al reviewed 177 patients with pelvic ring

    injuries treated with these screws and found that quality triplanar imaging decreased intraoperative and postoperative complications. They also recommend supplemental fixation of iliosacral screws with posterior plating in noncompliant patients.

     

     

     

    OrthoCash 2020

     

  47. A 78-year-old male undergoes the procedure shown in Figure A for treatment of a femoral neck fracture. As the patient passes through mid-rise during sitting to standing using the affected leg, what portion of the acetabulum experiences the highest contact pressures?

     

     

     

     

    1. Posterior inferior

    2. Anterior superior

    3. Posterior superior

    4. Directly superior

    5. Anterior inferior

     

    Corrent answer: 3

     

    Figure A is an AP radiograph of a hip hemiarthroplasty.

     

    Contact pressures from an insturmented hip endoprosthesis can have important implications in both implant positioning and rehabilitation protocols.

    Hodge et al implanted a pressure-measuring Moore-type endoprosthesis in a patient who had sustained a displaced fracture of the femoral neck. They used this prosthesis to determine the measurement and telemetry of contact pressures in the hip for 36 months post-operatively. The highest pressure, eighteen megapascals, was recorded while the patient was getting up from a chair using the affected leg and was localized in the posterior superior portion of the acetabulum. This can be important in the post-operative care of acetabular fractures, as patients are at increased risk of loss of fixation with greater acetabular contact forces. Interestingly, peak pressures in vivo were found to be considerably higher than previously measured pressures in vitro.

     

    Incorrect Answers:

    1,2,4,5,-These regions of the acetabulum have less contact pressure compared to the posterior superior portion.

     

     

     

    OrthoCash 2020

     

  48. For an above knee amputation, each of the following is a benefit of adductor myodesis EXCEPT:

    1. Allows preservation of greater femoral length

    2. Provides a soft tissue cushion beneath the osseous amputation

    3. Improves the position of the femur to allow more efficient ambulation

    4. Creates dynamic balance of the amputated femur

    5. Improves prosthetic fit Corrent answer: 1

      Adductor myodesis enhances clinical function following an above knee amputation. An example is provided in illustration A. Adductor myodesis balances the pull of the hip abductors and flexors that insert proximally This prevents an abducted and flexed deformity that encumbers ambulation. It also creates a soft tissue envelope that pads the distal bony amputation and enhances suction fitting of the prosthesis. Performing the myodesis does not preserve femoral length. A little extra distal femur is actually resected to facilitate the myodesis insertion.

       

      In an instructional course lecture, Pinzur et al review the goals and techniques of lower extremity amputations. To achieve a successful amputation, the surgeon must establish reasonable goals with the patient, manage the soft tissue envelope, create an osseous platform for weight-bearing, and facilitate physical rehabilitation.

       

       

       

       

       

      OrthoCash 2020

       

  49. A 21-year-old male sustains the injury shown in Figures A through D. Which of the following is the most appropriate definitive treatment of this injury?

     

     

     

     

     

     

     

     

    1. Spanning external fixation

    2. Lateral locking plate

    3. Lateral buttress plate

    4. Posteromedial buttress plate

    5. Medial bridging plate Corrent answer: 4

    Figures A through D show a medial tibial plateau fx, best classified as a Schatzker IV. This injury is high-energy in nature and often requires a staged approach incorporating initial reduction and spanning external fixation.

     

    Appropriate treatment of this injury involves a medial buttress plate to hold the medial tibial condyle in position. This fracture pattern can be associated with a knee dislocation (see Illustration A). In a fracture-dislocation, the femur often displaces with this medial condylar fragment and is involved with a significant rate of neurovascular injury/compartment syndrome. Along with a proper vascular exam, ankle brachial indices (ABI) must be immediately taken and if abnormal, further vascular testing is warranted.

     

     

     

     

     

    OrthoCash 2020

     

  50. Lipohemarthrosis of the knee is most likely secondary to which of the following?

    1. Seronegative monoarticular arthritis

    2. Patellar tendon rupture

    3. Medial meniscus tear

    4. Medial patellofemoral ligament rupture

    5. Occult fracture

     

    Corrent answer: 5

     

    Lipohemarthrosis is formed when an intraarticular fracture occurs and can be detected with arthrocentesis or imaging such as xray, MRI, ultrasound, or CT. It is most commonly seen with occult tibial plateau fractures but can be associated with any intra-articular fractures. Up to three layers are visible on an MRI (fat/serum/cellular parts of blood), and this separation may take up to 3 hours to appear after injury. An example of hemarthrosis as seen on CT is shown in Illustration A. The referenced article by Ahn et al is a review of MRI findings in intraarticular knee injuries. They note that detection of lipohemarthrosis on an MRI is very sensitive and specific for intraarticular fracture. The referenced article by Schick et al reports that MRI can be as sensitive as arthrocentesis in detecting lipohemarthrosis (occult fracture).

     

     

     

     

     

    OrthoCash 2020

     

  51. The Lauge-Hansen classification of ankle fractures identifies characteristic fracture patterns based on mechanism of injury. What is the mechanism for the fracture pattern shown in Figure A?

     

     

     

     

    1. Supination-External Rotation

    2. Pronation-External Rotation

    3. Pronation-Abduction

    4. Supination-Adduction

    5. Supination-Abduction

     

    Corrent answer: 4

     

    The 4 categories in the Lauge-Hansen classification are: supination-adduction, supination-external rotation, pronation-external rotation, and pronation-abduction.

     

    The typical fracture pattern of supination-adduction is demonstrated in figure A and consists of a tension failure (transverse) fracture of the lateral malleolus combined with a vertical fracture of the medial distal tibia. There is frequently comminution where the fracture begins on the tibial plafond. This is caused by compression from the medial talar dome.

     

     

     

    OrthoCash 2020

     

  52. A 35-year-old male sustains a posterior column/posterior wall acetabular fracture. Which of the following is the preferred approach for open treatment of this injury?

    1. Modified Stoppa approach

    2. Extended iliofemoral approach

    3. Kocher-Langenbeck approach

    4. Ilioinguinal approach

    5. Combined anterior and posterior approach Corrent answer: 3

    Operative treatment is indicated for most displaced acetabular fractures to allow early ambulatory function and to decrease the chance of post-traumatic arthritis. Among the various surgical approaches, the Kocher-Langenbeck allows direct exposure of both the posterior column and posterior wall.

    Indications for using this exposure include posterior wall fractures, posterior column fractures, combined posterior wall/posterior column fractures, and simple transverse fractures.

     

     

     

    OrthoCash 2020

     

  53. Excision of heterotopic bone about the forearm or elbow can be done with limited recurrence rates as early as which of the following after initial injury?

    1. Once ankylosis of the forearm or elbow occurs

    2. 6 weeks

    3. 6 months

    4. 12 months

    5. 18 months

     

    Corrent answer: 3

     

    Excision of heterotopic bone about the elbow and forearm was classically treated once the bone was mature and no further bone development was occuring (bone scan became negative). However, several studies have shown that earlier removal before this point in time is safe, when done in conjunction with radiation therapy (XRT).

     

    The referenced study by McAuliffe et al is a retrospective review of heterotopic ossification (HO) about the elbow followed by 1000 cGY (5 fractions over 1 week) of XRT as early as 3 months post-injury. They were able to achieve an average arc of motion > 100 degrees.

     

    The other referenced study by Beingessner et al is a review of HO excision of the forearm. They found that excision and XRT, followed by 6 weeks of indomethacin, led to an increase of forearm motion from an average of 17 degrees to 136 degrees when the excision was done at 4 months post-injury.

     

     

     

    OrthoCash 2020

     

  54. When placing a percutaneous retrograde pubic ramus screw for fixation of an acetabular fracture, which of the following radiographic views can best ensure that the screw does not exit the posterior aspect of the superior pubic ramus?

    1. AP pelvis

    2. Outlet obturator oblique view

    3. Inlet iliac oblique view

    4. Iliac oblique view

    5. Obturator oblique view Corrent answer: 3

    As reviewed in the referenced article by Starr et al, when placing a retrograde pubic rami screw, the pelvic inlet iliac oblique view will best determine the anteroposterior placement of the screw in the pubic ramus. To ensure placement outside of the joint, the outlet obturator oblique is best, but all other views should be incorporated into determination of the position of

    fixation, as the corridor for this screw placement is quite narrow. Illustration A shows a left sided inlet iliac view on a pelvic bone model.

     

     

     

     

     

    OrthoCash 2020

     

  55. A 25-year-old male is involved in a motor vehicle accident and sustains the injury seen in Figure A. He is intubated in the field and receives 2 liters of LR and continues to be tachycardic and hypotensive. A massive transfusion protocol is initiated. Which of the following is true regarding the transfusion of packed red blood cells, platelets, and fresh frozen plasma?

     

     

     

    1. PRBC should be transfused until Hgb>8

    2. PRBC, platelets, and FFP should be transfused in equal ratios

    3. Platelets and fresh frozen plasma should be given when INR >1.4, platelet count <100,000

    4. FFP is not needed unless INR>1.5

    5. Platelets should not be transfused unless platelet count <10,000 Corrent answer: 2

    High energy pelvic injuries such as the one seen in Figure A continues to be a source of high mortality in orthopaedics. Active involvment of the orthpaedic surgeon in managing these life threatening injuries remains critical. A sheet or pelvic binder needs to be emergently applied in this clinical scenario. An aggressive resuscitation protocol must also be initiated. The review article by Hak et al discussed the advances in prehospital, interventional, surgical, and critical care that have led to increase survival rates for pelvic injuries. Gonzalez et al found that initial coagulopathy in trauma patients was associated with decreased survival. They noted that hypothermia and acidosis was well managed but pre-ICU coagulopathy was the most difficult to treat. They recommended early FFP in a FFP:PRBC ratio of 1:1

     

     

     

    OrthoCash 2020

     

  56. A 48-year-old female sustains the injury seen in Figure A. Which of the following preoperative variables has been shown to be associated with improved outcomes following surgical treatment of this injury pattern?

     

     

     

    1. Patients with a heavier workload

    2. Patients receiving Worker's Compensation

    3. Gissane angle of 140°

    4. Böhler angle of > 15°

    5. Comminuted posterior facet Corrent answer: 4

    Figure A shows a displaced intra-articular calcaneus fracture. Studies have found in this fracture pattern a Böhler angle of greater than 0-15° is associated with improved outcomes.

     

    Buckley et al (2002) found that patients with the following preoperative variables have improved outcomes with ORIF vs non-operative treatment: women, non-workers' compensation, younger males (<30), patients with a higher Böhler angle (>0-15°), patients with a lighter workload, and fracture patterns with single, simple displaced intra-articular calcaneal fracture.

     

    Buckley et al (1992) found that anatomic or near anatomic reductions (<2mm stepoff) enhance outcomes while comminuted reductions or fractures without reduction produce long-term outcomes that are less satisfactory.

     

    Bajammal et al investigated intra-articular calcaneus fractures and reported

    that patients who were NOT receiving Workers' Compensation, were younger (less than twenty-nine years old), had a moderately lower Böhler angle (0 degrees to 14 degrees ), a comminuted fracture, a light workload, or an anatomic reduction or a step-off of < or =2 mm after surgical reduction (p = 0.04) scored significantly higher on the scoring scales after surgery compared with those who were treated nonoperatively.

     

    Illustration A shows the Böhler angle, which is an angle between the following two lines: 1) a line connecting anterior process and highest point on posterior articular surface, 2) a line connecting highest point on posterior articular surface and superior tuberosity. A normal Böhler angle is 20-40°. Flattening of the Böhler angle represents collapse of the posterior facet. Illustration B shows both the critical angle of Gissane (G) and the Boehler angle (B).

     

    Incorrect Answers:

    Answer 1. Patients with heavier workloads have worse outcomes with surgery. Answer 2. Patients with Worker's Compensation claims have worse outcomes with surgery.

    Answer 3. A normal angle of Gissane does not necessarily indicate posterior facet depression, and no improvements with surgical treatment have been shown.

    Answer 5. A comminuted posterior facet is associated with worse outcomes.

     

     

     

     

     

     

     

     

    OrthoCash 2020

     

  57. A 26-year-old male sustains a comminuted, intra-articular calcaneus fracture and subsequently undergoes operative intervention as shown in Figure A. Postoperatively in the recovery room, he presents with an isolated, fixed flexed great toe. What is the most likely etiology of this finding?

     

     

     

    1. Use of a lateral extensile approach to the calcaneus

    2. Calcaneal tuberosity varus malalignment

    3. Use of screws in the constant fragment that are too long

    4. Missed foot compartment syndrome

    5. Plantar nerve palsy Corrent answer: 3

    The flexor hallucis longus runs along the medial aspect of the hindfoot, medial to the posterior facet and inferior to middle facet just under sustentaculum tali(constant fragment) This tendon can be injured with poor drilling technique or use of screws that are too long during reduction and fixation of a calcaneus fracture using a lateral plate. If present, the great toe will be in a fixed, flexed position from the tendon being tethered over a screw.

     

    Figure A shows a calcaneus fracture with a lateral plate and screws.

     

     

     

    OrthoCash 2020

     

  58. All of the following are characteristic of end-stage septic shock EXCEPT?

    1. Increased systemic vascular resistance

    2. Decreased cardiac output

    3. Decreased pulmonary capillary wedge pressure

    4. Decreased central venous pressure

    5. Increased mixed venous oxygen saturation Corrent answer: 1

    Septic shock is associated with decreased systemic vascular resistance, decreased cardiac output, decreased pulmonary capillary wedge pressure, decreased central venous pressure, and increased mixed venous oxygen saturation. Septic shock is different from hypovolemic shock in that the systemic vascular resistance is decreased, whereas it is increased in hypovolemic shock. Septic shock is a medical emergency caused by decreased tissue perfusion and oxygen delivery as a result of severe infection and sepsis, though the microbe may be systemic or localized to a particular site. It can cause multiple organ dysfunction syndrome (formerly known as multiple organ failure) and death. Its most common victims are children, immunocompromised individuals, and the elderly, as their immune systems cannot deal with the infection as effectively as those of healthy adults. The mortality rate from septic shock is reported to be as high as 25%-50%.

     

     

     

    OrthoCash 2020

     

  59. A 27-year-old female sustains a twisting injury to her leg while rollerblading. Radiographs of the tibia and fibula are provide in Figures A and B. A closed reduction is performed and the patient is placed in a long leg cast. Radiographs following cast placement are provided in Figures C and D. The decision is made to proceed with closed treatment instead of operative. Which of the following is most likely to occur with nonoperative management?

     

     

     

     

     

     

     

     

    1. Malunion due to unacceptable coronal alignment

    2. Malunion due to unacceptable sagittal alignment

    3. Fracture displacement due to the mechanism of injury

    4. Fracture displacement due to the age of the patient

    5. Shortening due to the oblique nature of the tibia fracture Corrent answer: 5

    The radiographs demonstrate a distal third spiral tibia shaft with a proximal fibula fracture. The coronal and sagittal alignments are within acceptable limits. The oblique fracture is at risk of shortening, especially with a concomitant fibular fracture.

     

    Acceptable alignment for non-operative treatment of tibia fractures is defined as:

    <5 degrees varus-valgus angulation,

    <10 degrees anterior/posterior angulation

    >50% cortical apposition

    <1 cm shortening

    < 10 degrees rotational alignment

     

    Sarmiento et al. reviews fracture bracing for the treatment of long bones. With reference to tibial fractures, bracing is best for transverse fractures.

    Shortening is difficult to control in oblique fractures. However, shortening is

    usually less than 15 mm and does not result in functional limitations. He reports union in 97% of tibial fractures treated with bracing.

     

    One year follow-up radiographs are provided in Illustrations A and B. The patient presented in this question went on to functional healing.

     

     

     

     

     

     

     

     

    OrthoCash 2020

     

  60. A lateral distal femoral locking plate is not an appropriate implant for which of the following fractures?

     

     

     

     

     

     

     

     

     

    1. Figure A

    2. Figure B

    3. Figure C

    4. Figure D

    5. Figure E

     

    Corrent answer: 2

     

    Figure B demonstrates an isolated medial femoral condyle fracture. Lateral locked plating is not an appropriate technique for this fracture.

     

    The fracture shown in Figure B is an AO B type (partial articular fracture). This fracture is best treated with open reduction internal fixation through a medial approach, with lag screw and buttress plate fixation.

     

    Figures A, C, D and E show supracondylar distal femur fractures that can be treated with ORIF with a fixed-angle device such as lateral locked plating.

     

    Incorrect Answers:

    Answer 1. Fixed angle fixation is appropriate for comminuted extra-articular distal femur fractures.

    Answers 3, 4, 5. ixed angle fixation is appropriate for comminuted intraarticular distal femur fractures. Non-locked plating for type C (complete articular) distal femur fractures has been associated with varus malalignment.

     

     

    OrthoCash 2020

     

  61. A 70-year-old female with a history of poorly controlled diabetes mellitus presents with purulent ulcers along the plantar aspect of her right forefoot and exposed metatarsal bone. She elects to undergo an amputation. She is insensate to the midfoot bilaterally. Her ankle-brachial index (ABI) for her right posterior tibial artery is 0.4. Further preoperative evaluation demonstrates a transcutaneous oxygen pressure of 45 and an albumin of 3.4. Which of the following would be a contraindication to performing a Syme amputation (ankle disarticulation) in this patient?

    1. Albumin of 3.4

    2. Active osteomyelitis

    3. ABI of 0.4 for the posterior tibial artery

    4. Transcutaneous oxygen pressure of 45

    5. Peripheral neuropathy

     

    Corrent answer: 3

     

    A Syme amputation (ankle disarticulation) is a function-preserving amputation option that allows for terminal weight bearing, however strict criteria must be met for a patient to undergo successful Syme amputation. An ankle-brachial index (ABI) less than 0.5 for the posterior tibial artery in a patient with diabetes would be a contraindication for this procedure as success is dependent on the vascular supply of posterior tibial artery to the plantar flap and heel pad.

     

    Pinzur et al retrospectively reviewed their results when performing a single-stage Syme ankle disarticulation in patients with diabetes either for peripheral neuropathy or infection. Patients with ABIs less than 0.5 for the posterior tibial artery had significantly decreased healing rates and smokers had a three-fold increased risk of postoperative infection.

     

    Incorrect Answers:

    Answer 1: Albumin of 3.4 indicates adequate preoperative nutrition. Albumin of

    2.0 or less would be concerning for increased wound healing risk.

    Answer 2: Osteomyelitis was the indication for amputation in a number of diabetic patients in the referenced study, and should not preclude Syme amputation unless the hindfoot and/or distal tibia is involved

    Answer 4: Transcutaneous 02 pressure of 45 signifies adequate oxygenation and vascularity. Less than 30 is generally considered a cutoff for adequate vascularity.

    Answer 5: Peripheral neuropathy with recurrent ulceration was an indication for amputation in the referenced study.

     

     

     

    OrthoCash 2020

     

  62. A 35-year-old-male sustains the fracture seen in Figure A. Which of the following reduction forces must be applied to the proximal fragment to correct the deformity commonly seen in these fractures?

     

     

     

     

    1. Adduction and extension

    2. Abduction and extension

    3. Adduction and flexion

    4. Abduction and flexion

    5. External rotation

     

    Corrent answer: 1

     

    Figure A demonstrates a displaced subtrochanteric femur fracture with an intact lesser trochanter. The pull of iliopsoas on the lesser trochanter as well as the intact external rotators and gluteal musculature results in the the proximal fragment being in a flexed and externally rotated or abducted position (the most common post operative deformity). Reduction manuevers must be biologically friendly but also counteract the flexion/abduction moment. Lundy's review article discusses evaluation and treatment of subtrochanteric fractures. The review article details the various implants often used which include 95 degrees plates, femoral reconstruction nails, or trochanteric femoral nails with interlocking options. Lundy's article discourages the use of the 135 degree screw and side plate combo due to high failure rates in these fracture patterns. Bedi et al also review treatment of these fractures and discuss common

    problems of malunion, nonunion, and implant failure. The article reviews reduction techniques that are soft tissue friendly, as well as the use of appropriate implants in these fracture types.

     

     

     

    OrthoCash 2020

     

  63. A 12-year-old male sustains an ulnar fracture with an associated posterior-lateral radial head dislocation. After undergoing closed reduction, the radiocapitellar joint is noted to remain non-concentric. What is the most likely finding?

    1. Lateral ulnar collateral ligament disruption

    2. Anterior band of the medial collateral disruption

    3. Posterior band of the medial collateral ligament disruption

    4. Annular ligament interposition

    5. Anconeus muscle interposition Corrent answer: 4

    In pediatric Monteggia fractures the annular ligament is commonly interposed in the radiocapitellar joint.

     

    Bado initially described and classified Monteggia fractures. The most common injury pattern is an extension type 1 with anterior radial head dislocation and apex anterior ulnar shaft fracture. The apex of the ulna fracture determines the direction of the radial head subluxation or dislocation. Adults typically require ORIF of the ulna. These fractures in children are often treated non-operatively with closed reduction if the ulna fracture is transverse and stable. Type III is the one most commonly associated with irreducibility of the radial head because of interposition of the annular ligament. The incidence of posterior interosseous nerve injury is high with this lesion. The nerve deficit usually completely resolves rapidly and spontaneously.

     

    Tan et al reviewed their treatment of 35 children with Type I and Type III Monteggia fractures. All radial heads were explored and the interposed annular ligament was stretched out of the joint space. They noted that none of the patients had any recurrent dislocation or subluxation.

     

    Ring et al in their review stress the importance of an anatomic reduction of the ulna to restore the reduction of the radial head.

     

     

     

    OrthoCash 2020

  64. All of the following are factors associated with transfer of patients to Level 1 trauma centers EXCEPT:

    1. Male

    2. Medicaid insurance

    3. Injury severity score of 36

    4. Caucasian race

    5. One or more comorbidity Corrent answer: 4

      Caucasian race has not been found to be a predictor for transfer to a Level 1 trauma center.

       

      The retrospective case-control study by Koval et al found that African-American race, presence of medical comorbidity, medicaid insurance, and male gender are predictors for transfer of patients to a trauma center that have ISS scores less than 9.

       

      The article by Nathens et al found that lack of insurance was an independent predictor for transfer to a trauma center after adjusting for differences in injury severity. An injury severity score of 36 represents a patient that has sustained life-threatening polytrauma and should be transferred to a Level 1 trauma center.

       

       

       

      OrthoCash 2020

       

  65. A 34-year-old male falls from a roof and sustains a right elbow dislocation that is closed reduced in the emergency room. An AP radiograph is shown in Figure A. This injury pattern is at highest risk for which of the following?

     

     

     

    1. Anterior interosseous nerve palsy

    2. Varus posteromedial rotatory instability

    3. Posterior interosseous nerve palsy

    4. Valgus posterolateral rotatory instability

    5. Elbow instability when pushing oneself up from a seated position in a chair Corrent answer: 2

    Anteromedial coronoid facet fracture and LCL injury following an elbow dislocation is commonly associated with varus posteromedial rotatory instability. Varus and posteromedial rotation force on the forearm results in rupture of the LCL from its humeral origin. As the LCL ruptures, the medial coronoid process is fractured as it impacts against and under the medial trochlea. Fracture involvement of the sublime tubercle, where the MCL attaches, can lead to more instability. Ulnar neuropathy can be seen following this injury pattern but AIN and PIN nerve palsy do not commonly characterize this injury pattern.

     

    The review article by O'Driscoll highlights key points in diagnosis and management of capitellum, distal humerus, coronoid, and terrible triad injuries.

     

    The article by Doornberg and Ring is a Level 4 study of 18 patients that sustained varus posteromedial rotational injuries resulting in anteromedial facet coronoid fractures. They found that lack of fixation at injury or malunion of the anteromedial facet were significant predictors of suboptimal functional outcome and development of arthrosis.

     

    The anteromedial facet is highlighted in yellow as displayed in Illustration A. Illustration B depicts the lateral collateral ligament injury also evident during

    varus stress fluroscopic examination, due to tension failure of the LCL off its humeral origin during the various mechanism.

     

     

     

     

     

     

     

     

     

    OrthoCash 2020

     

  66. Which of the following factors is most associated with malrotation during antegrade or retrograde femoral nailing?

    1. Surgeon experience

    2. Level of primary fracture line

    3. Use of a piriformis starting portal

    4. Fracture comminution

    5. Closed reduction technique Corrent answer: 4

    Femoral malrotation after intramedullary nailing is unfortunately a possibility with either antegrade or retrograde nailing techniques. Malrotation and iatrogenic length changes are most common when comminution is present, as cortical reads are inherently limited.

    Hufner et al report that malrotation (internal or external >15 degrees) was seen in 22% of their patients via CT scan after intramedullary nailing. There was a significant difference depending on the time of surgery, with significantly more malrotation during the night shift. Increased fracture comminution also significantly increased malrotation rates.

     

    Incorrect Answers:

    1,2,3,5: No significant increases were seen with the other answers listed above.

     

     

     

    OrthoCash 2020

     

  67. What is the most common type of malalignment after intramedullary nailing of distal 1/3 extra-articular tibia fractures using a infrapatellar approach when compared with plating?

    1. Varus

    2. Valgus

    3. Translational

    4. Shortening

    5. Apex anterior

     

    Corrent answer: 2

     

    The most common type of malalignment after intramedullary nailing of distal 1/3 extra-articular tibia fractures using an infrapatellar approach when compared with plating is valgus malalignment.

     

    Fixation of distal one-third tibial shaft fractures can be successfully treated with either intramedullary nailing or plating. The literature describes advantages and disadvantages to both approaches, however intramedullary nailing has been shown to lead to increased rates of valgus malunion. Recent studies have shown that using a suprapatellar approach may decrease the incidence of valgus malalignement.

     

    Vallier et al performed a randomized prospective study to compare plate and nail stabilization for distal tibia shaft fractures by assessing complications and secondary procedures. One-hundred and four patients were randomized to either reamed intramedullary nailing, or medial distal tibia plate fixation.

    Primary angular malalignment was identified in 17 patients (16.3%). This included four patients treated with tibial plating (8.3%) and 13 patients treated with nails (23%, P = 0.02). Eight of these (7.7% of all patients) had malalignment between 6° and 10° of angulation. Valgus was the most common

    angular deformity, accounting for 70% of angular deformity cases.

     

    Avilucea et al. looked at the immediate postoperative alignment of distal tibia fractures (within 5 cm of the tibial plafond) treated with suprapatellar intramedullary nail (IMN) insertion compared with the infrapatellar technique. They found primary angular malalignment of ≥5 degrees occurred in 35 (26.1%) patients with infrapatellar IMN insertion and in 5 (3.8%) patients who underwent suprapatellar IMN insertion. They conclude suprapatellar IMN technique results in a significantly lower rate of malalignment compared with the infrapatellar IMN technique.

     

    Incorrect Answers:

    Answer 1, 3,4, and 5: Vallier et al. found that valgus was the most common angular deformity.

     

     

     

    OrthoCash 2020

     

  68. A 33-year-old secretary presents three months after a motor vehicle collision with a mild asymmetry to her sternal area and difficulty swallowing. She denies any complaints of respiratory distress or upper extremity paresthesias. Her upper extremity neurovascular exam shows no deficits. A 3-D computed tomography image is shown in Figure A. What is the most appropriate treatment for this patient?

     

     

     

     

    1. Nonoperative treatment with a sling and unrestricted activity in 3 months

    2. Open reduction in the operating room with thoracic surgery back-up

    3. Closed reduction in the office with local anesthetic

    4. Closed reduction in the operating room with thoracic surgery back-up

    5. Nonoperative treatment with immediate unrestricted active range of motion of the shoulder

    Corrent answer: 2

     

    The clinical presentation is consistent with a chronic sternoclavicular dislocation, which is defined as being greater than 3 weeks old. The 3D CT image shows posterior displacement of the medial clavicle relative to the sternum. Chronic anterior dislocations are recommended to be treated conservatively, especially if not symptomatic, but as this is a posterior dislocations, current recommendations are to treat them with reduction in order to avoid delayed issues with the medial clavicle interacting with the mediastinal structures.

     

    The review article by Wirth and Rockwood notes the following complications with posterior dislocation: respiratory distress, venous congestion or arterial insufficiency, brachial plexus compression, and myocardial conduction abnormalities. They recommend reconstruction of the costoclavicular ligaments with resection of the medial claviclar head as needed for unstable or symptomatic injuries.

     

     

     

    OrthoCash 2020

     

  69. A 35 year-old female presents after prolonged extrication from a motor vehicle collision complaining of severe pelvic pain. Physical examination reveals diminished perianal sensation. She is otherwise neurologically intact. Figures A through D are radiographs and representative CT cuts of her injury. Which of the following nerve roots has likely been injured by the acute trauma?

     

     

     

     

     

     

     

     

     

     

     

    1. L3

    2. L4

    3. L5

    4. S1

    5. S2

     

    Corrent answer: 5

     

    The clinical scenario is consistent with a high-energy sacral fracture. The radiographs in figures A and B demonstrate a sacral fracture with posterior displacement of the right hemipelvis seen on the inlet view. Figures C and D are axial and sagittal CT images which show a displaced fracture of the right

    hemisacrum along with a transvere fracture component through the S3 body . Diminished perianal sensation is concerning for an S2 nerve root injury.

     

    Mehta et al reviewed the current management of sacral fractures. They note that the S1 and S2 nerve roots are more likely to be injured with sacral fractures as they occupy 1/3 to 1/4 of the neural foramina, as opposed to S3 and S4, which only occupy 1/6 of the neural foramina.

     

    Robles reviewed the current literature to ascertain principles of evaluation and treatment for transverse sacral fractures. The author notes that injury to nerve roots S2 to S5 is manifested by impairment of urinary and anal continence and sexual function.

     

    The first illustration demonstrates the sacral nerve root dermatomal distribution. The second shows a pelvic cadaver dissection demonstrating the sacral nerve roots as they exit the foramina.

     

     

     

     

     

     

     

     

    OrthoCash 2020

     

  70. A 44-year-old male is struck by a vehicle while riding his bike. In the trauma bay, he complains of right shoulder pain . Upper extremity physical exam reveals no neurologic deficits, and an initial radiograph of the shoulder is shown in Figure A. A CT scan of the shoulder shows 1cm of posterior displacement of the tuberosity fragment. Which of the following is true regarding this injury?

     

     

     

    1. It is usually associated with a posterior shoulder dislocation

    2. The subscapularis muscle is the main deforming force

    3. Non-operative treatment of this displaced injury results in good long term shoulder function

    4. Open reduction and internal fixation is the treatment of choice

    5. Associated rotator cuff tears are uncommon Corrent answer: 4

    The radiograph in Figure A demonstrates a posteriorly displaced greater tuberosity fracture. These injuries are often associated with anterior shoulder dislocations, and concomitant rotator cuff tears. The subscapularis attaches to the lesser tuberosity, and is not a deforming force. Open reduction and internal fixation (ORIF) is usually the treatment of choice, and it is well accepted that more than 5mm of displacement is an indication for surgery in patients that require overhead function of the arm.

     

    Flatow et al evaluated 12 patients who were an average of five years status post ORIF of displaced greater tuberosity fractures. All fractures healed without postoperative displacement. Six patients had an excellent result and six had a good result.

     

    Platzer et al retrospectively analyzed functional and radiographic results of 52 patients with operative treatment of displaced greater tuberosity fractures at an average time of 5.5 years from surgery compared to 9 patients with equivalent injuries treated non-operatively. Evaluation of the results of the surgical study group and the nonoperative control group, patients with reduction and fixat ion of greater tuberosity fractures had significantly better

    results on shoulder function than did those with conservative treatment.

     

     

     

     

    OrthoCash 2020

     

  71. Which of the following is a recognized predictor of mortality after hip fracture?

    1. American Society of Anesthesiologist (ASA) classification

    2. Post-operative weight bearing status

    3. Fracture comminution

    4. Fixation device used

    5. Type of anesthetic used Corrent answer: 1

    American Society of Anesthesiologist (ASA) classification is predictive of post-surgical mortality in hip fracture patients.

     

    The ASA classification (detailed in Illustration A) was initially developed in 1963 and has been shown to be predictive of post-surgical mortality in hip fracture patients. Basic categories are as follows: 1= normal, healthy; 2= mild systemic disease; 3= severe systemic disease, not incapacitating; 4= severe incapacitating systemic condition, constant threat to life; 5= moribund patient; 6 = brain dead, organs being donated.

     

    Richmond et al. looked at 836 patients treated for a hip fracture and found that this injury is not associated with significant excess mortality among patients older than age 85. However, in younger patients, those with ASA classifications of 3 or 4 have significant excess mortality following hip fracture that persists up to 2 years after injury.

     

    Holt et al. investigated the relationship between a number of patient and management variables and mortality after surgery for fracture of the hip. Data relating to 18,817 patients were obtained from the Scottish Hip Fracture Audit database. They found that type of anesthetic did not adversely affect the 30 or 120 day mortality rate.

     

     

     

     

     

    OrthoCash 2020

     

  72. A 25-year-old male sustains an ankle fracture dislocation and undergoes open reduction and internal fixation. He returns to clinic five months following surgery complaining of continued ankle pain and instability with weight bearing. His immediate post-operative AP radiograph is seen in Figure A. Which of the following could have prevented this patient from developing persistent pain?

     

     

     

    1. Deep deltoid ligament repair

    2. Quadricortical syndesmotic screw fixation

    3. Restoration of fibular length and rotation

    4. Lateral collateral ligament complex repair

    5. Use of two syndesmotic screws Corrent answer: 3

    The patient presents with continued ankle pain and instability following open reduction and internal fixation. The radiograph in figure A demonstrates inadequate restoration of fibular length, likely leading to continued tibiotalar instability.

     

    Illustration A demonstrates fibular malreduction with dislocation of the fibula anterior to the tibial incisura. Illustration B shows a comminuted fibula fracture along with a measurement of length from an intact fibula. The arc from the lateral process of the talus to the peroneal groove of the distal fibula is known as the "dime" sign and should remain unbroken if fibular length has been restored. Illustration C demonstrates the use of a push-pull screw and lamina spreader to regain length intraoperatively for a comminuted fibula fracture.

     

    Chu and Weiner review management of malunions of the distal fibula. The authors state that restoration of fibular length, alignment and rotation leads to reduction of the talus, provides a buttress to talar motion in the setting of an incompetent deltoid, and allows the syndesmotic ligaments to heal at the appropriate tension.

     

    Wikeroy et al conducted a study of patients from a prior prospective, randomized control trial comparing different methods of syndesmotic fixation. There was no significant difference in outcomes between tricortical or quadricortical 3.5mm screw fixation, however worse outcomes were seen with associated posterior malleolar fractures, obesity, a difference in sydesmotic width of 1.5mm or greater, and a CT confirmed tibio-fibular synostosis.

     

    Sinha et al present a simple technique for fibular lengthening in the setting of distal fibula malunion. They found high union rates and improved AOFAS scores at short-term follow up with their technique.

     

     

     

     

     

     

     

     

     

     

     

    OrthoCash 2020

     

  73. A 34-year-old male presents with the right posterior wall acetabular fracture shown in Figure A. What is the most accurate method to test for hip stability in this patient?

     

     

     

    1. The Keith method

    2. The Moed method

    3. The Calkins method

    4. Dynamic fluoroscopic examination of the hip under anesthesia

    5. A history of associated hip dislocation Corrent answer: 4

    Dynamic fluoroscopic examination of the affected hip under anesthesia is considered the best method of predicting hip stability. Fragment size, which can be calculated using the Keith, Moed, or Calkins method, can be used to predict hip stability radiographically, however they are not as accurate. In general it is thought that posterior wall fractures involving less than 20% of the posterior wall are stable, whereas those involving greater than 40%-50% are unstable. Unfortunately, this leaves an indeterminent zone (20-40%) which does not provide guidance in treatment.

     

    Moed et al retrospectively reviewed 33 patients with posterior wall fractures who underwent dynamic fluoroscopic stress testing and compared the results of this testing to the Moed, Calkins, and Keith method of hip stability prediction. They found that the Moed method is the only reliable technique that is predictive of hip stability for small fracture fragments while also being predictive of instability for large fracture fragments. However, they also stated that there remain a substantial number of fractures involving 20% or more of the posterior wall that are both stable and unstable by examination under anesthesia. Therefore, they recommend dynamic fluoroscopic examination for assessment of hip stability in the presence of a posterior wall fracture.

     

    Moed et al in their second paper reviewed all patients with less than a ≤50% of the acetabular wall fracture, adequate imaging, and documented EUA results.

    The group looked at multiple variables including fracture fragment size,

    superior exit point of the fracture, center-edge angle, acetabular index, Tönnis angle, lateralized head sign, crossover sign, posterior wall sign, ischial spine sign, and hip version. Their conclusion was that no one variable was able to predict stability or instability and they continue to recommend EUA.

     

    Tornetta et al conducted a study in which dynamic fluoroscopic stress views were taken of 41 acetabular fractures that met the criteria for non-operative management to determine subtle signs of instability. Of the 41 fractures, 38 were found to be stable and 91% of these had good or excellent outcomes at

    2.7 years. They concluded that dynamic stress views can identify subtle instability in patients who would normally be considered for non-operative treatment.

     

    incorrect answers:

    1. > Keith Method - Depth of the fracture segment in injured hip is compared to the contralateral intact posterior wall depth at the level of the fovea

       

    2. > Moed - Depth of the fracture segment in the injured hip is compared to contralateral posterior wall depth at the level of the greatest amount of fracture involvement

       

    3. > Calkins - Length of posterior acetabular arc from each hip is compared at the level of the greatest amount of fracture involvement.

     

     

     

    OrthoCash 2020

     

  74. A 23-year-old male is an unrestrained driver in a motor vehicle accident and sustains an unstable pelvic ring fracture. During fluoroscopic-aided fixation, a lateral sacral view is used for proper placement of which of the following fixation methods?

    1. Anterior column percutaneous screw placement

    2. Posterior column percutaneous screw placement

    3. Posterior iliosacral plating

    4. Supra-acetabular pin placement

    5. Percutaneous iliosacral screw placement Corrent answer: 5

    The lateral sacral view is used to place percutaneous iliosacral screws. Sacral alar morphology has been shown to be variable from patient to patient.

    Therefore, intraoperative fluoroscopy is recommended. During placement of the screws, the L5 nerve root is at risk.

    Routt et al (1997) examined the sacral slope and sacral alar anatomy in cadavers and a series of patients. They determined that the pelvic outlet and lateral sacral plain films provide the best plain radiographic view of the sacral ala. They recommended routine usage of these views intraoperatively to guide screw placement.

     

    Routt et al (2000) reported on the early complications of percutaneous placement of iliosacral screws for treatment of posterior pelvic ring disruptions. While technically challenging, this technique leads to less blood loss and lower rates of infection compared to traditional open techniques.

     

    Barei et al described methods of anterior and posterior pelvic ring disruptions. They determined that successful placement depends on accurate closed reduction, excellent intraoperative fluoroscopic imaging, and detailed preoperative planning. Early treatment decreased hemorrhage, provides patient comfort, and allows early mobilization.

     

     

     

    OrthoCash 2020

     

  75. A 23-year-old female is an unrestrained driver in a motor vehicle collision, sustaining the injury shown in Figure A. She subsequently undergoes reduction and percutaneous bilateral iliosacral screw placement. Which of the following is the most likely neurologic complication associated with percutaneous iliosacral screw insertion?

     

     

     

     

    1. Weakness in knee extension

    2. Decreased patellar reflex

    3. Weakness in great toe extension

    4. Weakness in ankle plantar flexion

    5. Decreased Achilles reflex Corrent answer: 3

    Figure A shows an unstable bilateral pelvic ring injury. Percutaneous posterior iliosacral screw fixation places the L5 nerve root at risk as it courses across the sacral ala. Injury to the L5 nerve root would typically result in weakness in great toe extension and sensory changes on the dorsum of the foot. It is important to notice that L5 often partially innervates tibialis anterior along with L4, so weakness to ankle dorsiflexion may be present as well. Illustration A shows the post-operative films with bilateral iliosacral screws.

     

    Routt et al examined the sacral slope and sacral alar anatomy in cadavers and a series of consecutive patients. They determined that the pelvic outlet and lateral sacral plain films provide the best plain radiographic views of the sacral ala. They recommended routine usage of these views intraoperatively to guide screw placement.

     

    In another study, Routt et al reported on the early complications of percutaneous placement of iliosacral screws for treatment of posterior pelvic ring disruptions. While technically challenging, this technique leads to less blood loss and lower rates of infection compared to traditional open techniques.

     

    Illustration B displays the root diagrams for sensation, reflex, and motor of the L4-S1 nerves.

     

    Incorrect answers:

    1: Weakness to knee extension would be caused primarily by an injury to the L4 nerve root.

    2: Decreased patellar reflex would be caused primarily by an injury to the L4 nerve root.

    4: Weakness in ankle plantar flexion would be caused primarily by an injury to the S1 nerve root.

    5: Decreased Achilles reflex would be caused primarily by an injury to the S1 nerve root.

     

     

     

     

     

     

     

     

    OrthoCash 2020

     

  76. A 20-year-old patient presents after jumping from the window of a burning building with a sacral fracture. Which of the following fracture patterns seen in Figures A through E would give this patient the highest risk of associated nerve injury?

     

     

     

     

     

     

    1. Figure A

    2. Figure B

    3. Figure C

    4. Figure D

    5. Figure E

     

    Corrent answer: 1

     

    Answering this question relies on knowledge of the Denis classification of sacral fractures and their associated risks of nerve injury. Figure A represents a Denis Zone 3 (medial to the foramina) sacral fracture, which has the highest associated risk of nerve injury.

     

    Denis et al outlined a novel classification system of sacral fractures based on the position of the fracture line relative to the sacral foramina. The authors found a 56.7% incidence of nerve injury in fractures that extended medial to the sacral foramina (zone 3), compared with 28.4% for fractures through the

    foramina (zone 2), and 5.9% for fractures lateral to the foramina (zone 1).

     

    Mehta et al reviewed the current principles for management of sacral fractures. They note that bowel, bladder and sexual dysfunction occur in 76% of patients with zone 3 sacral fractures.

    Illustration A below demonstrates the Denis classification of sacral fractures. Incorrect Answers:

    1. Figure B shows a zone 1 sacral fracture, which has a 5.9% incidence of

      nerve injury

    2. Figure C shows a zone 2 sacral fracture, which has a 28.4% incidence of nerve injury

    3. Figure D shows a sacroiliac joint dislocation, not a sacral fracture

    4. Figure E shows a zone 1 sacral fracture with an associated iliac fracture (crescent fracture)

     

     

     

     

     

     

    OrthoCash 2020

     

  77. A 35-year-old male is involved in a motor vehicle accident and suffers the fracture shown in Figure A. This is an isolated shoulder injury, and he has no neurologic deficits on physical exam. CT scan of the scapula shows the glenoid to be translated medially 3mm, and anglulated 20 degrees from its anatomic axis. What is the most appropriate initial treatment for this injury?

     

     

     

    1. Immobilization in sling x 2 weeks then PT

    2. Immobilization in sling x 8 weeks then PT

    3. ORIF via a deltopectoral approach

    4. ORIF via a posterior approach

    5. ORIF via a lateral approach Corrent answer: 1

    The radiographs are consistent with a extra-articular glenoid neck fracture, which by definition is not significantly displaced. These fractures are best treated with a sling (2 weeks) and early mobilization. Significantly displaced fractures, have translational displacement greater than or equal to 1 cm or angulatory displacement greater than or equal to 40°. These typically need ORIF.

     

    A schematic of the fracture types is shown in Illustration A.

     

    McGahan et al review the epidemiology of scapula fractures and advocate conservative treatment with early mobilization.

     

    Van Noort et al reviewed 13 scapular neck fractures and found that nonoperative treatment in the absence of ipsilateral shoulder injury and associated neurological impairment lead to good functional outcomes, with or without significant translational displacement of the fracture.

     

     

     

     

     

    OrthoCash 2020

     

  78. A 68-year-old healthy active male presents after falling and sustaining an injury to his right knee. His medical history is significant only for osteoporosis. Radiographs and representative CT scan images are shown in Figures A-D. What is the most appropriate treatment method for this patient's injury?

     

     

     

     

     

     

     

     

     

     

     

    1. Traction and splinting

    2. Lag screw fixation followed by non-locking plate application

    3. Retrograde supracondylar nail fixation

    4. External fixation and percutaneous screw reduction of the fracture

    5. Lag screw fixation followed by locking plate application Corrent answer: 5

    The injury shown in Figures A-D represents a comminuted metaphyseal distal femur fracture with a sagittally oriented intra-articular split in osteoporotic bone. Because of the intra-articular nature of this injury, the best fixation construct for treatment of this fracture in an otherwise healthy and active patient is lag screw fixation followed by locked plate application.

     

    Egol et al performed a systematic review of the literature to compare and contrast the function and roles of conventional unlocked plates to locked plates in fracture fixation. They concluded that locked plates may increasingly be indicated for indirect fracture reduction, diaphyseal/metaphyseal fractures in osteoporotic bone, bridging severely comminuted fractures, and the plating of fractures where anatomical constraints prevent plating on the tension side of the bone.

     

    Perren et al discuss the treatment of osteoporotic fractures in the elderly population. They state that in this group of patients, plating with locked screws improve the biology and the mechanics of internal fixation. Furthermore, when this fixation method is used as an 'internal ex fix' (bridging construct) it may stimulate early callus formation because of the inherent flexibility of the construct.

     

    Illustrations A and B show intraoperative fluoroscopic images during fracture fixation. Illustration C shows an AP radiograph of the distal femur 3 months after fixation with a locked plate construct.

     

     

     

     

     

     

     

     

     

     

     

     

    OrthoCash 2020

     

  79. Which of the following is true regarding intimate partner violence (IPV)?

    1. Most patients do not have a fear of domestic retaliation upon reporting

    2. Victims of IPV rarely have a history of injury during the previous 12 months

    3. Interdisciplinary collaboration yields positive outcomes

    4. Emotional abuse is easily identifiable in patients suffering from IPV

    5. Victims of IPV feel comfortable discussing their issues to male physcians Corrent answer: 3

    Identification of IPV is essential in the orthopedic ambulatory setting. Discerning physical abuse is more straightforward as patients can have frequent visits with multiple, unexplained fractures. Emotional abuse is more difficult to discern. Communication and awareness is advocated. Most barriers to reporting include a lack of awareness of IPV, downplaying of the situation, fear of partner retaliation, concern for custody conflicts, shame, embarrassment and a reluctance to talk to male physicians.

     

    Interdisciplinary collaboration among healthcare workers was a predictor of positive treatment outcomes. Included were more accurate assessments of past history, more descriptive emotional symptoms as displayed by victims and written documentation of recommendations concerning intervention and linkage to community resources.

     

    Bhandari et al performed a cross-sectional study of 282 women who presented to fracture clinic at two Level-I trauma centers in Canada. The prevalence of abuse was found to be 32% while 8.5% were found to have a history of previous abuse in the past 12 months. Ethnicity, socioeconomic status, and injury patterns were not associated with abuse.

     

    Shields et al reviewed 153 cases of domestic violence victims who presented to two Emergency departments. They determined that positive treatment outcomes were correlated to the degree of interdisciplinary collaboration among treating health care providers.

     

     

     

    OrthoCash 2020

     

  80. A 35-year-old male suffers the injury seen in Figures A and B following a motor vehicle collision. He is initially taken to a local

    hospital. The treating surgeon, concerned that his hospital does not have a plastic surgeon available for soft-tissue coverage, arranges for transfer of the patient to a nearby level I trauma center for definitive care. Upon arrival at the definitive treatment center, the patient is taken for formal debridement and external fixator application. Which of the following options has the greatest effect on this patient's risk of infection?

     

     

     

     

     

     

     

    1. External fixator application

    2. Tetanus prophylaxis

    3. Operative debridement within 6 hours

    4. TIme to transfer to definitive trauma center

    5. Soft-tissue coverage within 48 hours

     

    Corrent answer: 4

     

    Figures A and B demonstrate an open, segmental tibia shaft fracture with extensive soft-tissue injury. Recent evidence has demonstrated that time to

    transfer to a definitive trauma center has a significant effect on the incidence of infection for high-energy, open lower extremity fractures.

     

    Pollak et. al analyzed a subgroup of 315 patients with high-energy, open lower extremity fractures from the Lower Extremity Assessment Project (LEAP study). Time to admission to a definitive trauma center for treatment was a significant, independent predictor of infection, with patients transferred 11-24 hours following injury having a significantly increased risk of major infection as compared with patients transferred within 3 hours of injury.

     

    Werner et. al reviewed the existing literature surrounding the urgency of surgical debridement for open fractures, specifically relating to the "6 hour rule". The authors found limited evidence in the current literature to support emergent debridement within 6 hours of injury, and recommend urgent debridement (within 24 hours) once the patient is physiologically stable, life threatening emergencies have been addressed, and adequate surgical staff and resources are available.

     

    Incorrect Answers:

    1. External fixator application, although important for limb stability and ongoing assessment of the soft-tissue envelope, has not been shown to affect the rate of infection for open fractures

    2. Tetanus prophylaxis is only effective against one infectious organism

    3. Time to debridement was not shown to affect the rate of infection in the referenced articles

    4. Time to soft-tissue coverage was not shown to have a statistically significant difference on the rate of infection in the level II study by Pollak et. al

     

     

     

    OrthoCash 2020

     

  81. A 38-year-old male suffers the injury shown in Figure A. During operative fixation, free osteoarticular fragments are encountered and reconstruction of these pieces is attempted. Postoperatively, which of the following will have the most beneficial effect on the healing potential of the surviving chondrocytes within these reconstructed articular segments?

     

     

     

    1. Gentle compressive loading of the affected joint through early range of motion exercises

    2. Strict joint immobilzation for three weeks

    3. Shear loading of the affected joint

    4. Joint distraction with a spanning external fixator for three weeks

    5. Glucosamine chondroitin sulfate supplementation Corrent answer: 1

    Figure A demonstrates a comminuted tibial plateau fracture with significant intra-articular involvement. Basic science evidence has demonstrated that post-operative gentle compressive loading may have a positive impact on articular cartilage healing; however, excessive shear loading may be detrimental.

     

    Irrgang et al provide guidelines for rehabilitation following surgical management of articular cartilage lesions of the knee. They state that after articular cartilage repair, exercises to enhance muscle function must be done in a manner which minimizes shear loading of the joint surfaces in the area of the lesion. The authors also discuss the benefits of gentle compressive loading and motion of the joint, and its positive effects on chondrocyte nutrition.

    Furthermore, they recommend a period of protected weight bearing as often being necessary, and that this should be followed by progressive loading of the joint.

    Illustration A is a diagram showing the different layers of joint cartilage.

     

     

     

     

     

     

    OrthoCash 2020

     

  82. When utilizing the pectoralis major tendon as a reference for restoring humeral height during shoulder hemiarthroplasty, at what level cephalad to the proximal edge of the tendon should the top of the prosthesis sit?

    1. 1.0 cm

    2. 2.4 cm

    3. 3.8 cm

    4. 5.6 cm

    5. 6.5 cm

     

    Corrent answer: 4

     

    The pectoralis major tendon is a reproducible structure from which the humeral height during shoulder arthroplasty can be based upon, even in fracture situations where the anatomy is distorted. The superior edge of the pectoralis major tendon at its insertion on the humerus is 5.6 cm below the top of the humeral head.

     

    The referenced study by Torrens et al is a cadaveric evaluation of the pectoralis major tendon and proximal humeral anatomy. They reported that the top of the head sits 5.6cm proximal to the superior border of the pectoralis major tendon.

     

    The referenced study by Greiner et al is a retrospective review of shoulder hemiarthroplasty; improved radiographic and clinical outcomes were reported when the pectoralis major tendon was used as a reference for humeral height.

    Improved outcomes were also reported with successful tuberosity healing and centering of the humeral head in the glenoid.

     

    The referenced study by Murachovsky et al is another cadaveric study that found the pectoralis major tendon is a reproducible means from which to base the humeral height. The distance reported was 5.6 +/- 0.5 cm in 40 shoulders.

     

     

     

    OrthoCash 2020

     

  83. A 28-year-old male sustains a midshaft fibula fracture after being kicked during a karate tournament and develops compartment syndrome isolated to the lateral compartment of his leg. If left untreated, which of the following sensory or motor deficits would be expected?

    1. Decreased sensation on the dorsum of his foot involving the hallux, 3rd, and 4th toes

    2. Inability to plantar flex the ankle

    3. Decreased sensation on the dorsum of his foot involving the first webspace

    4. Inability to dorsiflex the ankle

    5. Inability to abduct his toes Corrent answer: 1

    The clinical vignette describes a scenario of isolated compartment syndrome in the lateral compartment of the leg. The only nervous structure residing in the lateral compartment is the superficial peroneal nerve. In compartment syndrome of the lateral leg compartment, failure of prompt surgical fasciotomy would present as a sensory deficit of the superficial peroneal nerve presenting as numbness on the dorsum of his foot involving the hallux, 3rd, and 4th toes, as seen in Illustration A.

     

    Matsen et al discuss the poor results which can be a cause of late diagnosis and surgical decompression. They recommended compartment monitoring in equivocal cases as well as release of all four leg compartments when facing leg compartment syndrome. A diagram of a two-incision fasciotomy is shown in Illustration B.

     

    Olson et al provide a review of compartment syndrome for the lower extremity. They discuss a variety of injuries and medical conditions may initiate acute compartment syndrome, including fractures, bleeding disorders, and other trauma. Although the diagnosis is primarily a clinical one, they also recommend supplementation with compartment pressure measurements in equivocal cases.

     

     

     

     

     

    OrthoCash 2020

     

  84. A 33-year-old female sustains the injury shown in Figure A. Compared to antegrade nailing of this injury, retrograde nailing has been shown to have an increased amount of which of the following?

     

     

     

    1. Operative time

    2. Symptomatic distal interlocking screws

    3. Hip pain

    4. Union rate

    5. Final knee range of motion Corrent answer: 2

    Figure A shows a femoral shaft fracture, which can be appropriately treated with an intramedullary nail.

     

    The referenced article by Ostrum et al reported that retrograde nailing had an increased rate of symptomatic distal interlocking screws, an increased rate of need for dynamization, longer union time, and less thigh pain than antegrade nailing. Union rate and knee range of motion were not significantly different between these two techniques.

     

    Subsequent studies have found that union time does not significantly differ between antegrade and retrograde nailing.

     

     

     

    OrthoCash 2020

  85. A 35-year-old male undergoes closed reduction under sedation in the emergency department for a posterior hip dislocation with an associated posterior wall fracture. The post-reduction CT is seen in Figure A. What is the appropriate next step in management of this injury?

     

     

     

     

    1. Nonoperative management based on the size of the posterior wall fragment

    2. Operative management based on the size of the posterior wall fragment

    3. Operative management based on the history of hip dislocation

    4. Dynamic fluoroscopic stress exam under anesthesia in the obturator oblique view

    5. Dynamic fluoroscopic stress exam under anesthesia in the iliac oblique view Corrent answer: 4

    Joint stability is critical for successful nonoperative management of posterior wall acetabular fractures. Recent evidence has established that dynamic fluoroscopic stress examination is the best method to determine joint stability in the setting of a posterior wall fracture. The obturator oblique view allows for the best evaluation of hip joint stability during examination for posterior wall fractures.

     

    Grimshaw and Moed retrospectively reviewed the results of patients with posterior wall acetabular fractures managed nonoperatively after evaluation with dynamic fluoroscopic stress tests. At two year follow up, all had good to excellent Merle d’Aubigne clinical scores for hip function and no evidence of post-traumatic hip arthritis on AP pelvis radiographs.

     

    Tornetta retrospectively reviewed his results managing patients with dynamic fluoroscopic stress examination for acetabular fractures which met radiographic nonoperative criteria. Good-to-excellent clinical results were seen in 91% of patients managed nonoperatively.

     

    Tornetta reviewed management of acetabular fractures and Tornetta and Mostafavi separately reviewed management of hip dislocations. In both

    articles, emphasis is placed on dynamic examination of posterior wall fractures as instability has been seen with fractures comprising as little as 15% of the posterior wall.

     

    Illustration A demonstrates two fluoroscopic images from a dynamic stress exam of a patient with a posterior wall fracture. The image obtained in the obturator oblique view clearly demonstrates that the femoral head loses congruency with the acetabular dome.

     

    Incorrect Answers:

    Answers 1 & 2: Posterior wall fragment size less than 40% was historically used as an indirect measure of stability, however measurements of fragment size may be unreliable and instability has been seen with fractures much smaller than 40%

    Answer 3: A history of hip dislocation was thought to indicate a more unstable fracture. In the level IV study by Grimshaw and Moed, patients with an associated hip dislocation who were stable under stress exam had no significant difference in outcome with nonoperative management.

    Answer 5: The iliac oblique view is used to evaluate the anterior wall and posterior column. Displacement or instability of the posterior wall would not be seen with this view

     

     

     

     

     

     

    OrthoCash 2020

     

  86. A 64-year-old woman is thrown off a horse, sustaining the injury shown in Figures A and B. She undergoes surgical fixation as seen in Figures C through E. What is the most commonly reported complication of this procedure?

     

     

     

     

     

     

     

     

     

     

     

    1. Axillary nerve injury

    2. Valgus migration of the fracture

    3. Nonunion

    4. Hardware failure

    5. Screw penetration

     

    Corrent answer: 5

     

    The patient in the scenario has a 2-part proximal humerus fracture treated with a locking plate as seen in Figures A-E. The most common complication with the use of this implant is screw penetration. The terms screw cut out and penetration are often used interchangeably in the literature with cut out appearing more frequently in reports regarding intertrochanteric fractures.

    Owsley et al retrospectively reviewed 53 proximal humerus fractures treated with locking plates and the same post-operative protocol. The most common complication was screw cut out or penetration, followed by varus displacement. They concluded that 3 and 4-part fractures in patients over 60 years have a higher incidence of failure.

     

    Agudelo et al retrospectively reviewed 153 patients at a level-one trauma center treated with proximal humerus locking plates, investigating modes of failure for the implant. They determined that varus malreduction (head-shaft angle<120 degrees) was the most common mode of failure in their group.

     

     

     

    OrthoCash 2020

     

  87. A 25-year-old male sustained a closed midshaft femur fracture following a motor vehicle collision. He is taken to the operating room for supine intramedullary nail fixation of the fracture. Figure A is a lateral fluoroscopic view of the distal femur taken just prior to distal interlocking screw placement. What change in position (with the Carm stationary) would be expected to produce a perfect lateral view of the interlocking hole?

     

     

     

     

    1. Raising the leg

    2. Lowering the leg

    3. Internal (or external) rotation of the leg

    4. Abduction (or adduction) of the leg

    5. Fluoroscopic magnification

    Corrent answer: 4

     

    Interlocking of intramedullary nails using fluoroscopy requires attention to detail. A true lateral of the intramedullary nail is present when "perfect circle" views of interlocking holes are present. Once perfect circles are obtained, the drill can be advanced parallel to the fluoroscopic beam.

     

    Knowledge of the implications of the appearance of the interlocking holes when "perfect circles" are not present can be helpful in minimizing the number of manipulations and fluoroscopic exposure. Widening of the interlocking hole in the proximal-distal direction (as is seen in this case) signifies the need for an adjustment in the abduction/adduction plane. Similarly, widening of the interlocking hole in the anterior-posterior plane signifies the need for an adjustment in the internal/external rotation plane (Answer 3).

     

    Raising or lowering the leg (Answer A and B) should not have major effects of the appearance of the interlocking hole. Similarly, magnification of the C-arm (Answer D) will not affect the appearance. Internal/external rotation will result in widening in the anterior-posterior plane (as this dimension is currently adequate). Abduction or adduction will result in creating "perfect circles" (Answer 4). The position of the leg may hint to which of these is correct. If further widening occurs in the proximal-distal direction, the opposite maneuver will correct to the proper position.

     

     

     

    OrthoCash 2020

     

  88. A 46-year-old male is involved in a motor vehicle accident and suffers a proximal humerus fracture. Operative treatment is recommended, and plate fixation is performed through an extended anterolateral acromial approach. Which of the following structures is at increased risk of injury using this surgical exposure compared to the deltopectoral approach?

    1. Musculocutaneous nerve

    2. Posterior humeral circumflex artery

    3. Axillary nerve

    4. Cephalic vein

    5. Anterior humeral circumflex artery Corrent answer: 3

    The anterolateral acromial approach was developed to allow less invasive treatment of proximal humerus fractures. The plane of the avascular anterior

    deltoid raphe is utilized, and the axillary nerve is at particular risk of injury and must be identified and protected. With this approach, anterior dissection near the critical blood supply is avoided, substantial muscle retraction is minimized, and the lateral plating zone is directly accessed.

     

    Gardner et al evaluated 23 patients who had acute displaced fractures of the proximal humerus treated with the anterolateral acromial approach and either a locking plate or an intramedullary nail. At one year post-op, there were no axillary nerve deficits, and they found that the approach allowed direct access to the lateral fracture planes for fracture reduction and plate placement or safe nail and interlocking screw placement.

     

    Gardner et al, in another study, performed cadveric dissection using the extended anterolateral acromial approach and measured multiple parameters regarding the axillary nerve. The nerve was predictably found approximately 35 mm from the prominence of the greater tuberosity.

     

    Morgan et al performed a cadaveric study to describe the anatomic insertion point of the deltoid onto the proximal humerus. They found that the deltoid insertion is long and broad, and that placement of 4.5-mm plate would result in detaching 13.5 mm of its insertional footprint. This would leave half of the insertion still attached to the humerus.

     

    Illustration A shows the position of the axillary nerve in relation to the approach.

     

     

     

     

     

     

     

     

    OrthoCash 2020

     

  89. Which of the following statements is true regarding brake travel time after surgical treatment of complex lower extremity trauma?

    1. Brake travel time is significantly increased until 6 weeks after patient begins weight bearing

    2. Return of normal brake travel time takes longer after long bone fracture compared to articular fractures

    3. Normal brake travel time correlates with improved short musculoskeletal

      functional assessment scores

    4. Brake travel time is significantly reduced until 8 weeks after patient begins weight bearing

    5. Brake travel time returns to normal when weight bearing begins Corrent answer: 1

    Brake travel time (BTT) has been shown to be significantly increased until 6 weeks after initiation of weight bearing in both long bone and articular fractures of the right lower extremity.

     

    Egol et al used a computerized driving simulator to compare BTT in three groups of patients; 1)control group, 2)long bone lower extremity fracture group, 3)lower extremity articular fracture group. They concluded that BTT was significantly increased until 6 weeks after initiation of weight bearing in both long bone and articular fractures of the right lower extremity, and that short musculoskeletal functional assessment scores improved with respect to function and other indexes, but did not correlate with improvement in BTT.

     

    An earlier study by Egol et al looked at total brake time as it related to distance traveled by the automobile before braking at 6, 9, and 12 weeks after operative fixation of a right ankle fracture. When compared with controls, braking time was shown to return to normal by 9 weeks post-operatively, and no significant association was found between the functional scores and this normalization.

     

    Giddins et al provide a review of the literature, the law and the views of the major motor insurers related to driving after injury or operations.

     

     

     

    OrthoCash 2020

     

  90. What acetabular component is best appreciated on an obturator oblique radiograph of the pelvis as seen in Figure A?

     

     

     

    1. Ilioischial line

    2. Posterior column

    3. Posterior wall

    4. Anterior wall

    5. Sacroiliac joint

     

    Corrent answer: 3

     

    Letournel and Judet developed a schematic representation of the acetabulum as being contained within asymmetric long anterior and short posterior arms of an inverted “Y”.

     

    On the bony pelvis, the ilioischial component becomes that posterior column and the iliopectineal line becomes the anterior column. The Judet-Letournel classification system is based on this scheme. By careful evaluation of landmarks on a standard AP pelvis radiograph, as well as on 45-degree oblique obturator and iliac views, the extent of injury can be determined accurately.

     

    The AP view usually demonstrates the six fundamental landmarks relatively well as seen in illustration A. The obturator oblique view reveals additional information about the anterior column and posterior wall(see illustration A(B),

    B). In an obturator oblique view the x-ray beam is centered on and almost perpendicular to the obturator foramen. The iliac oblique view visualizes the posterior column and anterior wall (illustration A(C), C). This view also shows the best detail of the iliac wing as the radiographic beam is roughly perpendicular to the iliac wing. Inclusion of the opposite hip is essential for evaluation of symmetrical contours that may have slight individual variations

    and to evaluate the width of the normal articular cartilage in each view in a pelvic series (AP, Judet's) .

     

     

     

     

     

     

     

     

     

     

     

    OrthoCash 2020

     

  91. Which of the following fractures seen in Figures A through E would be amenable to fixation with a construct using tension band principles?

     

     

     

     

     

     

     

     

     

     

     

    1. Figure A

    2. Figure B

    3. Figure C

    4. Figure D

    5. Figure E

     

    Corrent answer: 1

     

    In order for a fracture to be successfully treated with tension band principles the bone must be eccentrically loaded, the construct must be applied on the tensile side, and the opposite cortex must be able to withstand compressive

    forces. Of the fractures seen in Figures A-E, the subtrochanteric fracture seen in Figure A best meets the criteria for stabilization according to tension band principles.

     

    Kinast et al retrospectively compared their results with 95 degree blade plate fixation of subtrochanteric fractures utilizing the blade plate as a dynamic tension band. They performed either wide exposure of the fracture site with autogenous bone grafting according to AO technique at the time (group 1), or indirect reduction techniques without bone grafting the medial side as advocated by Mast et al (group 2). The authors found 100% union rates at six months with indirect reduction techniques without bone grafting (group 2), and emphasize the key concepts of preservation of the medial soft tissues and intraoperative pretensioning of the plate.

     

    Illustration A demonstrates the principles of tension band fixation specifically applied to the femur. Within Illustration A, Figure C shows the correct application of a plate along the lateral cortex to resist tensile forces, along with incorrect application of the plate along the medial cortex (Figure D) or in a fracture pattern with an absent opposite cortex (Figure E). Illustration B shows an example of blade plate fixation of a subtrochanteric fracture.

     

    Incorrect Answers:

    Answer 2. Figure B demonstrates a comminuted proximal tibia fracture. Although the tibia is eccentrically loaded and an implant applied to anterior cortex could function as a tension band, the posterior comminution would lead to collapse.

    Answer 3. Figure C demonstrates a comminuted distal humeral shaft fracture. Again,the comminution prevents application of a tension band construct Answer 4: Figure D shows a valgus impacted proximal humerus fracture.

    Eccentric loading is absent for this type of fracture

    Answer 5: Figure E demonstrates a comminuted olecranon fracture with extension distal to the coronoid process. Although tension band constructs are commonly used for olecranon fractures, the comminution and distal extension of this fracture would prevent application of a tension band.

     

     

     

     

     

    OrthoCash 2020

     

  92. Which of the following is an advantage of computer-assisted navigation used to place medullary nail interlocking screws compared to a freehand techinque?

    1. Reduced fluoroscopy time

    2. More reliable placement of interlocking screws through the nail

    3. Reduced procedure time

    4. Increased quality of fluoroscopic images

    5. Improved accuracy of screw length Corrent answer: 1

    Computer-assisted navigation has been shown to reduce radiation exposure for surgeons when performing interlocking of medullary nails compared to freehand technique.

     

    Ricci et al compared two fluoroscopic navigation tracking technologies, optical and electromagnetic versus standard freehand fluoroscopic targeting, in a standardized foam block model for placement of interlocking screws. They found that fluoroscopy time (seconds) and number of fluoroscopy images were significantly less when using the computer-guided systems than for freehand-unguided insertion. Average distance of pin placement from the target in the foam blocks was significantly greater for controls than for each of the navigated systems.

     

    Suhm et al performed a prospective controlled clinical study to compare fluoroscopic guidance with fluoroscopy-based surgical navigation for distal locking of intramedullary implants. The surgical navigation group showed increased procedure time, but equivalent precision with reduced radiation exposure. There was no significant difference in the technical reliability

    between both groups.

     

     

     

     

    OrthoCash 2020

     

  93. An 82-year-old nursing home resident falls onto his elbow while rising from a seated position. He has pain and swelling at the elbow without evidence of instability. His radiographs show a comminuted displaced olecranon fracture involving 25% of the articular surface with global osteopenia. Which of the following treatment options has a low risk of complications and a high likelihood of a functional elbow outcome?

    1. Cast immobilization in 45 degrees of flexion for 8 weeks

    2. Closed reduction and percutaneous pinning

    3. ORIF with a tension band construct

    4. ORIF with a locking plate

    5. Cast immobilization in 90 degrees flexion Corrent answer: 5

    The patient in the scenario is a low-demand, elderly patient with a comminuted olecranon fracture. Recent studies have demonstrated good outcomes with

    non-operative treatment of olecranon fractures in this patient population.

     

    Hak (jaos'00) reviewed the treatment of olecranon fractures, recommending ORIF for displaced intra-articular fractures, either via tension band wiring or plate osteosynthesis based on fracture pattern and associated ligamentous disruption. For elderly, low-demand, osteoporotic patients, triceps advancement and fragment excision is an option in the case of severely comminuted, osteoporotic patients with a fracture involving <50% of the joint surface.

     

    Duckworth (jbjs'14) performed a single center retrospective review of low-demand patients with displaced olecranon fractures managed non-operatively with a short duration of immobilization followed by range of motion to tolerance. The authors found good short and long-term results with this treatment with patients experiencing minimal discomfort or loss of motion at long-term follow up.

     

    Incorrect answers:

    Answer 1: Cast immobilization in 45-90 degrees of flexion is advocated for non-displaced fractures. Furthermore, casting for 8 weeks is not indicated in nonoperative care of olecranon fractures secondary to the development of

    stiffness.

    Answers 2-4: Not the treatment in low demand patients with severe comminution and osteoporosis.

     

     

     

    OrthoCash 2020

     

  94. A 47-year-old male presents with a one month history of heel pain after starting marathon training. The heel is tender when squeezed. A foot radiograph is shown in Figure A, and an MRI is obtained which is shown in Figures B and C. What is the most likely diagnosis?

     

     

     

     

     

     

     

     

     

    1. Osteomyelitis

    2. Acute fracture

    3. Subtalar arthritis

    4. Achilles tendinitis

    5. Stress fracture

     

    Corrent answer: 5

     

    The patients history, physical exam, and MRI findings are most consistent with the diagnosis of a stress fracture. Low signal on T1 and increased signal on T2-weighted images is the classic appearance of a stress fracture on magnetic resonance images. Usually the fracture appears as a linear signal band that arises from the cortex of the bone and extends perpendicular to the surface of the bone. Figures A and B show a linear area of enhancement in the posterior aspect of the calcaneus signifying a likely stress reaction from recent overtraining.

     

    Boden et al review the pathogenesis and treatment of stress fractures. With regard to diagnosis, they state that it is primarily clinical, but imaging modalities such as plain radiography, scintigraphy, computed tomography, and magnetic resonance imaging may provide confirmation.

     

    Gehrmann et al review stress fractures of the foot, and state that with regards to calcaneal stress injuries, MRI studies should be obtained for definitive diagnosis when radiographs are negative, and pain persists. With regards to

    treatment, they state that these injuries usually heal with activity restrictions, heel-pad inserts, and protected weightbearing for a short period of time.

     

     

     

    OrthoCash 2020

     

  95. A 35-year-old right hand dominant man falls from a ladder and sustains the injury seen in Figure A. When discussing the risks and benefits of operative versus nonoperative treatment for his fracture, which of the following is true?

     

     

     

     

    1. No difference in shoulder function

    2. Higher risk of nonunion with operative management

    3. Higher risk of symptomatic malunion or nonunion with nonoperative management

    4. Earlier return to sport with nonoperative management

    5. No difference in union rates Corrent answer: 3

    Historically, displaced midshaft clavicle fractures, as seen in Figure A, were managed nonoperatively. Recent literature has demonsrated improved outcomes with operative management of these fractures.

     

    Khan et al review current concepts in the management of clavicle fractures. For displaced midshaft clavice fractures, operative treatment seems to result in improved patient and surgeon-based outcomes, decreased rates of malunion and nonunion, and shorter time to union.

     

    Kim and McKee review recent evidence regarding the management of clavicle fractures. For midshaft clavicle fractures, the incidence of nonunion and symptomatic malunion with nonoperative management is higher than

    previously believed. They state that risk factors include 100% displacement, comminution, increasing age and female gender.

     

    Incorrect Answers

    Answer 1. Recent randomized prospective trials have shown improved short term shoulder function with operative management of displaced midshaft clavicle fractures.

    Answer 2. Nonunion rates of 7-15% have been shown with nonoperative management versus 2% with operative fixation

    Answer 4. Earlier return to activities has been reported with operative management

    Answer 5. As with answer 2, there is a significantly higher rate of nonunion with nonoperative management

     

     

     

    OrthoCash 2020

     

  96. Assuming the images represent isolated injuries, which of the following Figures demonstrates a Pipkin II femoral head fracture?

     

     

     

     

     

     

     

     

     

     

     

     

     

     

    1. Figure A

    2. Figure B

    3. Figure C

    4. Figure D

    5. Figure E

     

    Corrent answer: 4

     

    Figure D represents a Pipkin II femoral head fracture, which is defined as a fracture which is superior to the fovea. Differentiation between Pipkin I and Pipkin II fractures can be important, as suprafoveal injuries often require surgical fixation. Illustration A demonstrates the Pipkin fracture types.

     

    Droll et al review femoral head fracture evaluation and treatment. They discuss non-operative indications (typically reserved for Pipkin I injuries) which include an anatomic or near anatomic reduction (<2 mm) of the femoral head fragment, a stable hip, and no interposed fragments preventing a congruent hip joint. They also state that an anterior surgical approach is preferred for fixation of Pipkin II type injuries.

     

    Pipkin discusses the evaluation and treatment of hip fracture dislocations as defined by the Stewart and Milford classification scheme. He focuses on Grade IV injuries, which at the time lacked an appropriate treatment algorithm.

    Incorrect Answers:

    1,2-Figures A and B show Pipkin IV fractures, due to the presence of an associated acetabular fracture.

    3-Figure C shows a Pipkin I fracture, as the fracture is infra-foveal

    5-Figure E shows an example of a Pipkin III injury due to the presence of an associated femoral neck fracture.

     

     

     

     

     

     

    OrthoCash 2020

     

  97. A 20-year-old female presents following a motor vehicle collision with the injuries seen in Figures A and B. She was initially hypotensive and tachycardic however she now has stable vital signs following a 2 liter bolus of saline and 2 units of packed red blood cells. Which of the following would indicate that this patient has occult end-organ hypoperfusion and should be further resuscitated prior to definitive fixation?

     

     

     

     

     

     

    1. Heart rate of 80 beats per minute

    2. Systolic blood pressure of 120 mmHg

    3. Base deficit of -1.8 mEq/L

    4. Serum lactate of 5 mmol/Liter

    5. Urine output of 40ml/hour Corrent answer: 4

    Figure A demonstrates an unstable pelvic fracture and subtrochanteric femur fracture and Figure B shows a scapular body and mutliple rib fractures. These high-energy fractures, along with the patient's initial hypotension and tacchycardia indicate a multiply injured patient in shock. Serum markers such as lactate (normal < 2 mmol/L) or base deficit are more sensitive markers of occult end-organ hypoperfusion and a serum lacate of 5mmol/Liter indicates an incompletely resuscitated patient.

    Porter et al review the endpoints of resuscitation and note a high incidence of patients (as much as 85%) in "compensated" shock despite normal conventional parameters such as vital signs and urine output. Compensated shock occurs secondary to shunting of tissue oxygenation and blood flow away from splanchnic organs and towards vital organs such as heart and brain.

     

    The review article by Elliott argues that serum lactate levels are the most reliable indicator of peripheral organ perfusion and tissue oxygenation. A base deficit between -2 and +2 is also an appropriate end point however may be non-specific in older patients with medical comorbidities leading to acid/base disturbances.

     

    Rossaint et al put forth evidence-based recommendations regarding control of bleeding and resuscitation of trauma patients. They strongly recommend based on moderate-quality evidence (Grade 1B) that both serum lactate and base deficit be used to monitor the extent and progression of bleeding and shock.

     

     

     

    OrthoCash 2020

     

  98. An 80-year-old female falls and sustains the fracture seen in Figure A. She is treated with an antegrade cephalomedullary nail. Which of the following led to the complication seen in Figure B?

     

     

     

     

     

     

    1. Nail with a lesser radius of curvature

    2. Nail with a greater radius of curvature

    3. Piriformis entry portal

    4. Trochanteric entry portal

    5. Lateral decubitus patient position Corrent answer: 2

    The image in Figure A shows an unstable intertrochanteric fracture and the image in Figure B shows perforation of the anterior cortex of the femur by the intramedullary implant. This complication is due to a mismatch of the curvature of the nail with the anterior bow of the femur, and was likely caused by a nail of a greater radius of curvature (eg, straighter than the femur).

     

    Egol et al measured the radius of curvature for 474 matched cadaveric femurs and found the average anterior radius of curvature to be 120cm (+/- 36cm). In contrast, the radii of curvature for the measured intramedullary nails ranged from 186cm to 300cm, demonstrating that the nails were straighter than the femurs. The authors advocate for a decreased radius of curvature (more curve) for intramedullary nails, especially larger diameter implants designed for fractures about the hip

     

    Ostrum and Levy present a case series of 3 patients with subtrochanteric

    fractures who had anterior penetration of the femoral cortex. They state that the mismatch in femoral bow between the bone and the implant is a contributing factor to distal femoral anterior cortex penetration in intramedullary nailing of subtrochanteric fractures.

     

    Simonian et al present 4 iatrogenic femoral neck fractures that occured during a series of 315 femoral nails. The authors attempted to reproduce the iatrogenic fractures with cadaveric femurs and felt that the iatrogenic fractures may be due to a combination of a valgus femoral neck and impingement from the AO insertion jig used at the time.

     

    Harper and Carson examined 14 cadaveric femurs and intramedullary implants at the time. Similar to Egol et al, they found a mismatch between the radius of curvature of the femurs and the intramedullary nails.

     

    Illustration A shows the difference between a lesser and greater radius of curvature. Illustration B demonstrates how to calculate radius of curvature based on an implant with an exaggerated bow. Illustration C shows the anterior bow of a synthetic femoral model compared with several intramedullary implants.

     

     

     

     

     

     

     

     

    OrthoCash 2020

     

  99. Bridge plating of the olecranon is MOST appropriate in which of the following clinical scenarios?

    1. Fixation of an olecranon osteotomy used for distal humerus surgery in a 24-year-old male

    2. Simple transverse olecranon fracture in 33-year-old female

    3. Comminuted olecranon fracture in 45-year-old male

    4. Severely comminuted proximal olecranon fracture in an osteoporotic 91-year-old female

    5. Aphophyseal elbow fracture in 6-year-old male Corrent answer: 3

    Bridge plating is most appropriately used for fixation of comminuted fractures which are not able to be fixed anatomically. Of the choices listed above, this would be most appropriate in a comminuted fracture in a 45-year-old male. In contrast, literature shows that severely comminuted, osteoporotic low-demand elderly are best treated with olecranon fracture excision and triceps advancement when possible.

     

    Bailey et al looked at the outcome of plate fixation of olecranon fractures. They concluded that plate fixation is effective for displaced olecranon fractures and leads to good functional outcome. There were low incidence of complications, although 20% did have removal of hardware due to irritation.

     

    Hak et al reviewed the fixation options for olecranon fractures. In their review, they state that comminuted olecranon fractures can be treated by limited-contact dynamic-compression plates. They also supported the use of supplemental bone graft to support comminuted depressed articular fragments. Fragment excision and triceps advancement is appropriate in selected cases in which open reduction seems unlikely to be successful, such as in osteoporotic elderly patients with severely comminuted fractures.

     

     

     

    OrthoCash 2020

     

  100. A 75-year-old ambulatory male who lives independently presents with the fracture shown in Figure A. Which of the following is true regarding timing of surgical fixation and post-operative mortality?

     

     

     

    1. 30 day mortality is decreased if surgery is delayed 4-7 days

    2. 1 year mortality is increased if surgery is delayed greater than 4 days

    3. Delay of surgery due to treatment of acute medical comorbidities has no effect on post-operative mortality rates

    4. 90 day mortality rate is decreased if surgery is delayed greater than 7 days

    5. Timing of surgical fixation has no statistically significant affect on postoperative mortality

    Corrent answer: 2

     

    Figure A shows a displaced femoral neck fracture.

     

    Moran et al studied 2660 elderly patients who underwent surgical treatment of a hip fracture to determine whether a delay in surgery affects postoperative mortality. The 30-day mortality for patients for whom the surgery had been delayed for more than 4 days was 10.7%, compared to 7.3% in those delayed 1-4 days. The group delayed >4 days also had significantly increased mortality at 90 days and 1 year. Patients who had been admitted with an acute medical comorbidity that required treatment prior to the surgery had a 30-day mortality of 17%, which was nearly 2.5 times greater than that for patients without and acute comorbidity. The study concluded that patients with medical comorbidities that delayed surgery had 2.5 times the risk of death within 30 days after the surgery compared with patients without comorbidities. Mortality was not increased when the surgery was delayed up to four days for patients who were otherwise fit for hip fracture surgery, however, a delay of more than four days significantly increased mortality.

    Healy et al examined 120 patients who underwent surgical treatment of 186 displaced femoral neck fractures with either internal fixation, hemiarthroplasty, or total hip arthroplasty. Arthroplasty was associated with more independent living, and was more cost-effective than internal fixation. There was no difference in rates of reoperation or mortality, but arthroplasty produced a longer interval to reoperation or death. They concluded that total hip arthroplasty was the best treatment for displaced fractures of the femoral neck in their series.

     

    Illustration A shows a scoring system developed by Rogmark et al to aid in decision making.

     

     

     

     

     

     

    OrthoCash 2020

     

  101. A 35-year-old male suffers an anterior column acetabular fracture during a motor vehicle collision, and subsequently undergoes percutaneous acetabular fixation. Intraoperatively, fluoroscopy is positioned to obtain an obturator oblique-inlet view while placing a supraacetabular screw. Which of the following screw relationships is best evaluated with this view?

    1. Intraarticular penetration of the screw

    2. Position of the screw cephalad to the sciatic notch

    3. Screw starting point at the anterior inferior iliac spine

    4. Screw starting point at the gluteal pillar

    5. Screw position between the inner and outer tables of the ilium Corrent answer: 5

    The obturator oblique-inlet view, as seen in Illustration A, best demonstrates the position of a supra-acetabular screw or pin relative to the tables of the ilium.

     

    Starr et al review their initial results and technique of closed or limited open reduction and percutaneous fixation of acetabular fractures. They defined two groups of patients who may benefit from this technique; elderly patients with multiple comorbidities to facilitate early mobilization and restore hip morphology, and young patients with elementary fracture patterns and multiple associated injuries.

     

    Starr et al describe their operative technique and outcomes for a case series of 3 patients using percutaneous acetabular fixation to augment open reduction of acetabular fractures. The authors state that, for placement of an anterior colum ramus screw, an iliac oblique-inlet (not obturator oblique-inlet) will ensure that the screw is within the medullary canal of the ramus and does not exit anterior or posterior.

     

    Gardner and Nork describe a technique for placement of a large femoral distractor in the supra-acetabular region to compress displaced posterior pelvic ring injuries. They note that the obturator oblique-inlet view is necessary to view the entire length of the pin as well as to ensure that pin remains in bone.

     

    Incorrect answers:

    Answer 1: Relationshiop of the screw to the acetabulum is best evaluated with the obturator oblique-outlet view as well as the iliac oblique view

    Answer 2: The iliac oblique view is used to ensure the trajectory of the screw is superior to the sciatic notch

    Answer 3: The Obturator oblique-outlet view, otherwise known as the "teepee" or "tear drop" view, is used to identify the start for supra-acetabular implant placement

    Answer 4: The gluteal pillar is not utilized as a start point when placing supraacetabular fixation, and the obturator oblique-inlet view would not be ideal to visualize this region of the pelvis

     

     

     

     

     

    OrthoCash 2020

     

  102. A 74-year-old man falls, sustaining the injury shown in Figures A through C. In surgical planning, what is the best surgical approach to treat this injury?

     

     

     

     

     

     

     

     

    1. Kocher-Langenbeck

    2. Watson-Jones

    3. Extended iliofemoral

    4. Ilioinguinal

    5. Hardinge

     

    Corrent answer: 4

     

    Figures A through C depict and AP pelvis and Judet views of a T-type fracture of the right acetabulum. The ilioinguinal approach provides access to the anterior wall and anterior column for fracture fixation, in addition to allowing fixation of the nondisplaced posterior transverse fracture line. The lateral femoral cutaneous nerve (LFCN) is at risk in the superficial part of the dissection. Another option for the approach would be the modified Stoppa, which would also allow excellent access to the anterior column as well as the internal aspect of the iliac wing and quadrilateral plate.

    Illustration A shows the five basic and 5 associated acetabular fractures. Incorrect Answers:

    Answer 1. Kocher-Langenbeck: access for posterior wall and column fractures

    Answer 2. Watson-Jones: anterolateral approach best for the hip, not the anterior column of the acetabulum.

    Answer 3. Extended iliofemoral: visualization for both column fractures Answer 5. Hardinge approach: lateral approach for THA

     

     

     

     

     

    OrthoCash 2020

     

  103. A 28-year-old female is struck by a motor vehicle while crossing the street and suffers the injury seen in Figure A. What technical adjunct could have prevented the operative complication seen in Figure B?

     

     

     

     

     

     

    1. Nail of a lesser radius of curvature

    2. Nail with a more distal Herzog curve

    3. Application of an anterior unicortical plate

    4. Nailing while in a hyperflexed position

    5. A more distal and medial nail entry site

    Corrent answer: 3

     

    The patient has a proximal tibia fracture that has been malreduced in procurvatum with anterior translation of the proximal fragment as seen in Figures A and B. Intramedullary nailing of proximal tibia fractures may result in malalignment such as valgus, procurvatum and anterior translation of the proximal fragment.Techniques such as anterior unicortical plating have been described to maintain reduction while placing a locked intramedullary implant.

     

    Nork et al retrospectively review their results using several different intraoperative adjuncts to maintain reduction while nailing proximal tibia fractures including anterior unicortical plates and femoral distractors. They report that, despite high rates of segmental comminution (59.5%) and open fractures (35.1%), acceptable alignment and primary union were achieved in more than 90% of patients.

     

    Dunbar et al describe their technique for application of a provisional unicortical plate through the traumatic wound to maintain reduction during intramedullary nailing of Gustilo-Anderson Type III open tibia fractures. The authors advocate that further soft tissue stripping during plate placement should be avoided, and found excellent results in terms of coronal and sagittal plane alignment for fractures treated with this technique.

     

    Matthews et al review their experience with unicortical plating to maintain reduction during intramedullary nailing of tibia fractures. Similar to the other referenced studies, the authors found the technique assisted with maintaining reduction intraoperatively.

     

    Illustrations A and B demonstrate maintenance of reduction of a proximal tibia fracture using an anterior unicortical plate as a reduction tool during intramedullary nailing.

     

    Incorrect Answers:

    Answer 1: Nail of a lesser radius of curvature. Unlike femoral nails, tibial nails are straight to match the coronal/sagittal axis of the tibia and therefore a nail with a lesser radius of curvature (more bend) would not match the anatomy and likely lead to malalignment

    Answer 2: Nail with a more distal Herzog curve. The proximal bend for tibial IM nails, known as the Herzog curve, may cause malalignment in proximal tibia fractures due to a phenomenon known as the "wedge effect". Nails with more distal Herzog curves have been implicated in anterior translation of the proximal fragment.

    Answer 4: Nailing in a hyperflexed position would cause extension of the proximal fragment due to tension on the extensor mechanism

    Answer 5: A more distal and medial entry site would likely lead to further deformity and possible nail cut out through the anterior cortex. A more proximal and lateral entry site is preferred for nailing proximal tibia fractures

     

     

     

     

     

     

     

     

     

    OrthoCash 2020

  104. A 32-year-old male sustains an APC-III pelvic ring disruption after a motor vehicle collision. Which of the following imaging techniques best describes the correct utilization of intraoperative flouroscopy for percutaneous iliosacral screw placement across S1?

    1. Inlet view helps best guide superior-inferior orientation

    2. AP pelvis best guides anterior-posterior screw orientation

    3. AP pelvis best guides superior-inferior screw orientation

    4. Outlet view best guides anterior-posterior screw orientation

    5. Outlet view best guides superior-inferior screw orientation Corrent answer: 5

    The outlet view best guides superior-inferior screw orientation during percutaneous S1 screw placement. This is due to the relative forward flexion of the sacrum and pelvis due to pelvic incidence. A lateral sacral view and an inlet pelvis view would best guide anterior-posterior screw orientation.

     

    Routt et al did a review of percutaneous techniques of pelvic surgery. Although anterior pelvic external fixation remains the most common form of percutaneous pelvic fixation, iliosacral screws have the advantage of stabilizing pelvic disruptions directly while diminishing operative blood loss and operative time. They stress importance of a thorough knowledge of pelvic osseous anatomy, injury patterns, deformities, and their fluoroscopic correlations for surgical success.

     

    Routt et al also looked at the complications that can result from percutaneous iliosacral screw placement. Complications ranged from inability for adequate imaging due to patient obesity, L5 nerve root injuries, fixation failure, and sacral nonunions. They support quality triplanar fluoroscopic imaging during iliosacral screw insertions to help accurately reduce injured posterior pelvic rings.

     

    Illustration A is an example of an outlet view image status post anterior pelvic ring plating and percutaneous iliosacral screw. This outlet view allows superior S1 neural foramen visualization to help guide screw placement and avoid nerve injury.

     

     

     

     

     

    OrthoCash 2020

     

  105. Which of the following methods accurately describes the measurement of tip-apex-distance as it relates to placement of a lag screw in the femoral head?

    1. Summation of the distance between the end of the screw and the apex of the femoral head on AP and lateral radiographs

    2. Distance from the acetabular teardrop to the tip of the screw on an AP radiograph of the hip

    3. Multiplication of the distance between the end of the screw and the apex of the femoral head on AP and lateral radiographs

    4. Distance from the center of the lesser trochanter to the tip of the screw on an AP hip radiograph

    5. Summation of the distance between the tip of the greater trochanter and end of the screw on AP and lateral hip radiographs

    Corrent answer: 1

     

    Tip-apex distance (TAD) as it relates to a lag screw in the femoral head is the summation of the distance between the end of the screw and the apex of the femoral head on AP and lateral radiographs, corrected for radiographic magnification of the image. This is shown in Illustration A.

     

    TAD is a useful intraoperative indicator of deep and central placement of the lag screw in the femoral head, regardless of whether a nail or a plate is chosen to fix a fracture. A TAD of <25mm has been shown to minimize the risk of fixation cut-out in stable and unstable intertrochanteric hip fractures.

     

    Baumgaertner et al. examined factors leading to the failure of sliding hip screws (SHS) in the treatment of 198 intertrochanteric fractures. They

    determined that the TAD is a reproducible, standard measurement to predict SHS failure. The average TAD for successful fractures was 24mm while the average TAD for failures was 38mm. No screw with a TAD <25mm failed.

     

    Geller et al. evaluated the TAD of intramedullary devices as an independent predictor of cut-out failure in intertrochanteric hip fractures. The authors found that there was a statistically significant increase in screw cut-out with TAD > 25mm as well as with increasing fracture severity. They concluded that TAD is a valuable consideration for intramedullary devices as well.

     

    Illustrations B and C show a lag screw with an excessive TAD, and subsequent failure of fixation.

     

    Incorrect Answers:

    Answers 2-5: TAD is defined as the sum of the distance between the end of the lag screw and the apex of the femoral head on the AP and lateral radiographs, corrected for radiographic magnification of the image.

     

     

     

     

     

     

     

     

     

     

    OrthoCash 2020

     

  106. A 25-year-old man presents one year after undergoing open reduction and internal fixation of the fracture seen in Figure A. Current radiographs demonstrate a united fracture with no evidence of ostenecrosis, subtalar or tibiotalar arthritis. Physical exam is notable for ambulation on the lateral border of the right foot with hindfoot varus, midfoot supination and diminished subtalar motion compared with the contralateral side. Which of the following is an option for reconstruction of this patient's deformity?

     

     

     

    1. Total ankle arthroplasty

    2. Lateral calcaneus closing wedge osteotomy

    3. Calcaneal neck opening wedge osteotomy

    4. Talar neck opening medial wedge osteotomy

    5. Triple arthrodesis

     

    Corrent answer: 4

     

    The most common position of talar neck malunion is varus and this deformity can lock the subtalar and transverse tarsal joints leading to diminished motion and ultimately subtalar arthrosis. Medial opening wedge osteotomy of the talar neck has been described to restore the anatomy of the talus and preserve hindfoot motion.

     

    Daniels et al performed a biomechanical study of hindfoot motion following medial osteotomy and varus malalignment of the talar neck. The authors found a direct correlation between the degree of varus malalignment and diminished forefoot and subtalar motion, and caution against compressive fixation of the medial talus in the presence of comminution.

     

    Monroe and Manoli present a case report of talar neck malunion and describe their technique for osteotomy of the talar neck with insertion of a tricortical iliac crest bone graft to correct the deformity. They found a significant improvement in the AOFAS ankle-hindfoot score and no evidence of

    osteonecrosis with follow-up greater than 4 years.

     

    Huang and Cheng retrospectively review their results with anatomic reconstruction of the talar neck in 9 patients who presented a minimum of four weeks after the initial injury with a malreduced or neglected talar neck fracture. They found favorable outcomes with delayed reconstruction as determined by the AOFAS ankle-hindfoot score at an average follow up of 53 months, however note that six patients subsequently developed radiographic hindfoot arthrosis.

     

    Figure A demonstrates a lateral radiograph of a comminuted talar neck fracture with subluxation of the subtalar joint.

     

    Incorrect Answers:

    Answer 1: Total ankle arthroplasty is reserved for patients with end-stage tibiotalar arthritis. This patient has an intact and functional tibiotalar joint Answer 2: Lateral calcaneus closing wedge osteotomy would correct some hindfoot varus, however would not address the talar neck deformity leading to diminished motion.

    Answer 3: Calcaneal neck opening wedge osteotomy would exacerbate the patient's deformity

    Answer 5: Triple arthrodesis would be not be a viable option in a patient with a preserved tibiotalar and subtalar joint.

     

     

     

    OrthoCash 2020

     

  107. Which of the following descriptions is true regarding APC-II (anterior-posterior compression) pelvic injuries as classified by Young and Burgess?

    1. Pubic symphysis diastasis, intact anterior sacroiliac ligaments, intact sacrotuberous ligament, intact posterior sacroiliac ligaments

    2. Pubic symphysis diastasis, torn anterior sacroiliac ligaments, intact sacrotuberous ligament intact posterior sacroiliac ligaments

    3. Pubic symphysis diastasis, intact anterior sacroiliac ligaments, torn sacrotuberous ligament, intact posterior sacroiliac ligaments

    4. Pubic symphysis diastasis, torn anterior sacroiliac ligaments, torn sacrotuberous ligament, intact posterior sacroiliac ligaments

    5. Pubic symphysis diastasis, intact anterior sacroiliac ligaments, torn sacrotuberous ligament, torn posterior sacroiliac ligaments

    Corrent answer: 4

    APC II injuries are unstable injuries and occur as a result of high-energy trauma. Anatomic structures which are injured or torn include the pubic symphysis, anterior iliosacral ligaments, and the sacrotuberous ligaments. The posterior sacroiliac ligaments are spared in APC-II injuries, and differentiate an APC-II injury from an APC-III injury, in which the posterior ligaments are also torn.

     

    Burgess et al review the classifications of pelvic ring disruptions and their association with mortality. They concluded that APC injuries required more blood replacement and were related to death more often than lateral compression, vertical shear, or combined mechanism pelvic injuries.

     

    Tile studied the anatomy of anterior to posterior pelvic ring injuries. Although the anterior structures, the symphysis pubis and the pubic rami, contribute to 40% to the stiffness of the pelvis, clinical and biomechanical studies have shown that the posterior sacroiliac complex is more important to pelvic-ring stability. The posterior sacroiliac ligamentous complex is more important to pelvic-ring stability than the anterior structures and therefore, the classification of pelvic fractures is based on the stability of the posterior lesion.

     

    Illustration A shows the APC classification.

     

     

     

     

     

     

    OrthoCash 2020

     

  108. Which of the following deformities is most likely to occur with dynamic hip screw fixation of unstable left sided standard obliquity hip fractures?

    1. Posterior spike displacement of the proximal fragment

    2. Anterior spike displacement of the proximal fragment

    3. Lateral displacement of the proximal fragment relative to the distal fragment

    4. Shortening of the proximal fragment relative to the distal fragment

    5. Medial displacement of the proximal fragment in relation to the distal fragment

    Corrent answer: 2

    Left-sided unstable intertrochanteric hip fractures are at increased risk of malreduction compared to unstable right-sided fractures fixed with dynamic hip screws. In left-sided fractures the rotational torque imparted to the proximal head and neck fragment can cause loss of reduction leading to potential failures of fixation. With these left sided injuries, the rotational torque can cause an anterior spike, whereas with right-sided injuries the rotational torque causes compression and reduction of the fracture. In addition, if a nail is used for these injuries and the proximal fracture fragment is not being held by the nail itself, this phenomenon can be seen as well.

     

    Mohan et al conducted a study to assess the effect of clockwise rotational torque onto the fracture configuration in unstable and stable intertrochanteric fractures fixed with a dynamic hip screw construct. They found that 11 out of 30 unstable fractures showed an anterior spike (flexion malreduction) in left-sided fixations due to clockwise torque. This malreduction was not present in right-sided or stable fractures.

     

    Illustrations A and B are images from Mohan et al's study showing the rotational affect on the fracture with placement of a dynamic hip screw.

     

     

     

     

     

     

     

     

    OrthoCash 2020

     

  109. A 45-year-old male sustains a proximal third tibia fracture as an isolated injury and elects to undergo operative treatment with intramedullary nailing. Post-operative radiographs show excessive procurvatum deformity. Which of the following operative techniques would have helped to best avoid the procurvatum deformity?

    1. Tibial nailing with increased knee flexion

    2. Lateral blocking screw in the proximal fragment

    3. Medial blocking screw in the proximal fragment

    4. Anterior blocking screw in the proximal fragment

    5. Posterior blocking screw in the proximal fragment Corrent answer: 5

    Posterior blocking screws in the proximal tibial segement help to avoid tibial procurvatum deformity and malunion. Proximal third tibia fractures are often times difficult to reduce anatomically due to the tendency for both valgus and flexion deformity at the fracture site. The posterior blocking screw helps to eliminate the tendency for the nail to be too posterior and cause the fracture to flex. Blocking screws should be placed on the concavity of the deformity to minimize the procurvatum and valgus deformities of this fracture pattern.

     

    Krettek et al looked at the importance of using blocking screws during intramedullary nailing of metaphyseal fractures using small diameter nails. They found less procurvatum deformity and malunions associated with using of blocking screws and found no complication with their utilization.

     

    Ricci et al reviewed fractures of the proximal third of the tibial shaft treated with intramedullary nails and blocking screws. Their results supported that blocking screws were effective in maintaining alignment of fractures of the proximal third of the tibial shaft treated by intramedullary nailing.

     

    Hiesterman et al review different reduction techniques to avoid malalignment, including the use of a proper starting point and insertion angle, blocking screws, unicortical plates, and a universal distractor.

     

    Illustration A shows a radiograph utilizing posterior proximal and distal blocking screws to prevent procurvatum deformity.

     

    Incorrect Answers:

    1. would exacerbate procurvatum deformity.

    2. would help avoid valgus deformity.

    3. would exacerbate valgus deformity.

    4. would exacerbate procurvatum deformity.

     

     

     

     

     

     

    OrthoCash 2020

     

  110. A 66-year-old male with a remote history of prostate cancer sustains a fall down a hill while gardening. During intramedullary nailing of his fracture, which intraoperative reduction maneuvers should take place to the proximal fragment to properly align the fracture?

     

     

     

     

     

     

    1. Flexion and internal rotation

    2. Extension and internal rotation

    3. Flexion and external rotation

    4. Extension and external rotation

    5. Abduction and internal rotation Corrent answer: 2

    Subtrochanteric fractures will cause a proximal fragment to be flexed, abducted, and externally rotated due to the imbalanced proximal muscular attachments. The proximal fragment would likely have to be extended, adducted, and internally rotated to obtain a proper reduction.

     

    Lundy did a review on subtrochanteric fractures. He reviews that these fractures can be effectively stabilized with 95 degrees plates, femoral reconstruction nails, or trochanteric femoral nails. Although intramedullary nails produce very stable constructs that are a great treatment option for this

    fracture, 135 degrees hip screw-plates are not suitable in the treatment of subtrochanteric femoral fractures due to the high risk of loss of fixation and fracture displacement.

     

    Illustration A depicts the applicable deforming muscle forces.

     

     

     

     

     

     

    OrthoCash 2020

     

  111. A 14-year-old presents on the request of her pediatrician for evaluation of her left hip. The patient reports having a recent history of lower abdominal pain, and as part of the work-up a KUB radiograph was obtained. The abdominal work-up was negative, and her pain has since resolved, however, the pediatrician noted an abnormal radiographic finding in the left hip and requested a formal orthopedic evaluation. The patient denies any history of hip trauma or pain. A left hip radiograph is shown in Figure A, and the the abnormality in question is indicated by the white arrow. The radiographic finding is most consistent with which of the following?

     

     

     

    1. Os acetabuli marginalis superior

    2. Fovea capitis

    3. Myositis ossficans

    4. Avascular necrosis.

    5. Acetabular fracture

     

    Corrent answer: 1

     

    The patients hip radiograph demonstrates an os acetabuli marginalis superior which is a benign accessory ossification center found in the superior aspect of the acetabulum. This can be commonly confused with an acute fracture or avascular necrosis. Although the os acetabuli marginalis superior occasionally persists into adult life, it usually fuses to the acetabulum by the time an individual reaches age 20.

     

    Caudle et al provide a case report of a a patient with a painful os acetabuli marginalis superior. This was successfully treated with resection of the fragment, and bone grafting. This was noted to be a very unusual source of hip pain in adolescents.

     

    Incorrect Answers:

    1. The fovea capitis is the depression on the head of the femur where the ligamentum teres inserts. This can appear as a small ossicle on the surface of femoral heads in skeletally immature patients. An example of this is shown in Illustration A.

    2. Myositis ossificans is soft tissue calcification which develops after trauma, or more rarely, surgery. An example of myositis ossificans around the hip is shown in Illustration B.

    3. Avascular necrosis of the femoral head classically occurs in patient with a history of alcoholism, steroid use, or sickle cell disease. Radiographs can demonstrate femoral head sclerosis, and eventually collapse of the articular surface. An example of femoral head AVN in a patient with sickle cell disease is shown in Illustration C.

    4. Acetabular fractures occur in the setting of trauma, and are relatively rare in the pediatric population. An example of a left sided acetabular fracture is shown in Illustration D. Illustration E shows a right sided acetabular fracture through the triradiate cartilage.

     

     

     

     

     

     

     

     

     

     

     

     

     

     

    OrthoCash 2020

     

  112. Which of the following tibial plateau fractures would be most appropriately treated by buttress plating alone?

     

     

     

     

     

     

     

     

     

     

     

     

     

    1. Figure A

    2. Figure B

    3. Figure C

    4. Figure D

    5. Figure E

     

    Corrent answer: 4

     

    Buttress plating is best indicated for simple partial articular fractures, such as shown in Figure D. Buttress plates can support a metaphyseal fragment and neutralize the shear and compressive forces across the cancellous bone. The actual buttress effect is only on the side of the plate. Pre-shaped plates are frequently used as buttress plates, as they conform to local anatomy, however exact contouring of the plate is necessary. Isolated buttress plating is not appropriate for articular depression fractures, such as those shown in Figures A, C, and E. Furthermore, buttress plating would provide inadequate fixation for a metadiaphyseal fracture, such as that shown in Figure B.

     

    Illustration A shows an example of a buttress plate used to treat a tibial plateau fracture.

     

     

     

     

     

    OrthoCash 2020

     

  113. A 73-year-old female presents with persistent right shoulder pain 3 months after undergoing open reduction and internal fixation for a right proximal humerus fracture. Which of the following could have best prevented the complication shown in the current radiograph shown in Figure A?

     

     

     

    1. Insertion of both cortical and locking screws into the humeral head

    2. Addition of a 20-gauge intraosseous tension band laterally through the greater tuberosity

    3. Treatment of the fracture with closed reduction and percutaneous k-wire fixation

    4. Addition of an inferomedial locking screw within the calcar

    5. Intramedullary nailing of the fracture Corrent answer: 4

    Figure A shows varus collapse and intra-articular joint penetration of the the proximal locking screws. This could have potentially been prevented by the addition of an inferomedial calcar screw, which would have provided greater strength to the fixation construct and resistance to fracture collpase.

    Illustration A is an immediate post-operative fluoro image of the fracture and shows the proximal humerus to be near anatomically aligned. The illustration also demonstrates simulated overlay of where the recommended inferomedial locking screw would be placed. Illustration B shows ORIF of a different proximal humerus fracture with placement of an inferomedial calcar screw.

     

    Konrad et al present a prospective, multicenter, observational study to evaluate the functional outcome and the complication rate after ORIF of proximal humeral fractures with use of a locking proximal humeral plate. At 1 year follow-up, they found an overall complication rate of 34%: most

    commonly due to screw perforation through the humeral head. Nineteen percent of the patients required re-operation within 1 year of their index surgery.

     

    Gardner et al discuss a technique of using a segment of fibula allograft, placed endosteally and incorporated into the locking construct, to aid in the reduction and restoration of the mechanical integrity of the medial column of the proximal humerus. An example of this fixation construct is shown in Illustration C.

     

    Gardner et al, in another study, sought determine what factors influence the maintenance of fracture reduction after locked plating of proximal humerus fractures, and particularly the role of medial column support. They found that achieving an anatomic or slightly impacted stable reduction, as well as meticulously placing a superiorly directed oblique locked screw in the inferomedial region of the proximal fragment, may achieve more stable medial column support and allow for better maintenance of reduction.

     

     

     

     

     

     

     

     

     

     

    OrthoCash 2020

     

  114. A 54-year-old female sustains a communited tibial shaft fracture from an accident at work. She undergoes simultaneous external fixation and ORIF using minimally invasive plate osteosynthesis. Following surgery, she complains of numbness along the dorsum of her medial and lateral foot. In which location (labeled A - E) on Figure A did percutaneous placement without careful dissection of a pin/screw likely cause her nerve injury?

     

     

     

    1. A

    2. B

    3. C

    4. D

    5. E

     

    Corrent answer: 5

     

    The above clinical scenario describes a post-op superficial peroneal nerve (SPN) deficit following ORIF of a tibial fracture using both external fixation and minimally invasive plate osteosynthesis (MIPO). The less invasive stabilization system (LISS) plate by Synthes is a system which utlizes the MIPO technique. This minimally invasive technique can increase the risk of damage to the SPN without careful identification of the nerve distally due to its close proximity to LISS plate holes 11-13.

     

    Deangelis et al studied the anatomy of the superficial peroneal nerve in relation to fixation of tibia fractures with the LISS plate using cadaveric dissections. They found that the superficial peroneal nerve is at significant risk during percutaneous screw placement in holes 11 through 13 of the 13-hole proximal tibia LISS plates. They recommended using an incision and careful dissection down to the plate in this region of distal locking screws to minimize the risk of damage to the nerve.

    Roberts et al also studied neurovascular anatomy of the leg in relation to screw placement, but did it in relation to locking screws used in intramedullary nailing. They concluded that intramedullary nail locking screws placed from a lateral-to-medial direction minimized the risk of injuries to the SPN and tibial neurovascular bundle. A disadvantage of lateral-to-medial locking screw placement was less resistance to nail bending forces.

     

     

     

    OrthoCash 2020

     

  115. What structure is 7cm from the acromion and at greatest risk of injury during a deltoid splitting approach for a proximal humerus fracture?

    1. Radial nerve

    2. Suprascapular nerve

    3. Axillary nerve

    4. Axillary artery

    5. Axillary vein

     

    Corrent answer: 3

     

    The axillary nerve is located approximately 7cm from the tip of the acromion. The axillary nerve comes off the of the brachial plexus (middle trunk, posterior division, posterior cord) carrying fibers from C5 and C6. The axillary nerve travels through the quadrangular space with the posterior circumflex humeral artery and vein to innervate the teres minor and deltoid muscles and supply sensation over the lateral shoulder.

     

    Based on the knowledge of the course of the axillary nerve, and potential complications regarding the vascular supply to the humeral head with the delto-pectoral approach, some authors are suggesting deltoid-splitting approach to the proximal humerus for reduction and fixation of proximal humeral fractures.

     

    Cetik et al evaluated 24 cadaver shoulders to better identify the course of the axillary nerve and identify the "safe zones" for deltoid-splitting incisions. They found the distance from the tip of the acromion to the axillary nerve varied depending on patient specific factors like arm length and they identified a "safe area" where dissection was free of injury to the axillary nerve (Illustrations A and B).

     

     

     

     

     

     

     

     

    OrthoCash 2020

  116. A 48-year-old active female runner underwent percutaneous screw fixation of a minimally displaced femoral neck fracture six months ago. There were no immediate post-operative complications, and she was progressed to full weight bearing three months after surgical fixation. Initial post-operative radiographs, and radiographs taken 3 months post-op revealed anatomic reduction of the fracture with no shortening. At her latest clinic visit she reports severe right groin pain, and difficulty ambulating. A current radiograph is shown in Figure A. What is the most appropriate surgical treatment?

     

     

     

     

    1. Hemiarthroplasty

    2. In situ dynamic hip screw revision fixation

    3. Core decompression and bone grafting

    4. Valgus intertrochanteric osteotomy with blade fixation

    5. Open reduction, bone grafting, and revision percutaneous screw fixation Corrent answer: 4

    The clinical presentation is consistent with a femoral neck nonunion, which is supported by the presence of new varus collapse and shortening which was not noted on prior radiographs. The most appropriate method to treat this complication is valgus intertrochanteric osteotomy of the femur with blade plate fixation.

     

    Valgus intertrochanteric osteotomies function by making a vertical fracture more horizontal, converting shear into compressive forces. It also helps correct the varus position of the fracture nonunion.

     

    Watson et al performed a retrospective review of the complications associated

    with combination femoral neck/shaft fractures and found 13 patients who had healing complications develop after their index surgical procedure. Eight femoral neck nonunions occurred, and these healed after either valgus intertrochanteric osteotomy (seven patients) or compression hip screw fixation (one patient).

     

    Jackson et al state that nonunion may occur in up to 43% of displaced intracapsular femoral neck fractures. The authors present an evidence-based algorithm regarding procedures for treatment of femoral neck nonunion and the roles of refixation, osteotomy, grafting, and prosthetic replacement when indicated.

     

    Angelini et al provide a review on salvage procedures after failed fixation of hip fractures. The authors state that in the setting of a nonunion in the younger patients with a well-preserved hip joint, treatment should typically involve revision internal fixation with or without osteotomy or bone grafting. They conclude that overall, salvage of nonunions of femoral neck and intertrochanteric hip fractures in properly selected patients can provide patients with good to excellent results.

     

    Figure A shows a femoral neck nonunion with varus collapse. Illustration A shows an example of a valgus intertrochanteric osteotomy with blade fixation.

     

     

     

     

     

     

    OrthoCash 2020

     

  117. A 32-year-old male sustains a posterior wall acetabulum fracture as the result of a high-speed motor vehicle collision. Improved patient-reported outcomes after surgical treatment are associated with which of the following variables?

    1. Increased age

    2. Increased hip flexion-extension arc

    3. Immediate weight-bearing

    4. Increased hip muscle strength

    5. Decreased stride length Corrent answer: 4

    Patient functional outcomes after acetabular fractures have been shown to be related to postoperative hip strength, regardless of surgical approach.

     

    The reference by Borrelli et al evaluated muscle strength and outcomes after acetabular surgery via an anterior approach. They report that hip extension strength was affected least (6%), whereas abduction, adduction, and flexion strength was affected to a greater degree. They note that hip muscle strength after operative treatment of a displaced acetabular fracture directly influences patient outcome.

     

    The reference by Engsberg et al is a review of patients that underwent ORIF of acetabular fractures through anterior or posterior approaches. They report that maximizing hip muscle strength may improve gait, and improvement in hip muscle strength and gait is likely to improve functional outcome. Worsening functional outcomes were correlated with decreased gait kinematics and stride length.

     

     

     

    OrthoCash 2020

     

  118. A 22-year-old male undergoes retrograde intramedullary nailing for the injury seen in Figure A. Which of the following would place branches of the femoral nerve and deep femoral artery at greatest risk during placement of the interlocking screw seen in Figure B?

     

     

     

     

     

     

    1. Anterior to posterior placement above the lesser trochanter

    2. Anterior to posterior placement below the lesser trochanter

    3. Lateral to medial placement above the lesser trochanter

    4. Lateral to medial placement below the lesser trochanter

    5. Open placement with blunt dissection down to bone Corrent answer: 2

    Branches of the deep femoral artery and femoral nerve are most at risk during placement of anterior to posterior interlocking screws below the level of the lesser trochanter as seen in Figure B.

     

    Riina et al performed a cadaveric study examining the neurovascular structures at risk during proximal interlocking of retrograde femoral nails and found that the first division of the femoral nerve crossed the femur on average approximately 4cm distal to the piriformis fossa. In addition, the femoral artery was closest to the medial aspect of the femur 4cm distal to the lesser trochanter. The authors recommend placement of interlocking screws proximal to the lesser trochanter to minimize risk of iatrogenic neurovascular injury.

     

    Brown et al attempt to define relative safe zones (RSZs) for placement of both lateral-medial and anterior-posterior interlocking screws of femoral nails in patients with acetabular fractures with reference to the location of the femoral neurvascular structures at the level of the lesser trochanter. The authors found that the available safe zone for placement of anterior-posterior screws decreased more than 50% in the setting of hematoma from acetabular fracture, and they recommend blunt dissection to bone and use of a single incison between the proximal interlocking holes to minimize risk to these structures.

     

    Handolin et al present two case reports of injury to the deep femoral artery with proximal interlocking screws during retrograde nailing. Based on their experiences, the authors advocate for blunt dissection to bone to avoid entrapment of critical structures when places these screws.

     

    Illustration A shows a cadaveric dissection and illustration of the numerous branches of the deep femoral artery and femoral nerve as they cross the femur from medial to lateral on the anterior surface just below lesser trochanter. The view is of the anterior thigh from the medial aspect. The sartorius has been cut and reflected and the retractors are beneath the rectus femoris.

     

     

     

     

     

    OrthoCash 2020

     

  119. A 23-year-old male is involved in a motor vehicle accident and sustains a left open femur fracture, right open humeral shaft fracture, and an LC-II pelvic ring injury. Which of the following best describes the radiographic findings associated with this pelvic injury pattern using the Young-Burgess Classification system?

    1. Crescent fracture located on the side of impact

    2. Widened anterior SI joint, disrupted sacrotuberous and sacrospinous ligaments with intact posterior SI ligaments

    3. Complete SI disruption with lateral displacement

    4. Sacral compression fracture on side of impact with transverse pubic rami fractures

    5. Open-book injury with contralateral sacral compression fracture Corrent answer: 1

    Lateral compression type II fractures (as described by the Young-Burgess Classification System) are associated with a crescent fracture of the iliac wing located on the side of impact. A representative CT scan image and illustration of this injury are shown in Illustrations A and B respectively. A table describing each pelvic injury and their associated complications is shown in Illustration C. Illustration D shows each Young-Burgess pelvic injury type.

    Burgess et al discuss the effectiveness of a treatment protocol as determined by their pelvic injury classification and hemodynamic status. The injury classification system was based on lateral compression, anteroposterior compression, vertical shear, and combined mechanical injury types. They found that their classification-based treatment protocols reduce the morbidity and mortality related to pelvic ring disruption.

     

    Tile discusses acute pelvic trauma and his classification system for pelvic injuries (ie. Types A, B, and C). He states that any classification system must be seen only as a general guide to treatment, and that the management of each patient requires careful, individualized decision making.

     

    Incorrect Answers:

    Answer 2: This describes an APC-II injury Answer 3: This describes an APC-III injury Answer 4: This describes and LC-I injury

    Answer 5: This describes an LC-III injury (ie. "wind-swept pelvis")

     

     

     

     

     

     

     

     

     

     

     

    OrthoCash 2020

     

  120. A 65-year-old male presents with continued left hip and thigh pain, and inability to bear full weight after undergoing ORIF of a left proximal femur fracture 3 months ago. Current radiographs are shown in Figure A. The patient denies any fevers, or other systemic signs of illness. Which of the following would have potentially decreased the risk of excess fracture collapse and implant failure in this patient?

     

     

     

    1. Use of a six-hole 135 degree compression plate

    2. Addition of iliac crest autograft to the fracture site

    3. Application of long strut allografts around the fracture site

    4. Placement of a cephalomedullary nail

    5. Addition of an 7.3mm de-rotation screw in the femoral head Corrent answer: 4

    The patient is presenting with a reverse obliquity peritrochanteric fracture nonunion, as shown in Figure A. 135-degree compression plate implants (such as the one used in this patient) are designed to stabilize intertrochanteric femoral fractures, and they may be unable to resist the deforming forces inherent in subtrochanteric fractures. When this device is used, the distal fragment often displaces medially and proximally as the fracture settles. The proximal fragment also may rotate on the compression screw because the plate design allows only for one screw in the proximal fragment.

    Cephalomedullary nails, such as that shown in Illustration A, have been shown to offer biomechanical superiority and diminished risk of implant failure when compared to plating of these injuries.

     

    Lundy provides a review article on the evaluation and treatment of subtrochanteric femur fractures. He states that these fractures can be effectively stabilized with 95° plates, femoral reconstruction nails, or trochanteric femoral nails with interlocking options. With regards to plates, he states that a 135° hip screw-plate is not suitable in the treatment of subtrochanteric femoral fractures, and that use of these implants may result in loss of fixation and fracture displacement.

     

    Menezes et al reviewed 155 consecutive patients who were treated with a proximal femoral nail from 1997 to 2001 to determine the rate of implant specific complications. They concluded that low rates of femoral shaft fractures and failure of fixation support the use of the proximal femoral nail for treatment of unstable trochanteric and subtrochanteric fractures.

     

    Robinson et al used the long Gamma nail to treat a consecutive series of 302 local patients who had sustained a subtrochanteric fracture during low-energy trauma over an 8 year period. They found that trochanteric-entry cephalomedullary nails are associated with an acceptable rate of perioperative complications and favorable functional outcomes.

     

    Incorrect Answers:

    1. Increasing the length of the the 135-degree compression plate to 6-holes will not improve its biomechanical properties in this fracture pattern.

    2. Addition of autograft would not improve the biomechanical stability of the

      fracture, and is not appropriate during the index procedure.

    3. Long strut allografts are not indicated in the initial treatment of reverse obliquity subtrochanteric fractures.

    5-Addition of a de-rotation screw would not change the stability of the fixation construct.

     

     

     

     

     

     

    OrthoCash 2020

     

  121. A 34-year-old female sustains a proximal third tibia fracture as an isolated injury and elects to undergo operative treatment with intramedullary nailing. Which of the following operative techniques would help to best avoid a procurvatum deformity of the tibia?

    1. Semiextended position during nailing

    2. Lateral blocking screws in proximal tibia fragment

    3. Use of a radiolucent triangle to flex the knee

    4. Anterior blocking screw in the proximal tibia fragment

    5. Medial parapatellar arthrotomy avoiding the patellar tendon Corrent answer: 1

    The semiextended position for intramedullay nailing of proximal tibia shaft fractures has shown to cause less complications of flexion deformity and malunion. Proximal third tibia fractures are often times difficult to reduce anatomically due to the tendency for both valgus and flexion deformity at the fracture site. The semiextended position helps to eliminate the tendency for

    the fracture to flex, due to the avoidance of excessive knee flexion during the reduction. Illustration A is an example of semiextended positioning for tibial nailing. Illustration B shows radiograph utilizing posterior blocking screws to prevent procurvatum deformity.

     

    Tornetta et al looked at the semiextended intramedullary nailing of proximal tibia fractures. Of the 24 patients nailed using this technique, 19 had no anterior angulation, while only 5 had less than 5 degrees of flexion deformity.

     

    In their study, Kubiak et al also advocate the semiextended position for tibial nailing. They note better control of fracture alignment and stabilization especially in the sagittal plane, and better ability to obtain imaging and maintenance of reduction during nail insertion and locking.

     

    Incorrect answers:

    Answer 2 would help to avoid valgus deformity. Answer 3 would worsen the flexion deformity.

    Answer 4 would worsen the flexion deformity and drive the nail posterior in distal segment.

    Answer 5 would not affect flexion deformity.

     

     

     

     

     

     

     

     

    OrthoCash 2020

     

  122. Which of the following cannulated screw configurations used in the treatment of subcapital femoral neck fractures is optimal?

    1. Inverted triangle pattern with the inferior screw posterior to midline and adjacent to the calcar

    2. Inverted triangle pattern with the inferior screw anterior to midline and adjacent to the calcar

    3. Triangle pattern with the superior screw posterior to midline and adjacent to the calcar

    4. Inverted triangle pattern with the inferior screw posterior to midline and central in the femoral neck

    5. Inverted triangle pattern with the inferior screw anterior to midline and central in the femoral neck

    Corrent answer: 1

     

    The strongest portion of the femoral neck is the posterior inferior neck in the region of the femoral calcar. The optimal biomechanical configuration includes an inverted triangle pattern with the single screw in the inferior aspect of the femoral neck adjacent to the calcar.

     

    Booth et al performed a cadaveric study comparing central versus calcar (cortical-adjacent) fixation. The results demonstrated significant improved stability, load, stiffness, and displacement in all tested parameters for the group with calcar-adjacent screw fixation.

     

    Lindequist and Törnkvist performed a Level 4 study of 72 femoral neck fractures. They found that all 5 of their nonunions had screws placed greater than 3mm from the femoral calcar. Additionally, 16 of 18 fractures healed in the group of displaced fractures where both the fixating screws were placed within 3 mm from the femoral neck cortex.

     

    Gurusamy et al performed a Level 4 study of 395 patients undergoing femoral neck fixation. They found a reduced spread of the screws on the lateral view was associated with an increased risk of nonunion of the fracture.

     

    Illustration A depicts the optimal configuration of an inverted triangle with the single screw being inferior and all of the screws being cortical adjacent.

     

     

     

     

     

     

    OrthoCash 2020

  123. A 63-year-old female sustained a distal radius and associated ulnar styloid fracture 3 months ago after being involved in a motor vehicle collision. Radiographs obtained at the time of injury are shown in Figure A. She underwent open reduction and fixation of the distal radius fracture, and current radiographs are shown in Figure B. At the time of the index operation, there was no distal radioulnar joint instability after plating of the radius. Which of the following is true post-operatively regarding this patient's ulnar styloid fracture?

     

     

     

     

     

     

     

    1. Worse outcomes on the Mayo wrist score are expected without fixation

    2. Chronic distal radioulnar joint instability can be expected to occur without fixation

    3. Wrist function depends on the level of ulnar styloid fracture and initial displacement

    4. Grip strength and wrist range of motion are improved with fixation

    5. There is no adverse effect on wrist function or stability without fixation Corrent answer: 5

    An accompanying ulnar styloid fracture in patients with stable fixation of a distal radial fracture has no apparent adverse effect on wrist function or stability of the distal radioulnar joint.

     

    Kim et al evaluated 138 patients who underwent surgical treatment of an unstable distal radial fracture, without fixation of an associated ulnar styloid fracture if present. Postoperative evaluation included measurement of grip strength and wrist range of motion; calculation of the modified Mayo wrist score; as well as testing for instability of the distal radioulnar joint at a mean of 19 postoperatively. They did not find a significant relationship between wrist functional outcomes and ulnar styloid fracture level or the amount of displacement.

     

    af Ekenstam et al performed prospective and randomized study of two different treatments of extraarticular Colles' fracture with a fractured ulnar styloid. In one group, the ulnar styloid was left alone, and in the other group it was transfixed and/or the triangular ligament was repaired after closed reduction of the fractured radius. They concluded that repair of the ulnar styloid complex in extraarticular fractures of the distal radius is not better than conventional treatment.

     

     

     

    OrthoCash 2020

     

  124. Each of the following are guidelines for management of a domestic violence victim EXCEPT:

    1. Socioeconomic status should not preclude evaluation for domestic violence

    2. Interview the patient outside the presence of other non-medical personnel

    3. Federal law mandates photographs be taken of injuries

    4. Document your opinion if the patient’s injuries are not consistent with the offered explanation

    5. Physicians should check requirements to see if there is mandatory reporting statute in their state

      Corrent answer: 3

       

      It is important to fully document the abuse as it has been described to you, however there is no federal law mandating photographic documentation of domestic violence injuries. Photographs may be taken but only with the patient's permission. Disclosure of a diagnosis of abuse to any third party and reporting it to the authorities should be done only with the abused patient’s knowledge and consent, unless there is a mandatory reporting statue in the particular state of practice. Being a female, age 19-29, pregnant, or of a low

      socioeconomic status (<$10,000 per/yr) increase one's risk for domestic abuse. Pregnancy is the highest risk factor for abuse with 40% to 60% of battered women reporting that they were abused during pregnancy.

      Interviewing should be done outside the presence of others to minimize potential interference by the abusive spouse/partner.

       

      The review article by Zillmer outlines that as many as 35% of women presenting to ERs for trauma care have injuries that are a result of domestic violence.

       

       

       

      OrthoCash 2020

       

  125. An otherwise healthy 34-year-old female undergoes an elective L5-S1 microdiscectomy. At her 4 week followup, she is noted to have drainage from her wound. Labs reveal a CRP of 30 mg/L (normal < 3) and an ESR of 88 mm/hr (normal < 20). The patient undergoes irrigation and debridement followed by a 6-week course of IV vancomycin (intra-op cultures positive for MRSA). At 20-week followup, the patient had another set of CRP and ESR drawn. Which set of values would indicate that the patient's infection has been successfully treated?

    1. CRP 20 mg/L, ESR 40 mm/hr

    2. CRP 15 mg/L, ESR 20 mm/hr

    3. CRP 6 mg/L, ESR 10 mm/hr

    4. CRP 1 mg/L, ESR 25 mm/hr

    5. CRP 8 mg/L, ESR 5 mm/hr

     

    Corrent answer: 4

     

    A normalized C-reactive protein (CRP) value (< 3 mg/L) would be the most likely value seen in a patient with a resolution of a post-op wound infection. Erythrocyte sedimentation rate (ESR) has been shown to decrease at a much slower place and its normalization is much less specific for infection resolution.

     

    CRP has been found to correlate with clinical response in patients treated with antibiotics for wound infection following spinal surgery. ESR has been found to remain elevated despite the presence of resolution of clinical symptoms and normalized CRP in these patients. It must be recognized that CRP can be elevated for reasons other than infection, including a slight increase with aging, rise during late pregnancy, and abnormal levels with liver failure.

    Similarly, ESR can be increased with pregnancy, some renal diseases, cancer, and anemia. Additionally, it can be decreased in sickle cell, leukemia, congestive heart failure, and hyperviscosity. Normal ESR values are between

    0-15 mm/hr for men under age 50 and between 0 and 20 mm/hr and for women under age 50.

     

    Khan et al. studied 21 patients with postoperative wound infections after spinal surgery. Of 21 patients they found resolution of all postoperative wound infections at the 20-week mark, and found that all patient had normalized CRPs (mean 0.8 mg/dL). However, they found ESR remained elevated (mean 29 mm/hr) despite normalized CRPs and complete resolution of signs and symptoms of infection.

     

    Fang et al. reviewed 48 patients with post-operative spinal surgery wound infections and compared to a random control group of 95 uninfected spinal surgery patients. They noted that Age >60 years, smoking, diabetes, previous surgical infection, increased body mass index, and alcohol abuse were all statistically significant preoperative risk factors for developing a post-op wound infection.. The most likely procedure to be complicated by an infection was a combined anterior/posterior spinal fusion performed in a staged manner under separate anesthesia. Infections were primarily monomicrobial and the most common organism cultured from the wounds was Staphylococcus aureus.

     

    Illustration A shows the CRP trends in the study by Khan et al. with postoperative wound infections after spinal surgery treated with IV antibiotics. ER refers to early responder while LR refers to late responder. At the 4-week time point, 16 patients have shown clinical improvement with no fevers, wound drainage, erythema, or need for wound packing. The other 5 patients, referred to as late responders, have shown one or more clinical signs of infection in addition to having an elevated CRP. At 20 weeks, both early and late responders both have shown normalized CRP levels and no clinical evidence of infection. Illustration B shows the ESR trends in the those which demonstrated incomplete normalization of ESR values despite resolution of infection.

     

    Incorrect Answers:

    Answer 1, 2, 3, 5: CRP levels > 3 mg/L are elevated and indicate incomplete response to infection

     

     

     

     

     

     

     

     

    OrthoCash 2020

     

  126. A 25-year-old man is struck by car while crossing the street. His injuries include the closed left tibial shaft fracture shown in Figure A. He is a smoker, but is otherwise healthy. Intramedullary nailing is performed without initial complications. Which of the following puts this patient at greatest risk for tibial nonunion?

     

     

     

    1. Use of anti-inflammatories post-operatively

    2. Post-operative gapping at the fracture site

    3. Presence of an associated fibular fracture

    4. History of smoking

    5. Mechanism of injury Corrent answer: 2

    Post-operative gapping at the fracture site significantly increased the risk of reoperation due to nonunion or malunion.

     

    Bhandari et al performed a retrospective study to identify which prognostic factors were associated with an increased risk of reoperation for nonunion in surgically treated tibial shaft fractures. They examined over 200 fractures, and found the presence of an open fracture wound (RR 4.32), lack of cortical continuity between the fracture ends following fixation (RR 8.33), and the presence of a transverse fracture (RR 20.0) were the three variables most predicitive of reoperation.

     

    Audige et al analyzed 416 patients with operatively treated tibial shaft fractures who were followed for at least 6 months. They found that the greatest risk for delayed healing or nonunion was the presence of an open injury, fractures of the distal 1/3 of the tibia, and postoperative gapping at the

    fracture site (The risk of healing problems was doubled for fractures of the distal shaft and for fractures showing a postoperative diastasis).

     

     

     

    OrthoCash 2020

     

  127. Figure A is a radiograph of a healthy, independent 51-year-old male. He is treated with immediate open reduction internal fixation to prevent which of the following complications?

     

     

     

     

    1. Fracture non-union

    2. Avascular necrosis

    3. Skin necrosis

    4. Plantar flexion weakness

    5. Ankle stiffness

     

    Corrent answer: 3

     

    Figure A shows an avulsion fracture of the calcaneal tuberosity. Immediate open reduction and internal fixation is required to prevent wound complications.

     

    Displaced avulsion fractures of the calcaneal tuberosity should be managed urgently to prevent necrosis of the soft tissues overlying the heel. In these injuries, the Achilles tendon is securely attached to the fractured tuberosity. Urgent closed reduction and casting is usually not possible due to the power and proximal pull of the triceps surae. Surgical fixation is required. The best treatment modality is open reduction and bone-to-bone fixation with screws. Closed reduction and percutaneous pinning fixation is not strong enough to provide a stable fixation of the tuberosity.

    Lui reported on avulsion fractures of the bony insertion of the Achilles tendon at the calcaneus. He stated that screw fixation alone is not sufficient for repair of these injuries. His technique involved two suture anchors used capture the small bone fragment to the calcaneus. This allowed for the pull of the triceps surae to be neutralized and early physical therapy.

     

    Hess et al. looked at a case series of calcaneal tuberosity avulsion fractures that were treated in a delayed fashion. All three patients with posterior tuberosity calcaneal avulsion fractures developed skin necrosis because of a delay in treatment.

     

    Figure A shows a displaced posterior tuberosity calcaneal avulsion fracture. Illustration A shows skin breakdown overlying the posterior tuberosity calcaneal avulsion fracture.

     

    Incorrect Answers:

    Answer 1: The amount of displacement is an indication for fixation, however urgent treatment does not improve union rates with these fractures.

    Answer 2: Tuberosity calcaneal avulsion fractures rarely disrupt the blood supply to the avulsion fragment and are not associated with avascular necrosis.

    Answer 3: Plantar flexion weakness is a known complication of these injuries despite many treatment options.

    Answer 4: Ankle stiffness is most commonly related to surgical fixation methods and post-operative immobilization and delayed rehabilitation.

     

     

     

     

     

    OrthoCash 2020

     

  128. A 22-year-old male presents 4 weeks following open reduction and internal fixation of his unstable ankle fracture. He has had three days of increasing pain, swelling and the new onset of purulent drainage from the mid-portion of the lateral incision. Laboratory values, including white blood cell count, sedimentation rate, and C-reactive protein are elevated. Current radiographs are seen in Figures A and B. On examination the wound probes deep and likely involves the lateral plate. What is the best step in management at this time?

     

     

     

    1. Suppression with broad spectrum oral antibiotics until fracture healing

    2. Suppression with broad spectrum intravenous antibiotics until fracture healing

    3. Surgical debridement, removal of internal fixation, culture specific antibiotics, casting until fracture healing

    4. Surgical debridement, maintenance of internal fixation, culture specific antibiotics until fracture healing

    5. Wound culture in the office and suppression with culture specific antibiotics until fracture healing

    Corrent answer: 4

     

    The patient is presenting with an acute deep infection following open reduction and internal fixation of an unstable ankle fracture. Recent studies have shown that a protocol of early aggressive surgical debridement, maintenance of internal fixation and culture specific antibiotics can be effective at achieving fracture healing.

     

    Management of early postoperative infection following open reduction and internal fixation can be challenging. Effective treatment typically involves a combination of surgical debridement and culture specific antibiotics. Removing internal fixation prior to fracture healing can lead to additional insult to the soft tissue and ongoing inflammation secondary to fracture instability. Recently published protocols have shown effective treatment with maintenance of implants and culture specific antibiotics following early, aggressive surgical debridement.

    Berkes et al. performed a multi-center retrospective study of 121 patients with acute postoperative infection (defined as less than 6 weeks from surgery) following internal fracture fixation. The authors demonstrated a 71% rate of success (defined by maintenance of implants until fracture healing) with a protocol of debridement and suppression with culture specific antibiotics. Risk factors for failure of this technique include open fractures and the use of an intramedullary nail for fracture fixation.

     

    Figures A and B show an ankle status post open reduction and internal fixation of a lateral malleolus fracture. There are no signs of loosening of fixation or cortical erosions concerning for osteomyelitis.

     

    Illustrations A and B are weight bearing X-rays that demonstrate the same fracture, now healed, after debridement and culture specific antibiotics.

    Illustrations C and D demonstrate the same fracture after elective removal of implants at 10 months following the index procedure. Intraoperative cultures at the time of hardware removal were negative for recurrent infection.

     

    Incorrect answers:

    Answers 1 and 2: Broad spectrum antibiotics without surgical debridement would not be effective in dealing with the infection in this clinical scenario with purulence tracking to the level of the plate

    Answer 3: Surgical debridement and antibiotics would control the infection adequately, however removal of fracture fixation in an unstable fracture would lead to instability, soft tissue inflammation and likely malunion or nonunion that would require complex revision

    Answer 5: Although culture specific antibiotics are an improvement over broad spectrum, again surgical debridement in conjunction with antibiotics is the most appropriate choice in this scenario. Intraoperative deep cultures in a sterile environment are preferred over cultures obtained in the office.

     

     

     

     

     

     

     

     

    OrthoCash 2020

     

  129. A 28-year-old male sustained an ankle injury 3 months ago, and was treated with closed management and splinting; a current x-ray is shown in Figure A. Which of the following is the most important factor in deciding between a joint sacrificing and a joint preserving operation for this patient at this time?

     

     

     

    1. Workers' Compensation involvement

    2. Gender

    3. Date of injury

    4. Degree of tibiotalar arthritis

    5. Degree of deformity Corrent answer: 4

    This patient presents with malunion after sustaining a bimalleolar ankle fracture. Surgical treatment options consist of osteotomy for deformity correction with internal fixation (joint preserving) versus fusion (joint sacrificing) with the primary determinant being the amount of ankle arthritis present.

     

    When treating ankle malunions, the decision to perform deformity correction and preserve the joint versus fusing the joint is dependent on signs of progressive, advanced tibiotalar arthritis on radiographs.

     

    Yablon and Leach followed 26 patients following corrective fibular osteotomy following malunion and noted excellent results at a mean follow-up of 7 years. All but 3 returned to preinjury level of activity and had desired outcomes following deformity correction.

    Reidsma et al. prospectively followed 57 patients with a minimum follow-up of 10 years following corrective osteotomy and fixation for ankle fracture malunions. Good to excellent results were obtained for 85% of the cohort. The authors concluded that those ankle fracture malunions with none to minimal arthritic changes should still receive corrective osteotomy to prevent further progression of arthritis.

     

    Yablon et al. following 53 patients for 6-9 months, reported on the importance of anatomic restoration of the lateral malleolus when fixing bimalleolar ankle fractures. With anatomic reduction and fixation, no progression of arthritis was noted with return to function.

    Figure A is an AP radiograph of a right ankle fracture malunion. Incorrect answers:

    Answers 1-3, 5: Factors such as age, gender, and degree of deformity are not

    as important as the presence of advanced arthritis that may suggest requiring a joint sacrificing fusion procedure. The date of injury may portend a poor outcome (the longer from the date of injury/subsequent malunion), but is not the most important factor in deciding between corrective osteotomy versus fusion.

     

     

     

    OrthoCash 2020

     

  130. A 45-year-old male presents after falling off of a ladder. Radiograph is shown in Figure A. Which of the following is the appropriate sequence in management?

     

     

     

    1. Closed reduction, splint application, computed tomography (CT) scan, delayed open reduction and internal fixation

    2. Closed reduction, cast application, close observation

    3. Splint application, CT scan, application external fixator, delayed open reduction and internal fixation

    4. Splint application, application external fixator, CT scan, delayed open reduction and internal fixation

    5. Splint application, acute open reduction and internal fixation Corrent answer: 4

    This patient has sustained a pilon fracture, with severe comminution and impaction at the articular surface.

     

    The correct sequence of management includes (1) immediate splinting, (2) application of an external fixator, (3) restoration of length alignment and rotation with temporizing external fixation, (4) computed tomography, followed by (5) definitive fixation once soft tissues are amenable.

     

    Tornetta and Gorup analyzed the use of preoperative CT scans in comparison to radiographs in preparation for fixing pilon fractures. The authors noted increased recognition of intra-articular fragments, comminution and noted a high percentage of operative planning changes following CT analysis.

    Furthermore, the authors recommended CT scans AFTER external fixation, for even better fragment characterization.

     

    Marsh et al. in their instructional course lecture provide tips and tricks in successful management of pilon fractures. One of the highlighted points include staged, delayed treatment of pilon fractures via spanning external fixator as well as highlighting the importance of obtaining the CT after restoring length and alignment.

     

    Figure A exhibits a radiograph of a comminuted, impacted, shortened pilon fracture.

     

    Incorrect answers:

    Answer 1: With such a short, impacted fracture, external fixation as a temporizing measure is recommended to help calm the soft tissue envelope. Answer 2: Closed treatment is not accepted in this type of fracture, due to the high incidence of arthritis and malunion.

    Answer 3: CT scan should be obtained AFTER external fixation, not before. Answer 5: This high-energy injury likely has severe soft tissue injury, which is not amenable to acute fixation.

     

     

    OrthoCash 2020

     

  131. A 45-year-old male with long-standing diabetes sustains the injury shown in Figure A. He has a BMI of 38, established peripheral neuropathy, and his most recent HbA1c is 8.8. What is the most appropriate definitive management option of Figures B through F?

     

     

     

     

     

     

     

     

    1. Figure B

    2. Figure C

    3. Figure D

    4. Figure E

    5. Figure F

     

    Corrent answer: 4

     

    Open reduction and internal fixation (ORIF) remains the mainstay of treatment for ankle fractures in patients with diabetes.

     

    ORIF for ankle fractures in diabetics can be augmented with increased density of fixation to account for notable, pathologic bone. Specifically, multiple quadricortical syndesmotic screws, bicortical medial malleolar screws, and stiffer plates are all viable options. Furthermore, due to delayed healing properties, prolonged immobilization may also be required to avoid fixation failure.

     

    Guo et al. performed a cohort controlled comparison between diabetics and non-diabetic patients with operative ankle fractures. Although they hypothesized that there would be more complications in the diabetic group, there was no statistical differences in fixation failure or complications when adhering to treatment principles for diabetics (including prolonged non-weight bearing for 10-12 weeks and increased density of fixation).

     

    Chaudhary et al. review the notable complications following ankle fracture treatment in patients with diabetes. In reviewing the literature, the authors recommend ORIF with meticulous soft tissue handling, increased density of fixation, and prolonged immobilization as the mainstay of diabetic ankle fracture treatment. External fixation and frames, while treatment options, should be reserved for salvage or infectious clinical scenarios.

     

    Figure A depicts a bimalleolar ankle fracture. Figure B depicts a cam walker.

    Figure C depicts a cast. Figure D standard fixation for ankle fracture for a patient without diabetes. Figure E exhibits definitive fixation with additional screws to increase construct stability. Figure F depicts a ring fixator.

     

    Incorrect Answers:

    Answers 1,2: Non-operative treatment is not appropriate for this fracture pattern

    Answer 3: Figure D, typically used for those without diabetes is less appropriate for this patient/clinical scenario.

    Answer 5: While external fixation and a ring fixator may be utilized, ORIF should be attempted first, however, as it provides stiffer fixation.

     

     

     

    OrthoCash 2020

     

  132. Distraction bone block arthrodesis alone would most likely help a patient suffering from a painful calcaneus fracture malunion with all of the following except:

    1. Low talar declination angle

    2. Hindfoot varus alignment

    3. Subtalar arthritis

    4. Talonavicular subluxation

    5. Peroneal impingement

       

      Corrent answer: 5

       

      Distraction bone block arthrodesis in isolation would be unlikely to improve pain related to peroneal impingement. Lateral wall exostectomy would likely be needed for this, and should be concomitantly performed in most cases.

       

      Calcaneal fracture malunions demonstrate several common patterns. Patients may manifest pain from anterior tibiotalar impingement (a result of a low talar declination angle), difficulty with shoe wear due to shortening and widening of the hindfoot, painful talonavicular subluxation, subfibular impingement, and post-traumatic subtalar osteoarthritis. The distraction bone block arthrodesis procedure was developed to eliminate the pain of subtalar arthritis while simultaneously normalizing hindfoot height by inserting a contoured structural bone graft into the subtalar joint. The procedure can correct pathologic hindfoot varus/valgus and restore a normal talocalcaneal angle, thereby improving symptomatic post-traumatic pes planus. The procedure alone does not address lateral wall blowout causing subfibular or peroneal impingement, which requires lateral wall exostectomy. Lateral wall exostectomy therefore should be done in addition to address this issue.

      Carr et al. first reported the use of subtalar distraction bone block arthrodesis for salvage of post-traumatic subtalar arthritis following calcaneus fractures. In their 16 patient series, distraction of the subtalar joint with insertion of the bone block allowed for correction of talocalcaneal angle, restoration of hindfoot height, and improvement in symptoms related to tibiotalar impingement.

       

      Clare et al. evaluated a treatment protocol for calcaneal malunions based upon the classification of Stephens and Sanders. Type II and III malunions were treated with subtalar bone-block arthrodesis and other concomitant procedures. They found that their treatment protocol was effective for pain relief, re-establishing a plantigrade foot, and improving function. The most difficult component of the case was restoration of calcaneal height.

       

      Braley et al. evaluated isolated lateral decompression in the treatment of symptomatic calcaneal malunions without concomitant subtalar arthrodesis. These 11 patients had persistent lateral sided pain and their malunions did demonstrate subtalar involvement. The authors reported 9 of the 11 patients had a satisfactory outcome with lateral decompression alone. They concluded that a lateral decompression in management of symptomatic malunions with lateral-sided symptoms is an essential consideration.

       

      Incorrect Answers:

      Answer 1: Talar declination angle describes the relative plantar- or dorsiflexion of the talus relative to the ground.

      Answer 2: Hindfoot alignment can be improved with distraction arthrodesis. Answer 3: Arthrodesis addresses painful subtalar arthritis.

      Answer 4: Restoration of the normal talocalcaneal angle can be achieved, which can also normalize the talonavicular relationship.

       

       

       

      OrthoCash 2020

       

  133. A 31-year-old patient has had activity related lateral ankle pain for 4 months. She underwent the procedure shown in Figure A approximately 8 months ago. What surgical technique has most likely resulted in this patients pain?

     

     

     

    1. Failure to recognize the most distal screw penetrating the joint surface

    2. Low posterior plating with prominent distal screw

    3. Failure to recognize an associated syndesmosis disruption

    4. Fracture malreduction causing shortening of the fibula

    5. Failure to use a longer plate with the lag screw positioned outside the plate Corrent answer: 2

    Figure A shows a low posterior plate with a prominent screw head in the most distal hole of the plate. This fixation technique is correlated with peroneal pathology, which usually presents months after fixation when the patient increases their activity level.

     

    The two most common fixation techniques of lateral malleolus fractures are (1) lag screw plus lateral neutralizing plating and (2) posterolateral antiglide plating. The disadvantages of the lateral plating includes the risk of intraarticular screws distally, prominent lateral hardware, and poor distal screw fixation. To overcome these complications, posterolateral antiglide plating allows for bicortical distal fixation with no articular perforation. However, low placement of the plate with a prominent screw head in the most distal hole is associated with symptomatic peroneal pathology. If the most distal screw is not prominent, or absent, this is less likely to cause peroneal complications.

     

    Weber et al. examined the effect of antiglide plate and screw positioning on peroneal tendon pathology. They showed that low posterior plating and large screw heads caused significant retromalleolar pain in most patients. To decrease peroneal pathology, they state that the distal end of the plate should stay proximal to the osteosynovial peroneal groove. Radiologically this level

    corresponds to the junction of the proximal and middle thirds of the lateral malleolus.

     

    Figure A shows a posterior positioned 5 hole 1/3 tubular plate. There is no articular screw penetration and the fracture is healed in an anatomical position. The distal aspect of the plate is is the distal third of the lateral malleolus.

     

    Incorrect Answers:

    Answer 1: The most distal screws rarely penetrates the joint with the use of fibular antiglide plates. In addition, there is no evidence of screw penetration in this patient.

    Answer 3: A missed syndesmosis disruption would usually show some radiographic findings. The tibiofibular clear space is usually most sensitive, which is measured radiographically by the distance from the lateral border of the posterior malleolus in the distal tibia to the medial border of the fibula. As a general rule, it is considered normal if the measurement is less than approximately 6 mm on both AP and mortise views.

    Answer 4: The fracture reduction looks anatomic. The talocrural angle, 'dime' sign and “Shenton's line” of the ankle all normal.

    Answer 5: The construct used to fix this isolated lateral trans-syndesmotic fracture is acceptable. The one-third tubular plate, which is placed posterolaterally on the fibula as an antiglide plate, indirectly reduces the fracture and acts as a buttress to resist the posterior and proximal displacement of the distal fragment. Insertion of a lag screw through the plate is a described technique.

     

     

     

    OrthoCash 2020

     

  134. A 35-year-old painter falls from a ladder sustaining an isolated fracture of his left calcaneus. Months later at follow-up, he is noted to have pain and a catching sensation in his medial foot with active flexion of the great toe. What is the most likely initial injury leading to this complication?

    1. Displaced calcaneal beak fracture

    2. Displaced fracture of the calcaneal tuberosity

    3. Comminuted posterior facet fracture

    4. Fracture of the sustentaculum tali

    5. Lateral wall blowout fracture Corrent answer: 4

    A known complication of fractures of the sustentaculum tali is stenosis (delayed) or injury (acute) of the flexor hallucis longus (FHL) tendon. Stenosis

    can cause pain and popping with great toe flexion.

     

    Fractures of the calcaneus often occur after falls from height, and in addition, may be associated with vertebral fractures due to the high-impact mechanism. The FHL tendon runs directly underneath the sustentaculum tali on the medial calcaneus and can be injured causing frank tears or delayed stenosis. These fractures may be missed on ankle or foot plain films alone, and advanced imaging should be ordered if clinical suspicion for calcaneus fractures exists given mechanism and location of pain/swelling.

     

    Komiya et al. present a case report of direct impalement of the FHL tendon in the tunnel under the sustentaculum tali. Though this particular complication is quite rare, such a report highlights the relevant anatomy and structures at risk as well as demonstrates the need for a good clinical exam as the injury was not noted on imaging but the concern was raised on physical exam of FHL involvement.

     

    Della Rocca et al. report their 19-patient series on operatively managed isolated sustentaculum tali fractures. They report a high rate of associated ipsilateral foot injuries (14 patients) and describe fixation using a medial approach and retracting the flexor tendons and neurovascular bundle.

     

    Illustration A is an axial CT image from Della Rocca (2009) et al. showing a representative sustentaculum tali fracture

     

    Incorrect Answers:

    Answer 1 - beak fractures are posterior, anatomically related to the Achilles tendon insertion, not the FHL tendon.

    Answer 2 - FHL tendon is not associated with the tuberosity.

    Answer 3 - FHL tendon runs anterior to the posterior facet and would unlikely be involved in that injury.

    Answer 5 - FHL tendon runs medially, not laterally.

     

     

     

     

     

    OrthoCash 2020

     

  135. A 30-year-old male patient involved in a hang-gliding accident sustains a knee dislocation with multiligamentous knee injury and transection of his peroneal nerve. He undergoes multiple reconstructive surgeries. Two years later, he continues to have a foot drop and dynamic tendon transfer is recommended. This treatment most commonly involves transferring a tendon from which native insertion point to which new insertion point?

    1. Plantar distal phalanges to medial navicular

    2. Medial navicular to dorsal lateral cuneiform

    3. Plantar 1st metatarsal to dorsal lateral cuneiform

    4. 5th metatarsal base to dorsal medial cuneiform

    5. Plantar distal phalanx of the hallux to dorsal distal phalanx of hallux Corrent answer: 2

    Dynamic tendon transfer to restore active dorsiflexion of the foot involves transferring the posterior tibial tendon (PTT) insertion on the medial navicular to the dorsal lateral cuneiform.

     

    Common peroneal nerve (CPN) injuries following traumatic knee dislocation are common, with an incidence of 25-40%. CPN palsy is characterized by foot drop

    due to loss of ankle dorsiflexors with a steppage gait and eventual development of a supinated equinovarus foot secondary to the unopposed pull of the PTT. Nonsurgical management involves use of an ankle-foot orthosis and physical therapy. Surgical options include acute primary repair, nerve grafting with either autologous sural nerve or nerve conduits and dynamic tendon transfer. The PTT is harvested from its insertion at the navicular, passed through the interosseous membrane (IOM) and anchored to the lateral cuneiform (see Illustration A). The classic bridle procedure involves concomitant anastamosis of the PTT to the tibialis anterior (TA) and peroneus longus (PL) tendons.

     

    Garozzo et al reported a case series of 62 patients with post-traumatic CPN palsy who underwent a one-stage procedure consisting of nerve repair and PTT transfer. Nerve repair combined with PTT transfer improved postoperative outcomes compared to nerve repair alone. At 2-year follow up, neural regeneration was demonstrated in 90% of patients. The authors hypothesized that poor outcomes following nerve repair alone are due to force imbalance between the functioning flexors and paralyzed extensors, which is somewhat equalized by performing a PTT transfer at time of repair.

     

    Niall et al reviewed 55 patients with traumatic knee dislocation and reported a 41% incidence of CPN injury, exclusively associated with dislocations involving disruption of the posterior cruciate ligament (PCL) and posterolateral corner (PLC). Complete neurologic recovery was found in only 21% of patients. The best prognosis was found with lesions in continuity, less than 7cm of nerve involvement, and short conduction block and muscle activity on nerve conduction and EMG studies.

     

    Vigasio et al described a dynamic tendon transfer technique for traumatic complete CPN injury, involving transfer of the PTT to the TA rerouted to a new origin at the lateral cuneiform to restore ankle dorsiflexion and flexor digitorum longus (FDL) to the extensor digitorum longus (EDL) and extensor hallucis longus (EHL) to restore digit dorsiflexion. Rerouting the TA towards the transferred PTT ensures the PTT harvest length is sufficient. This avoids excessive tensioning of the PTT, which may limit tendon excursion and result in a static tenodesis rather than dynamic function, as well as the need for PTT lengthening which may decrease strength of the transfer

     

    Illustration A is a series of intraoperative photographs demonstrating PTT transfer from Garg et al. An incision is made distal to the medial malleolus and the PTT is harvested subperiosteally (A). The PTT is delivered through a second incision ~15cm proximal to the medial malleolus (B-C). The PTT is then passed through the interosseous membrane and out a third incision over the anterior

    fibula (D). Lastly, the PTT is passed through a fourth incision over the dorsal midfoot and anchored to the lateral cuneiform (E).

     

    Incorrect Responses:

    Answer 1: Transferring the FDL (insertion = plantar distal phalanges) to the medial navicular is used for correction of flexible flatfoot deformity arising from PTT insufficiency. Some surgeons transfer the FDL to the medial navicular at the time of PTT transfer to the dorsum of the foot, to compensate for loss of PTT function and minimize risk of flatfoot development.

    Answers 3: Transferring the PL (insertion = plantar 1st metatarsal) is not recommended, as this muscle is innervated by the CPN via the superficial peroneal nerve and therefore would not be functional.

    Answer 4: Transferring the peroneus brevis (PB; insertion = 5th metatarsal base) is not recommended, this muscle is innervated by the CPN via the superficial peroneal nerve and therefore would not be functional.

    Answer 5: Transferring the flexor hallucis longus (FHL; insertion = plantar distal phalanx of the hallux) to the insertion of the EHL (dorsal distal phalanx of hallux) is recommended for correction of claw toe deformity and would not help restore foot dorsiflexion in this patient.

     

     

     

     

     

     

    OrthoCash 2020

     

  136. Which of the following represents the most common complication following operative treatment of the injury shown in Figure A?

     

     

     

     

     

     

     

     

     

     

     

     

     

    1. Figure B

    2. Figure C

    3. Figure D

    4. Figure E

    5. Figure F

     

    Corrent answer: 3

     

    This patient has a displaced talar neck fracture. The most common complication is post-traumatic arthritis.

     

    Complications after treatment of displaced talar neck fractures are common. Both tibiotalar and subtalar arthritis occur, with subtalar arthritis being the most common. Osteonecrosis is also common, but post-traumatic arthritis is the most common.

     

    Lindvall et al. reviewed 26 displaced talar fractures treated with internal fixation and found that post-traumatic arthritis was the most common complication, occurring in 100% of patients. Osteonecrosis was found in 13 of 26 patients (50%).

     

    Vallier et al. reviewed patients presenting with talar neck fractures and found post-traumatic arthritis in 21 of 39 patients (54%). Osteonecrosis was found in 19 of 39 patients (49%).

     

    Figure A shows a displaced talar neck fracture. Figure B shows talar neck nonunion (arrow) and osteonecrosis of the talar body. Figure C is an AP of the same patient shown in Figure B and again shows osteonecrosis of the talar body. Figure D shows subtalar arthritis after internal fixation of a talar neck fracture via medial malleolar osteotomy. Figure E shows a clinical photo of a patient with a varus malunion after talar neck fracture. Figure F shows a wound dehiscence.

     

    Incorrect answers:

    Answers 1, 2, 4, 5. These are all known complications of surgical treatment of displaced talar neck fractures but occur less frequently than post-traumatic arthritis.

     

     

     

    OrthoCash 2020

     

  137. A 25-year-old woman began training for a marathon and she reports a 2-week history of heel pain. She has pain throughout the day that worsens with prolonged weight-bearing. On exam, the location of

    maximal tenderness is indicated by the white arrow in Figure A. The patient denies point tenderness at the location of the yellow arrow in Figure A. Which of the following MRI images (Figures B to F) would you expect to find in this patient?

     

     

     

     

     

     

     

     

     

     

     

     

     

     

     

     

     

    1. Figure B

    2. Figure C

    3. Figure D

    4. Figure E

    5. Figure F

     

    Corrent answer: 2

     

    The clinical presentation is consistent with a stress fracture of the calcaneus. The T1 MRI shows a fracture line within the calcaneus which is consistent with

    a calcaneal stress fracture.

     

    Calcaneal stress fractures most commonly occur in long-distance runners and military recruits. Usually, they are caused by overload and inability of bone formation to match resorption. Patients usually begin a rapid increase of activity level from a prior sedentary lifestyle. Patients complain of heel pain that persists throughout the day. The pain is located along the medial and lateral walls of the calcaneus. Diagnosis can be made with radiographs 2-3 weeks after symptom onset by the appearance of a sclerotic line. If the diagnosis is uncertain, MRI may be obtained.

     

    Gehrmann et al. performed a review of stress fractures in the foot. They report that most studies of calcaneal stress fractures are from military recruits.

    Primary treatment includes rest, avoidance of weight-bearing, and physical therapy. Patients usually return to full duty between 8 and 10 weeks. Recruits with inadequate treatment had a recurrence of symptoms. They conclude that most studies are from the 1940’s and 1950’s and lack any significant detail into specific treatment protocols.

     

    Sormaala et al. performed a retrospective study in which they reviewed MRIs in all military recruits who had exercise-induced heel pain, over an eight-year period. They report that only 15% of injuries were visible on radiographs and a portion of patients had stress fractures of another tarsal bone. They conclude that a majority of calcaneal stress fractures occur in the posterior part of the bone, but some fractures may be found in the middle and anterior parts. They recommend obtaining an MRI if a radiograph is negative in a patient with exercise-induced foot or heel pain.

     

    Figure A demonstrates the most common area of tenderness and pain of a calcaneal stress fracture, indicated by the white arrow, while the yellow arrow points to the area of tenderness consistent with plantar fasciitis. Figure B is a sagittal T2-weighted MRI demonstrating plantar fasciitis. Figure C is a sagittal T1-weighted MRI demonstrating a calcaneal stress fracture. Figure D is an axial T1-weighted MRI demonstrating a mass on the medial aspect of the calcaneus consistent with a ganglion cyst. Figure E is a sagittal T1-weighted MRI of the foot demonstrating an intraosseous lipoma within the calcaneus. Figure F is a sagittal T2-weighted MRI demonstrating insertional Achilles tendinopathy.

    Illustration A demonstrates the most common etiologies of plantar foot pain.

     

    Incorrect Answers:

    Answer 1: Figure B, a sagittal T2-weighted MRI, demonstrates plantar fasciitis. Symptoms of plantar fasciitis are sharp heel pain, usually worse in the morning. Tenderness is usually found on the plantar aspect of the foot.

    Answer 3: Figure D, an axial T1-weighted MRI, demonstrates a ganglion cyst in the tarsal tunnel. Symptoms of tarsal tunnel syndrome include sharp burning pains in the foot and parasthesias and numbness in the plantar foot.

    Answer 4: Figure E, a sagittal T1-weighted MRI, demonstrates an intraosseous lipoma. Intraosseous lipomas are usually incidental findings.

    Answer 5: Figure F, a sagittal T2-weighted MRI, demonstrates insertional Achilles tendinitis. This usually presents with pain over the insertion of the Achilles. It may have a relapsing and remitting course, which worsens with activity.

     

     

     

     

     

     

    OrthoCash 2020

     

  138. A 35-year-old male fell and sustained an open talar neck fracture. He underwent operative fixation of his fracture. He presents at 2 months after surgery. He denies any constitutional symptoms and his pain is well controlled. On exam, his wounds are well healed with no erythema. Imaging is shown in Figure A. What can the patient be told about his condition?

     

     

     

    1. Hawkins sign is positive. The likelihood of developing osteonecrosis is high

    2. Hawkins sign is positive. The likelihood of developing osteonecrosis is low

    3. Hawkins sign is negative. The likelihood of developing osteonecrosis is high

    4. Hawkins sign is negative. The likelihood of developing osteonecrosis is low

    5. He has developed chondrolysis Corrent answer: 2

    This patient has a positive Hawkins sign, which signifies that he is unlikely to develop osteonecrosis.

     

    A subchondral radiolucency of the talar dome after a talar neck fracture is known as the Hawkins sign. It is an indication that the talar body is viable. It usually appears by 6-8 weeks after injury and is best seen on the mortise view. If the Hawkins sign is present, it is unlikely that the patient will develop osteonecrosis.

     

    Early writes a review on talus fracture management. He reports that talar neck fractures occur through the extra-articular portion of the talus and represent nearly half of talus fractures. The mechanism of injury is a combined ankle dorsiflexion followed by axial compression of the tibiotalar joint. He concludes that anatomic reduction gives the patient the best chance of a good outcome.

    Leduc et al. wrote a review on posttraumatic avascular necrosis of the talus. They note that AVN is diagnosed on plain radiography by the absence of the Hawkins sign. They report that CT can also reveal characteristic talar AVN patterns and can be used to confirm radiographic findings. CT helps to assess subtle depression, collapse, fragmentation, and arthritic changes. MRI remains the most sensitive technique for detecting osteonecrosis of the talus, especially in the early stages. They conclude that although there are many published treatments of AVN of the talus, outcome studies are still lacking.

     

    Tezval et al. performed a retrospective review to determine the prognostic reliability, sensitivity, and specificity of the Hawkins sign. They found that in all patients who developed osteonecrosis, none had the Hawkins sign present. In all patients that exhibited a Hawkins sign, none developed osteonecrosis. They determined the sensitivity of the Hawkins sign to be 100% while the specificity was 57.5%. They conclude that if a full or partial positive Hawkins sign is detected, it is unlikely that AVN will develop.

     

    Figure A demonstrates the Hawkins sign with subchondral radiolucency which is noted by arrows in Illustration A.

     

    Incorrect Answers:

    Answers 1, 3, 4: The patient has a positive Hawkins sign, therefore the chance of developing osteonecrosis is low.

    Answer 5: Chondrolysis is characterized by rapid destruction of articular cartilage on both sides of the joint which leads to loss of joint space

     

     

     

     

     

    OrthoCash 2020

     

  139. A 25-year-old male presents to the emergency department after a lawnmower accident with traumatic loss of his great toe. On examination, his wound is grossly contaminated with soil. In addition to a cephalosporin and an aminoglycoside, penicillin is given. Which of the following is true with regards to the organism that penicillin is targeting in this injury?

    1. It is an Aerobic, Gram-positive rod

    2. It is an Anaerobic, Gram-positive coccus

    3. It is an Anaerobic, Gram-negative rod

    4. It is Catalase positive

    5. It may cause botulism Corrent answer: 5

    The organism being covered with penicillin is Clostridia spp. Clostridium botulinum is a Gram Positive Bacilli that is the cause of botulism.

     

    Clostridia spp, is a Gram-positive, obligate anaerobic spore-forming rod that is found in soil and gut flora. It produces gas by the fermentation of glucose and

    may cause gas gangrene. Common bacteria of this genus are C. perfringens (most common), C. tetani (causes tetanus), C. difficile, and C. botulinum (causes botulism). If wounds are grossly contaminated with soil, penicillin is given to cover against Clostridia.

     

    Decoster et al. performed a review of traumatic foot wounds. They report that lawnmower injuries to the foot are relatively common. IV antibiotic therapy should be initiated with a broad-spectrum cephalosporin and an aminoglycoside to provide coverage against Gram-negative organisms.

    Penicillin should be given to protect against Clostridial infection. They conclude that irrigation and debridement is indicated initially followed by packing of open wounds. Repeat debridements are necessary as nonviable tissue demarcates. If major reconstructive procedures are necessary, they should be delayed as they have a high rate of failure if performed too soon.

     

    Cross et al. wrote a review on treatment principles in the management of open fractures and they note that in open fractures with soil contamination, additional coverage should be added for anaerobic bacteria, typically Clostridia. Another member of the genus Clostridia is C. tetani, the causative agent of tetanus. Vaccine status for tetanus must also be assessed in these situations as well.

     

    Illustration A is a radiograph demonstrating soft tissue swelling and subcutaneous emphysema, consistent with gas gangrene. Illustration B is a clinical photograph of gas gangrene evidenced by edema, discoloration, ecchymosis, and hemorrhagic bullae. Illustration C is a pathology slide of C. perfringens, a Gram-positive rod.

     

    Incorrect Answers:

    Answers 1, 2, 3: Clostridia are Gram-positive, obligate anaerobes. Answer 4: Clostridia are catalase negative.

     

     

     

     

     

     

     

     

     

     

    OrthoCash 2020

     

  140. All of the following are ways that a negative pressure dressing is beneficial to wound healing EXCEPT:

    1. Accelerated granulation tissue formation

    2. Removes excess proteins and electrolytes from wound

    3. Reducing anaerobic colonization

    4. Causes cells to release vascular endothelial growth factor by mechanical force

    5. Causes an increase in capillary afterload Corrent answer: 5

      Negative pressure dressings or vacuum-assisted wound closures (VAC) apply a negative pressure to the wound bed which allows a decrease in capillary afterload which produces a better inflow of blood.

       

      VAC dressings exert their positive effects on wound healing in multiple ways. Firstly, they remove interstitial fluids, which have been found to contain inhibitory factors that suppress the formation of fibroblasts, vascular endothelial cells, and keratinocytes. This also eliminates the formation of any superficial purulence or slime which also reduces the potential for anaerobic colonization. Removal of excess fluid also pulls out excess proteins and electrolytes to help maintain and osmotic and oncotic gradient. VACs allow arterioles to dilate which produces a proliferation of granular tissue. Also, there is a decrease in capillary afterload, (the pressure against which the heart must overcome to eject blood), which promotes better inflow of blood. Lastly, applying a mechanical force to the surrounding soft tissues allows the edges of the wound to be drawn towards the center, uniformly. This decreases the size of the wound over time. The micromechanical forces exerted on individual cells causes a release in local growth factors, such as vascular endothelial growth factor (VEGF), which stimulate wound healing.

       

      Herscovici et al. applied VAC dressings to 21 consecutive patients with open, high-energy soft tissue injuries. They found that wounds averaged 4.1 sponge changes and the device was used for an average of 19.3 days. 12 wounds avoided the need for further treatment. Only 9 patients required free tissue transfer. They concluded that the VAC is a viable treatment adjunct for the treatment of open, high-energy injuries.

       

      Clare et al. reported their experience with the VAC dressing in the treatment of non-healing, diabetic and dysvascular wounds. They retrospectively reviewed 17 patients with non-healing wounds of the lower extremity. 9 had diabetes and 8 had severe peripheral vascular disease. The average length of treatment

      was 8.2 weeks and 14/17 wounds successfully healed, and only 3 failed VAC treatment. They concluded that the VAC dressing is an acceptable option for wound care of the lower extremity.

      Illustration A is a photo of an open wound being treated with a VAC dressing. Incorrect Answers:

      Answers 1, 2, 3, and 4 are all ways that VAC dressings are beneficial to wound

      healing.

       

       

       

       

       

       

      OrthoCash 2020

       

  141. Which of the following is true regarding anterior sternoclavicular joint dislocations?

    1. Reduction may result in tracheal injury

    2. They are usually stable following closed reduction

    3. They require fusion to hold the reduction

    4. They are rarely symptomatic when left unreduced

    5. They should be treated acutely with medial clavicle excision Corrent answer: 4

    From the Bicos article, “Anterior SC joint instability should primarily be treated conservatively. The patients should be informed that there is a high risk of persistent instability with nonoperative or operative care, but that the persistent instability will be well tolerated and have little functional impact in the vast majority. Therefore, operative intervention for anterior SC joint instability is mainly cosmetic in nature."

     

     

     

    OrthoCash 2020

  142. An otherwise healthy 45-year-old female slips and falls with immediate right ankle pain. Stress examination of the right ankle is shown in Figure A. Which of the following is the most important for achieving a satisfactory outcome following open reduction internal fixation for this injury?

     

     

     

     

    1. Weight-bearing before 3 weeks.

    2. Medial clear space >5mm

    3. Talocrural angle of 83 degrees

    4. Tibiofibular clear space of >6mm

    5. Talar tilt of >5 degrees

     

    Corrent answer: 3

     

    Anatomic reduction of a rotational ankle fracture is considered the most important factor in achieving a satisfactory outcome. A talocrural angle of 83 degrees suggests an anatomic reduction has been achieved.

     

    Unstable rotational ankle fractures should be treated surgically in order to achieve anatomic reduction (if the fracture is not overly comminuted) as well as restore length, rotation, and alignment. Satisfactory outcomes can be best achieved when these surgical goals are achieved. Postoperative protocols

    regarding immobilization and weight bearing may be somewhat variable depending on surgeon preference, fracture pattern, modifiable patient factors, and non-modifiable patient factors.

     

    Lin et al. performed a systematic review of randomized studies looking at postoperative immobilization and rehabilitation following ankle fractures. They found that after surgical fixation, starting exercise in a removable brace improved pain, ankle range of motion, and improved activity limitations but led to a higher rate of adverse events. Early weight-bearing improved ankle range of motion as well. They concluded that there is limited evidence to support removable braces, early weight-bearing, and no immobilization following surgical fixation of ankle fractures.

     

    Reidsma et al. retrospectively reviewed 57 malunited ankle fractures treated with revision osteotomy with a minimum of 10 years of follow-up. They found that 85% of patients had good or excellent outcomes and that prolonged time to reconstructive surgery led to a worse outcome. They concluded that reconstructive surgery should be performed early in the setting of a malunited ankle fracture even with early arthritic changes.

     

    Figure A shows a displaced Weber B fibula fracture with medial joint space widening on stress examination. Illustration A shows intraoperative imaging following open reduction internal fixation of the same patient. Illustration B demonstrates the talocrural angle and medial clear space parameters in an anatomically reduced ankle.

     

    Incorrect Answers:

    Answer 1: Nonweightbearing for at least 6 weeks is common; however, some studies have shown improved outcomes in range of motion if weight-bearing is started early in the immobilization period.

    Answer 2, 4, and 5: Postoperatively the parameters of the medial clear space

    <4mm, tibiofibular clear space <6mm, and <5 degrees of talar tilt signifies anatomic correction of length, rotation, and alignment.

     

     

     

     

     

     

     

     

    OrthoCash 2020

     

  143. Which of the following arteries supplies the surgical flap in the extensile open treatment of the injury shown in Figure A?

     

     

     

    1. Lateral calcaneal branch of the anterior tibial artery

    2. Lateral calcaneal branch of the peroneal artery

    3. Lateral malleolar branch of the peroneal artery

    4. Lateral malleolar branch of the dorsalis pedis artery

    5. Lateral malleolar branch of the anterior tibial artery Corrent answer: 2

    This patient has a displaced calcaneal fracture that is commonly treated through a lateral extensile approach. The flap of the lateral extensile approach is supplied by the lateral calcaneal branch of the peroneal artery.

     

    Intraarticular fractures of the calcaneus represent approximately 2% of all fractures and are commonly the result of high-energy trauma such as motor vehicle accidents and falls. Successful operative treatment of these injuries depends on the anatomic reduction of the articular surface; restoration of the alignment, height, and length of the calcaneus; and avoidance of complications. These fractures are usually treated with a lateral extensile approach. The artery which provides blood supply to this flap is the lateral calcaneal branch of the peroneal artery. It is critical to maintain the integrity of this vessel in order to avoid complications.

    Borrelli et al. performed a study to describe the arterial blood supply of the subcutaneous tissues of the lateral hindfoot and to define the relationships between these arteries and the lateral extensile incision. The lateral calcaneal artery appeared to be responsible for the blood supply to the corner of the flap.

     

    Figure A demonstrates a displaced fracture of the calcaneus. Illustration A depicts the arteries on the lateral foot (PA: peroneal artery, LCA: lateral calcaneal artery, LMA: lateral malleolar artery, LTA: lateral tarsal artery). Illustration B (Borrelli et al.) is a lateral radiograph of the hindfoot and ankle that demonstrates skin staples placed along the surgical incision and vascular clips placed along the path of each artery. Illustration C is a clinical photograph of the lateral extensile approach.

     

    Incorrect Answers:

    Answer 1: The lateral calcaneal artery is a branch of the peroneal artery, not the anterior tibial artery.

    Answers 3-5: The lateral malleolar artery is a branch of the anterior tibial artery. It does not supply the flap of the lateral extensile approach.

     

     

     

     

     

     

     

     

     

     

     

    OrthoCash 2020

     

  144. A 65-year-old patient sustains the injury shown in Figure A. His medical history is significant for hypertension, Type 2 diabetes and dialysis dependent chronic kidney failure. A postoperative radiograph is shown in Figure B. Based on his risk factors, what is his most likely post operative mortality at two years after surgery?

     

     

     

     

    1. 13%

    2. 25%

    3. 45%

    4. 60%

    5. 100%

    Corrent answer: 3

     

    Based on the injury sustained and the risk factors (namely chronic renal failure) shown, the patient will likely have a postoperative mortality of 45% at 2-years post operatively.

     

    Hip fractures often occur older patients from low energy type mechanisms of injury. In femoral neck fractures, the relative pre-injury mobility is the most significant determining factor for the postoperative survival.

     

    Bhattacharya et al. reviewed factors affecting acute mortality in patients undergoing orthopaedic procedures. They found that patients with history of chronic renal failure had an univariate mortality rate of 9%. The most predictive factors of death were: chronic renal failure, CHF, COPD, hip fracture and an age greater than 70 years of age.

     

    Karaeminogullari et al. reviewed clinical outcomes of operative treatment of hip fractures in patients on chronic hemodialysis. A total of 29 patients sustained 32 hip fractures. The mortality rate, with an average follow up of 23 months, was found to be 45%. They found a significant association between age and risk of mortality.

     

    Figure A shows an AP radiograph demonstrative of a displaced femoral neck fracture. Figure B shows a postoperative radiograph with an appropriately placed hip hemi-arthroplasty. Illustration V is a video that provides a brief overview of the evaluation and management of femoral neck fractures.

     

    Incorrect Answers

    1, 2, 4, 5: Based on the current literature, the post operative mortality rates at close to 2 years of follow up are 45%

     

     

     

    OrthoCash 2020

     

  145. A 25-year-old male presents following a motor vehicle collision with a Glasgow Coma Scale of 7. Subsequent imaging in the trauma bay demonstrates a bifrontal cerebral contusion, an L4 burst fracture, multiple rib fractures, an LC-1 type pelvic ring injury, a femoral shaft fracture, and an open ipsilateral tibial shaft fracture. He is intubated and an intracranial pressure monitor is placed which consistently measures 30mm Hg. He is normotensive with a lactate of 1.5 after 2 liters of crystalloid and 1 unit of packed red blood cells. Which of his injuries would most dictate a temporizing approach with external

    fixation of his femoral shaft fracture instead of reamed intramedullary nailing?

    1. L4 burst fracture

    2. Bifrontal cerebral contusion

    3. Open ipsilateral tibia fracture

    4. LC1 pelvic ring injury

    5. Rib fractures

     

    Corrent answer: 2

     

    In the setting of a severe closed head injury such as a bifrontal cerebral contusion with elevated intracranial pressures, external fixation of a femoral shaft fracture is indicated to limit the risk of intraoperative hypotension and decreased cerebral perfusion pressure.

     

    Immediate reamed nailing remains the standard treatment for the vast majority of femoral shaft fractures, however patients with multiple injuries with incomplete resuscitation and patients with severe intracranial trauma may benefit from a damage control approach with external fixation.

     

    Anglen et al retrospectively reviewed the intracranial pressure (ICP) and cerebral perfusion pressure (CPP) in patients undergoing femoral nailing. The authors found a significant decrease in intraoperative CPP, especially in those patient undergoing femoral nailing in the first 24 hours, however they were unable to demonstrate a link between the decreased CPP and poor patient outcomes.

     

    Pietropaoli et al examined the effects of intraoperative hypotension on patients with blunt head trauma. The authors found that 32% of patients experienced intraoperative hypotension (systolic blood pressure less than 90mm Hg) and those patients with a hypotensive episode had an 82% mortality and significantly worse outcomes on the Glasgow Outcomes Scale.

     

    McKee et al conducted a retrospective cohort study comparing matched groups of patients with femoral shaft fractures with and without a closed head injury. In contrast to previous studies, the authors found no significant difference in outcome between the groups including mortality, hospital length of stay or neuropsychologic testing.

     

    Illustration A shows a femoral shaft fracture treated with external fixation. Incorrect Answers:

    Answer 1, 3-5: Immediate reamed nailing would not change the outcome of any of these injuries

     

     

     

     

     

     

    OrthoCash 2020

     

  146. A 22-year-old left hand dominant laborer sustains the injury shown in Figures A and B as the result of a fall from a ladder. Which of the following has been shown to be true regarding operative versus nonoperative treatment of this injury?

     

     

     

     

     

     

    1. Decreased chance of nonunion with nonoperative treatment

    2. Improved Constant and DASH scores with operative treatment at all time points

    3. Increased symptomatic malunion rate with operative treatment

    4. No change in shoulder abduction strength

    5. Increased time to union with operative treatment Corrent answer: 2

    Surgical management of displaced, shortened clavicle fractures is associated with a decreased rate of nonunion and malunion. General recommendations for surgical treatment include shortening of greater than 2 centimeters.

     

    Kim et al. review clavicle fracture treatment history and current indications. They report that although previous thought was that nearly all clavicle fractures should be treated nonoperatively, outcomes can be improved with fixation of certain clavicle fracture patterns.

     

    COTS et al. performed a multicenter, randomized controlled trial of 132 patients with a displaced midshaft fracture of the clavicle. They found that Constant and DASH scores were improved in the operative fixation group at all points in time, with union time being 28 weeks in the nonoperative group and

    16 weeks in the operative group. Malunion was higher in the nonoperative group as well.

     

    McKee et al. reviewed 30 patients who underwent closed treatment of a displaced midshaft clavicle fracture. They found that range of motion of the shoulder was maintained but the strength of the shoulder was decreased to 81% for flexion, 82% for maximum abduction, 81% for maximum external rotation, and 85% for maximum internal rotation. Endurance for these movements was also significantly decreased as compared to the contralateral side.

     

    Figure A shows a clinical photo of a patient with a clavicle fracture. Figure B shows a displaced, comminuted clavicle fracture.

     

    Incorrect Answers:

    Answer 1: Operative treatment increases the union rate.

    Answer 3: Operative treatment decreases the rate of symptomatic malunion. Answer 4: Operative treatment increases shoulder abduction strength.

    Answer 5: Time to union is decreased with operative treatment.

     

     

     

    OrthoCash 2020

     

  147. Figures A and B are radiographs of a 20-year old male athlete that sustained a high impact tackle during a football game. What percentage of these injuries will present with an associated vascular injury?

     

     

     

     

     

     

    1. 10%

    2. 20%

    3. 40%

    4. 70%

    5. 90%

     

    Corrent answer: 3

     

    Approximately 40% of low-velocity anterior knee dislocations are associated with popliteal vascular injury.

    Anterior knee dislocations, which are the most common of all directional dislocations, are produced by a hyperextension mechanism. This causes the tibia to translate anterior to the femur and the popliteal vessels to stretch, causing intimal tears.

     

    Wascher et al. reviewed the association of vascular injury with traumatic knee dislocations. They showed that 50% of all knee dislocations spontaneously reduce. However, patients who present with reduced knee dislocations have a similar risk of vascular injury (~ 40%) and other concurrent injuries as those who present with a dislocated knee.

     

    Levy et al. reviewed the timing of treatment of multiligament-injured knee injuries arising from acute knee dislocations. They suggest that early operative treatment of the multiligament-injured knee yields improved functional and clinical outcomes compared with nonoperative management or delayed surgery. They noted that repair of the PLC, either acute or delayed, may yield higher revision rates compared with reconstruction options.

     

    Figures A and B show AP and lateral radiographs of an anterior knee dislocation. A video is provided that gives a brief overview of knee dislocations.

     

     

     

    OrthoCash 2020

     

  148. A 45-year-old female presents to the office wearing a right upper arm splint with radiographs shown in Figure A and B. She sustained an isolated closed injury to the right arm 9 days ago. Her soft-tissues and neurological examination are normal. What would be the most appropriate treatment for this injury?

     

     

     

     

     

     

    1. Continue current splint for 6 weeks

    2. Continue current splint for 3 weeks and transition to hanging arm sling for additional 3 weeks

    3. Transition to functional brace for additional 6-8 weeks

    4. Open reduction internal fixation with compression plating

    5. Staged procedure with humeral external fixator, then open reduction internal fixation with compression plating

    Corrent answer: 3

     

    Figures A and B show radiographs on a minimally displaced humeral shaft fracture. The most appropriate treatment for this injury would be functional bracing (Sarmiento) for an additional 6-8 weeks or until healed.

     

    Commonly accepted parameters for closed treatment include less than 30 degrees of varus angulation, 20 degrees of anterior/posterior angulation, and 3 cm of shortening. Functional bracing has become the gold standard for humeral shaft fractures as it consistently shows excellent healing results as well as preventing the complication of shoulder +/- elbow stiffness associated with joint spanning splints or slings.

     

    Sarmiento et al. treated 922 patients with humeral diaphysis fractures with a prefabricated brace. They found a 97% rate of union with the use of the brace. In addition, only 2% of the patients had lost more than 25 degrees of shoulder motion at the time of brace removal.

     

    Koch et al. reviewed 67 humeral shaft fractures that were treated by Sarmiento bracing in a 15-year period. Fifty-eight cases (87%) had healed clinically at a mean of 10 weeks. Among 9 patients with delayed or nonunion leading to operative intervention, there were 6 cases with transverse fractures

     

    Figures A and B show a moderately displaced right humeral shaft fracture with 13 degrees of AP angulation, 10 degrees of varus/valgus angulation and no shortening, treated in a coaptation splint. A nondisplaced proximal humeral fracture is also seen. Illustration A shows an image taken of a patient wearing the sarmiento brace.

     

    Incorrect Answers:

    Answer 1,2: Joint spanning splints or slings have not shown to be superior to functional bracing. They are associated with joint stiffness post removal.

    Answer 4: Operative indications are: associated vascular injuries, bilateral humeral shaft fractures, polytrauma patient (including paraplegia), injury to the brachial plexus, pathological fractures, floating elbow, and floating shoulder.

    Answer 5: Staged operative procedure would be indicated in open fractures or significant deformity with soft-tissue swelling.

     

     

     

     

     

    OrthoCash 2020

     

  149. Which of the following is an appropriate initial step in the management of a multiply injured patient with an unstable pelvic ring fracture and hemodynamic instability?

    1. Application of an external fixator

    2. Pelvic angiography

    3. Pelvic packing

    4. Application of a pelvic binder

    5. Percutaneous Iliosacral screws Corrent answer: 4

    Patients with multiple injuries including a pelvic ring fracture who present with hemodynamic instability should have a pelvic binder or circumferential pelvic sheet placed as part of their initial resuscitation.

     

    A systematic approach to search for sources of bleeding and control ongoing hemorrhage is necessary for patients who present with hemodynamic changes in the setting of a pelvic ring fracture. Management of continued hypotension after pelvic binder placement is controversial and varies among trauma centers.

     

    Krieg et al. prospectively evaluated 16 patients with unstable pelvic ring injuries initially managed with a novel circumferential compression device. The authors found substantial reduction in pelvic width with the use of this

    compressive device in patients with volume expanding pelvic ring fractures.

     

    Croce et al. retrospectively compared patients with unstable pelvic ring injuries who were treated with either emergent pelvic fixation (EPF) or a pelvic orthotic device (POD). The authors found that those patients treated with POD had decreased transfusion requirements and shorter length of hospital stay.

     

    Routt et al describe their technique for circumferential pelvic antishock sheeting (CPAS). The authors provide an illustrative case and discuss the potential advantages of sheet application versus other techniques of pelvic stabilization.

     

    Illustration A is the initial AP radiograph of a patient with a pelvic fracture and hemodynamic instability. The pelvic binder was placed in the field prior to arrival. Illustration B demonstrates the same patient in the angiography suite after removal of the pelvic binder. Note the increased widening of bilateral SI joints, greater on the left than the right.

     

    Incorrect Answers:

    Answer 1: External fixation of pelvic ring fractures can be used to assist with resuscitation but pelvic binder application should be attempted first

    Answer 2: The use of pelvic angiography is controversial and institution specific however some centers utilize pelvic angiography as part of the algorithm for management of ongoing hemorrhage.

    Answer 3: Pelvic packing is utilized in some centers to control ongoing pelvic hemorrhage however it is not used as initial management of patients with hemodynamic instability

    Answer 5: Percutaneous iliosacral screws can also be utilized as a form of resuscitation however they should not be used as as first line of management

     

     

     

     

     

     

     

     

    OrthoCash 2020

     

  150. A 19-year-old female sustains the injury shown in Figures A thru C as the result of a motor vehicle collision. Which of the following is the most common cause of death with this type of pelvic injury pattern?

     

     

     

     

     

     

     

     

    1. Hypovolemic shock

    2. Spinal injury

    3. Solid organ rupture

    4. Acute respiratory distress syndrome

    5. Closed head injury Corrent answer: 5

    The injury pattern described in the question and images is a lateral compression pelvic ring injury. Of the choices provided, the most common associated cause of death is a closed head injury.

     

    Pelvic ring disruptions are the result of high energy blunt trauma and are associated with other significant injuries in greater than 50% of the cases. These injuries may involve neurovascular structures and other organ systems.

     

    Burgess et al. retrospectively reviewed their pelvic ring injuries and reported their classification system based upon the vector of force involved and the quantification of disruption from that force, i.e., lateral compression, anteroposterior compression, vertical shear, and combined mechanical injury. Overall blood replacement was highest in anterior-posterior patterns. Mortality was also highest in anteroposterior patterns. The most common identifiable cause of death in patients with lateral compression fractures is closed head injury. In contrast, the identifiable cause of death in patients with anteroposterior compression injuries is combined pelvic and visceral injury.

    Watnik et al. reviewed lower urinary tract injuries and noted that they occur in as much as 25% of patients with pelvic ring disruptions. They also report that early repair of bladder injury can facilitate the placement anterior pelvic fixation, in efforts to minimize infection.

     

    Smith et al. reviewed hemodynamically unstable pelvic ring fracture patients and found that there is a positive association of blood replacement requirements and mortality. They also reported that death within the first 24 hours after admission was most often a result of acute blood loss while death after the first day was most often caused by multi-organ failure.

     

    Figure A shows an AP pelvic radiograph with evident anterior pelvic ring fractures. Figure B and C are axial CT cuts showing the posterior and anterior ring fractures, respectively. This fracture pattern is consistent with a lateracl compression mechanism.

     

    Incorrect Answers:

    Answer 1-4: These options are less commonly reported as causes of death than closed head injury in a lateral pelvic ring injury mechanism.

     

     

     

    OrthoCash 2020

     

  151. A 67-year-old male is involved in a motor vehicle accident and presents with the closed orthopedic injuries shown in Figures A and B. He is also noted to have a grade 1 splenic laceration and lung contusion. He is cleared by the trauma team, and undergoes early total care with reamed femoral and tibial nailing. A tourniquet is used for the tibial nailing portion of the case, and the tibial isthmus is over reamed to accept a larger nail. The use of a tourniquet in this case has been most clearly shown to be associated with which of the following?

     

     

     

    1. Tibia shaft necrosis post-operatively

    2. Increased pulmonary morbidity post-operatively

    3. Increased cortical bone temperature during reaming

    4. Increased nonunion rates

    5. Decreased pain post-operatively Corrent answer: 2

    In patients with multitrauma, combining reamed femoral nailing with fracture fixation (ie. tibial shaft) under tourniquet control has been shown to increase pulmonary morbidity.

     

    Limb reperfusion after tourniquet ischemia causes pulmonary microvascular injury. Similarly, microembolization, like that associated with reamed femoral nailing, can induce pulmonary microvascular injury. Both processes result in increased pulmonary capillary membrane permeability and edema, and ultimately increased pulmonary morbidity.

     

    Karunakar et al showed in a canine model that there is no significant difference in the heat generated during reaming with and without a tourniquet. The factor that made the most difference was related to the size of the reamer used compared with the diameter of the isthmus. They concluded that the risk of

    thermal necrosis appears to be related more to the process of intramedullary reaming than to the tourniquet.

     

    Giannoudis and associates performed a prospective randomized trial on 34 patients to measure the rise of temperature during reaming of the tibia before intramedullary nailing with and without the use of a tourniquet. The factor that generated the most heat was using large reamers (11 mm to 12 mm) in a patient with a small isthmus (8 mm to 9 mm). Use of a tourniquet, steroid use, and knee flexion during reaming were not shown to be associated with diaphyseal necrosis after reamed tibial nailing.

     

    Pollak et al evaluated the association between femoral nailing followed by tourniquet ischemia and clinical lung injury. They reviewed 72 patients with femoral shaft fractures and tibial or ankle fractures requiring internal fixation over a six year period. All femoral shaft fractures were treated with reamed intramedullary nails, and the patients were divided into groups, based on whether the tibial or ankle injury was managed surgically with or without a tourniquet. They noted increased pulmonary morbidity in the group where a tourniquet was used.

     

    Figure A shows a femoral shaft fracture at the junction of the middle and proximal one-third of the femoral shaft. Figure B shows a contralateral tibial shaft fracture.

     

    Incorrect Answers:

    1-Tourniquet use has not been shown to lead to thermal necrosis of the bone during reaming of the tibial shaft.

    1. The cortical temperature does not increase to a greater degree with tourniquet use versus non-tourniquet use during reaming.

    2. There is no evidence that use of a tourniquet affects tibial shaft union rates after reaming and intramedullary nailing.

    3. Tourniquet use has been associated with increased pain post-operatively.

     

     

     

     

    OrthoCash 2020

     

  152. A 24-year-old male sustains the isolated injuries shown in Figures A and B during a high-speed motor vehicle accident. On physical examination, the overlying skin is intact and there is no evidence of a Morel-Lavallée lesion. Which of the following surgical techniques is considered to have the highest rate of fracture malreduction with this combined injury?

     

     

     

     

     

     

    1. Antegrade cephalomedullary nail

    2. Retrograde intramedullary nail and 3 cannulated screws

    3. Retrograde intramedullary nail and sliding hip screw

    4. Antegrade intramedullary nail and 3 cannulated screws

    5. Plate fixation of the diaphyseal fracture and 3 cancellous screws Corrent answer: 1

    Figures A and B show displaced ipsilateral femoral neck and shaft fractures. Fixation with a single implant, such as an antegrade cephalomedullary device, has been shown to have the highest rate of fracture malreduction with displaced fractures.

     

    Approximately 5% of femoral shaft fractures are accompanied by ipsilateral neck fractures. Fixation methods to stabilize these fractures may be accomplished by using a single implant or two separate implants. Single implant techniques are thought to reduce operative time and blood loss by simultaneously fixing nondisplaced fractures. With displaced fractures, single implant techniques have been shown to have a higher rate of malreduction of

    at least one of the two fractures. Two separate implant devices are recommended in these scenarios.

     

    Bedi et al. examined a retrospective cohort of 40 patients with ipsilateral femoral neck and shaft fractures. They showed that using a single cephalomedullary device for fixation of both femoral shaft and neck fractures led to a significantly higher rate of fracture malreduction (3 of 9), in comparison to a staged, two implant strategy (0 of 28) (P = 0.01).

     

    Peljovich et al. reviewed the presentation and management of ipsilateral femoral neck and shaft fractures. To reduce complications of AVN, malunion and nonunion, they suggest obtaining anatomic reduction and rigid fixation of the femoral neck fracture first with 3 cannulated screws, blade plate, or sliding hip screw. The shaft fracture can then be reduced and stabilized with either retrograde intramedullary femoral nailing or plating.

     

    Incorrect Answers:

    Answers 2,3,4,5: Fixation of femoral neck and shaft fractures using two separate implant devices has a lower rate of malreduction. No study to date has conclusively demonstrated superiority of any particular combination of devices in long-term studies.

     

     

     

    OrthoCash 2020

     

  153. A 35-year-old patient sustains an upper extremity injury after a motor vehicle collision. Radiographs are shown in Figures A and B. Which treatment modality will optimize internal stability of the elbow?

     

     

     

     

    1. Open reduction and internal fixation with k-wires

    2. Open reduction and internal fixation with tension band wiring

    3. Open reduction and internal fixation with plate fixation

    4. Open reduction and internal fixation with an intramedullary screw

    5. Cast immobilization

     

    Corrent answer: 3

     

    Based on the radiographs shown, the patient has a comminuted trans-olecranon fracture dislocation of the elbow. Stability will be optimized with internal fixation in the form of a plating construct.

     

    Trans-olecranon fracture dislocations often occur in the young secondary to high energy mechanisms. Direct blows often lead to high levels of comminution; the distal humerus is driven into the proximal ulna and olecranon, thereby leading to a concomitant dislocation. In this setting, the optimal treatment is use of a plate and screw fixation construct to provide mechanical stability.

     

    Veillette et al. review the epidemiology, pathophysiology and treatment options for olecranon fractures. They highlight that because olecranon injuries involve the articular surface, fixation constructs should allow anatomic restoration of the joint surface. Additionally, fixation constructs should allow for early mobilization in order to prevent stiffness of the elbow.

     

    Mortazavi et al. retrospectively reviewed eight patients who sustained anterior trans-olecranon fracture dislocations of the elbow. Based on their experience, they recommend use of plate fixation to optimize stability of the elbow.

    Additionally, they indicate that satisfactory results can be obtained if the greater sigmoid notch is appropriately reduced.

     

    Figures A and B show the AP and lateral radiographs of a trans-olecranon fracture dislocation of the elbow. Severe comminution of the olecranon can be seen on the lateral radiograph in Figure B.

     

    Incorrect Answers:

    Answers 1, 2, 4: K-wires alone would not provide optimal stability and allow for early motion. A tension-band construct or IM screw fixation is contraindicated with this degree of comminution.

    Answer 5: Use of cast immobilization would lead to high levels of stiffness and is not indicated for this patient.

     

     

     

    OrthoCash 2020

  154. A 25-year-old male sustained an isolated injury to his right foot after a fall from height. On examination, he has moderate swelling and pain over the dorsum of the foot. The overlying skin is intact. Radiographs of the foot are seen in Figures A and B. A CT scan image is seen Figures C. When consenting the patient for open reduction and internal fixation of this injury, what would you document as the most common complication?

     

     

     

     

     

     

     

    1. Symptomatic implants

    2. Subtalar arthritis

    3. Tibiotalar arthritis

    4. Malunion

    5. Wound dehiscence

     

    Corrent answer: 2

     

    This patient has sustained a displaced talar neck fracture. The most common complication associated with this injury is post-traumatic arthritis of the subtalar and tibiotalar joint.

     

    Talar neck fractures are the most common fracture of the talus. They occur most frequently during forced dorsiflexion of the foot with axial load. Displaced fractures are best treated with anatomic reduction and stable internal fixation.

    Complications of surgery include nonunion or malunion, post-traumatic arthritis, infection, osteonecrosis, and secondary surgery. The prevalence of post-traumatic subtalar arthritis has been shown to be approximately 50-100% with these injuries.

     

    Lindvall et al. examined a retrospective cohort of twenty-six displaced talar neck fractures that had been treated with open reduction and stable internal fixation. They reported that 26/26 fractures developed post-traumatic arthritis and pain within 48 months of follow-up after fixation. They concluded that post-traumatic arthritis is a more common complication than osteonecrosis following operative treatment.

     

    Vallier et al retrospectively reviewed the records of 39 fractures of the talar neck treated with open reduction and internal fixation. Twenty-one (54%) of thirty-nine patients had development of posttraumatic arthritis, which was more common after comminuted fractures (p < 0.07) and open fractures (p = 0.09).

     

    Figures A and B show plain radiographs of a Hawkins II talar neck fracture. There is dislocation of the subtalar joint. The tibiotalar and talonavicular joints appear congruent. Figures C is a CT scan imaging showing some comminution at the fracture site.

     

    Incorrect Answers:

    Answer 1: Symptomatic implants ~ 18%

    Answer 3: Tibiotalar arthritis ~ 33%

    Answer 4: Malunion ~ 25 %

    Answer 5: Wound dehiscence ~ 3 %

     

     

     

     

    OrthoCash 2020

     

  155. A 23-year-old man presents with the injury seen in Figure A after a motor vehicle collision. He undergoes the treatment seen in Figure B. Which of the following statements is most accurate when comparing his treatment with open reduction and internal fixation?

     

     

     

     

     

     

    1. Higher rates of radial nerve injury

    2. Higher total complication rate

    3. Lower rates of nonunion

    4. Lower rates of shoulder impingement

    5. Lower rates of malunion Corrent answer: 2

    Antegrade intramedullary (IM) nailing of humeral shaft fractures has been found to be associated with increased complication rates when compared with open reduction and internal fixation (ORIF).

     

    Operative treatment of humeral shaft fractures remains controversial, with prospective randomized studies demonstrating small differences between IM nailing and ORIF.

     

    Heineman et al. (2010) conducted a meta-analysis of prospective randomized studies comparing IM nailing with ORIF for humeral shaft fractures. The authors found no significant difference between the two treatment modalities for either their primary outcome (complications) or any of the secondary outcomes (nonunion, infection, nerve palsy, re-operation)

     

    Heineman et al. (2012) have recently conducted an update on their meta-analysis to include more recent randomized studies. With the inclusion of these newer studies the author found a statistically significant increase in total complication rate with the use of IM nailing compared with ORIF.

     

    Figure A demonstrates a displaced, transverse humeral shaft fracture. Figure B demonstrates antegrade IM nailing of a humeral shaft fracture

     

    Incorrect Answers:

    Answer 1: Radial nerve injury has not been shown to be different between IM nailing and ORIF

    Answer 3: No difference in union rates between the two modalities in prospective studies

    Answer 4: Higher rates of shoulder impingement have been seen with IM nailing in some studies

    Answer 5: No difference in rates of malunion between IM nailing and ORIF

     

     

     

    OrthoCash 2020

     

  156. Elevated interleukin 6 (IL-6) is most closely associated to which of the following clinical outcomes in orthopedic trauma patients?

    1. Decreased mortality rates

    2. Increased mortality rates

    3. Decreased osteomyelitis infection rates

    4. Increased rhabdomyolysis rates

    5. Increased compartmental syndrome rates Corrent answer: 2

    Elevated levels of Interleukin 6 (IL-6) is most closely associated with higher injury severity scores and increased mortality rates in polytrauma orthopaedic patients.

     

    Hyperstimulation of the inflammatory system by major trauma is considered to be the key element in the pathogenesis of severe inflammatory response syndrome and multi-organ dysfunction syndrome. IL-6 is a complex acute-reactant cytokinase that is expressed by cells in response to tissue injury. IL-6 levels are associated with injury severity, complications, and mortality.

    Patients with the most severe injuries have the highest IL-6 serum levels.

     

    Sears et al. reviewed the markers of inflammation in major trauma. They suggest that interleukin-6 and human leukocyte antigen-DR class II molecules appear to have the greatest potential for use in predicting the clinical course and outcome in trauma patients. Early identification of traumatic patients, based on inflammatory markers and genomic predisposition, could help to guide intervention and treatment.

     

    Pape et al measured the perioperative concentrations of interleukin-6 in sixty-eight blunt trauma patients with non-life threatening pelvic fractures. Release of proinflammatory cytokines were higher in patients undergoing surgical procedures that cause increased blood loss. The release of markers seems to be related to the type and magnitude of surgery, rather than to the duration of the procedure.

     

    Illustration A shows a diagram of the acute inflammatory response after major trauma

     

    Incorrect Answers:

    Answer 1: Mortality rates have shown to be increased with elevated levels of IL-6 in early trauma.

    Answer 3: Osteomyelitis is a complication of open fracture trauma. Elevated levels of IL-6 and CRP are seen with this infection.

    Answer 4: IL-6 is not commonly measured in rhabdomyolysis. Creatine kinase is commonly elevated with musculoskeletal injury.

    Answer 5: Elevated IL-6 levels are not closely correlated with incidence incidence of compartmental syndrome.

     

     

     

     

     

    OrthoCash 2020

     

  157. A 26-year-old female sustained a closed femoral shaft fracture which was treated as shown in Figure A. She subsequently developed a painful oligotrophic nonunion and an exchange nailing was performed 6 months later. However, she has had persistent pain in the thigh since time of revision surgery seven months ago, but denies any fevers, chills, or night sweats. Current examination reveals a mildly tender right thigh with no obvious deformity. Laboratory studies for infectious markers are normal, but her radiograph in Figure B continues to show an oligotrophic nonunion. Which of the following factors has most likely contributed to her current presentation?

     

     

     

    1. One pack of cigarettes per day for last 8 years

    2. Positive intraoperative cultures for coagulase-negative Staphylococcus at the time of nail exchange

    3. Iliac crest bone grafting at the time of nail exchange

    4. Exchange nail was dynamically interlocked

    5. A larger diameter nail was used in the exchange Corrent answer: 1

    The clinical presentation and radiographs are consisted with persistent oligotrophic non-union of the femur, after reamed nail exchange. Of the following factors, cigarette smoking has been shown to have the biggest effect on non-union in this population.

     

    Reamed intramedullary nail exchange has become widely accepted as the treatment of choice for most diaphyseal femoral nonunions. The use of reamed exchange nailing are thought to allow for more biomechanical stability via larger diameter nails and improved biologics with bone reaming. These will help to generate bone healing and bridge fracture gaps. Smoking has been shown clinically and experimentally to affect union rate and fracture callus strength. Patients with femoral shaft non-unions should be forewarned of the its effect on fracture healing in nail exchange.

     

    Hak et al retrospectively reviewed a cohort of 23 patients treated with reamed nail exchange for symptomatic femoral shaft non-unions. 15 of the patients were smokers and only 10 (66.7 percent) went on to heal after reamed nail exchange. This was in comparison to all 8 non-smoking patients who healed without complication after exchange reamed nailing. They suggest that smokers should be referred to an appropriate smoking cessation program before revision nail exchange is considered.

    Figure A shows a proximal third femoral shaft fracture that was treated with intramedullary nailing. The oblique fracture line remains evident with a small callus response at the fracture site at 8 months post-operative. Figure B shows an radiograph of the exchange femoral nail 7 months after revision sugary.

    There is increased fracture callus at fracture site, however the fracture line remains evident.

     

    Incorrect Answers:

    Answer 2: Intraoperative coagulase-negative staph. cultures are likely contaminant. The patient denies infectious symptoms and inflammatory markers for infection are negative. In addition, the radiographs do not suggest infection as there is no cortex irregularity, bone lysis, or implant loosening.

    Answer 3: Bone grafting at the time of nail exchange has shown to increase bone union.

    Answer 4: Dynamic interlocked screws were shown to increase bone union greater than static interlocked screws at the time nail exchange.

    Answer 5: Bone grafting at the time of nail exchange has shown to increase bone union.

     

     

     

    OrthoCash 2020

     

  158. The sacrospinous and sacrotuberous ligaments are disrupted in which of the following injury patterns?

    1. Ischial tuberosity avulsion fracture

    2. Type I anterior-posterior compression pelvic ring injury

    3. Type II lateral compression pelvic ring injury

    4. Type II anterior-posterior compression pelvic ring injury

    5. Both column acetabular fracture Corrent answer: 4

    Type II anterior-posterior compression (APC) pelvic ring injuries have disruption of the symphysis pubis, as well as disruption of the anterior SI ligaments, sacrotuberous ligament, and sacrospinous ligament. The alternative pattern of disruption of the pelvic floor ligaments is sometimes seen as an avulsion injury from the bony attachments of these structures (sacrum or ischium).

     

    The progression of this pattern to involve the posterior SI joint ligaments creates a type III pattern, which is vertically and rotationally unstable.

     

    Tile published a comprehensive review of pelvic ring injuries, focusing on the

    anatomy and pathology of these injuries. He noted that the posterior SI complex is the most important to pelvic ring stability, which is the reason why these are generally classified by the grade of posterior injury. He also reminds the reader that these classification systems are not a substitute for individualized treatment decision making.

     

    Illustration A shows an APC III pelvic ring injury, with significant symphyseal diastasis and SI joint widening. Illustration B shows the ligamentous anatomy of the pelvic ring.

     

    Incorrect Answers:

    Answer 1: These injuries are generally seen in sports-related trauma, with avulsion of the hamstring(s) from the ischium.

    Answer 2: This injury pattern does not have any involvement of these two ligaments by definition.

    Answer 3: This injury pattern involves inward rotation of the hemipelvis, thus protecting these structures. A type III LC injury may have a contralateral rupture of these ligaments, however.

    Answer 5: This injury pattern typically does not involve rupture of these ligaments unless an ipsilateral pelvic ring injury is seen.

     

     

     

     

     

     

     

     

    OrthoCash 2020

     

  159. A 34-year-old man is involved in a motorcycle accident and sustains a closed tibia fracture and multiple rib fractures. A radiograph is provided in Figure A. During surgical treatment of this fracture, which of the following techniques will help facilitate a successful reduction and intramedullary fixation?

     

     

     

    1. Hyperflexion to help prevent apex anterior angulation

    2. A medial parapatellar incision to help prevent valgus angulation

    3. Starting point just lateral to the medial tibial eminence to help prevent valgus angulation

    4. A medially placed blocking screw to help prevent valgus angulation

    5. Suprapatellar nailing technique to help prevent apex anterior angulation Corrent answer: 5

    The clinical scenario and radiograph demonstrate a proximal tibia fracture amenable to intramedullary nail fixation. The suprapatellar nailing technique helps prevent apex anterior or procurvatum deformity typical of these injuries.

     

    Proximal third tibial shaft fractures are often difficult to reduce anatomically due to the tendency for both valgus and flexion deformity at the fracture site. Many different techniques have been devised to overcome the deforming forces. These include (1) Poller blocking screws posterior and lateral to the intramedullary nail (IMN), (2) utilizing a semi-extended knee position during IMN of proximal tibia fractures, (3) use of a suprapatellar approach for IMN,

    1. usage of a slightly more lateral starting point during conventional IMN, and

    2. application of unicortical plate.

     

    Lang et al. review surgical management of proximal tibia fractures. The typical

    deformity is flexion and valgus. An appropriate starting point just medial to the lateral tibial eminence will help facilitate the appropriate position of the intramedullary nail.

     

    Hiesterman et al. review the management of extraarticular proximal tibia fractures. They elaborate on the anatomic considerations of the classic procurvatum and valgus deformity. The procurvatum deformity results from the extensor mechanism inserting on the proximal fragment, which can be neutralized in the semi-extended suprapatellar nailing technique. The valgus deformity comes from a combination of the intramedullary anatomy of the tibia and the insertion of the pes anserinus muscles on the proximal fragment. They emphasize that attaining and maintaining a proper reduction and utilizing an appropriate starting point is critical for the treatment of these fractures.

     

    Figure A demonstrates an extra-articular proximal tibia fracture. Illustration A demonstrates the proper start for a proximal tibia fracture which should be just medial to the lateral tibial eminence on a perfect AP view and just anterior to the articular margin sagittally (as borrow from Hiesterman et al.).

     

    Incorrect Answers:

    Answer 1: Hyperflexion of the knee can accentuate an apex anterior deformity if measures are not taken to prevent it such as posterior blocking screw placement or a unicortical plate.

    Answer 2: The medial parapatellar approach is generally not used for tibial nailing, although a lateral parapatellar approach has been advocated by some to help obtain a more lateral start point.

    Answer 3: The proper nail starting point is just medial to the lateral tibial eminence.

    Answer 4: A laterally placed blocking screw will help prevent valgus angulation when placed in the proximal segment.

     

     

     

     

     

     

    OrthoCash 2020

  160. A 21-year-old male is brought to the emergency department with multiple gun shot wounds. Initial radiographic evaluation discovers a femoral shaft fracture, distal tibia fracture, and the injury shown in Figure A. Figure B shows a single entry wound located at the left distal humerus. Systemic injuries include multiple abdominal bullet wounds with associated intra-abdominal free fluid. Using the 'damage-control' approach to orthopaedic trauma, what would be the best initial management for the injury seen in Figure A?

     

     

     

     

     

    1. Closed reduction and splinting

    2. Irrigation and debridement, then splinting

    3. Irrigation and debridement, then spanning external fixation

    4. Open reduction and internal fixation with a compression plate

    5. Irrigation and debridement, then intramedullary nailing of the humerus Corrent answer: 1

    Figure A shows a distal third humeral fracture caused by a low-velocity bullet. According to concepts of 'damage control' orthopaedics, this fracture would be best managed initially with closed reduction and splinting.

     

    Low-velocity gun shot wounds are not usually considered open fractures. Therefore, according to the principles of damage control orthopaedics (DCO), GSW and closed humerus fractures can be treated initially with closed reduction and splinting. The concept of DCO is to limit further systemic inflammatory insult after major trauma. The extent of injury is closely tied to elevated levels of serum inflammatory markers (e.g. IL-6 and IL-8) as well as lactate and acid/base deficits. The "second-hit" hypothesis of DCO is based on the cumulative effect of the systemic inflammatory response on the body, which is increased by the additional release of inflammatory markers with early surgery.

     

    Roberts et al. reviewed the concepts of damage control orthopaedics. They suggest that the goal of DCO is to avoid provoking a severe inflammatory response through the stabilization and control of injury rather than repair. To further quantify the impact of the 'second-hit' hypothesis, the authors assessed the role of femoral nailing in early treatment. Specifically, the inflammatory response induced by reamed femoral nailing was shown to increase IL-6 levels within the venous system.

     

    Hildebrand et al. discussed the treatment of extremity injury in damage control orthopaedics. They suggest that most upper extremity injuries can be stabilized early with closed reduction and simple splints or slings. For closed below knee fractures with stable soft tissues, splinting is the single best option.

     

    Figure A shows a distal third humeral fracture caused by a low-velocity bullet. Figure B shows a small entry wound from a low-velocity bullet. Illustration A, in contrast, shows a humerus fracture caused by a shot-gun injury. Shot-gun injuries are traditionally treated as open fractures, which require irrigation and debridement and spanning external fixation according to the DCO approach.

     

    Incorrect Answers:

    Answer 2: Low velocity gun shot wounds are not usually considered open

    fractures and do not require formal irrigation and debridement.

    Answer 3: Humerus fractures stabilized with a spanning external fixator may be considered with open humeral fractures, vascular injury or a floating elbow. Answer 4,5: Invasive procedures to fix the humerus acutely will increase the release of systemic inflammatory markers. For most upper extremity injuries, simple external stabilization with a splint or a sling is recommended initially.

    Definitive fixation would be considered at a later date if there is an associated lower extremity fracture, high level athlete, significant displacement, nonunion or patient preference. The timing of surgery would be based on skin and soft tissue status, and their systemic response after injury.

     

     

     

     

     

     

    OrthoCash 2020

     

  161. A 25-year-old is involved in a motor vehicle accident and sustains an isolated upper extremity injury. A representative radiograph is shown in Figure A. What physical exam findings are most predictive of functional outcomes?

     

     

     

    1. Presence of open fractures

    2. Asymmetry of pulses

    3. Ecchymosis of the upper shoulder girdle

    4. Swelling of the shoulder

    5. Neurologic compromise of the extremity Corrent answer: 5

    Based on the radiograph and mechanism of injury, this patient has sustained a scapulothoracic dissociation. In the context of this injury, neurologic status of the extremity is the exam finding most predictive of functional outcome.

     

    Scapulothoracic dissociation is a traumatic injury to the scapulothoracic articulation that results from lateral traction to the shoulder girdle. It is often associated with significant chest wall, heart and lung trauma. A hallmark physical exam finding is decreased or absent pulses in the affected extremity. Because of the significant traction placed on the brachial plexus, functional outcome is dependent on the extent of neurologic injury. In patients with a complete brachial plexopathy, forequarter amputation should be considered.

     

    Clements et al. review the diagnosis and management of scapulothoracic dissociation. They indicate that this entity is defined by subclavian or axillary vascular disruption, lateralization of the scapula, separation of the clavicular articulations, and nerve root or brachial plexus injury. For patients with flail extremities, an above elbow amputation is recommended

     

    Figure A shows an AP radiograph of a patient with scapulothoracic dissociation. Note the significant lateral displacement of the scapula and scapular body comminution

    Illustration A shows a coronal 3D reconstruction of a CT scan of the chest. Note the significant lateral displacement of the scapula relative to the chest wall.

    There is greater than 1 cm displacement relative to the vertebral spinous processes. A video is provided that reviews diagnosis and management of scapulothoracic dissociation.

     

    Incorrect Answers

    Answers 1, 2, 3, 4: While these may all be seen in the setting of this injury, neurological status of the extremity is the most predictive of functional outcomes.

     

     

     

     

     

     

    OrthoCash 2020

     

  162. Which of the following figures depicts a lateral compression mechanism of injury?

     

     

     

     

     

     

     

     

     

     

     

    1. Figure A

    2. Figure B

    3. Figure C

    4. Figure D

    5. Figure E

     

    Corrent answer: 2

     

    Figure B depicts a left lateral compression mechanism of injury or more specifically, an LC-II injury in the Young-Burgess classification system of pelvic fractures. Figure B depicts superior and inferior pubic rami fractures and an ipsilateral "crescent" fracture. The "crescent" injury involves a combination of ligamentous injury at inferior portion of SI joint, and vertical fracture of the posterior ilium which extends thru iliac crest.

     

    Proper diagnosis of pelvic ring fractures including LC-type fractures is made via radiographs including AP, inlet, and outlet views of the pelvis. CT of the pelvis can be an adjunct to the diagnosis of pelvic ring fractures and is commonly

    obtained in the general trauma workup, but is not necessary to make the diagnosis.

     

    Tile et al. describes his classification system of pelvis ring fractures focusing on the importance of the posterior sacroiliac ligaments for stabilization. This classification system is A-C with a Tile C being a fracture with complete disruption of the posterior sacroiliac ligaments and thus the most instability.

     

    Lefaivre et al. retrospectively review CT scans of LC-1 pelvic injuries to more specifically categorize the CT findings of that fracture pattern. A large portion were found to have complete disruption of the sacrum.

     

    Koo et al. measured the interobserver reliability of both the Tile and Young-Burgess classification systems. They found that CT scans of the pelvis increased the reliability of the assessment of pelvic stability.

     

    Illustration A is pelvic radiograph that is labelled to help distinguish the crescent fracture from the SI joint.

     

    Incorrect Answers:

    Answer 1: Figure A shows an APC-II (anterior-posterior compression) injury. Answer 3: Figure C shows a vertical shear injury.

    Answer 4: Figure D is a CT scan of an APC-III injury.

    Answer 5: Figure E is a CT scan of an anterior wall acetabular fracture.

     

     

     

     

     

     

    OrthoCash 2020

  163. A 35-year-old female presents with the orthopaedic injuries shown in Figures A-D following a high-speed motor vehicle collision. She is also found to have a right-sided diaphragmatic hernia (Figure E) and a stable subarachnoid hemorrhage. The femoral and tibial plateau fractures are open with no gross contamination, and there is an ipsilateral Morel-Lavelle lesion of the left thigh. What is the most appropriate initial management of the patient’s injuries in addition to debridement and irrigation of the open injuries?

     

     

     

     

     

     

     

     

     

     

     

    1. Application of a knee immobilizer, splinting of the ankle and forearm

    2. External fixation of the femur and tibial plateau, splinting of the ankle and forearm

    3. Retrograde intramedullary nailing of the femur, limited internal fixation of the tibial plateau, splinting of the ankle and forearm

    4. External fixation of the femur, ORIF of the tibial plateau, splinting of the ankle and forearm

    5. Retrograde intramedullary nailing of the femur, ORIF of the tibial plateau, ORIF of the ankle and forearm

    Corrent answer: 2

     

    In the setting of a poly-trauma patient with an open high-energy tibial plateau fracture, the best initial treatment is irrigation and debridement with external fixation of the femur and tibial plateau along with splinting of the ankle and forearm.

     

    Early total care of high-energy peri-articular and intra-articular proximal tibia fractures is wrought with an unacceptably high risk of wound complications. Acute internal fixation through compromised soft tissues, whether due to severe swelling or contaminated open wounds, substantially increases the risk of delayed wound healing, dehiscence, and infection. Staged fixation utilizing immediate spanning external fixation is recommended in the setting of these injuries and in a poly-trauma patient, as it often offers at least a provisional reduction, serves to restore the length and alignment of the proximal tibia, and thereby promotes soft tissue healing. The external fixator further enables resuscitation of the patient, access to the soft tissues for wound care, and facilitates pre-operative planning for definitive fixation.

     

    Parekh et al. retrospectively evaluated perioperative complications following staged internal fixation of high-energy distal femoral and proximal tibia fractures. At final follow-up, there were 8 (16%) deep infections, all in open fractures, with one that required below-knee amputation. The authors concluded that temporary bridging external fixation offers a decreased risk for wound complications over immediate internal fixation and is therefore suitable for patients who are unfit for initial definitive fixation.

     

    Berkson et al. reviewed temporary external fixation of complex high-energy tibial plateau fractures. The authors report that knee-spanning external fixation serves to restore tibial length, obtain provisional reduction via ligamentotaxis, and maintain alignment, thereby supporting more expedient soft tissue recovery and mitigating wound complications. They concluded that, in the setting of open fractures, acute debridement and irrigation, application of

    knee-spanning external fixation, and serial debridement is indicated to minimize the risk of infection.

     

    Haidukewych et al. reviewed the indications, techniques, results, and complications regarding the management of complex intra-articular and periarticular tibial plateau fractures. They highlight the benefits of and recommend

    staged treatment with initial external fixation and delayed definitive fixation to minimize wound complications in high-energy tibial plateau fractures with a compromised soft-tissue envelope.

     

    Lichte et al. present a current review of the evidence supporting damage control orthopedics (DCO). The authors conclude that DCO is more appropriate in the critically ill polytrauma patient to minimize the risk of a “second hit” phenomenon with the plan for conversion to definitive fixation within the 7-14 day window.

     

    Figures A and B show a displaced distal femoral shaft fracture with an ipsilateral bicondylar tibial plateau fracture. Figure C shows an ipsilateral bimalleolar ankle fracture-dislocation. Figure D shows an ipsilateral displaced distal radius and distal ulna fracture. Figure E shows a right-sided diaphragmatic hernia.

     

    Incorrect Answers:

    Answer 1: A knee immobilizer would not provide sufficient stability for such a significantly length-unstable injury.

    Answer 3: While there are advocates of limited internal fixation in the acute setting, acute retrograde nailing of the femur is not recommended in a polytraumatized patient with a closed head injury and major abdominal trauma.

    Answers 4 and 5: Acute open reduction and internal fixation of a high-energy open tibial plateau fracture is not recommended as it is associated with a substantially increased risk of wound complications.

     

     

     

    OrthoCash 2020

     

  164. A 34-year-old male presents with a closed left leg injury after falling off a 20ft ladder. His injury radiographs are shown in Figure A. He is taken for intramedullary nail (IMN) fixation. Which of the following correctly combines techniques used to decrease the incidence of the most common deformities associated with this fracture pattern?

     

     

     

     

     

     

     

     

     

     

     

     

     

    1. Starting point in Figure B with blocking screw in Figure D

    2. Starting point in Figure B with blocking screw in Figure E

    3. Starting point in Figure C with blocking screw in Figure D

    4. Starting point in Figure C with blocking screw in Figure E

    5. Starting point in Figure C with blocking screw in Figure F Corrent answer: 4

    The patient has sustained a proximal third tibial shaft fracture, which has a tendency to fall into valgus and procurvatum during IMN fixation. A more lateralized starting point (Figure C) and use of a posterior blocking screw (Figure E) are techniques which have been shown to minimize these deformities, respectively.

     

    Proximal third tibial shaft fractures are subject to the deforming forces exerted by the extensor mechanism and pes anserinus, commonly resulting in apex anterior (procurvatum) and valgus angulation of the fracture, respectively.

    During IMN fixation, these deformities may be mitigated by techniques including use of the semi-extended approach, lateralization of the starting point, and the application of blocking screws placed in the posterior half of tibia within the coronal plane and in the lateral concave side of the proximal fragment within the sagittal plane. Unicortical plate fixation may also be used

    to maintain fracture alignment.

     

    Ricci et al. describes the technique and outcomes for use of blocking screws with tibial IMN fixation in proximal third tibial shaft fractures. The authors found less than 5 degrees of angular deformity in all patients in the planes in which blocking screws had been used to control alignment. Ten of the twelve patients went on to uneventful union, and all patients had maintained their alignment at final follow-up. The authors concluded that blocking screws are effective to help obtain and maintain alignment of proximal third tibial shaft fractures treated with IMN nail fixation.

     

    Krettek et al. evaluated the mechanical effects of blocking screws in supplementing IMN fixation of both proximal and distal third tibial shaft fractures. The authors noted that with the addition of blocking screws, the deformation of the bone construct segment decreased by 25% for proximal fractures and by 57% for distal fractures. The authors concluded that blocking screws can increase the primary stability of proximal and distal metaphyseal tibial fractures after IMN fixation.

     

    Figure A shows the AP and lateral radiographs of a proximal third tibial shaft fracture with characteristic procurvatum and valgus alignment.

    Figure B is an AP radiograph of the knee showing the standard entry IMN entry point, at the medial aspect of the lateral tibial spine.

    Figure C is an AP radiograph showing a more lateralized starting point. Figure D is a lateral radiograph demonstrating an incorrect location of the blocking screw within the anterior half of the proximal segment.

    Figure E is a lateral radiograph demonstrating an appropriately posterior location of the blocking screw within the proximal segment.

    Figure F is an AP radiograph demonstrating an incorrect location of the blocking screw within the medial (instead of lateral) aspect of the proximal tibia in the sagittal plane.

     

    Incorrect Answers:

    Answer 1: Figure B is the standard starting point of a tibia IMN. For a proximal third tibial fracture, lateralizing the starting point would be recommended to mitigate valgus deformity. Figure D shows the blocking screw incorrectly placed in the anterior half of the tibia, which would drive the IMN posterior and leave the fracture susceptible to procurvatum deformity.

    Answer 2: Figure B is the standard starting point of a tibia IMN. For a proximal third tibial fracture, lateralizing the starting point would be recommended to mitigate valgus deformity. Figure E is however the correct location for placement of a blocking screw in the coronal plane.

    Answer 3: Figure C is the appropriate lateralized starting point to reduce the

    chance of valgus deformity for a proximal third tibial fracture. Figure D shows the blocking screw incorrectly placed in the anterior half of the tibia, which would drive the IMN posterior and leave the fracture susceptible to procurvatum deformity.

    Answer 5: Figure C is the appropriate lateralized starting point to reduce the chance of valgus deformity for a proximal third tibial fracture. Figure D shows the blocking screw incorrectly placed in the medial aspect of the proximal fragment, which would drive the IMN lateral and leave the fracture susceptible to valgus deformity.

     

     

     

    OrthoCash 2020

     

  165. A 25-year-old patient is involved in a motor vehicle accident. An isolated orthopaedic injury is sustained to the upper extremity with no compromise of skin integrity or neurovascular function. A radiograph of the injury is shown in Figure A. The patient is interested in pursuing surgical intervention. What is a reported outcome of surgery when compared to nonoperative management at 1 year postoperatively?

 

 

 

 

  1. Increased rates of symptomatic nonunion

  2. Similar rates of symptomatic nonunion

  3. No differences in cosmetic results

  4. Increased functional outcome scores

  5. Improved range of motion of the shoulder Corrent answer: 4

Surgical fixation of this significantly displaced clavicle fracture (greater than 100%) has been associated with greater functional outcomes up to 1 year after injury compared to non-operative measures

 

Clavicle fractures are often seen in young active patients as the result of falls or direct compressive trauma. Non-operative management of midshaft clavicle

Figure B

Figure C

Figure D

Figure E

 

Correct answer: 3

 

There are 4 main types of plating techniques: 1. Bridging 2. Neutralization 3. Dynamic Compression 4. Buttress plating. Plates can utilize locking or non-locking screws.

 

Buttress plating is appropriate for a Shatzker Type I (see illustration C), as it can prevent collapse and axial deformity from shear or bending forces.

 

Karunakar et al showed that there was no significant difference between split depression tibial plateau fractures (Shatzker II) fixed with either a buttress plate with rafting screws versus a periarticular plate with built in rafting screw hole options, similar to the commonly used pre-contoured periarticular locking plate.

 

 

 

 

Percutaneous placement of a lateral proximal tibial locking plate that extends down to the distal third of the leg is associated with postoperative decreased sensation of which of the following distributions?

Medial hindfoot

Lateral hindfoot

First dorsal webspace

Dorsal midfoot

Plantar foot

 

Correct answer: 4

 

Placement of long lateral tibial plates have been shown to have a risk of iatrogenic injury to the superficial peroneal nerve, which has a sensory distribution to the dorsal foot. This risk is seen especially with percutaneous approaches, such as those used with the LISS plate.

 

The first reference by Deangelis et al found a risk of superficial peroneal injury with percutaneous screw placement of holes 11-13 in the LISS plate.

 

The second referenced article by Roberts et al noted a slightly increased distance to the neurovascular bundle when interlocking tibial nails in a lateral to medial direction (compared to medial to lateral locking) and slightly increased biomechanical strength when locking in a medial to lateral direction.

 

The third referenced article by Wolinsky et al notes a risk of iatrogenic injury to the deep peroneal nerve and anterior tibial artery with an anterolateral approach to the distal tibia, but notes the superficial peroneal nerve is safe with an appropriate exposure.

 

 

 

 

A 64-year-old female sustains a nondisplaced distal radius fracture and undergoes closed treatment using a cast. Three months after the fracture she reports an acute loss of her ability to extend her thumb. What is the most likely etiology of her new loss of function?

Posterior interosseous nerve entrapment

Extensor pollicis longus rupture

Extensor pollicis longus entrapment

Distal radius malunion

Intersection syndrome

 

Correct answer: 2

 

According to the referenced article by Jupiter and Fernandez, the most common scenario of extensor pollicis longus rupture after a distal radius fracture is when the fracture is non or minimally displaced. The hypothesis is that the rupture happens at an area of relative hypovascularity and healing callus can aggravate this area, leading to a degenerative tear.

Hove et al reported an incidence of delayed tendon rupture after distal radius fracture of 0.3 percent. In their series of 18 extensor pollicis longus tendon ruptures, 15 were treated with tendon transfers. They reported good results: nearly 100% patient satisfaction, all patients were able to elevate the thumb to the level of the palm, and full independent index finger movements.

 

 

 

 

Presence of diabetes-induced peripheral neuropathy has been shown to be an independent risk factor for postoperative complications of which of the following injuries?

Distal radius fractures

Patella fractures

Metatarsal fractures

Ankle fractures

Distal femoral fractures Correct answer: 4

Presence of peripheral neuropathy has important implications in treating ankle fractures in diabetic patients. Increased immobilization periods, attention to tight glucose control, and adjunct/alternative operative techniques may be necessary for an optimal outcome.

 

The first referenced article by Chaudry et al is an excellent review of diabetic ankle fractures.

 

The second reference by Costigan et al noted that peripheral neuropathy is the most significant risk factor for postoperative complications, followed closely by lack of pedal pulses preoperatively.

 

The last referenced article by Jones et al noted a significantly higher complication rate in diabetics with operative ankle fractures, and reported that neuroarthropathy is a significant risk factor for postoperative complications.

 

 

 

 

A 45-year-old male falls off his motorcycle and injures his arm. AP and lateral radiographs reveal a proximal ulnar shaft fracture, 30 degrees apex anterior, and a radial head dislocation. Which direction is the radial head most likely dislocated?

Lateral

Posterior

Posterolateral

Anterior

Anteromedial

 

Correct answer: 4

 

A Monteggia fracture with apex anterior ulnar shaft fracture is associated with an anterior radial head dislocation (See Illustration A).

 

Bado initially described and classified these injuries. The most common injury pattern in children is an extension (type 1) with anterior radial head dislocation and apex anterior ulnar shaft fracture, whereas the type II variety is most common in adults. The apex of the ulna fracture determines the direction of the radial head subluxation or dislocation. Adults typically require ORIF of the ulna.

 

These fractures in children are often treated non-operatively with closed reduction if the ulna fracture is transverse and stable as illustrated by Fowles' case series, but may also require pinning if the fracture is oblique and unstable.

 

 

 

 

 

A 34-year-old male sustains a traumatic injury to his foot following a motorcycle accident. The patient's neurovascular status necessitates the amputation demonstrated in figures A through C. One year following the amputation, the patient complains of difficulty with gait and deformity of the ankle. Which of the following statements best describes the forces resulting in this deformity?

 

 

Tight posterior capsule tissues of the ankle

Neuropraxia of the deep peroneal nerve

Unopposed pull of gastrocnemius-soleus only

Unopposed pull of gastrocnemius-soleus, posterior tibialis, and peroneus

brevis

Unopposed pull of gastrocnemius-soleus and posterior tibialis Correct answer: 5

The clinical photograph and radiograph demonstrate a modified Lisfranc amputation. The loss of the peroneus longus, peroneus brevis, EHL, EDL, and peroneus tertius insertions result in an equinovarus deformity due to the pull of the gastroc-soleus complex, posterior tibialis, and anterior tibialis.

 

Several surgical techniques have been described to address or prevent equinovarus deformities after Lisfranc amputation. Open or percutaneous achilles tendon lengthening, open gastrocnemius recession, or endoscopic gastrocnemius recession have shown to address the equinus deformity. Split tibialis anterior tendon transfer (STATT), 4th and 5th digit flexor-to-extensor tendons tenodesis, as well as peroneus brevis (PB) to peroneus longus (PL) tendon transfers have shown to address the varus deformity.

 

Early et al. state the Lisfranc amputation should be considered when there is inadequate soft tissue coverage for transmetatarsal amputation or instability at the LisFranc joint. In addition, they comment that the deforming forces, the gastroc-soleus complex and posterior tibialis, are primarily innervated by the S1 nerve root.

 

Figures A through C show clinical and radiographic images of a modified Lisfranc amputation. The classic Lisfranc amputation describes transaction of the first, third, fourth, and fifth tarsometatarsal joints. The second TMT is usually left in place, as it provides stability for the middle cuneiform. These radiographs show a midfoot amputation carried out more transversely across the bones of the midfoot.

 

Incorrect Answers:

Answers 1-4: The equinus deformity occurs after the elimination of extensor digitorum longus and extensor hallucis longus muscles, which cause an imbalance between the posterior compartment and the anterior compartment muscles that across the ankle. The varus deformity occurs when tibialis anterior and posterior compartment muscles overpower the eversion strength of the peroneus brevis muscle.

 

 

 

A 32-year-old female sustains the injury shown in Figure A. What is the most reliable method to evaluate the competence of the deltoid ligament?

 

 

Medial ankle tenderness

Medial ankle ecchymosis

Squeeze test

Stress radiography of the ankle

Canale view radiograph Correct answer: 4

Figure A shows a lateral malleolar fracture from a rotational-type injury. Evaluation of the medial structures (deltoid ligament) is important for therapeutic reasons, as medial sided instability will portend a poor prognosis if treated nonoperatively.

 

The referenced study by McConnell et al showed that physical exam is a poor indicator of medial ankle injury and that stress radiography is needed for proper medial ankle evaluation. All ankles found to be stable via stress examination healed with an intact mortise.

 

The study by Gill et al showed that gravity stress radiographs are equivalent to manual stress radiographs of the ankle when evaluating rotational ankle fractures. This was true when both pronation-external rotation and supination-external rotation ankle fractures were examined.

 

Illustration A shows the proper way to obtain a gravity stress radiograph.

 

 

 

 

A 34-year-old male is involved in a motor vehicle collision and sustains several orthopaedic injuries. Figure A shows a red line representating a fracture of the proximal femur. This fracture orientation is most often present when found concomitantly with which of the following orthopaedic injuries?

 

 

Ipsilateral acetabular fracture

lumbar spine burst fracture

Ipsilateral femoral shaft fracture

Anterior-posterior compression pelvic injury

Ipsilateral calcaneus fracture Correct answer: 3

Femoral neck fractures are seen less than 10% of the time with femoral shaft fractures, but they are frequently missed on initial evaluation. When present, the pattern is typically nondisplaced, vertical, and basicervical.

 

The review article by Peljovich and Patterson note that the femoral shaft component of the combined injury is typically in the middle third and is often comminuted.

 

The article by Tornetta et al reports that they reduced the delay in diagnosis of concomitant femoral neck fractures by 91% by instituting a protocol that included: dedicated AP internal rotation plain radiograph, a fine (2-mm) cut CT scan through the femoral neck, an intraoperative fluoroscopic lateral radiograph prior to fixation, as well as postoperative AP and lateral radiographs of the hip in the operating room prior to awakening the patient.

 

The article by Wiss et al noted that 18% of their ipsilateral femoral neck/shaft cohort developed a symptomatic varus nonunion requiring a valgus osteotomy. It is significant to note that in this study, the shaft fractures were fixed prior to definitive neck stabilization, and the review article by Peljovich emphasizes that the neck fracture should be treated first and the shaft fracture second.

 

 

 

 

A 74-year-old female falls from a standing height and sustains the fracture shown in Figure A. The occurrence of this injury most increases her risk of sustaining which of the following fractures?

 

 

Sacral fracture

Hip fracture

Distal radius fracture

Distal fibula fracture

Distal humerus fracture Correct answer: 2

Clinton et al performed a Level 2 study of 8049 women that demonstrated that proximal humeral fractures independently increased the risk of a subsequent hip fracture. The risk was more than five times in the first year after the humeral fracture but was not associated with a significant increase in the hip fracture risk in subsequent years.

 

Johnell et al performed a Level 4 review that found that men and women had an increased risk of hip, forearm and spine fractures following a prior spine, hip or shoulder fracture.

 

Schousboe et al performed a Level 2 investigation of 9516 community-dwelling elderly women and found that 521 hip fractures occurred after 10 years of follow-up. They found that prior non-spine fractures, non-hip fractures, and prevalent moderate to severe radiographic vertebral fractures were modestly associated with incident hip fracture.

 

 

 

 

A 47-year-old male sustained a comminuted calcaneus fracture in a motorcyle accident. He subsequently develops the post-traumatic condition shown in Figure A. All of the following would be indications for a subtalar distraction arthrodesis using a bone graft instead of an in-situ subtalar arthrodesis EXCEPT:

 

Decreased calcaneus height

Decreased talocalcaneal angle

Decreased talar declination angle

Presence of a collapsed subtalar joint from AVN

Presence of full ankle dorsiflexion with no tibiotalar impingement Correct answer: 5

Subtalar distraction arthrodesis using a bone graft is not indicated in a patient with full ankle dorsiflexion and no tibiotalar impingement.

 

The radiograph demonstrates post-traumatic subtalar arthritis with deformity. Subtalar arthrodesis in situ would not correct the deformity. Substantial loss of heel height may lead to symptomatic anterior tibiotalar impingement and is considered the most appropriate indication for subtalar distraction arthrodesis. Measurement of the talocalcaneal angle is made using a line representing the long axis of the talus and its intersection with the longitudinal axis of the calcaneus. It is demonstrated in Illustration A. The talar declination angle is formed by the axis of the talus to the plane of support and is demonstrated as angle H in Illustration B. The talocalcaneal height is measured from the dome of the talus to the base of the calcaneus displayed as Line K in Illustration C. Each of these measurements along with longitudinal arch are often decreased in patients with post-traumatic subtalar arthritis and are discussed as deformities that can be corrected with bone block subtalar distraction arthrodesis. Chandler et al and Trnka et al each performed Level 4 studies that concluded that distraction arthrodesis was most appropriate in the setting anterior ankle impingement.

 

 

 

 

 

 

A 30-year-old male sustains a right shoulder injury with initial radiographs shown in Figures A and B. What single piece of additional information would best assist in determining this patient's functional outcome?

 

 

Lower extremity injury

Neurological deficit

Contralateral upper extremity injury

Proximal humerus fracture

Worker's compensation

 

Correct answer: 2

 

Figures A and B show a scapulothoracic dissociation, with significant lateralization of the scapula and widening of the acromioclavicular joint to over 4 cm (Figure A). This can be thought of as an internal disarticulation of the scapulothoracic association and acromioclavicular joints.

 

The referenced article by Riess et al revealed that functional outcomes are

worse with brachial plexus injuries and concomitant scapulothoracic dissociation than with isolated brachial plexus injuries. In fact, at 2 year followup, only 57% of the dissociation patients could carry anything over 5 lb with the injured side.

 

The other referenced article by Zelle et al found that complete brachial plexus injuries portended the worst outcome for scapulothoracic dissociation injuries.

 

 

 

 

During a dual incision fasciotomy of the leg, the soleus is elevated from the tibia to allow access to which of the following compartments?

Superficial posterior

Deep posterior

Lateral

Anterior

Mobile wad

 

Correct answer: 2

 

The soleus is elevated/released from the posterior tibia during the medial approach to allow access to the deep posterior compartment. Release of this compartment cannot be done without proper elevation of the soleus. The superficial posterior compartment mass is primarily located in the proximal half of the leg, while the deep posterior musculature is located in the distal 2/3 of the leg.

 

Illustration A depicts the musculature and septums compartmentalizing the lower leg. The transverse intermuscular septum separates the deep from the superficial posterior compartments.

 

 

 

 

Longitudinal radioulnar dissociation, including Essex Lopresti fractures, requires disruption of the interosseous membrane (IOM). The interosseous membrane (IOM) consists of all of the following ligaments EXCEPT?

Central band ligament

Accessory band ligament

Annular ligament

Dorsal oblique accessory cord ligament

Distal oblique bundle ligament Correct answer: 3

The IOM includes 5 types of ligaments: central band, accessory band, distal oblique bundle, proximal oblique cord, and dorsal oblique accessory cord. The annular ligament is not a part of the IOM.

 

The IOM bridges the radius and ulna and acts as a hinge for rotation of radius about ulna. The central portion is thickened, and forms the central band which is the most important ligament for IOM load distribution characteristics.

 

Noda et al, in a cadaver study, identified the precise anatomical insertions and attachment points of each of the 5 IOM components. They found the most distal and proximal ends of the radial origin of the central band were 53% and 64% of total radial length from the tip of the radial styloid, whereas those of the ulnar insertion were 29% and 44% of total ulnar length from the ulnar head.

Pfaeffle et al also performed a cadaveric biomechanical study applying compressive loads to specimens with IOMs that are intact, cut, or cut/reconstructed with flexor carpi radialis allografts. They found that reconstruction of the IOM can restore the normal load transfer chararcteristics and that the central band of the IOM is the most important portion of the IOM to be reconstructed.

 

Illustration A shows the ligaments of IOM membrane: CB = central band, AB = accessory band, distal oblique bundle, proximal oblique cord, and dorsal oblique accessory cord

 

Incorrect Answers:

The annular ligament is a strong band of fibers, which encircles the head of the radius and keeps the radius in contact with the radial notch of the ulna, but is not a part of the IOM.

 

 

 

 

 

During surgical treatment of an olecranon fracture with a tension band construct as seen in Figure A, what nerve is at risk with over penetration of the proximal anterior cortex of the ulna with the Kirchner wire?

 

 

Median nerve

Anterior interosseous nerve

Posterior interosseous nerve

Ulnar nerve

Radial nerve

 

Correct answer: 2

 

An inability to flex the thumb interphalangeal joint or the index finger distal interphalangeal joint is indicative of an anterior interosseous nerve palsy/injury.

 

This has been reported with tension band fixation of olecranon fractures, especially with overpenetration of the anterior cortex of the proximal ulna by the Kirschner wire. Anterior interosseous nerve palsy can also be possible by overpenetration of drill bits or screws through the anterior cortex. Initial treatment of this nerve injury is observation.

 

The referenced article by Parker et al reports a case of anterior interosseous nerve palsy after such tension band fixation. They note that this was a direct injury from the Kirchner wires.

 

The referenced article by Adams et al reviews nerve injuries about the elbow. They note that an appreciation of the complex anatomy of the region and an understanding of treatment options are necessary for surgeons who treat elbow injuries.

Figure A is a lateral radiograph of a olecranon fracture fixed with a tension band construct.

 

Incorrect Answers: The other nerves are not associated with overpenetration of the far cortex in an olecranon tension band fixation.

 

 

 

 

In an isolated ankle syndesmotic injury, the fibula is unstable in the incisura fibularis of the tibia. In what direction is the fibula most unstable?

Anterior-posterior

Medial-lateral

Proximal-distal

Varus-valgus

Equivalent instability in all axes Correct answer: 1

In an ankle syndesmosis injury, the fibula is most unstable in an anterior and posterior direction. This is whether or not there is an accompanying ankle fracture. Most commonly, the fibula will subluxate in an anterior-posterior direction in an ankle fracture model.

 

The first referenced article by Xenos et al found that stress lateral radiographs have more interobserver reliability than stress AP/mortise radiographs and that two syndesmotic screws are stronger than one.

 

The referenced article by Candal-Couto et al is a biomechanical study that found more anterior-posterior instability in a syndesmosis injury model, and more ankle instability is noted with syndesmosis injury and a concomitant deltoid injury.

 

The referenced article by Zalavras et al is an excellent review article on ankle syndesmosis injuries.

 

 

 

 

A 27-year-old male sustains closed injuries to his right foot in a motor vehicle collision. He is a nonsmoker. A radiograph and computed tomography scan are provided in Figures A and B. All of the following

are prognostic of a superior outcome with operative treatment EXCEPT:

 

 

 

Male

Works as an attorney

Nonsmoker

Twenty-seven years old

He was injured while off his job Correct answer: 1

The clinical presentation, radiographs, and CT are consistent with a displaced and severely depressed intra-articular calcaneus fracture.

 

Rammelt et al reviews the operative treatment of calcaneal fractures. Notably

poorer outcomes are associated with open fractures, delayed reduction after more than 14 days and individual risk factors such as high body mass index and smoking.

 

The two studies by Buckley et al are both data taken from a multicenter, prospective, randomized study of calcaneal fractures. Women who were treated operatively achieved superior clinical outcomes compared to those women treated nonoperatively. Patients who were not on workers compensation and less than 29 years old and those that had a moderate Bohler's angle (0-14 degrees), less fracture comminution, more sedentary jobs, and an anatomic reduction reported superior clinical outcomes than those treated nonoperatively.

 

Figure A is a lateral radiograph demonstrating an intra-articular joint depression calcaneus fracture. Figure B is a coronal CT scan demonstrating intra-articular displacement of the posterior facet.

 

Answers 2-5 are all incorrect because these are predictive of superior outcome with operative treatment.

 

 

 

 

A 25-year-old male involved in a motorcyle accident sustains the injury seen in Figures A and B. After initial adequate debridement of nonviable tissue, which of the following irrigation methods and devices should be used?

 

 

Antibiotic solution applied by low pressure gravity flow device

Antibiotic solution applied by high pressure pulsatile flow device

Saline solution applied by low pressure gravity flow device

Saline solution applied by high pressure pulsatile flow device

Antibiotic solution applied by high pressure pulsatile flow device followed by low pressure gravity flow device

Correct answer: 3

 

Initial wound treatment is critical in the treatment of open fractures and contaminated wounds. Figures A and B reveal a Type IIIB open tibia fracture with obvious gross contamination and devitalized tissue. Systemic antibiotics and tetanus prophylaxis should be administered immediately upon arrival, and

the lower extremity should be splinted while the remaining trauma workup takes place. Once cleared, this injury should be adequately debrided of all devitalized tissue and subsequently irrigated with a saline solution to reduce the bacterial count. Some evidence suggests that high-pressure pulsatile lavage damages bone structure and disrupts soft tissue.

 

In an animal model, Hassinger et al showed that high-pressure pulsatile lavage caused deeper penetration of bacteria and results in greater bacterial retention in soft tissue when compared with low-pressure lavage.

 

Owens et al, in a sheep model of contaminated soft tissue, compared low and high pressure lavage with normal saline solution, bacitracin solution, castile soap, and benzalkonium chloride. At 48 hours the group treated by low pressure lavage and saline showed the lowest rebound in bacterial counts.

 

 

 

 

A patient presenting with scapulothoracic dissocation and ipsilateral extremity neurologic injury is most likely to have which of the following outcomes?

Glenohumeral arthritis

Return of 3/5 motor strength in distal extremity

Full return of extremity sensory function only

Flail extremity

Death

 

Correct answer: 4

 

Scapulothoracic dissociation is a high-energy injury resulting from massive traction injury to the shoulder girdle with disruption of the scapulothoracic articulation. The most common long term result from this injury is complete loss of motor and sensory function of the extremity (flail limb), with death in the acute or semi-acute period also common.

 

The referenced study by Althausen et al found that outcomes from this injury were: a flail extremity in 52%, early amputation in 21%, and death in 10%.

 

The other referenced study by Ebraheim et al found that 12/15 patients had a complete brachial plexus injury and that none recovered any function (the other 3 patients died in the acute period).

 

 

 

Vascular complications are most commonly seen with which of the following fractures about the knee?

 

 

 

 

Figure A

Figure B

Figure C

Figure D

Figure E

 

Correct answer: 2

 

Figure B represents a medial plateau fracture/dislocation pattern (Schatzker IV). This fracture typically requires more energy to occur than the corresponding lateral plateau injury, which is due to the more dense bone on the medial side. A fracture-dislocation of the knee must be suspected with these injuries, as the femur will sometimes follow the displaced medial tibial condyle. Along with a proper vascular exam, ankle brachial indices (ABI) must be immediately taken and if abnormal further vascular testing is warranted.

Furthermore, these injuries which are a hybrid of a dislocation and a fracture will often have a benign appearance on radiographs, but a high rate of vascular complications.

 

Berkson et al present a review article and they stress the importance of safeguarding tissue vascularity and while emphasizing joint reduction and restoration of the mechanical axis of the limb.

 

Ottolenghi et al in their review article showed a vascular injury rate of 2% for

tibial plateau fractures.

 

Stark et al in their review showed a very high incidence of acute compartment syndrome in Schatzker IV and VI injuries. In their series, 18% of Schatzker VI and 53% of Shatzker IV fractures developed compartment syndrome.

 

 

 

 

A 26-year-old male wrestler suffers the elbow injury shown in Figure A. On physical exam he is neurologically intact and has a palpable radial pulse. He is treated with closed reduction in the emergency room. In order to optimize his clinical outcomes, which of the following treatment and rehabilitation protocols should be avoided?

 

 

Immediate active and active-assist range of motion through a stable arc

Initial splinting and immobilization for 4 weeks followed by physical therapy

Initial splinting in 90 degrees of flexion with neutral forearm rotation

A range of motion protocol that limits full extension in the early phases of rehab

Light duty use of the affected arm immediately following immobilization Correct answer: 2

Prolonged splinting following simple elbow dislocations of greater than 2 weeks after reduction can lead to chronic stiffness and poor outcomes and should be avoided. All of the remaining answers are appropriate in the rehabilitation of these injuries.

 

Simple elbow dislocations are second only to the shoulder in rates of joint dislocation. Closed reduction with early rehabilitation has proven the most effective treatment for these injuries when the elbow is stable. Rehabilitation for a simple elbow dislocations should include initial splinting in 90 degrees of flexion with neutral forearm rotation and immediate active and active-assist range of motion through a stable arc. Full extension should be avoided initially

in rehab. The patient should be encouraged to use the affected arm for light duty immediately following immobilization.

 

Mehloff et al. determined that elbows splinted longer than 2 weeks had decreased ROM at 1 year. They further describe the benefits of using the affected arm for light duty after 2 weeks of rest.

 

Smith et al. describe the effectiveness of immediate active and active-assist ROM exercises after reduction.

 

O'Driscoll et al. describe how to test an elbow for posterolateral instability, which can occur after an elbow dislocation. They emphasize the elbow is most unstable in extension immediately after injury. Therefore, the elbow should be splinted in 90 degrees of flexion with neutral forearm rotation until therapy begins.

 

Figure A is a lateral radiograph of the elbow which shows a simple posterior elbow dislocation. The anatomic description is based on the anatomic location of olecranon relative to humerus.

 

 

 

 

Increasing the oxygen gradient for diffusion is the mechanism of action for which of the following methods of treatment of lower extremity trauma?

Open fasciotomy

Percutaneous fasciotomy

High-dose anti-inflammatories

Hyperbaric oxygen therapy

Negative pressure wound therapy Correct answer: 4

Hyperbaric oxygen therapy (HBO) allows patients to breathe 100% oxygen in a chamber under conditions of increased barometric pressure.

 

This tremendous partial pressure of oxygen supports gas diffusion for a much greater distance than under normal conditions, thus delivering oxygen to relatively ischemic and hypoxic tissues. Trauma-related indications for HBO therapy include carbon monoxide intoxication, gas gangrene, crush injury, compartment syndrome, necrotizing fasciitis, treatment of chronic osteomyelitis, support of grafts and flaps, and burns. Contraindications relate to issues of gas exchange, oxygen sensitivity, and technical safety.

The cited reference is a useful review article of hyperbaric oxygen therapy in extremity trauma.

 

 

 

 

All of the following indicators of resuscitation may be within normal limits for a trauma patient that is in "compensated" shock EXCEPT:

Systolic blood pressure

Urine output

Heart rate

Serum lactate

Mean arterial pressure Correct answer: 4

Historically, normal blood pressure, heart rate, and urine output have been endpoints to signal complete resuscitation in the polytrauma patient.

 

The review article by Porter et al states that there is a high incidence of patients (as much as 85%) in "compensated" shock despite normal vital signs and urine output parameters. Compensated shock is secondary to a maldistribution of blood flow and tissue oxygenation as splanchnic organs have less distribution of the cardiac output compared to the heart and the brain.

 

The article by Elliott is also a review, and it states that serum lactate is the best indicator of peripheral organ perfusion and tissue oxygenation. It also states that base deficit and gastric mucosal pH are appropriate end points to determine the complete resuscitation of trauma patients.

 

 

 

 

A 25-year-old man sustains the fracture seen in Figure A and is seen in pre-op holding prior to surgery. What position of his lower extremity would result in the lowest intracapsular hip pressure?

 

Neutral rotation and flexion

External rotation and flexion

Internal rotation and extension

External rotation and extension

Internal rotation and flexion Correct answer: 2

The femoral neck fracture seen in Figure A is considered by most to be a surgical emergency in a 25-year old due to the at risk blood supply of the femoral head. Due to the intra-capsular hematoma and hemarthrosis that occurs, the involved extremity is often found to be in external rotation and flexion as the intra-capsular volume is the highest in this position. Debate exists of the significance of the pressure caused by the fracture hemarthrosis as it is believed by some to cause a local compartment syndrome adding further insult to the already tenuous blood supply. Advocates of early fixation have proposed that reduction maneuvers without capsulotomy can compromise the circulation of the femoral head by increasing the hip joint pressure.

Maruenda et al showed in their study of 34 consecutive patients with femoral neck fractures that the mean intracapsular pressure was the highest with the hip in extension and internal rotation.

 

Bonnaire et al in their prospective study of 55 patients with intracapsular femoral neck fractures found the lowest pressure to be at 70 degrees of

flexion. Both Maruenda and Bonnaire's studies showed no significant intracapsular pressure difference based on fracture displacement.

 

Corollary to adult fracture patients, it is also noted that pediatric patients with a septic hip hold their hip in a flexed and externally rotated position to maximize intracapsular volume as shown in Illustration A.

 

 

 

 

 

A 15-year-old male falls from his bicycle and lands directly on his left hip. He is unable to bear weight and presents to the emergency department with the AP pelvis radiograph seen in Figure A. Which of the following radiographic views could aid in classifying this patient's fracture pattern?

 

 

Iliac oblique (Judet) view

Traction AP of the hip with the leg in neutral rotation

Outlet view of the pelvis

Traction AP of the hip with the leg internally rotated 15°

Traction AP of the hip with the leg externally rotated 15° Correct answer: 4

The ability to define the location of a femoral neck fracture aids in determining the optimal treatment. The addition of a "traction-internal rotation" view (as described in Answer 4) to standard hip radiographs may assist with classification of femoral neck fractures by accounting for the anteversion of the femoral neck.

 

Koval et al performed a study in which proximal femur fractures were classified with standard radiographs (AP pelvis, AP hip, cross-table lateral), followed by the addition of a physician-assisted view with traction and 15 degrees of internal rotation. The inclusion of the traction-internal rotation view led to increased agreement in classification between the authors, and was especially helpful for differentiating displaced femoral neck fractures versus stable intertrochanteric fractures.

 

 

 

 

A 25-year-old female sustains the isolated fracture seen in Figure

A. The patient elects to have nonoperative management. When compared to operative treatment, which of the following is true of the clinical outcome following nonoperative management?

 

Long arm cast immobilization is necessary with nonoperative management

Twenty degree loss of forearm rotation is expected with nonoperative management

Loss of wrist motion is expected with nonoperative management

Loss of elbow motion is expected with nonoperative management

Equivalent clinical outcomes Correct answer: 5

The clinical presentation is consistent for an isolated ulnar fracture, or "night stick" fracture. For minimally displaced and angulated fractures, nonoperative management has equivalent clinical outcomes to surgical treatment.

 

Isolated ulnar shaft fractures with greater than 10 degrees of angulation or 50% displacement can result in loss of forearm rotation if treated nonoperatively. For minimally displaced fracture, nonoperative treatment results in union with good function. Time to union has been found to be greater in nonoperative groups, but clinical outcomes equivalent.

 

Pollock et al. compared two patient groups treated with short term immobilization or long term cast treatment and found no significant difference between wrist and elbow motion between the two groups. The short term immobilization group in their study had a shorter time to union and lower non union rate as well.

 

 

 

 

A comminuted proximal humerus fracture is treated with a shoulder hemiarthroplasty as shown in Figure A. The superior border of the pectoralis major tendon can be used to determine accurate restoration of which of the following?

 

Humeral prosthesis height and retroversion

Humeral prosthesis offset and retroversion

Humeral prosthesis head-neck angle and height

Humeral prosthesis stem width and height

Humeral prosthesis stem length and retroversion Correct answer: 1

The height of the prosthesis is best determined by the superior border of the pectoralis major insertion (PMI), which has been shown in several clinical and cadaveric studies to be the most reliable instrument to assess humeral prosthesis height. In addition, the PMI can be used to position the implant in regards to retroversion, as the distance and angle of the PMI to the posterior fin of the implant has been investigated and reported (see second reference below). No evidence exists to differentiate this against using the forearm as a landmark for retroversion, however.

 

The referenced study by Murchavosky et al found that in cadaveric dissection, pectoralis major tendons inserted 5.6 cm distal to the superior aspect of the humeral head.

 

The referenced study by Torrens et al also found that the insertion point was

5.6 cm distal to the superior aspect of the humeral head and very reproducible regarding its relationship to retroversion. The mean distance of the PMI to the

posterior fin of the prosthesis was 1.06 cm. The mean angle between the PMI and the posterior fin of the prosthesis was 24.65 degrees.

 

The referenced study by Greiner et al found that clinical results from utilizing this reference in fracture cases improved radiographic and clinical outcomes; they also reported that clinical outcomes primarily depended on tuberosity healing.

 

Figure A shows a shoulder hemiarthroplasty.

 

 

 

 

A 35-year-old male involved in a high-speed motor vehicle collision presents to the trauma bay hypotensive and with a clinically unstable pelvis. A pelvis radiograph is shown in Figure A. He is placed in a pelvic binder, and his blood pressure normalizes temporarily. An abdominal CT demonstrates free fluid and air in the intraperitoneal cavity, and a laparotomy is indicated. What is the most appropriate next step in orthopaedic management?

 

 

Percutaneous SI screw placement

External fixation placement

Pubic symphysis plating

Posterior pelvic plating

Anterior sacroiliac plating Correct answer: 2

In the management of patients with multiple injuries, controversy often arises as to the appropriate method of initial pelvic stabilization. It is generally agreed upon that applying an external frame is appropriate in the setting of an unstable patient with intraperitoneal fluid and labile blood pressure. Ex-fix placement can support hemodynamic stabilization and assist the general surgeons with their laparotomy procedure. Plate or screw fixation of the pelvis should be delayed because the laparotomy takes precedence in a patient who is hemodynamically unstable, and internal fixation in the presence of bowel contamination can result in increased rates of infection.

 

Furthermore, Tile noted increased septic complications with intrapelvic hardware fixation in the setting of intraperitoneal soft tissue damage and bleeding. Angiography and embolization may help with the pelvic bleeding, but will not stabilize the pelvis during the laparotomy. The review article by Tile et al discusses the assessment of the patient with a pelvic injury, and summarizes the various methods of temporary and definitive pelvic fixation.

 

 

 

 

A 32-year-old female sustains an isolated midshaft clavicle fracture, as shown in Figure A. Her clinical exam does not reveal skin tenting or neurovascular injury, but shortening is measured at 2.6 cm. Which of the following treatment methods has been shown to have the lowest rate of nonunion and symptomatic malunion?

 

 

Open reduction and internal fixation with plating

Open reduction and percutaneous pinning

Closed reduction and percutaneous pinning

Closed reduction and external fixation

Nonoperative treatment with a sling and early range of motion Correct answer: 1

Figure A shows a left clavicle fracture with significant shortening. Open reduction and internal fixation with plate and screw constructs of displaced, shortened clavicle fractures has been shown to lead to the best patient reported functional outcomes as well as have the least incidence of nonunion and symptomatic malunion. Factors associated with poor functional outcome as well as nonunion in these injuries include fracture shortening >2cm, fracture comminution, fracture displacement > 100%, female gender, and advancing age.

 

The referenced article by Khan et al is an excellent review of the indications, treatment methods and outcomes of clavicle fractures.

 

 

 

 

A 78-year-old female falls and sustains the fracture seen in Figure A. Surgical treatment is pursued with open reduction internal fixation with a lateral locking plate. Postoperative radiographs are provided in Figure B. What is the most common complication with this mode of fixation?

 

Infection

Osteonecrosis

Axillary artery injury

Screw cut-out

Axillary nerve injury Correct answer: 4

Screw cut-out is the most common complication following open reduction and internal fixation of 3 and 4 part proximal humerus fractures. A radiograph of this phenomenon in this patient at 5 months is shown in illustration A. This complication ultimately results from fracture settling and axial compression allowing the screws to penetrate the joint.

 

Owsley and Gorczyca evaluated a single surgeon's experience treating 53 patients with displaced proximal humeral fracture with a standard post-op regime. They found that screw cut-out was the most common complication following fixation of 3 and 4 part fractures with locking plate technology.

 

Frankhauser et al. describe their results with a "new" proximal humeral locking plate (AO/ASIF technology) in 28 patient with 29 fractures. They describe good to excellent clinical/radiographic results by following radiographs and patient related Constant scores.

 

 

 

 

 

A 72-year-old female presents to your office with a 12-month old painful nonunion of a 2-part (surgical neck) fracture of the proximal humerus. Prior to her injury, she denied shoulder pain and an excellent range of motion. Since the injury, she has persistent debilitating pain and dysfunction with motion above the shoulder level. X-Rays show good bone stock, no significant shoulder arthritis or avascular

necrosis, and well-positioned tuberosities. Which of the following is the most optimal next step in management?

Closed reduction and percutaneous pinning

Shoulder hemiarthroplasty

Superior capsular reconstruction (SCR)

Open reduction and internal fixation (ORIF) with or without bone grafting

Reverse total shoulder replacement with lattismus dorsi transfer to assist with internal rotation

Correct answer: 4

 

Treatment of 2 part proximal humerus malunion in patient with good bone stock with evidence of arthritis or osteonecrosis can be effectively treated with ORIF with locked plating with or without bone grafting.

 

Treatment of a chronic nonunion of the proximal humerus in the elderly should be treated with fixation when possible. Critical attention should be paid to correct all deformities: tuberosity positioning, articular surface realignment, soft tissue balancing, rotator cuff repair (when needed), and treatment of soft tissue contractures. Attempts at arthroplasty are generally recommended only when the fracture has eroded enough to prevent successful fixation, if the tuberosities have resorbed, if the rotator cuff has a pre-existing tear, or other findings are present that would limit the success rate of fixation.

 

Quadlbauer et al. evaluated the results of ORIF with locked plating without bone grafting for 9 patients with proximal humerus non-unions with a mean 31-month followup. They noted that all 9 patients went on achieve bony union with improvement in ROM in all planes except fo adduction. They concluded that ORIF with locked plating without bone grafting is a reasonable and safe option for treating proximal humerus nonunion with high union rates and minimal risk of complications

 

Cadet et al. in a JAAOS review article discussed the various management options for proximal humerus non-unions. The authors noted that several recent series did demonstrate union in >90% of patients treated with reconstruction using locking plates and autogenous bone graft. They discuss that while reverse shoulder arthroplasty (rTSA) has also showed excellent clinical results, its should be reserve for patients with nonviable humeral heads, severe humeral osteoporosis

 

Incorrect Answers:

Answer 1: There is no role for closed reduction in the setting of a non-union Answer 2: While rTSA has shown great results with, similar results have not

recently been shown for shoulder hemiarthroplasty

Answer 3: SCR is reserved for a patient with irreparable superior rotator cuffs who are considered too young to undergo arthroplasty

Answer 5: While rTSA is an effective treatment option here, the addition of a lattismus dorsi transfer would assist with EXTERNAL rotation not INTERNAL rotation.

 

 

 

 

There is a risk of impaired forearm rotation after tension band fixation of an olecranon fracture with which of the following?

Ipsilateral proximal humerus fracture

Protrusion of Kirschner wire fixation through the volar cortex of the proximal ulna

Use of ulnar intramedullary Kirschner wire fixation

Olecranon fracture comminution

Lack of triceps tendon repair Correct answer: 2

Impaired pronation/supination can be seen if the K-wire is advanced either too radial or too far through the volar (anterior) cortex of the proximal ulna. The anterior interosseous nerve is also at risk with overpenetration. Conversely, migration and loosening of the K-wire is reduced with involvement of the anterior cortex.

 

The referenced study by Candal-Couto et al is a cadaveric study that found that K-wire insertion in less than 30 degrees in an ulnar direction led to impingement of the K-wire on the radial head/neck, biceps or supinator. They recommend placing these wires away from this danger zone in order to minimize rotation blocks.

 

The referenced study by Matthews et al is a case series of two patients who had limited forearm rotation after K-wire fixation. The etiology of limited rotation was found to be from direct overpenetration of the K-wire, which led to a mechanical block.

 

 

 

 

When performing a saline load test to evaluate for a traumatic arthrotomy of the knee, a mininum of how much saline should be utilized?

30 mL

50 mL

75 mL

100 mL

155 mL

 

Correct answer: 5

 

A saline load test is commonly utilized to evaluate for intraarticular penetration of superficial lacerations (traumatic arthrotomies). In this technique, a large gauge needle is used (18 ga) and saline is injected into the knee with passive ROM provided. Saline egress from the soft tissue injury is a positive test (traumatic arthrotomy present).

 

The study by Nord et al found that the volumes of saline that were needed in order to effectively diagnose 75%, 90%, 95%, and 99% of the knee

arthrotomies were 110, 145, 155, and 175 mL, respectively.

 

The other referenced study by Keese et al found that 50 mL of saline successfully identified only 46% of known knee arthrotomies and that 194 mL was needed to reach a 95% identification rate.

 

 

 

 

Regarding the role of the orthopaedic surgeon in addressing domestic and family violence, all of the following statements are true EXCEPT:

Report all cases of child abuse, as this is required by all states

Report all cases of adult spousal or intimate partner abuse, as this is required by all states

Hospitalize elderly victims who are in immediate danger and help develop a plan to ensure their safety

Advocate for appropriate legislation and public policy on violence and abuse related to health care

Orthopedic surgeons are responsible for knowing the reporting laws and procedures for suspected abuse

Correct answer: 2

 

Reporting requirements for adult spousal or intimate partner abuse is not standardized among states and it is the responsibility of the orthopaedic surgeon to understand the laws of his or her state. The AAOS Advisory statement gives information to assist in meeting the ethical and legal

obligations on Domestic and Family Violence and Abuse.

 

Domestic and family violence affects over 10% of the US population (approximately 32 million Americans). Child abuse and neglect contributed to 1,400 fatalities in 2002 and there was 565,747 reports of suspected elder abuse.

 

Reporting of suspected child abuse is required in all states. The orthopaedic surgeon should hospitalize elderly victims who are in immediate danger and help develop a plan to insure their safety.

 

 

 

 

A 42-year-old female sustains the injury shown in Figure A. What other anatomic structure is most commonly injured with this fracture?

 

 

Volar long radiolunate ligament

Radioscaphocapitate ligament

Dorsal radioulnar ligaments

Ligament of Testut and Kuentz

Scapholunate ligament

 

Correct answer: 3

Figure A shows a fracture of the distal 1/3 of the radius. This fracture is often associated with dislocation of the distal radioulnar joint (DRUJ). When the two injuries are present together, it is commonly referred to as a Galeazzi fracture.

 

DRUJ instability is a result of injury to the volar and dorsal radioulnar ligaments which are the primary stabilizers of this joint. Imaging often shows widening of the DRUJ on AP view, dorsal or volar displacement on lateral view and radial shortening. Treatment of a Galeazzi injury is operative, consisting of open reduction and internal fixation of the radius with a plate and screw construct, followed by intraoperative assessment of DRUJ alignment.

 

The referenced study by Giannoulis et al is an excellent review of Galeazzi fractures and treatment methods.

 

Ward et al found in a cadaver study that the most significant increases in translation of the wrist occurred after sectioning the dorsal radioulnar ligament in pronation and after sectioning the palmar radioulnar ligament in supination.

 

This patient's AP radiograph is shown in illustration A, and this shows an obvious DRUJ dislocation.

 

Incorrect answers

Choice 1,2,4 and 5 are not associated with injury of the DRUJ

 

 

 

 

A 34-year-old man sustains a twisting injury to his left ankle playing soccer. Radiographs from the ER are provided in figures A and

B. Four hours later, he undergoes open reduction internal fixation. An intraoperative fluoroscopy image is provided in figure C. Which of the following is the best method to assess the integrity of the syndesmosis?

 

 

Measurement of medial clear space widening

Measurement of the tibiofibular overlap

Anterior drawer test with comparison to the contralateral ankle

External rotation stress radiograph

Evaluation of the syndesmosis on preoperative CT scan

Correct answer: 4

 

The radiograph demonstrates a Weber B ankle fracture. A dynamic external rotation stress test with the ankle dorsiflexed to 90 degrees is the most accurate way to evaluate the integrity of the syndesmosis.

 

Nielson et al evaluated 70 ankle fractures with radiographs and MRI. Neither measurements of the tibiofibular clear space nor the tibiofibular overlap correlated with syndesmotic injury on MRI. Medial clear space widening of more than 4 mm occurred with MRI evidence of disruption of the deltoid and the tibiofibular ligaments.

 

Nielson et al used this same cohort of ankle fractures in a separate study to evaluate whether the level of the fibular fracture correlated with syndesmotic incompetence. They found no correlation. The level of fracture on the fibula cannot be used to accurately predict disruption of the syndesmosis.

 

Ebraheim et al reviewed a series of Weber B fibula fractures with deltoid injury and syndesmotic disruption. Their findings concluded that the surgeon's dynamic assessment of syndesmotic stability was more predictive of syndesmotic stability than any imaging parameters including radiographs and CT scans.

 

 

 

 

A 45-year-old male sustains a Gustilo and Anderson Type II open transverse humeral shaft fracture. He undergoes the treatment shown in Figures A and B. What is the advantage of this treatment choice as compared to antegrade intramedullary nailing?

 

 

Decreased risk of post-operative elbow pain

Decreased risk of radial nerve injury

Decreased risk of reoperation

Decreased risk of infection

Decreased risk of blood loss Correct answer: 3

Humeral shaft fractures treated with antegrade intramedullary nailing (IMN) have a higher risk of receiving an additional operation compared to those treated with plate fixation as seen in Figures A and B. Additionally, there is a higher incidence of shoulder impingement following IMN compared to plate fixation. There has been no difference found between the incidence of infection, elbow pain or radial nerve injury when comparing the two treatment options. Blood loss is not lower in an open case compared to nailing.

 

In a meta-analysis of 3 randomized studies comparing compression plate fixation to IMN fixation of humeral shaft fractures, Bhandari et al, found a significant relative risk reduction of 74% to reoperation with the use of plates and screws versus IMN. They also found a relative risk reduction of 90% in shoulder impingement with the use of plates and screws versus IMN. No significant difference was found between the two regarding rates of infection and/or radial nerve palsies.

 

In a prospective randomized study comparing IMN fixation to plate fixation in humeral shaft fractures, McCormack et al, also showed a significantly higher rate or reoperation in patients treated with IMN fixation. They also showed no difference in functional outcome or pain scores between the two groups. There were more nonunions in the IMN group in these studies, but the numbers were small and it did not reach statistical significance. Previous studies have shown higher nonunion rates in the IMN patients.

 

 

 

 

A 27-year-old man sustains a Gustilo and Anderson type II open tibia fracture during a motorcycle accident. He had a full course of tetanus vaccination as an infant and child. He also had a tetanus booster vaccination 12 months ago when he began a new job. In addition to intravenous antibiotics, what tetanus prophylaxis should be administered?

No prophylaxis required

Tetanus vaccine

Tetanus immune globulin

Tetanus vaccine and tetanus immune globulin

Tetanus vaccine and tetanus immune globulin with a booster vaccine required 6 months from now

Correct answer: 1

 

Any time an open wound is encountered, the tetanus prophylaxis protocol should be initiated in the emergency room. The correct treatment depends upon the severity of the wound and the patient's tetanus vaccine status.

Treatment may entail no further action, vaccination, or vaccination and administration of the tetanus immune globulin. The tetanus vaccine, booster, and immune globulin are used to enhance the immune response to clostridium tetani, a gram positive bacillus found in soil. In this case, the patient's tetanus had been updated within the past 5 years so he does not need an update of the vaccination or immune globulin.

 

Illustration A is a concise table that can be used as an algorithm to provide appropriate tetanus prophylaxis. Of note, this may vary from nation to nation.

 

 

 

 

 

A 67-year-old woman slips on the ice while retrieving her mail and lands on her outstretched left hand. She complains of wrist pain and deformity. On physical exam she has no sensation of the volar thumb, index, and middle fingers. Radiographs are provided in Figure

A. Two hours following closed reduction, the deformity is corrected, but the numbness and wrist pain is worsening. Which of the following interventions should be taken?

 

Evaluation of volar compartment pressures with a needle monitor

Icing and elevation of the arm with follow-up evaluation in 8 hours

Immediate EMG evaluation of the left upper extremity

Closed reduction, carpal tunnel release, and sugar tong splinting

Emergent open reduction internal fixation with carpal tunnel release Correct answer: 5

The radiograph demonstrates a displaced distal radius fracture along with a scaphoid fracture, and the patient is presenting with neurologic deficits of acute carpal tunnel syndrome. This is a surgical emergency requiring release of the carpal tunnel to prevent permanent dysfunction.

 

Dyer et al performed a retrospective cohort comparison of patients with distal radius fractures. Fracture translation was the most significant risk factor for development of acute carpal tunnel syndrome. Ipsilateral extremity fracture and multi-extremity trauma were trending, but not statistically significant risk factors.

 

In a review article, Schnetzler describes the pathogenesis, evaluation, and treatment of acute carpal tunnel syndrome.

 

 

 

Which of the following techniques has not been shown to prevent valgus angulation during intramedullary nailing of proximal one-third tibia fractures?

Use of a blocking screw lateral to midline in the proximal segment

Use of femoral distractor

Use of a lateral tibial nail starting point

Use of supplementary plate and screw fixation

Use of a suprapatellar nailing portal Correct answer: 5

Proximal tibial shaft fractures treated with intramedullary nails are most commonly malreduced with apex anterior and valgus deformities. Several techniques are available to overcome this malalignment: proximal and lateral nail starting point, usage of a femoral distractor or temporary plating, suprapatellar nailing, and lateral parapatellar approaches. Suprapatellar nailing portals have not been shown to affect coronal angulation - the concept is to affect the apex anterior deformity.

 

A final technical trick is the usage of blocking (Poller) screws - the referenced article by Ricci et al had 100% correction and maintenance of reduction with usage of blocking screws without other adjunct techniques. These should be placed in the lateral aspect of the proximal and distal fragments when needed. Remember, the blocking screws should go in the concavity of the deformity, or where you don't want the nail to go.

 

The referenced study by Krettek et al is a biomechanical evaluation of blocking screws in a tibial model that showed significantly increased strength when they were utilized.

 

Illustration A demonstrates proper AP screws placed lateral to the nail (white large arrows) to prevent valgus deformation, and ML screws placed posterior to the nail (white small arrows) to prevent procurvatum.

 

 

 

 

A 25-year-old female falls from her horse and injures her left wrist. There are no open wounds and the hand is neurovascularly intact. Radiographs are provided in Figures A-C. Which of the following will best achieve anatomic reduction, restore function, and prevent future degenerative changes of the wrist?

 

 

Long arm cast above the elbow for 6 weeks

Long arm cast for 3 weeks followed by a short arm cast for 3 additional weeks

Closed reduction and external fixation

Closed reduction and percutaneous pinning

Open reduction and internal fixation Correct answer: 5

The radiographs demonstrate an intra-articular distal radius fracture with dorsal angulation. Most importantly, there is a large dorsal articular fragment. Open reduction of this fragment is best acheived through an open approach.

The choice of dorsal vs. volar approach is debatable, but most importantly the articular surface, radial height, and volar tilt must be reduced anatomically.

 

Yu et al reviewed 104 distal radius fractures treated with either dorsal or volar plating. There was no difference in the rate of tendon irritation or rupture between the groups. Volar plating was associated with significantly more neuropathic pain complications.

 

Illustrations A and B demonstrate the open reduction and internal fixation of this fracture with a dorsal plate.

 

 

 

 

 

Of all the pelvic ring injury types, anteroposterior compression type III pelvic ring injuries have the highest rate of which of the following?

Head injury

Pulmonary injury

Traumatic amputation

Need for transfusion

Upper extremity fractures Correct answer: 4

Of the pelvic ring injuries, APC type III have the highest rate of mortality, blood loss, and need for transfusion. They also have a high rate of urogenital injury and abdominal organ injury. Lateral compression injuries (especially type III) have the highest rate of head injury. Vertical shear and combined injuries also have significant rates of concomitant injuries. The referenced article by Dalal et al is a review of Shock Trauma's pelvic ring injuries; they found significant increases in associated injuries as the grade of pelvic ring injury increased, regardless of mechanism/pattern. The aforementioned information was also found to be true with their patient review.

 

 

 

 

A 34-year-old male is an inpatient at a rehabilitation hospital after sustaining severe lower extremity injuries in a motor vehicle collision. As a result, his energy expenditure while ambulating is 40% above baseline after being fitted with an appropriate prosthetic prescription. What is this patient's most likely lower extremity amputation level?

Unilateral long transtibial

Unilateral average transtibial

Bilateral transtibial

Unilateral transfemoral

Bilateral transfemoral

 

Correct answer: 3

 

Bilateral transtibial amputee patients have an energy expenditure that is 40% above baseline levels.

 

The comparative study by Huang et al measured energy consumption by indirect calorimetry and found that mean oxygen consumption was 9% higher for a unilateral transtibial amputee, 49% higher for a unilateral transfemoral amputee (Miller's Review states 65%), and 280% higher for a bilateral transfemoral amputee.

 

The study by Traugh et al measured oxygen consumption in 9 transfemoral amputees. They found that crutch walking and prosthetic ambulation require the same amount of energy expenditure. Miller's Review also states that a long

transtibial amputation expends only 10% above baseline compared to an average length transtibial amputation which expends energy 25% above baseline.

 

 

 

 

A 34-year-old female presents to the trauma bay with hemodynamic instability following a motor vehicle collision. A chest radiograph shows a left-sided hemothorax and her pelvis radiograph is shown in Figure A. Which of the following is the next most appropriate step in managment?

 

 

Circumferential pelvic sheeting

Retrograde urethrogram to evaluate for associated urologic injury

Emergent transport to OR for pelvic anterior external fixator placement

CT scan to assess for occult femoral neck fracture

Bedside posterior pelvic C-clamp application Correct answer: 1

Unstable anteroposterior compression (APC) pelvic fractures are most appropriately managed with a pelvic binder or circumferential pelvic sheeting as described by Routt et al in the emergency room prior to definitive treatment. Illustration A demonstrates the utility of circumferential wrapping for the case shown in Figure A. Rapid, temporary fixation of unstable pelvic fracture patients with hemodynamic instability can be performed in the trauma bay. Pelvic binders can remain in place during further diagnostic tests such as

pelvic vessel angiography.

 

Bottlang et al performed a cadaveric study in JBJS of Young-Burgess type-II and III anteroposterior compression fractures and found that a pelvic binder reduced rotation instability by 61%.

 

The study by Krieg et al followed 16 patients treated with pelvic binders and found that the binder reduced the pelvic fracture displacement by 9% which closely approximated the reduction achieved with definitive fixation.

 

The Bottlang article published in JOT is a cadaveric study which determined that 180 +/- 50 Newtons of circumferential compression is needed to stabilize an unstable pubis symphysis diastasis. Tile Type A pelvic fractures are stable and include avulsion, iliac-wing, anterior-arch fracture due to a direct blow, or transverse sacrococcygeal fractures.

 

 

 

 

 

Excision of the 1 centimeter talar fragment shown in Figure A would lead to complete incompetence of which of the following structures?

 

Bifurcate ligament

Inferior peroneal retinaculum

Lateral talocalcaneal ligament

Arcuate ligament

Posterior talofibular ligament Correct answer: 3

Figure A shows a lateral process talar fracture. Excision of a fragment of approximately 1 cubic centimeter is reported for acute injury as well as painful nonunion/fibrous union. Excision of this piece is shown to sacrifice the entire lateral talocalcaneal ligament as well as 10% of the anterior and posterior talofibular ligaments.

 

Langer et al performed a cadaveric biomechanical analysis of such an excision and found that ankle and subtalar instability were not created with simple excision of this fragment.

 

DiGiovanni et al found that only three ligaments attach to the lateral process of the talus: lateral talocalcaneal, anterior talofibular, and posterior talofibular, with the lateral talocalcaneal ligament inserting closest to the tip/apex of the lateral process.

 

Illustration A shows an anatomical drawing of the ligaments of the lateral ankle. Illustration B shows a cadaveric image of the ligaments of the lateral ankle. The numbers in Illustration B correspond to the following structures: 1-calcaneofibular ligament, 2-lateral talocalcaneal ligament, 3-anterior talofibular ligament, 4-peroneal tubercle.

 

 

 

 

 

Isolated exchange reamed interlocking nailing is most likely indicated as the next step in treatment for which of the following clinical scenarios:

Tibial shaft nonunion with a 4cm bone defect

Infected tibial shaft nonunion

Hypertrophic diaphyseal tibial nonunion

Atrophic tibial shaft nonunion

Hypertrophic metadiaphyseal distal tibia nonunion Correct answer: 3

If a hypertrophic nonunion is present, it is most likley a mechanical issue. Tibial diaphyseal hypertrophic nonunions (Illustration A) have approximately an 85-90% incidence of union with exchange reamed nailing. A nonunion that has bone loss or appears atrophic (Illustration B) will usually require improved mechanical stability as well as biological stimulation in the form of either autograft or an osteoinductive substance like BMP. A bone defect of up to 5-6cm in length can usually achieve union with bone grafting. In the presence of an infected nonunion, the infectious process needs to be addressed prior to the introduction of any revision hardware. If a patient does not show radiographic signs of tibial fracture union for 9 months and does not have progression toward healing for 3 consecutive months, then revision surgery would be indicated.

 

Tempelman et al looked at 71 tibial shaft fractures treated with nonlocked or dynamically locked IM nails and found a loss of alignment in 11% of the fractures that were not transverse in nature. They concluded that these nailing techniques should not be used in the treatment of spiral or oblique tibial shaft fractures.

 

Incorrect Answers:

1-A 4cm bone defect could not be corrected with exchanged nailing alone, and would either need extensive grafting or bone transport

2-An infected tibial shaft nonunion would require infection clearance prior to exchanged nailing

4-Atrophic nonunions typically need biologic stimulation in the form of acute grafting or insertion of a BMP type substance

5-Hypertrophic metadiaphyseal distal tibial nonunions can be treated with isolated exchanged nailing, however this does not have the same success as diaphyseal injuries. It is difficult to acheive appropriate stability to allow for fracture healing in the metadiaphyseal region, and other modalities such as plating need to be considered.

 

 

 

 

A 58-year-old man injures his knee in a high-speed motor vehicle collision. Radiographs and CT are shown in Figures A thru C. What is the most appropriate surgical plan based on the images provided?

 

 

ORIF with medial and lateral plating with grafting of metaphyseal defect

ORIF with lateral plating with grafting of metaphyseal defect

ORIF with medial plating

ORIF with lateral plating

Percutaneous articular fragment reduction and screw fixation Correct answer: 2

The injury is a Schatzker type 2 with a significant split and depressed lateral tibial plateau fracture. The anteromedial cortex and medial plateau remain intact and connected to the tibial shaft making this a partial articular fracture (AO type B) and obviating the need for medial plate fixation. The best fixation strategy includes reduction of the articular surface with metaphyseal support with bone graft or bone substitute and a lateral plate for buttress support and subchondral screw support of the articular fragment.

Marsh et al revisited the OTA and AO fracture and dislocation classifications to unify coding of these fractures. Illustration A shows the Schatzker classification of tibial plateau fractures.

 

 

 

 

 

Anterior penetration of an iliosacral screw through the sacral ala would most likely lead to weakness of which of the following movements?

Hip flexion

Hip adduction

Knee extension

Ankle plantarflexion

Great toe dorsiflexion Correct answer: 5

Penetration of an iliosacral screw through the sacral ala would injure the ipsilateral L5 nerve root (great toe dorsiflexion). This can be avoided with proper understanding of the sacral anatomy as well as iliosacral screw starting points. The three required views for placement of this screw are: lateral sacral, pelvic inlet, and pelvic outlet.

 

The referenced study by Ziran et al is an excellent review of fluoroscopic evaluation of screw placement. They reported that the anterior border of the S1 body is best seen with overlap of the S1 and S2 anterior cortex while the superior aspect of the S1 foramen is best seen with overlap of the S2 foramen on the superior pubic ramus.

 

The referenced study by Routt et al reviewed 177 patients with pelvic ring

injuries treated with these screws and found that quality triplanar imaging decreased intraoperative and postoperative complications. They also recommend supplemental fixation of iliosacral screws with posterior plating in noncompliant patients.

 

 

 

 

A 78-year-old male undergoes the procedure shown in Figure A for treatment of a femoral neck fracture. As the patient passes through mid-rise during sitting to standing using the affected leg, what portion of the acetabulum experiences the highest contact pressures?

 

 

Posterior inferior

Anterior superior

Posterior superior

Directly superior

Anterior inferior

 

Correct answer: 3

 

Figure A is an AP radiograph of a hip hemiarthroplasty.

 

Contact pressures from an insturmented hip endoprosthesis can have important implications in both implant positioning and rehabilitation protocols.

Hodge et al implanted a pressure-measuring Moore-type endoprosthesis in a patient who had sustained a displaced fracture of the femoral neck. They used this prosthesis to determine the measurement and telemetry of contact pressures in the hip for 36 months post-operatively. The highest pressure, eighteen megapascals, was recorded while the patient was getting up from a chair using the affected leg and was localized in the posterior superior portion of the acetabulum. This can be important in the post-operative care of acetabular fractures, as patients are at increased risk of loss of fixation with greater acetabular contact forces. Interestingly, peak pressures in vivo were found to be considerably higher than previously measured pressures in vitro.

 

Incorrect Answers:

1,2,4,5,-These regions of the acetabulum have less contact pressure compared to the posterior superior portion.

 

 

 

 

For an above knee amputation, each of the following is a benefit of adductor myodesis EXCEPT:

Allows preservation of greater femoral length

Provides a soft tissue cushion beneath the osseous amputation

Improves the position of the femur to allow more efficient ambulation

Creates dynamic balance of the amputated femur

Improves prosthetic fit Correct answer: 1

Adductor myodesis enhances clinical function following an above knee amputation. An example is provided in illustration A. Adductor myodesis balances the pull of the hip abductors and flexors that insert proximally This prevents an abducted and flexed deformity that encumbers ambulation. It also creates a soft tissue envelope that pads the distal bony amputation and enhances suction fitting of the prosthesis. Performing the myodesis does not preserve femoral length. A little extra distal femur is actually resected to facilitate the myodesis insertion.

 

In an instructional course lecture, Pinzur et al review the goals and techniques of lower extremity amputations. To achieve a successful amputation, the surgeon must establish reasonable goals with the patient, manage the soft tissue envelope, create an osseous platform for weight-bearing, and facilitate physical rehabilitation.

 

 

 

 

A 21-year-old male sustains the injury shown in Figures A through D. Which of the following is the most appropriate definitive treatment of this injury?

 

 

Spanning external fixation

Lateral locking plate

Lateral buttress plate

Posteromedial buttress plate

Medial bridging plate Correct answer: 4

Figures A through D show a medial tibial plateau fx, best classified as a Schatzker IV. This injury is high-energy in nature and often requires a staged approach incorporating initial reduction and spanning external fixation.

 

Appropriate treatment of this injury involves a medial buttress plate to hold the medial tibial condyle in position. This fracture pattern can be associated with a knee dislocation (see Illustration A). In a fracture-dislocation, the femur often displaces with this medial condylar fragment and is involved with a significant rate of neurovascular injury/compartment syndrome. Along with a proper vascular exam, ankle brachial indices (ABI) must be immediately taken and if abnormal, further vascular testing is warranted.

 

 

 

 

Lipohemarthrosis of the knee is most likely secondary to which of the following?

Seronegative monoarticular arthritis

Patellar tendon rupture

Medial meniscus tear

Medial patellofemoral ligament rupture

Occult fracture

 

Correct answer: 5

 

Lipohemarthrosis is formed when an intraarticular fracture occurs and can be detected with arthrocentesis or imaging such as xray, MRI, ultrasound, or CT. It is most commonly seen with occult tibial plateau fractures but can be associated with any intra-articular fractures. Up to three layers are visible on an MRI (fat/serum/cellular parts of blood), and this separation may take up to 3 hours to appear after injury. An example of hemarthrosis as seen on CT is shown in Illustration A. The referenced article by Ahn et al is a review of MRI findings in intraarticular knee injuries. They note that detection of lipohemarthrosis on an MRI is very sensitive and specific for intraarticular fracture. The referenced article by Schick et al reports that MRI can be as sensitive as arthrocentesis in detecting lipohemarthrosis (occult fracture).

 

 

 

 

The Lauge-Hansen classification of ankle fractures identifies characteristic fracture patterns based on mechanism of injury. What is the mechanism for the fracture pattern shown in Figure A?

 

 

Supination-External Rotation

Pronation-External Rotation

Pronation-Abduction

Supination-Adduction

Supination-Abduction

 

Correct answer: 4

 

The 4 categories in the Lauge-Hansen classification are: supination-adduction, supination-external rotation, pronation-external rotation, and pronation-abduction.

 

The typical fracture pattern of supination-adduction is demonstrated in figure A and consists of a tension failure (transverse) fracture of the lateral malleolus combined with a vertical fracture of the medial distal tibia. There is frequently comminution where the fracture begins on the tibial plafond. This is caused by compression from the medial talar dome.

 

 

 

 

A 35-year-old male sustains a posterior column/posterior wall acetabular fracture. Which of the following is the preferred approach for open treatment of this injury?

Modified Stoppa approach

Extended iliofemoral approach

Kocher-Langenbeck approach

Ilioinguinal approach

Combined anterior and posterior approach Correct answer: 3

Operative treatment is indicated for most displaced acetabular fractures to allow early ambulatory function and to decrease the chance of post-traumatic arthritis. Among the various surgical approaches, the Kocher-Langenbeck allows direct exposure of both the posterior column and posterior wall.

Indications for using this exposure include posterior wall fractures, posterior column fractures, combined posterior wall/posterior column fractures, and simple transverse fractures.

 

 

 

 

Excision of heterotopic bone about the forearm or elbow can be done with limited recurrence rates as early as which of the following after initial injury?

Once ankylosis of the forearm or elbow occurs

6 weeks

6 months

12 months

18 months

 

Correct answer: 3

 

Excision of heterotopic bone about the elbow and forearm was classically treated once the bone was mature and no further bone development was occuring (bone scan became negative). However, several studies have shown that earlier removal before this point in time is safe, when done in conjunction with radiation therapy (XRT).

 

The referenced study by McAuliffe et al is a retrospective review of heterotopic ossification (HO) about the elbow followed by 1000 cGY (5 fractions over 1 week) of XRT as early as 3 months post-injury. They were able to achieve an average arc of motion > 100 degrees.

 

The other referenced study by Beingessner et al is a review of HO excision of the forearm. They found that excision and XRT, followed by 6 weeks of indomethacin, led to an increase of forearm motion from an average of 17 degrees to 136 degrees when the excision was done at 4 months post-injury.

 

 

 

 

When placing a percutaneous retrograde pubic ramus screw for fixation of an acetabular fracture, which of the following radiographic views can best ensure that the screw does not exit the posterior aspect of the superior pubic ramus?

AP pelvis

Outlet obturator oblique view

Inlet iliac oblique view

Iliac oblique view

Obturator oblique view Correct answer: 3

As reviewed in the referenced article by Starr et al, when placing a retrograde pubic rami screw, the pelvic inlet iliac oblique view will best determine the anteroposterior placement of the screw in the pubic ramus. To ensure placement outside of the joint, the outlet obturator oblique is best, but all other views should be incorporated into determination of the position of

fixation, as the corridor for this screw placement is quite narrow. Illustration A shows a left sided inlet iliac view on a pelvic bone model.

 

 

 

 

A 25-year-old male is involved in a motor vehicle accident and sustains the injury seen in Figure A. He is intubated in the field and receives 2 liters of LR and continues to be tachycardic and hypotensive. A massive transfusion protocol is initiated. Which of the following is true regarding the transfusion of packed red blood cells, platelets, and fresh frozen plasma?

 

PRBC should be transfused until Hgb>8

PRBC, platelets, and FFP should be transfused in equal ratios

Platelets and fresh frozen plasma should be given when INR >1.4, platelet count <100,000

FFP is not needed unless INR>1.5

Platelets should not be transfused unless platelet count <10,000 Correct answer: 2

High energy pelvic injuries such as the one seen in Figure A continues to be a source of high mortality in orthopaedics. Active involvment of the orthpaedic surgeon in managing these life threatening injuries remains critical. A sheet or pelvic binder needs to be emergently applied in this clinical scenario. An aggressive resuscitation protocol must also be initiated. The review article by Hak et al discussed the advances in prehospital, interventional, surgical, and critical care that have led to increase survival rates for pelvic injuries. Gonzalez et al found that initial coagulopathy in trauma patients was associated with decreased survival. They noted that hypothermia and acidosis was well managed but pre-ICU coagulopathy was the most difficult to treat. They recommended early FFP in a FFP:PRBC ratio of 1:1

 

 

 

 

A 48-year-old female sustains the injury seen in Figure A. Which of the following preoperative variables has been shown to be associated with improved outcomes following surgical treatment of this injury pattern?

 

Patients with a heavier workload

Patients receiving Worker's Compensation

Gissane angle of 140°

Böhler angle of > 15°

Comminuted posterior facet Correct answer: 4

Figure A shows a displaced intra-articular calcaneus fracture. Studies have found in this fracture pattern a Böhler angle of greater than 0-15° is associated with improved outcomes.

 

Buckley et al (2002) found that patients with the following preoperative variables have improved outcomes with ORIF vs non-operative treatment: women, non-workers' compensation, younger males (<30), patients with a higher Böhler angle (>0-15°), patients with a lighter workload, and fracture patterns with single, simple displaced intra-articular calcaneal fracture.

 

Buckley et al (1992) found that anatomic or near anatomic reductions (<2mm stepoff) enhance outcomes while comminuted reductions or fractures without reduction produce long-term outcomes that are less satisfactory.

 

Bajammal et al investigated intra-articular calcaneus fractures and reported

that patients who were NOT receiving Workers' Compensation, were younger (less than twenty-nine years old), had a moderately lower Böhler angle (0 degrees to 14 degrees ), a comminuted fracture, a light workload, or an anatomic reduction or a step-off of < or =2 mm after surgical reduction (p = 0.04) scored significantly higher on the scoring scales after surgery compared with those who were treated nonoperatively.

 

Illustration A shows the Böhler angle, which is an angle between the following two lines: 1) a line connecting anterior process and highest point on posterior articular surface, 2) a line connecting highest point on posterior articular surface and superior tuberosity. A normal Böhler angle is 20-40°. Flattening of the Böhler angle represents collapse of the posterior facet. Illustration B shows both the critical angle of Gissane (G) and the Boehler angle (B).

 

Incorrect Answers:

Answer 1. Patients with heavier workloads have worse outcomes with surgery. Answer 2. Patients with Worker's Compensation claims have worse outcomes with surgery.

Answer 3. A normal angle of Gissane does not necessarily indicate posterior facet depression, and no improvements with surgical treatment have been shown.

Answer 5. A comminuted posterior facet is associated with worse outcomes.

 

 

 

 

 

A 26-year-old male sustains a comminuted, intra-articular calcaneus fracture and subsequently undergoes operative intervention as shown in Figure A. Postoperatively in the recovery room, he presents with an isolated, fixed flexed great toe. What is the most likely etiology of this finding?

 

Use of a lateral extensile approach to the calcaneus

Calcaneal tuberosity varus malalignment

Use of screws in the constant fragment that are too long

Missed foot compartment syndrome

Plantar nerve palsy Correct answer: 3

The flexor hallucis longus runs along the medial aspect of the hindfoot, medial to the posterior facet and inferior to middle facet just under sustentaculum tali(constant fragment) This tendon can be injured with poor drilling technique or use of screws that are too long during reduction and fixation of a calcaneus fracture using a lateral plate. If present, the great toe will be in a fixed, flexed position from the tendon being tethered over a screw.

 

Figure A shows a calcaneus fracture with a lateral plate and screws.

 

 

 

 

All of the following are characteristic of end-stage septic shock EXCEPT?

Increased systemic vascular resistance

Decreased cardiac output

Decreased pulmonary capillary wedge pressure

Decreased central venous pressure

Increased mixed venous oxygen saturation Correct answer: 1

Septic shock is associated with decreased systemic vascular resistance, decreased cardiac output, decreased pulmonary capillary wedge pressure, decreased central venous pressure, and increased mixed venous oxygen saturation. Septic shock is different from hypovolemic shock in that the systemic vascular resistance is decreased, whereas it is increased in hypovolemic shock. Septic shock is a medical emergency caused by decreased tissue perfusion and oxygen delivery as a result of severe infection and sepsis, though the microbe may be systemic or localized to a particular site. It can cause multiple organ dysfunction syndrome (formerly known as multiple organ failure) and death. Its most common victims are children, immunocompromised individuals, and the elderly, as their immune systems cannot deal with the infection as effectively as those of healthy adults. The mortality rate from septic shock is reported to be as high as 25%-50%.

 

 

 

 

A 27-year-old female sustains a twisting injury to her leg while rollerblading. Radiographs of the tibia and fibula are provide in Figures A and B. A closed reduction is performed and the patient is placed in a long leg cast. Radiographs following cast placement are provided in Figures C and D. The decision is made to proceed with closed treatment instead of operative. Which of the following is most likely to occur with nonoperative management?

 

 

Malunion due to unacceptable coronal alignment

Malunion due to unacceptable sagittal alignment

Fracture displacement due to the mechanism of injury

Fracture displacement due to the age of the patient

Shortening due to the oblique nature of the tibia fracture Correct answer: 5

The radiographs demonstrate a distal third spiral tibia shaft with a proximal fibula fracture. The coronal and sagittal alignments are within acceptable limits. The oblique fracture is at risk of shortening, especially with a concomitant fibular fracture.

 

Acceptable alignment for non-operative treatment of tibia fractures is defined as:

<5 degrees varus-valgus angulation,

<10 degrees anterior/posterior angulation

>50% cortical apposition

<1 cm shortening

< 10 degrees rotational alignment

 

Sarmiento et al. reviews fracture bracing for the treatment of long bones. With reference to tibial fractures, bracing is best for transverse fractures.

Shortening is difficult to control in oblique fractures. However, shortening is

usually less than 15 mm and does not result in functional limitations. He reports union in 97% of tibial fractures treated with bracing.

 

One year follow-up radiographs are provided in Illustrations A and B. The patient presented in this question went on to functional healing.

 

 

 

 

 

A lateral distal femoral locking plate is not an appropriate implant for which of the following fractures?

 

 

 

Figure A

Figure B

Figure C

Figure D

Figure E

 

Correct answer: 2

 

Figure B demonstrates an isolated medial femoral condyle fracture. Lateral locked plating is not an appropriate technique for this fracture.

 

The fracture shown in Figure B is an AO B type (partial articular fracture). This fracture is best treated with open reduction internal fixation through a medial approach, with lag screw and buttress plate fixation.

 

Figures A, C, D and E show supracondylar distal femur fractures that can be treated with ORIF with a fixed-angle device such as lateral locked plating.

 

Incorrect Answers:

Answer 1. Fixed angle fixation is appropriate for comminuted extra-articular distal femur fractures.

Answers 3, 4, 5. ixed angle fixation is appropriate for comminuted intraarticular distal femur fractures. Non-locked plating for type C (complete articular) distal femur fractures has been associated with varus malalignment.

 

 

 

A 70-year-old female with a history of poorly controlled diabetes mellitus presents with purulent ulcers along the plantar aspect of her right forefoot and exposed metatarsal bone. She elects to undergo an amputation. She is insensate to the midfoot bilaterally. Her ankle-brachial index (ABI) for her right posterior tibial artery is 0.4. Further preoperative evaluation demonstrates a transcutaneous oxygen pressure of 45 and an albumin of 3.4. Which of the following would be a contraindication to performing a Syme amputation (ankle disarticulation) in this patient?

Albumin of 3.4

Active osteomyelitis

ABI of 0.4 for the posterior tibial artery

Transcutaneous oxygen pressure of 45

Peripheral neuropathy

 

Correct answer: 3

 

A Syme amputation (ankle disarticulation) is a function-preserving amputation option that allows for terminal weight bearing, however strict criteria must be met for a patient to undergo successful Syme amputation. An ankle-brachial index (ABI) less than 0.5 for the posterior tibial artery in a patient with diabetes would be a contraindication for this procedure as success is dependent on the vascular supply of posterior tibial artery to the plantar flap and heel pad.

 

Pinzur et al retrospectively reviewed their results when performing a single-stage Syme ankle disarticulation in patients with diabetes either for peripheral neuropathy or infection. Patients with ABIs less than 0.5 for the posterior tibial artery had significantly decreased healing rates and smokers had a three-fold increased risk of postoperative infection.

 

Incorrect Answers:

Answer 1: Albumin of 3.4 indicates adequate preoperative nutrition. Albumin of

2.0 or less would be concerning for increased wound healing risk.

Answer 2: Osteomyelitis was the indication for amputation in a number of diabetic patients in the referenced study, and should not preclude Syme amputation unless the hindfoot and/or distal tibia is involved

Answer 4: Transcutaneous 02 pressure of 45 signifies adequate oxygenation and vascularity. Less than 30 is generally considered a cutoff for adequate vascularity.

Answer 5: Peripheral neuropathy with recurrent ulceration was an indication for amputation in the referenced study.

 

 

 

 

A 35-year-old-male sustains the fracture seen in Figure A. Which of the following reduction forces must be applied to the proximal fragment to correct the deformity commonly seen in these fractures?

 

 

Adduction and extension

Abduction and extension

Adduction and flexion

Abduction and flexion

External rotation

 

Correct answer: 1

 

Figure A demonstrates a displaced subtrochanteric femur fracture with an intact lesser trochanter. The pull of iliopsoas on the lesser trochanter as well as the intact external rotators and gluteal musculature results in the the proximal fragment being in a flexed and externally rotated or abducted position (the most common post operative deformity). Reduction manuevers must be biologically friendly but also counteract the flexion/abduction moment. Lundy's review article discusses evaluation and treatment of subtrochanteric fractures. The review article details the various implants often used which include 95 degrees plates, femoral reconstruction nails, or trochanteric femoral nails with interlocking options. Lundy's article discourages the use of the 135 degree screw and side plate combo due to high failure rates in these fracture patterns. Bedi et al also review treatment of these fractures and discuss common

problems of malunion, nonunion, and implant failure. The article reviews reduction techniques that are soft tissue friendly, as well as the use of appropriate implants in these fracture types.

 

 

 

 

A 12-year-old male sustains an ulnar fracture with an associated posterior-lateral radial head dislocation. After undergoing closed reduction, the radiocapitellar joint is noted to remain non-concentric. What is the most likely finding?

Lateral ulnar collateral ligament disruption

Anterior band of the medial collateral disruption

Posterior band of the medial collateral ligament disruption

Annular ligament interposition

Anconeus muscle interposition Correct answer: 4

In pediatric Monteggia fractures the annular ligament is commonly interposed in the radiocapitellar joint.

 

Bado initially described and classified Monteggia fractures. The most common injury pattern is an extension type 1 with anterior radial head dislocation and apex anterior ulnar shaft fracture. The apex of the ulna fracture determines the direction of the radial head subluxation or dislocation. Adults typically require ORIF of the ulna. These fractures in children are often treated non-operatively with closed reduction if the ulna fracture is transverse and stable. Type III is the one most commonly associated with irreducibility of the radial head because of interposition of the annular ligament. The incidence of posterior interosseous nerve injury is high with this lesion. The nerve deficit usually completely resolves rapidly and spontaneously.

 

Tan et al reviewed their treatment of 35 children with Type I and Type III Monteggia fractures. All radial heads were explored and the interposed annular ligament was stretched out of the joint space. They noted that none of the patients had any recurrent dislocation or subluxation.

 

Ring et al in their review stress the importance of an anatomic reduction of the ulna to restore the reduction of the radial head.

 

 

 

All of the following are factors associated with transfer of patients to Level 1 trauma centers EXCEPT:

Male

Medicaid insurance

Injury severity score of 36

Caucasian race

One or more comorbidity Correct answer: 4

Caucasian race has not been found to be a predictor for transfer to a Level 1 trauma center.

 

The retrospective case-control study by Koval et al found that African-American race, presence of medical comorbidity, medicaid insurance, and male gender are predictors for transfer of patients to a trauma center that have ISS scores less than 9.

 

The article by Nathens et al found that lack of insurance was an independent predictor for transfer to a trauma center after adjusting for differences in injury severity. An injury severity score of 36 represents a patient that has sustained life-threatening polytrauma and should be transferred to a Level 1 trauma center.

 

 

 

 

A 34-year-old male falls from a roof and sustains a right elbow dislocation that is closed reduced in the emergency room. An AP radiograph is shown in Figure A. This injury pattern is at highest risk for which of the following?

 

Anterior interosseous nerve palsy

Varus posteromedial rotatory instability

Posterior interosseous nerve palsy

Valgus posterolateral rotatory instability

Elbow instability when pushing oneself up from a seated position in a chair Correct answer: 2

Anteromedial coronoid facet fracture and LCL injury following an elbow dislocation is commonly associated with varus posteromedial rotatory instability. Varus and posteromedial rotation force on the forearm results in rupture of the LCL from its humeral origin. As the LCL ruptures, the medial coronoid process is fractured as it impacts against and under the medial trochlea. Fracture involvement of the sublime tubercle, where the MCL attaches, can lead to more instability. Ulnar neuropathy can be seen following this injury pattern but AIN and PIN nerve palsy do not commonly characterize this injury pattern.

 

The review article by O'Driscoll highlights key points in diagnosis and management of capitellum, distal humerus, coronoid, and terrible triad injuries.

 

The article by Doornberg and Ring is a Level 4 study of 18 patients that sustained varus posteromedial rotational injuries resulting in anteromedial facet coronoid fractures. They found that lack of fixation at injury or malunion of the anteromedial facet were significant predictors of suboptimal functional outcome and development of arthrosis.

 

The anteromedial facet is highlighted in yellow as displayed in Illustration A. Illustration B depicts the lateral collateral ligament injury also evident during

varus stress fluroscopic examination, due to tension failure of the LCL off its humeral origin during the various mechanism.

 

 

 

 

 

 

Which of the following factors is most associated with malrotation during antegrade or retrograde femoral nailing?

Surgeon experience

Level of primary fracture line

Use of a piriformis starting portal

Fracture comminution

Closed reduction technique Correct answer: 4

Femoral malrotation after intramedullary nailing is unfortunately a possibility with either antegrade or retrograde nailing techniques. Malrotation and iatrogenic length changes are most common when comminution is present, as cortical reads are inherently limited.

Hufner et al report that malrotation (internal or external >15 degrees) was seen in 22% of their patients via CT scan after intramedullary nailing. There was a significant difference depending on the time of surgery, with significantly more malrotation during the night shift. Increased fracture comminution also significantly increased malrotation rates.

 

Incorrect Answers:

1,2,3,5: No significant increases were seen with the other answers listed above.

 

 

 

 

What is the most common type of malalignment after intramedullary nailing of distal 1/3 extra-articular tibia fractures using a infrapatellar approach when compared with plating?

Varus

Valgus

Translational

Shortening

Apex anterior

 

Correct answer: 2

 

The most common type of malalignment after intramedullary nailing of distal 1/3 extra-articular tibia fractures using an infrapatellar approach when compared with plating is valgus malalignment.

 

Fixation of distal one-third tibial shaft fractures can be successfully treated with either intramedullary nailing or plating. The literature describes advantages and disadvantages to both approaches, however intramedullary nailing has been shown to lead to increased rates of valgus malunion. Recent studies have shown that using a suprapatellar approach may decrease the incidence of valgus malalignement.

 

Vallier et al performed a randomized prospective study to compare plate and nail stabilization for distal tibia shaft fractures by assessing complications and secondary procedures. One-hundred and four patients were randomized to either reamed intramedullary nailing, or medial distal tibia plate fixation.

Primary angular malalignment was identified in 17 patients (16.3%). This included four patients treated with tibial plating (8.3%) and 13 patients treated with nails (23%, P = 0.02). Eight of these (7.7% of all patients) had malalignment between 6° and 10° of angulation. Valgus was the most common

angular deformity, accounting for 70% of angular deformity cases.

 

Avilucea et al. looked at the immediate postoperative alignment of distal tibia fractures (within 5 cm of the tibial plafond) treated with suprapatellar intramedullary nail (IMN) insertion compared with the infrapatellar technique. They found primary angular malalignment of ≥5 degrees occurred in 35 (26.1%) patients with infrapatellar IMN insertion and in 5 (3.8%) patients who underwent suprapatellar IMN insertion. They conclude suprapatellar IMN technique results in a significantly lower rate of malalignment compared with the infrapatellar IMN technique.

 

Incorrect Answers:

Answer 1, 3,4, and 5: Vallier et al. found that valgus was the most common angular deformity.

 

 

 

 

A 33-year-old secretary presents three months after a motor vehicle collision with a mild asymmetry to her sternal area and difficulty swallowing. She denies any complaints of respiratory distress or upper extremity paresthesias. Her upper extremity neurovascular exam shows no deficits. A 3-D computed tomography image is shown in Figure A. What is the most appropriate treatment for this patient?

 

 

Nonoperative treatment with a sling and unrestricted activity in 3 months

Open reduction in the operating room with thoracic surgery back-up

Closed reduction in the office with local anesthetic

Closed reduction in the operating room with thoracic surgery back-up

Nonoperative treatment with immediate unrestricted active range of motion of the shoulder

Correct answer: 2

 

The clinical presentation is consistent with a chronic sternoclavicular dislocation, which is defined as being greater than 3 weeks old. The 3D CT image shows posterior displacement of the medial clavicle relative to the sternum. Chronic anterior dislocations are recommended to be treated conservatively, especially if not symptomatic, but as this is a posterior dislocations, current recommendations are to treat them with reduction in order to avoid delayed issues with the medial clavicle interacting with the mediastinal structures.

 

The review article by Wirth and Rockwood notes the following complications with posterior dislocation: respiratory distress, venous congestion or arterial insufficiency, brachial plexus compression, and myocardial conduction abnormalities. They recommend reconstruction of the costoclavicular ligaments with resection of the medial claviclar head as needed for unstable or symptomatic injuries.

 

 

 

 

A 35 year-old female presents after prolonged extrication from a motor vehicle collision complaining of severe pelvic pain. Physical examination reveals diminished perianal sensation. She is otherwise neurologically intact. Figures A through D are radiographs and representative CT cuts of her injury. Which of the following nerve roots has likely been injured by the acute trauma?

 

 

 

L3

L4

L5

S1

S2

 

Correct answer: 5

 

The clinical scenario is consistent with a high-energy sacral fracture. The radiographs in figures A and B demonstrate a sacral fracture with posterior displacement of the right hemipelvis seen on the inlet view. Figures C and D are axial and sagittal CT images which show a displaced fracture of the right

hemisacrum along with a transvere fracture component through the S3 body . Diminished perianal sensation is concerning for an S2 nerve root injury.

 

Mehta et al reviewed the current management of sacral fractures. They note that the S1 and S2 nerve roots are more likely to be injured with sacral fractures as they occupy 1/3 to 1/4 of the neural foramina, as opposed to S3 and S4, which only occupy 1/6 of the neural foramina.

 

Robles reviewed the current literature to ascertain principles of evaluation and treatment for transverse sacral fractures. The author notes that injury to nerve roots S2 to S5 is manifested by impairment of urinary and anal continence and sexual function.

 

The first illustration demonstrates the sacral nerve root dermatomal distribution. The second shows a pelvic cadaver dissection demonstrating the sacral nerve roots as they exit the foramina.

 

 

 

 

 

A 44-year-old male is struck by a vehicle while riding his bike. In the trauma bay, he complains of right shoulder pain . Upper extremity physical exam reveals no neurologic deficits, and an initial radiograph of the shoulder is shown in Figure A. A CT scan of the shoulder shows 1cm of posterior displacement of the tuberosity fragment. Which of the following is true regarding this injury?

 

It is usually associated with a posterior shoulder dislocation

The subscapularis muscle is the main deforming force

Non-operative treatment of this displaced injury results in good long term shoulder function

Open reduction and internal fixation is the treatment of choice

Associated rotator cuff tears are uncommon Correct answer: 4

The radiograph in Figure A demonstrates a posteriorly displaced greater tuberosity fracture. These injuries are often associated with anterior shoulder dislocations, and concomitant rotator cuff tears. The subscapularis attaches to the lesser tuberosity, and is not a deforming force. Open reduction and internal fixation (ORIF) is usually the treatment of choice, and it is well accepted that more than 5mm of displacement is an indication for surgery in patients that require overhead function of the arm.

 

Flatow et al evaluated 12 patients who were an average of five years status post ORIF of displaced greater tuberosity fractures. All fractures healed without postoperative displacement. Six patients had an excellent result and six had a good result.

 

Platzer et al retrospectively analyzed functional and radiographic results of 52 patients with operative treatment of displaced greater tuberosity fractures at an average time of 5.5 years from surgery compared to 9 patients with equivalent injuries treated non-operatively. Evaluation of the results of the surgical study group and the nonoperative control group, patients with reduction and fixat ion of greater tuberosity fractures had significantly better

results on shoulder function than did those with conservative treatment.

 

 

 

 

 

Which of the following is a recognized predictor of mortality after hip fracture?

American Society of Anesthesiologist (ASA) classification

Post-operative weight bearing status

Fracture comminution

Fixation device used

Type of anesthetic used Correct answer: 1

American Society of Anesthesiologist (ASA) classification is predictive of post-surgical mortality in hip fracture patients.

 

The ASA classification (detailed in Illustration A) was initially developed in 1963 and has been shown to be predictive of post-surgical mortality in hip fracture patients. Basic categories are as follows: 1= normal, healthy; 2= mild systemic disease; 3= severe systemic disease, not incapacitating; 4= severe incapacitating systemic condition, constant threat to life; 5= moribund patient; 6 = brain dead, organs being donated.

 

Richmond et al. looked at 836 patients treated for a hip fracture and found that this injury is not associated with significant excess mortality among patients older than age 85. However, in younger patients, those with ASA classifications of 3 or 4 have significant excess mortality following hip fracture that persists up to 2 years after injury.

 

Holt et al. investigated the relationship between a number of patient and management variables and mortality after surgery for fracture of the hip. Data relating to 18,817 patients were obtained from the Scottish Hip Fracture Audit database. They found that type of anesthetic did not adversely affect the 30 or 120 day mortality rate.

 

 

 

 

A 25-year-old male sustains an ankle fracture dislocation and undergoes open reduction and internal fixation. He returns to clinic five months following surgery complaining of continued ankle pain and instability with weight bearing. His immediate post-operative AP radiograph is seen in Figure A. Which of the following could have prevented this patient from developing persistent pain?

 

Deep deltoid ligament repair

Quadricortical syndesmotic screw fixation

Restoration of fibular length and rotation

Lateral collateral ligament complex repair

Use of two syndesmotic screws Correct answer: 3

The patient presents with continued ankle pain and instability following open reduction and internal fixation. The radiograph in figure A demonstrates inadequate restoration of fibular length, likely leading to continued tibiotalar instability.

 

Illustration A demonstrates fibular malreduction with dislocation of the fibula anterior to the tibial incisura. Illustration B shows a comminuted fibula fracture along with a measurement of length from an intact fibula. The arc from the lateral process of the talus to the peroneal groove of the distal fibula is known as the "dime" sign and should remain unbroken if fibular length has been restored. Illustration C demonstrates the use of a push-pull screw and lamina spreader to regain length intraoperatively for a comminuted fibula fracture.

 

Chu and Weiner review management of malunions of the distal fibula. The authors state that restoration of fibular length, alignment and rotation leads to reduction of the talus, provides a buttress to talar motion in the setting of an incompetent deltoid, and allows the syndesmotic ligaments to heal at the appropriate tension.

 

Wikeroy et al conducted a study of patients from a prior prospective, randomized control trial comparing different methods of syndesmotic fixation. There was no significant difference in outcomes between tricortical or quadricortical 3.5mm screw fixation, however worse outcomes were seen with associated posterior malleolar fractures, obesity, a difference in sydesmotic width of 1.5mm or greater, and a CT confirmed tibio-fibular synostosis.

 

Sinha et al present a simple technique for fibular lengthening in the setting of distal fibula malunion. They found high union rates and improved AOFAS scores at short-term follow up with their technique.

 

 

 

 

 

 

A 34-year-old male presents with the right posterior wall acetabular fracture shown in Figure A. What is the most accurate method to test for hip stability in this patient?

 

The Keith method

The Moed method

The Calkins method

Dynamic fluoroscopic examination of the hip under anesthesia

A history of associated hip dislocation Correct answer: 4

Dynamic fluoroscopic examination of the affected hip under anesthesia is considered the best method of predicting hip stability. Fragment size, which can be calculated using the Keith, Moed, or Calkins method, can be used to predict hip stability radiographically, however they are not as accurate. In general it is thought that posterior wall fractures involving less than 20% of the posterior wall are stable, whereas those involving greater than 40%-50% are unstable. Unfortunately, this leaves an indeterminent zone (20-40%) which does not provide guidance in treatment.

 

Moed et al retrospectively reviewed 33 patients with posterior wall fractures who underwent dynamic fluoroscopic stress testing and compared the results of this testing to the Moed, Calkins, and Keith method of hip stability prediction. They found that the Moed method is the only reliable technique that is predictive of hip stability for small fracture fragments while also being predictive of instability for large fracture fragments. However, they also stated that there remain a substantial number of fractures involving 20% or more of the posterior wall that are both stable and unstable by examination under anesthesia. Therefore, they recommend dynamic fluoroscopic examination for assessment of hip stability in the presence of a posterior wall fracture.

 

Moed et al in their second paper reviewed all patients with less than a ≤50% of the acetabular wall fracture, adequate imaging, and documented EUA results.

The group looked at multiple variables including fracture fragment size,

superior exit point of the fracture, center-edge angle, acetabular index, Tönnis angle, lateralized head sign, crossover sign, posterior wall sign, ischial spine sign, and hip version. Their conclusion was that no one variable was able to predict stability or instability and they continue to recommend EUA.

 

Tornetta et al conducted a study in which dynamic fluoroscopic stress views were taken of 41 acetabular fractures that met the criteria for non-operative management to determine subtle signs of instability. Of the 41 fractures, 38 were found to be stable and 91% of these had good or excellent outcomes at

2.7 years. They concluded that dynamic stress views can identify subtle instability in patients who would normally be considered for non-operative treatment.

 

incorrect answers:

> Keith Method - Depth of the fracture segment in injured hip is compared to the contralateral intact posterior wall depth at the level of the fovea

 

> Moed - Depth of the fracture segment in the injured hip is compared to contralateral posterior wall depth at the level of the greatest amount of fracture involvement

 

> Calkins - Length of posterior acetabular arc from each hip is compared at the level of the greatest amount of fracture involvement.

 

 

 

 

A 23-year-old male is an unrestrained driver in a motor vehicle accident and sustains an unstable pelvic ring fracture. During fluoroscopic-aided fixation, a lateral sacral view is used for proper placement of which of the following fixation methods?

Anterior column percutaneous screw placement

Posterior column percutaneous screw placement

Posterior iliosacral plating

Supra-acetabular pin placement

Percutaneous iliosacral screw placement Correct answer: 5

The lateral sacral view is used to place percutaneous iliosacral screws. Sacral alar morphology has been shown to be variable from patient to patient.

Therefore, intraoperative fluoroscopy is recommended. During placement of the screws, the L5 nerve root is at risk.

Routt et al (1997) examined the sacral slope and sacral alar anatomy in cadavers and a series of patients. They determined that the pelvic outlet and lateral sacral plain films provide the best plain radiographic view of the sacral ala. They recommended routine usage of these views intraoperatively to guide screw placement.

 

Routt et al (2000) reported on the early complications of percutaneous placement of iliosacral screws for treatment of posterior pelvic ring disruptions. While technically challenging, this technique leads to less blood loss and lower rates of infection compared to traditional open techniques.

 

Barei et al described methods of anterior and posterior pelvic ring disruptions. They determined that successful placement depends on accurate closed reduction, excellent intraoperative fluoroscopic imaging, and detailed preoperative planning. Early treatment decreased hemorrhage, provides patient comfort, and allows early mobilization.

 

 

 

 

A 23-year-old female is an unrestrained driver in a motor vehicle collision, sustaining the injury shown in Figure A. She subsequently undergoes reduction and percutaneous bilateral iliosacral screw placement. Which of the following is the most likely neurologic complication associated with percutaneous iliosacral screw insertion?

 

 

Weakness in knee extension

Decreased patellar reflex

Weakness in great toe extension

Weakness in ankle plantar flexion

Decreased Achilles reflex Correct answer: 3

Figure A shows an unstable bilateral pelvic ring injury. Percutaneous posterior iliosacral screw fixation places the L5 nerve root at risk as it courses across the sacral ala. Injury to the L5 nerve root would typically result in weakness in great toe extension and sensory changes on the dorsum of the foot. It is important to notice that L5 often partially innervates tibialis anterior along with L4, so weakness to ankle dorsiflexion may be present as well. Illustration A shows the post-operative films with bilateral iliosacral screws.

 

Routt et al examined the sacral slope and sacral alar anatomy in cadavers and a series of consecutive patients. They determined that the pelvic outlet and lateral sacral plain films provide the best plain radiographic views of the sacral ala. They recommended routine usage of these views intraoperatively to guide screw placement.

 

In another study, Routt et al reported on the early complications of percutaneous placement of iliosacral screws for treatment of posterior pelvic ring disruptions. While technically challenging, this technique leads to less blood loss and lower rates of infection compared to traditional open techniques.

 

Illustration B displays the root diagrams for sensation, reflex, and motor of the L4-S1 nerves.

 

Incorrect answers:

1: Weakness to knee extension would be caused primarily by an injury to the L4 nerve root.

2: Decreased patellar reflex would be caused primarily by an injury to the L4 nerve root.

4: Weakness in ankle plantar flexion would be caused primarily by an injury to the S1 nerve root.

5: Decreased Achilles reflex would be caused primarily by an injury to the S1 nerve root.

 

 

 

 

 

A 20-year-old patient presents after jumping from the window of a burning building with a sacral fracture. Which of the following fracture patterns seen in Figures A through E would give this patient the highest risk of associated nerve injury?

 

 

Figure A

Figure B

Figure C

Figure D

Figure E

 

Correct answer: 1

 

Answering this question relies on knowledge of the Denis classification of sacral fractures and their associated risks of nerve injury. Figure A represents a Denis Zone 3 (medial to the foramina) sacral fracture, which has the highest associated risk of nerve injury.

 

Denis et al outlined a novel classification system of sacral fractures based on the position of the fracture line relative to the sacral foramina. The authors found a 56.7% incidence of nerve injury in fractures that extended medial to the sacral foramina (zone 3), compared with 28.4% for fractures through the

foramina (zone 2), and 5.9% for fractures lateral to the foramina (zone 1).

 

Mehta et al reviewed the current principles for management of sacral fractures. They note that bowel, bladder and sexual dysfunction occur in 76% of patients with zone 3 sacral fractures.

Illustration A below demonstrates the Denis classification of sacral fractures. Incorrect Answers:

Figure B shows a zone 1 sacral fracture, which has a 5.9% incidence of

nerve injury

Figure C shows a zone 2 sacral fracture, which has a 28.4% incidence of nerve injury

Figure D shows a sacroiliac joint dislocation, not a sacral fracture

Figure E shows a zone 1 sacral fracture with an associated iliac fracture (crescent fracture)

 

 

 

 

 

A 35-year-old male is involved in a motor vehicle accident and suffers the fracture shown in Figure A. This is an isolated shoulder injury, and he has no neurologic deficits on physical exam. CT scan of the scapula shows the glenoid to be translated medially 3mm, and anglulated 20 degrees from its anatomic axis. What is the most appropriate initial treatment for this injury?

 

Immobilization in sling x 2 weeks then PT

Immobilization in sling x 8 weeks then PT

ORIF via a deltopectoral approach

ORIF via a posterior approach

ORIF via a lateral approach Correct answer: 1

The radiographs are consistent with a extra-articular glenoid neck fracture, which by definition is not significantly displaced. These fractures are best treated with a sling (2 weeks) and early mobilization. Significantly displaced fractures, have translational displacement greater than or equal to 1 cm or angulatory displacement greater than or equal to 40°. These typically need ORIF.

 

A schematic of the fracture types is shown in Illustration A.

 

McGahan et al review the epidemiology of scapula fractures and advocate conservative treatment with early mobilization.

 

Van Noort et al reviewed 13 scapular neck fractures and found that nonoperative treatment in the absence of ipsilateral shoulder injury and associated neurological impairment lead to good functional outcomes, with or without significant translational displacement of the fracture.

 

 

 

 

A 68-year-old healthy active male presents after falling and sustaining an injury to his right knee. His medical history is significant only for osteoporosis. Radiographs and representative CT scan images are shown in Figures A-D. What is the most appropriate treatment method for this patient's injury?

 

 

 

Traction and splinting

Lag screw fixation followed by non-locking plate application

Retrograde supracondylar nail fixation

External fixation and percutaneous screw reduction of the fracture

Lag screw fixation followed by locking plate application Correct answer: 5

The injury shown in Figures A-D represents a comminuted metaphyseal distal femur fracture with a sagittally oriented intra-articular split in osteoporotic bone. Because of the intra-articular nature of this injury, the best fixation construct for treatment of this fracture in an otherwise healthy and active patient is lag screw fixation followed by locked plate application.

 

Egol et al performed a systematic review of the literature to compare and contrast the function and roles of conventional unlocked plates to locked plates in fracture fixation. They concluded that locked plates may increasingly be indicated for indirect fracture reduction, diaphyseal/metaphyseal fractures in osteoporotic bone, bridging severely comminuted fractures, and the plating of fractures where anatomical constraints prevent plating on the tension side of the bone.

 

Perren et al discuss the treatment of osteoporotic fractures in the elderly population. They state that in this group of patients, plating with locked screws improve the biology and the mechanics of internal fixation. Furthermore, when this fixation method is used as an 'internal ex fix' (bridging construct) it may stimulate early callus formation because of the inherent flexibility of the construct.

 

Illustrations A and B show intraoperative fluoroscopic images during fracture fixation. Illustration C shows an AP radiograph of the distal femur 3 months after fixation with a locked plate construct.

 

 

 

 

 

 

Which of the following is true regarding intimate partner violence (IPV)?

Most patients do not have a fear of domestic retaliation upon reporting

Victims of IPV rarely have a history of injury during the previous 12 months

Interdisciplinary collaboration yields positive outcomes

Emotional abuse is easily identifiable in patients suffering from IPV

Victims of IPV feel comfortable discussing their issues to male physcians Correct answer: 3

Identification of IPV is essential in the orthopedic ambulatory setting. Discerning physical abuse is more straightforward as patients can have frequent visits with multiple, unexplained fractures. Emotional abuse is more difficult to discern. Communication and awareness is advocated. Most barriers to reporting include a lack of awareness of IPV, downplaying of the situation, fear of partner retaliation, concern for custody conflicts, shame, embarrassment and a reluctance to talk to male physicians.

 

Interdisciplinary collaboration among healthcare workers was a predictor of positive treatment outcomes. Included were more accurate assessments of past history, more descriptive emotional symptoms as displayed by victims and written documentation of recommendations concerning intervention and linkage to community resources.

 

Bhandari et al performed a cross-sectional study of 282 women who presented to fracture clinic at two Level-I trauma centers in Canada. The prevalence of abuse was found to be 32% while 8.5% were found to have a history of previous abuse in the past 12 months. Ethnicity, socioeconomic status, and injury patterns were not associated with abuse.

 

Shields et al reviewed 153 cases of domestic violence victims who presented to two Emergency departments. They determined that positive treatment outcomes were correlated to the degree of interdisciplinary collaboration among treating health care providers.

 

 

 

 

A 35-year-old male suffers the injury seen in Figures A and B following a motor vehicle collision. He is initially taken to a local

hospital. The treating surgeon, concerned that his hospital does not have a plastic surgeon available for soft-tissue coverage, arranges for transfer of the patient to a nearby level I trauma center for definitive care. Upon arrival at the definitive treatment center, the patient is taken for formal debridement and external fixator application. Which of the following options has the greatest effect on this patient's risk of infection?

 

 

 

External fixator application

Tetanus prophylaxis

Operative debridement within 6 hours

TIme to transfer to definitive trauma center

Soft-tissue coverage within 48 hours

 

Correct answer: 4

 

Figures A and B demonstrate an open, segmental tibia shaft fracture with extensive soft-tissue injury. Recent evidence has demonstrated that time to

transfer to a definitive trauma center has a significant effect on the incidence of infection for high-energy, open lower extremity fractures.

 

Pollak et. al analyzed a subgroup of 315 patients with high-energy, open lower extremity fractures from the Lower Extremity Assessment Project (LEAP study). Time to admission to a definitive trauma center for treatment was a significant, independent predictor of infection, with patients transferred 11-24 hours following injury having a significantly increased risk of major infection as compared with patients transferred within 3 hours of injury.

 

Werner et. al reviewed the existing literature surrounding the urgency of surgical debridement for open fractures, specifically relating to the "6 hour rule". The authors found limited evidence in the current literature to support emergent debridement within 6 hours of injury, and recommend urgent debridement (within 24 hours) once the patient is physiologically stable, life threatening emergencies have been addressed, and adequate surgical staff and resources are available.

 

Incorrect Answers:

External fixator application, although important for limb stability and ongoing assessment of the soft-tissue envelope, has not been shown to affect the rate of infection for open fractures

Tetanus prophylaxis is only effective against one infectious organism

Time to debridement was not shown to affect the rate of infection in the referenced articles

Time to soft-tissue coverage was not shown to have a statistically significant difference on the rate of infection in the level II study by Pollak et. al

 

 

 

 

A 38-year-old male suffers the injury shown in Figure A. During operative fixation, free osteoarticular fragments are encountered and reconstruction of these pieces is attempted. Postoperatively, which of the following will have the most beneficial effect on the healing potential of the surviving chondrocytes within these reconstructed articular segments?

 

Gentle compressive loading of the affected joint through early range of motion exercises

Strict joint immobilzation for three weeks

Shear loading of the affected joint

Joint distraction with a spanning external fixator for three weeks

Glucosamine chondroitin sulfate supplementation Correct answer: 1

Figure A demonstrates a comminuted tibial plateau fracture with significant intra-articular involvement. Basic science evidence has demonstrated that post-operative gentle compressive loading may have a positive impact on articular cartilage healing; however, excessive shear loading may be detrimental.

 

Irrgang et al provide guidelines for rehabilitation following surgical management of articular cartilage lesions of the knee. They state that after articular cartilage repair, exercises to enhance muscle function must be done in a manner which minimizes shear loading of the joint surfaces in the area of the lesion. The authors also discuss the benefits of gentle compressive loading and motion of the joint, and its positive effects on chondrocyte nutrition.

Furthermore, they recommend a period of protected weight bearing as often being necessary, and that this should be followed by progressive loading of the joint.

Illustration A is a diagram showing the different layers of joint cartilage.

 

 

 

 

 

When utilizing the pectoralis major tendon as a reference for restoring humeral height during shoulder hemiarthroplasty, at what level cephalad to the proximal edge of the tendon should the top of the prosthesis sit?

1.0 cm

2.4 cm

3.8 cm

5.6 cm

6.5 cm

 

Correct answer: 4

 

The pectoralis major tendon is a reproducible structure from which the humeral height during shoulder arthroplasty can be based upon, even in fracture situations where the anatomy is distorted. The superior edge of the pectoralis major tendon at its insertion on the humerus is 5.6 cm below the top of the humeral head.

 

The referenced study by Torrens et al is a cadaveric evaluation of the pectoralis major tendon and proximal humeral anatomy. They reported that the top of the head sits 5.6cm proximal to the superior border of the pectoralis major tendon.

 

The referenced study by Greiner et al is a retrospective review of shoulder hemiarthroplasty; improved radiographic and clinical outcomes were reported when the pectoralis major tendon was used as a reference for humeral height.

Improved outcomes were also reported with successful tuberosity healing and centering of the humeral head in the glenoid.

 

The referenced study by Murachovsky et al is another cadaveric study that found the pectoralis major tendon is a reproducible means from which to base the humeral height. The distance reported was 5.6 +/- 0.5 cm in 40 shoulders.

 

 

 

 

A 28-year-old male sustains a midshaft fibula fracture after being kicked during a karate tournament and develops compartment syndrome isolated to the lateral compartment of his leg. If left untreated, which of the following sensory or motor deficits would be expected?

Decreased sensation on the dorsum of his foot involving the hallux, 3rd, and 4th toes

Inability to plantar flex the ankle

Decreased sensation on the dorsum of his foot involving the first webspace

Inability to dorsiflex the ankle

Inability to abduct his toes Correct answer: 1

The clinical vignette describes a scenario of isolated compartment syndrome in the lateral compartment of the leg. The only nervous structure residing in the lateral compartment is the superficial peroneal nerve. In compartment syndrome of the lateral leg compartment, failure of prompt surgical fasciotomy would present as a sensory deficit of the superficial peroneal nerve presenting as numbness on the dorsum of his foot involving the hallux, 3rd, and 4th toes, as seen in Illustration A.

 

Matsen et al discuss the poor results which can be a cause of late diagnosis and surgical decompression. They recommended compartment monitoring in equivocal cases as well as release of all four leg compartments when facing leg compartment syndrome. A diagram of a two-incision fasciotomy is shown in Illustration B.

 

Olson et al provide a review of compartment syndrome for the lower extremity. They discuss a variety of injuries and medical conditions may initiate acute compartment syndrome, including fractures, bleeding disorders, and other trauma. Although the diagnosis is primarily a clinical one, they also recommend supplementation with compartment pressure measurements in equivocal cases.

 

 

 

 

A 33-year-old female sustains the injury shown in Figure A. Compared to antegrade nailing of this injury, retrograde nailing has been shown to have an increased amount of which of the following?

 

Operative time

Symptomatic distal interlocking screws

Hip pain

Union rate

Final knee range of motion Correct answer: 2

Figure A shows a femoral shaft fracture, which can be appropriately treated with an intramedullary nail.

 

The referenced article by Ostrum et al reported that retrograde nailing had an increased rate of symptomatic distal interlocking screws, an increased rate of need for dynamization, longer union time, and less thigh pain than antegrade nailing. Union rate and knee range of motion were not significantly different between these two techniques.

 

Subsequent studies have found that union time does not significantly differ between antegrade and retrograde nailing.

 

 

 

A 35-year-old male undergoes closed reduction under sedation in the emergency department for a posterior hip dislocation with an associated posterior wall fracture. The post-reduction CT is seen in Figure A. What is the appropriate next step in management of this injury?

 

 

Nonoperative management based on the size of the posterior wall fragment

Operative management based on the size of the posterior wall fragment

Operative management based on the history of hip dislocation

Dynamic fluoroscopic stress exam under anesthesia in the obturator oblique view

Dynamic fluoroscopic stress exam under anesthesia in the iliac oblique view Correct answer: 4

Joint stability is critical for successful nonoperative management of posterior wall acetabular fractures. Recent evidence has established that dynamic fluoroscopic stress examination is the best method to determine joint stability in the setting of a posterior wall fracture. The obturator oblique view allows for the best evaluation of hip joint stability during examination for posterior wall fractures.

 

Grimshaw and Moed retrospectively reviewed the results of patients with posterior wall acetabular fractures managed nonoperatively after evaluation with dynamic fluoroscopic stress tests. At two year follow up, all had good to excellent Merle d’Aubigne clinical scores for hip function and no evidence of post-traumatic hip arthritis on AP pelvis radiographs.

 

Tornetta retrospectively reviewed his results managing patients with dynamic fluoroscopic stress examination for acetabular fractures which met radiographic nonoperative criteria. Good-to-excellent clinical results were seen in 91% of patients managed nonoperatively.

 

Tornetta reviewed management of acetabular fractures and Tornetta and Mostafavi separately reviewed management of hip dislocations. In both

articles, emphasis is placed on dynamic examination of posterior wall fractures as instability has been seen with fractures comprising as little as 15% of the posterior wall.

 

Illustration A demonstrates two fluoroscopic images from a dynamic stress exam of a patient with a posterior wall fracture. The image obtained in the obturator oblique view clearly demonstrates that the femoral head loses congruency with the acetabular dome.

 

Incorrect Answers:

Answers 1 & 2: Posterior wall fragment size less than 40% was historically used as an indirect measure of stability, however measurements of fragment size may be unreliable and instability has been seen with fractures much smaller than 40%

Answer 3: A history of hip dislocation was thought to indicate a more unstable fracture. In the level IV study by Grimshaw and Moed, patients with an associated hip dislocation who were stable under stress exam had no significant difference in outcome with nonoperative management.

Answer 5: The iliac oblique view is used to evaluate the anterior wall and posterior column. Displacement or instability of the posterior wall would not be seen with this view

 

 

 

 

 

A 64-year-old woman is thrown off a horse, sustaining the injury shown in Figures A and B. She undergoes surgical fixation as seen in Figures C through E. What is the most commonly reported complication of this procedure?

 

 

 

Axillary nerve injury

Valgus migration of the fracture

Nonunion

Hardware failure

Screw penetration

 

Correct answer: 5

 

The patient in the scenario has a 2-part proximal humerus fracture treated with a locking plate as seen in Figures A-E. The most common complication with the use of this implant is screw penetration. The terms screw cut out and penetration are often used interchangeably in the literature with cut out appearing more frequently in reports regarding intertrochanteric fractures.

Owsley et al retrospectively reviewed 53 proximal humerus fractures treated with locking plates and the same post-operative protocol. The most common complication was screw cut out or penetration, followed by varus displacement. They concluded that 3 and 4-part fractures in patients over 60 years have a higher incidence of failure.

 

Agudelo et al retrospectively reviewed 153 patients at a level-one trauma center treated with proximal humerus locking plates, investigating modes of failure for the implant. They determined that varus malreduction (head-shaft angle<120 degrees) was the most common mode of failure in their group.

 

 

 

 

A 25-year-old male sustained a closed midshaft femur fracture following a motor vehicle collision. He is taken to the operating room for supine intramedullary nail fixation of the fracture. Figure A is a lateral fluoroscopic view of the distal femur taken just prior to distal interlocking screw placement. What change in position (with the Carm stationary) would be expected to produce a perfect lateral view of the interlocking hole?

 

 

Raising the leg

Lowering the leg

Internal (or external) rotation of the leg

Abduction (or adduction) of the leg

Fluoroscopic magnification

Correct answer: 4

 

Interlocking of intramedullary nails using fluoroscopy requires attention to detail. A true lateral of the intramedullary nail is present when "perfect circle" views of interlocking holes are present. Once perfect circles are obtained, the drill can be advanced parallel to the fluoroscopic beam.

 

Knowledge of the implications of the appearance of the interlocking holes when "perfect circles" are not present can be helpful in minimizing the number of manipulations and fluoroscopic exposure. Widening of the interlocking hole in the proximal-distal direction (as is seen in this case) signifies the need for an adjustment in the abduction/adduction plane. Similarly, widening of the interlocking hole in the anterior-posterior plane signifies the need for an adjustment in the internal/external rotation plane (Answer 3).

 

Raising or lowering the leg (Answer A and B) should not have major effects of the appearance of the interlocking hole. Similarly, magnification of the C-arm (Answer D) will not affect the appearance. Internal/external rotation will result in widening in the anterior-posterior plane (as this dimension is currently adequate). Abduction or adduction will result in creating "perfect circles" (Answer 4). The position of the leg may hint to which of these is correct. If further widening occurs in the proximal-distal direction, the opposite maneuver will correct to the proper position.

 

 

 

 

A 46-year-old male is involved in a motor vehicle accident and suffers a proximal humerus fracture. Operative treatment is recommended, and plate fixation is performed through an extended anterolateral acromial approach. Which of the following structures is at increased risk of injury using this surgical exposure compared to the deltopectoral approach?

Musculocutaneous nerve

Posterior humeral circumflex artery

Axillary nerve

Cephalic vein

Anterior humeral circumflex artery Correct answer: 3

The anterolateral acromial approach was developed to allow less invasive treatment of proximal humerus fractures. The plane of the avascular anterior

deltoid raphe is utilized, and the axillary nerve is at particular risk of injury and must be identified and protected. With this approach, anterior dissection near the critical blood supply is avoided, substantial muscle retraction is minimized, and the lateral plating zone is directly accessed.

 

Gardner et al evaluated 23 patients who had acute displaced fractures of the proximal humerus treated with the anterolateral acromial approach and either a locking plate or an intramedullary nail. At one year post-op, there were no axillary nerve deficits, and they found that the approach allowed direct access to the lateral fracture planes for fracture reduction and plate placement or safe nail and interlocking screw placement.

 

Gardner et al, in another study, performed cadveric dissection using the extended anterolateral acromial approach and measured multiple parameters regarding the axillary nerve. The nerve was predictably found approximately 35 mm from the prominence of the greater tuberosity.

 

Morgan et al performed a cadaveric study to describe the anatomic insertion point of the deltoid onto the proximal humerus. They found that the deltoid insertion is long and broad, and that placement of 4.5-mm plate would result in detaching 13.5 mm of its insertional footprint. This would leave half of the insertion still attached to the humerus.

 

Illustration A shows the position of the axillary nerve in relation to the approach.

 

 

 

 

 

Which of the following statements is true regarding brake travel time after surgical treatment of complex lower extremity trauma?

Brake travel time is significantly increased until 6 weeks after patient begins weight bearing

Return of normal brake travel time takes longer after long bone fracture compared to articular fractures

Normal brake travel time correlates with improved short musculoskeletal

functional assessment scores

Brake travel time is significantly reduced until 8 weeks after patient begins weight bearing

Brake travel time returns to normal when weight bearing begins Correct answer: 1

Brake travel time (BTT) has been shown to be significantly increased until 6 weeks after initiation of weight bearing in both long bone and articular fractures of the right lower extremity.

 

Egol et al used a computerized driving simulator to compare BTT in three groups of patients; 1)control group, 2)long bone lower extremity fracture group, 3)lower extremity articular fracture group. They concluded that BTT was significantly increased until 6 weeks after initiation of weight bearing in both long bone and articular fractures of the right lower extremity, and that short musculoskeletal functional assessment scores improved with respect to function and other indexes, but did not correlate with improvement in BTT.

 

An earlier study by Egol et al looked at total brake time as it related to distance traveled by the automobile before braking at 6, 9, and 12 weeks after operative fixation of a right ankle fracture. When compared with controls, braking time was shown to return to normal by 9 weeks post-operatively, and no significant association was found between the functional scores and this normalization.

 

Giddins et al provide a review of the literature, the law and the views of the major motor insurers related to driving after injury or operations.

 

 

 

 

What acetabular component is best appreciated on an obturator oblique radiograph of the pelvis as seen in Figure A?

 

Ilioischial line

Posterior column

Posterior wall

Anterior wall

Sacroiliac joint

 

Correct answer: 3

 

Letournel and Judet developed a schematic representation of the acetabulum as being contained within asymmetric long anterior and short posterior arms of an inverted “Y”.

 

On the bony pelvis, the ilioischial component becomes that posterior column and the iliopectineal line becomes the anterior column. The Judet-Letournel classification system is based on this scheme. By careful evaluation of landmarks on a standard AP pelvis radiograph, as well as on 45-degree oblique obturator and iliac views, the extent of injury can be determined accurately.

 

The AP view usually demonstrates the six fundamental landmarks relatively well as seen in illustration A. The obturator oblique view reveals additional information about the anterior column and posterior wall(see illustration A(B),

B). In an obturator oblique view the x-ray beam is centered on and almost perpendicular to the obturator foramen. The iliac oblique view visualizes the posterior column and anterior wall (illustration A(C), C). This view also shows the best detail of the iliac wing as the radiographic beam is roughly perpendicular to the iliac wing. Inclusion of the opposite hip is essential for evaluation of symmetrical contours that may have slight individual variations

and to evaluate the width of the normal articular cartilage in each view in a pelvic series (AP, Judet's) .

 

 

 

 

 

 

Which of the following fractures seen in Figures A through E would be amenable to fixation with a construct using tension band principles?

 

 

 

Figure A

Figure B

Figure C

Figure D

Figure E

 

Correct answer: 1

 

In order for a fracture to be successfully treated with tension band principles the bone must be eccentrically loaded, the construct must be applied on the tensile side, and the opposite cortex must be able to withstand compressive

forces. Of the fractures seen in Figures A-E, the subtrochanteric fracture seen in Figure A best meets the criteria for stabilization according to tension band principles.

 

Kinast et al retrospectively compared their results with 95 degree blade plate fixation of subtrochanteric fractures utilizing the blade plate as a dynamic tension band. They performed either wide exposure of the fracture site with autogenous bone grafting according to AO technique at the time (group 1), or indirect reduction techniques without bone grafting the medial side as advocated by Mast et al (group 2). The authors found 100% union rates at six months with indirect reduction techniques without bone grafting (group 2), and emphasize the key concepts of preservation of the medial soft tissues and intraoperative pretensioning of the plate.

 

Illustration A demonstrates the principles of tension band fixation specifically applied to the femur. Within Illustration A, Figure C shows the correct application of a plate along the lateral cortex to resist tensile forces, along with incorrect application of the plate along the medial cortex (Figure D) or in a fracture pattern with an absent opposite cortex (Figure E). Illustration B shows an example of blade plate fixation of a subtrochanteric fracture.

 

Incorrect Answers:

Answer 2. Figure B demonstrates a comminuted proximal tibia fracture. Although the tibia is eccentrically loaded and an implant applied to anterior cortex could function as a tension band, the posterior comminution would lead to collapse.

Answer 3. Figure C demonstrates a comminuted distal humeral shaft fracture. Again,the comminution prevents application of a tension band construct Answer 4: Figure D shows a valgus impacted proximal humerus fracture.

Eccentric loading is absent for this type of fracture

Answer 5: Figure E demonstrates a comminuted olecranon fracture with extension distal to the coronoid process. Although tension band constructs are commonly used for olecranon fractures, the comminution and distal extension of this fracture would prevent application of a tension band.

 

 

 

 

Which of the following is an advantage of computer-assisted navigation used to place medullary nail interlocking screws compared to a freehand techinque?

Reduced fluoroscopy time

More reliable placement of interlocking screws through the nail

Reduced procedure time

Increased quality of fluoroscopic images

Improved accuracy of screw length Correct answer: 1

Computer-assisted navigation has been shown to reduce radiation exposure for surgeons when performing interlocking of medullary nails compared to freehand technique.

 

Ricci et al compared two fluoroscopic navigation tracking technologies, optical and electromagnetic versus standard freehand fluoroscopic targeting, in a standardized foam block model for placement of interlocking screws. They found that fluoroscopy time (seconds) and number of fluoroscopy images were significantly less when using the computer-guided systems than for freehand-unguided insertion. Average distance of pin placement from the target in the foam blocks was significantly greater for controls than for each of the navigated systems.

 

Suhm et al performed a prospective controlled clinical study to compare fluoroscopic guidance with fluoroscopy-based surgical navigation for distal locking of intramedullary implants. The surgical navigation group showed increased procedure time, but equivalent precision with reduced radiation exposure. There was no significant difference in the technical reliability

between both groups.

 

 

 

 

 

An 82-year-old nursing home resident falls onto his elbow while rising from a seated position. He has pain and swelling at the elbow without evidence of instability. His radiographs show a comminuted displaced olecranon fracture involving 25% of the articular surface with global osteopenia. Which of the following treatment options has a low risk of complications and a high likelihood of a functional elbow outcome?

Cast immobilization in 45 degrees of flexion for 8 weeks

Closed reduction and percutaneous pinning

ORIF with a tension band construct

ORIF with a locking plate

Cast immobilization in 90 degrees flexion Correct answer: 5

The patient in the scenario is a low-demand, elderly patient with a comminuted olecranon fracture. Recent studies have demonstrated good outcomes with

non-operative treatment of olecranon fractures in this patient population.

 

Hak (jaos'00) reviewed the treatment of olecranon fractures, recommending ORIF for displaced intra-articular fractures, either via tension band wiring or plate osteosynthesis based on fracture pattern and associated ligamentous disruption. For elderly, low-demand, osteoporotic patients, triceps advancement and fragment excision is an option in the case of severely comminuted, osteoporotic patients with a fracture involving <50% of the joint surface.

 

Duckworth (jbjs'14) performed a single center retrospective review of low-demand patients with displaced olecranon fractures managed non-operatively with a short duration of immobilization followed by range of motion to tolerance. The authors found good short and long-term results with this treatment with patients experiencing minimal discomfort or loss of motion at long-term follow up.

 

Incorrect answers:

Answer 1: Cast immobilization in 45-90 degrees of flexion is advocated for non-displaced fractures. Furthermore, casting for 8 weeks is not indicated in nonoperative care of olecranon fractures secondary to the development of

stiffness.

Answers 2-4: Not the treatment in low demand patients with severe comminution and osteoporosis.

 

 

 

 

A 47-year-old male presents with a one month history of heel pain after starting marathon training. The heel is tender when squeezed. A foot radiograph is shown in Figure A, and an MRI is obtained which is shown in Figures B and C. What is the most likely diagnosis?

 

 

 

Osteomyelitis

Acute fracture

Subtalar arthritis

Achilles tendinitis

Stress fracture

 

Correct answer: 5

 

The patients history, physical exam, and MRI findings are most consistent with the diagnosis of a stress fracture. Low signal on T1 and increased signal on T2-weighted images is the classic appearance of a stress fracture on magnetic resonance images. Usually the fracture appears as a linear signal band that arises from the cortex of the bone and extends perpendicular to the surface of the bone. Figures A and B show a linear area of enhancement in the posterior aspect of the calcaneus signifying a likely stress reaction from recent overtraining.

 

Boden et al review the pathogenesis and treatment of stress fractures. With regard to diagnosis, they state that it is primarily clinical, but imaging modalities such as plain radiography, scintigraphy, computed tomography, and magnetic resonance imaging may provide confirmation.

 

Gehrmann et al review stress fractures of the foot, and state that with regards to calcaneal stress injuries, MRI studies should be obtained for definitive diagnosis when radiographs are negative, and pain persists. With regards to

treatment, they state that these injuries usually heal with activity restrictions, heel-pad inserts, and protected weightbearing for a short period of time.

 

 

 

 

A 35-year-old right hand dominant man falls from a ladder and sustains the injury seen in Figure A. When discussing the risks and benefits of operative versus nonoperative treatment for his fracture, which of the following is true?

 

 

No difference in shoulder function

Higher risk of nonunion with operative management

Higher risk of symptomatic malunion or nonunion with nonoperative management

Earlier return to sport with nonoperative management

No difference in union rates Correct answer: 3

Historically, displaced midshaft clavicle fractures, as seen in Figure A, were managed nonoperatively. Recent literature has demonsrated improved outcomes with operative management of these fractures.

 

Khan et al review current concepts in the management of clavicle fractures. For displaced midshaft clavice fractures, operative treatment seems to result in improved patient and surgeon-based outcomes, decreased rates of malunion and nonunion, and shorter time to union.

 

Kim and McKee review recent evidence regarding the management of clavicle fractures. For midshaft clavicle fractures, the incidence of nonunion and symptomatic malunion with nonoperative management is higher than

previously believed. They state that risk factors include 100% displacement, comminution, increasing age and female gender.

 

Incorrect Answers

Answer 1. Recent randomized prospective trials have shown improved short term shoulder function with operative management of displaced midshaft clavicle fractures.

Answer 2. Nonunion rates of 7-15% have been shown with nonoperative management versus 2% with operative fixation

Answer 4. Earlier return to activities has been reported with operative management

Answer 5. As with answer 2, there is a significantly higher rate of nonunion with nonoperative management

 

 

 

 

Assuming the images represent isolated injuries, which of the following Figures demonstrates a Pipkin II femoral head fracture?

 

 

 

 

Figure A

Figure B

Figure C

Figure D

Figure E

 

Correct answer: 4

 

Figure D represents a Pipkin II femoral head fracture, which is defined as a fracture which is superior to the fovea. Differentiation between Pipkin I and Pipkin II fractures can be important, as suprafoveal injuries often require surgical fixation. Illustration A demonstrates the Pipkin fracture types.

 

Droll et al review femoral head fracture evaluation and treatment. They discuss non-operative indications (typically reserved for Pipkin I injuries) which include an anatomic or near anatomic reduction (<2 mm) of the femoral head fragment, a stable hip, and no interposed fragments preventing a congruent hip joint. They also state that an anterior surgical approach is preferred for fixation of Pipkin II type injuries.

 

Pipkin discusses the evaluation and treatment of hip fracture dislocations as defined by the Stewart and Milford classification scheme. He focuses on Grade IV injuries, which at the time lacked an appropriate treatment algorithm.

Incorrect Answers:

1,2-Figures A and B show Pipkin IV fractures, due to the presence of an associated acetabular fracture.

3-Figure C shows a Pipkin I fracture, as the fracture is infra-foveal

5-Figure E shows an example of a Pipkin III injury due to the presence of an associated femoral neck fracture.

 

 

 

 

 

A 20-year-old female presents following a motor vehicle collision with the injuries seen in Figures A and B. She was initially hypotensive and tachycardic however she now has stable vital signs following a 2 liter bolus of saline and 2 units of packed red blood cells. Which of the following would indicate that this patient has occult end-organ hypoperfusion and should be further resuscitated prior to definitive fixation?

 

 

Heart rate of 80 beats per minute

Systolic blood pressure of 120 mmHg

Base deficit of -1.8 mEq/L

Serum lactate of 5 mmol/Liter

Urine output of 40ml/hour Correct answer: 4

Figure A demonstrates an unstable pelvic fracture and subtrochanteric femur fracture and Figure B shows a scapular body and mutliple rib fractures. These high-energy fractures, along with the patient's initial hypotension and tacchycardia indicate a multiply injured patient in shock. Serum markers such as lactate (normal < 2 mmol/L) or base deficit are more sensitive markers of occult end-organ hypoperfusion and a serum lacate of 5mmol/Liter indicates an incompletely resuscitated patient.

Porter et al review the endpoints of resuscitation and note a high incidence of patients (as much as 85%) in "compensated" shock despite normal conventional parameters such as vital signs and urine output. Compensated shock occurs secondary to shunting of tissue oxygenation and blood flow away from splanchnic organs and towards vital organs such as heart and brain.

 

The review article by Elliott argues that serum lactate levels are the most reliable indicator of peripheral organ perfusion and tissue oxygenation. A base deficit between -2 and +2 is also an appropriate end point however may be non-specific in older patients with medical comorbidities leading to acid/base disturbances.

 

Rossaint et al put forth evidence-based recommendations regarding control of bleeding and resuscitation of trauma patients. They strongly recommend based on moderate-quality evidence (Grade 1B) that both serum lactate and base deficit be used to monitor the extent and progression of bleeding and shock.

 

 

 

 

An 80-year-old female falls and sustains the fracture seen in Figure A. She is treated with an antegrade cephalomedullary nail. Which of the following led to the complication seen in Figure B?

 

 

Nail with a lesser radius of curvature

Nail with a greater radius of curvature

Piriformis entry portal

Trochanteric entry portal

Lateral decubitus patient position Correct answer: 2

The image in Figure A shows an unstable intertrochanteric fracture and the image in Figure B shows perforation of the anterior cortex of the femur by the intramedullary implant. This complication is due to a mismatch of the curvature of the nail with the anterior bow of the femur, and was likely caused by a nail of a greater radius of curvature (eg, straighter than the femur).

 

Egol et al measured the radius of curvature for 474 matched cadaveric femurs and found the average anterior radius of curvature to be 120cm (+/- 36cm). In contrast, the radii of curvature for the measured intramedullary nails ranged from 186cm to 300cm, demonstrating that the nails were straighter than the femurs. The authors advocate for a decreased radius of curvature (more curve) for intramedullary nails, especially larger diameter implants designed for fractures about the hip

 

Ostrum and Levy present a case series of 3 patients with subtrochanteric

fractures who had anterior penetration of the femoral cortex. They state that the mismatch in femoral bow between the bone and the implant is a contributing factor to distal femoral anterior cortex penetration in intramedullary nailing of subtrochanteric fractures.

 

Simonian et al present 4 iatrogenic femoral neck fractures that occured during a series of 315 femoral nails. The authors attempted to reproduce the iatrogenic fractures with cadaveric femurs and felt that the iatrogenic fractures may be due to a combination of a valgus femoral neck and impingement from the AO insertion jig used at the time.

 

Harper and Carson examined 14 cadaveric femurs and intramedullary implants at the time. Similar to Egol et al, they found a mismatch between the radius of curvature of the femurs and the intramedullary nails.

 

Illustration A shows the difference between a lesser and greater radius of curvature. Illustration B demonstrates how to calculate radius of curvature based on an implant with an exaggerated bow. Illustration C shows the anterior bow of a synthetic femoral model compared with several intramedullary implants.

 

 

 

 

 

Bridge plating of the olecranon is MOST appropriate in which of the following clinical scenarios?

Fixation of an olecranon osteotomy used for distal humerus surgery in a 24-year-old male

Simple transverse olecranon fracture in 33-year-old female

Comminuted olecranon fracture in 45-year-old male

Severely comminuted proximal olecranon fracture in an osteoporotic 91-year-old female

Aphophyseal elbow fracture in 6-year-old male Correct answer: 3

Bridge plating is most appropriately used for fixation of comminuted fractures which are not able to be fixed anatomically. Of the choices listed above, this would be most appropriate in a comminuted fracture in a 45-year-old male. In contrast, literature shows that severely comminuted, osteoporotic low-demand elderly are best treated with olecranon fracture excision and triceps advancement when possible.

 

Bailey et al looked at the outcome of plate fixation of olecranon fractures. They concluded that plate fixation is effective for displaced olecranon fractures and leads to good functional outcome. There were low incidence of complications, although 20% did have removal of hardware due to irritation.

 

Hak et al reviewed the fixation options for olecranon fractures. In their review, they state that comminuted olecranon fractures can be treated by limited-contact dynamic-compression plates. They also supported the use of supplemental bone graft to support comminuted depressed articular fragments. Fragment excision and triceps advancement is appropriate in selected cases in which open reduction seems unlikely to be successful, such as in osteoporotic elderly patients with severely comminuted fractures.

 

 

 

 

A 75-year-old ambulatory male who lives independently presents with the fracture shown in Figure A. Which of the following is true regarding timing of surgical fixation and post-operative mortality?

 

30 day mortality is decreased if surgery is delayed 4-7 days

1 year mortality is increased if surgery is delayed greater than 4 days

Delay of surgery due to treatment of acute medical comorbidities has no effect on post-operative mortality rates

90 day mortality rate is decreased if surgery is delayed greater than 7 days

Timing of surgical fixation has no statistically significant affect on postoperative mortality

Correct answer: 2

 

Figure A shows a displaced femoral neck fracture.

 

Moran et al studied 2660 elderly patients who underwent surgical treatment of a hip fracture to determine whether a delay in surgery affects postoperative mortality. The 30-day mortality for patients for whom the surgery had been delayed for more than 4 days was 10.7%, compared to 7.3% in those delayed 1-4 days. The group delayed >4 days also had significantly increased mortality at 90 days and 1 year. Patients who had been admitted with an acute medical comorbidity that required treatment prior to the surgery had a 30-day mortality of 17%, which was nearly 2.5 times greater than that for patients without and acute comorbidity. The study concluded that patients with medical comorbidities that delayed surgery had 2.5 times the risk of death within 30 days after the surgery compared with patients without comorbidities. Mortality was not increased when the surgery was delayed up to four days for patients who were otherwise fit for hip fracture surgery, however, a delay of more than four days significantly increased mortality.

Healy et al examined 120 patients who underwent surgical treatment of 186 displaced femoral neck fractures with either internal fixation, hemiarthroplasty, or total hip arthroplasty. Arthroplasty was associated with more independent living, and was more cost-effective than internal fixation. There was no difference in rates of reoperation or mortality, but arthroplasty produced a longer interval to reoperation or death. They concluded that total hip arthroplasty was the best treatment for displaced fractures of the femoral neck in their series.

 

Illustration A shows a scoring system developed by Rogmark et al to aid in decision making.

 

 

 

 

 

A 35-year-old male suffers an anterior column acetabular fracture during a motor vehicle collision, and subsequently undergoes percutaneous acetabular fixation. Intraoperatively, fluoroscopy is positioned to obtain an obturator oblique-inlet view while placing a supraacetabular screw. Which of the following screw relationships is best evaluated with this view?

Intraarticular penetration of the screw

Position of the screw cephalad to the sciatic notch

Screw starting point at the anterior inferior iliac spine

Screw starting point at the gluteal pillar

Screw position between the inner and outer tables of the ilium Correct answer: 5

The obturator oblique-inlet view, as seen in Illustration A, best demonstrates the position of a supra-acetabular screw or pin relative to the tables of the ilium.

 

Starr et al review their initial results and technique of closed or limited open reduction and percutaneous fixation of acetabular fractures. They defined two groups of patients who may benefit from this technique; elderly patients with multiple comorbidities to facilitate early mobilization and restore hip morphology, and young patients with elementary fracture patterns and multiple associated injuries.

 

Starr et al describe their operative technique and outcomes for a case series of 3 patients using percutaneous acetabular fixation to augment open reduction of acetabular fractures. The authors state that, for placement of an anterior colum ramus screw, an iliac oblique-inlet (not obturator oblique-inlet) will ensure that the screw is within the medullary canal of the ramus and does not exit anterior or posterior.

 

Gardner and Nork describe a technique for placement of a large femoral distractor in the supra-acetabular region to compress displaced posterior pelvic ring injuries. They note that the obturator oblique-inlet view is necessary to view the entire length of the pin as well as to ensure that pin remains in bone.

 

Incorrect answers:

Answer 1: Relationshiop of the screw to the acetabulum is best evaluated with the obturator oblique-outlet view as well as the iliac oblique view

Answer 2: The iliac oblique view is used to ensure the trajectory of the screw is superior to the sciatic notch

Answer 3: The Obturator oblique-outlet view, otherwise known as the "teepee" or "tear drop" view, is used to identify the start for supra-acetabular implant placement

Answer 4: The gluteal pillar is not utilized as a start point when placing supraacetabular fixation, and the obturator oblique-inlet view would not be ideal to visualize this region of the pelvis

 

 

 

 

A 74-year-old man falls, sustaining the injury shown in Figures A through C. In surgical planning, what is the best surgical approach to treat this injury?

 

 

Kocher-Langenbeck

Watson-Jones

Extended iliofemoral

Ilioinguinal

Hardinge

 

Correct answer: 4

 

Figures A through C depict and AP pelvis and Judet views of a T-type fracture of the right acetabulum. The ilioinguinal approach provides access to the anterior wall and anterior column for fracture fixation, in addition to allowing fixation of the nondisplaced posterior transverse fracture line. The lateral femoral cutaneous nerve (LFCN) is at risk in the superficial part of the dissection. Another option for the approach would be the modified Stoppa, which would also allow excellent access to the anterior column as well as the internal aspect of the iliac wing and quadrilateral plate.

Illustration A shows the five basic and 5 associated acetabular fractures. Incorrect Answers:

Answer 1. Kocher-Langenbeck: access for posterior wall and column fractures

Answer 2. Watson-Jones: anterolateral approach best for the hip, not the anterior column of the acetabulum.

Answer 3. Extended iliofemoral: visualization for both column fractures Answer 5. Hardinge approach: lateral approach for THA

 

 

 

 

A 28-year-old female is struck by a motor vehicle while crossing the street and suffers the injury seen in Figure A. What technical adjunct could have prevented the operative complication seen in Figure B?

 

 

Nail of a lesser radius of curvature

Nail with a more distal Herzog curve

Application of an anterior unicortical plate

Nailing while in a hyperflexed position

A more distal and medial nail entry site

Correct answer: 3

 

The patient has a proximal tibia fracture that has been malreduced in procurvatum with anterior translation of the proximal fragment as seen in Figures A and B. Intramedullary nailing of proximal tibia fractures may result in malalignment such as valgus, procurvatum and anterior translation of the proximal fragment.Techniques such as anterior unicortical plating have been described to maintain reduction while placing a locked intramedullary implant.

 

Nork et al retrospectively review their results using several different intraoperative adjuncts to maintain reduction while nailing proximal tibia fractures including anterior unicortical plates and femoral distractors. They report that, despite high rates of segmental comminution (59.5%) and open fractures (35.1%), acceptable alignment and primary union were achieved in more than 90% of patients.

 

Dunbar et al describe their technique for application of a provisional unicortical plate through the traumatic wound to maintain reduction during intramedullary nailing of Gustilo-Anderson Type III open tibia fractures. The authors advocate that further soft tissue stripping during plate placement should be avoided, and found excellent results in terms of coronal and sagittal plane alignment for fractures treated with this technique.

 

Matthews et al review their experience with unicortical plating to maintain reduction during intramedullary nailing of tibia fractures. Similar to the other referenced studies, the authors found the technique assisted with maintaining reduction intraoperatively.

 

Illustrations A and B demonstrate maintenance of reduction of a proximal tibia fracture using an anterior unicortical plate as a reduction tool during intramedullary nailing.

 

Incorrect Answers:

Answer 1: Nail of a lesser radius of curvature. Unlike femoral nails, tibial nails are straight to match the coronal/sagittal axis of the tibia and therefore a nail with a lesser radius of curvature (more bend) would not match the anatomy and likely lead to malalignment

Answer 2: Nail with a more distal Herzog curve. The proximal bend for tibial IM nails, known as the Herzog curve, may cause malalignment in proximal tibia fractures due to a phenomenon known as the "wedge effect". Nails with more distal Herzog curves have been implicated in anterior translation of the proximal fragment.

Answer 4: Nailing in a hyperflexed position would cause extension of the proximal fragment due to tension on the extensor mechanism

Answer 5: A more distal and medial entry site would likely lead to further deformity and possible nail cut out through the anterior cortex. A more proximal and lateral entry site is preferred for nailing proximal tibia fractures

 

 

 

 

 

A 32-year-old male sustains an APC-III pelvic ring disruption after a motor vehicle collision. Which of the following imaging techniques best describes the correct utilization of intraoperative flouroscopy for percutaneous iliosacral screw placement across S1?

Inlet view helps best guide superior-inferior orientation

AP pelvis best guides anterior-posterior screw orientation

AP pelvis best guides superior-inferior screw orientation

Outlet view best guides anterior-posterior screw orientation

Outlet view best guides superior-inferior screw orientation Correct answer: 5

The outlet view best guides superior-inferior screw orientation during percutaneous S1 screw placement. This is due to the relative forward flexion of the sacrum and pelvis due to pelvic incidence. A lateral sacral view and an inlet pelvis view would best guide anterior-posterior screw orientation.

 

Routt et al did a review of percutaneous techniques of pelvic surgery. Although anterior pelvic external fixation remains the most common form of percutaneous pelvic fixation, iliosacral screws have the advantage of stabilizing pelvic disruptions directly while diminishing operative blood loss and operative time. They stress importance of a thorough knowledge of pelvic osseous anatomy, injury patterns, deformities, and their fluoroscopic correlations for surgical success.

 

Routt et al also looked at the complications that can result from percutaneous iliosacral screw placement. Complications ranged from inability for adequate imaging due to patient obesity, L5 nerve root injuries, fixation failure, and sacral nonunions. They support quality triplanar fluoroscopic imaging during iliosacral screw insertions to help accurately reduce injured posterior pelvic rings.

 

Illustration A is an example of an outlet view image status post anterior pelvic ring plating and percutaneous iliosacral screw. This outlet view allows superior S1 neural foramen visualization to help guide screw placement and avoid nerve injury.

 

 

 

 

Which of the following methods accurately describes the measurement of tip-apex-distance as it relates to placement of a lag screw in the femoral head?

Summation of the distance between the end of the screw and the apex of the femoral head on AP and lateral radiographs

Distance from the acetabular teardrop to the tip of the screw on an AP radiograph of the hip

Multiplication of the distance between the end of the screw and the apex of the femoral head on AP and lateral radiographs

Distance from the center of the lesser trochanter to the tip of the screw on an AP hip radiograph

Summation of the distance between the tip of the greater trochanter and end of the screw on AP and lateral hip radiographs

Correct answer: 1

 

Tip-apex distance (TAD) as it relates to a lag screw in the femoral head is the summation of the distance between the end of the screw and the apex of the femoral head on AP and lateral radiographs, corrected for radiographic magnification of the image. This is shown in Illustration A.

 

TAD is a useful intraoperative indicator of deep and central placement of the lag screw in the femoral head, regardless of whether a nail or a plate is chosen to fix a fracture. A TAD of <25mm has been shown to minimize the risk of fixation cut-out in stable and unstable intertrochanteric hip fractures.

 

Baumgaertner et al. examined factors leading to the failure of sliding hip screws (SHS) in the treatment of 198 intertrochanteric fractures. They

determined that the TAD is a reproducible, standard measurement to predict SHS failure. The average TAD for successful fractures was 24mm while the average TAD for failures was 38mm. No screw with a TAD <25mm failed.

 

Geller et al. evaluated the TAD of intramedullary devices as an independent predictor of cut-out failure in intertrochanteric hip fractures. The authors found that there was a statistically significant increase in screw cut-out with TAD > 25mm as well as with increasing fracture severity. They concluded that TAD is a valuable consideration for intramedullary devices as well.

 

Illustrations B and C show a lag screw with an excessive TAD, and subsequent failure of fixation.

 

Incorrect Answers:

Answers 2-5: TAD is defined as the sum of the distance between the end of the lag screw and the apex of the femoral head on the AP and lateral radiographs, corrected for radiographic magnification of the image.

 

 

 

 

 

A 25-year-old man presents one year after undergoing open reduction and internal fixation of the fracture seen in Figure A. Current radiographs demonstrate a united fracture with no evidence of ostenecrosis, subtalar or tibiotalar arthritis. Physical exam is notable for ambulation on the lateral border of the right foot with hindfoot varus, midfoot supination and diminished subtalar motion compared with the contralateral side. Which of the following is an option for reconstruction of this patient's deformity?

 

Total ankle arthroplasty

Lateral calcaneus closing wedge osteotomy

Calcaneal neck opening wedge osteotomy

Talar neck opening medial wedge osteotomy

Triple arthrodesis

 

Correct answer: 4

 

The most common position of talar neck malunion is varus and this deformity can lock the subtalar and transverse tarsal joints leading to diminished motion and ultimately subtalar arthrosis. Medial opening wedge osteotomy of the talar neck has been described to restore the anatomy of the talus and preserve hindfoot motion.

 

Daniels et al performed a biomechanical study of hindfoot motion following medial osteotomy and varus malalignment of the talar neck. The authors found a direct correlation between the degree of varus malalignment and diminished forefoot and subtalar motion, and caution against compressive fixation of the medial talus in the presence of comminution.

 

Monroe and Manoli present a case report of talar neck malunion and describe their technique for osteotomy of the talar neck with insertion of a tricortical iliac crest bone graft to correct the deformity. They found a significant improvement in the AOFAS ankle-hindfoot score and no evidence of

osteonecrosis with follow-up greater than 4 years.

 

Huang and Cheng retrospectively review their results with anatomic reconstruction of the talar neck in 9 patients who presented a minimum of four weeks after the initial injury with a malreduced or neglected talar neck fracture. They found favorable outcomes with delayed reconstruction as determined by the AOFAS ankle-hindfoot score at an average follow up of 53 months, however note that six patients subsequently developed radiographic hindfoot arthrosis.

 

Figure A demonstrates a lateral radiograph of a comminuted talar neck fracture with subluxation of the subtalar joint.

 

Incorrect Answers:

Answer 1: Total ankle arthroplasty is reserved for patients with end-stage tibiotalar arthritis. This patient has an intact and functional tibiotalar joint Answer 2: Lateral calcaneus closing wedge osteotomy would correct some hindfoot varus, however would not address the talar neck deformity leading to diminished motion.

Answer 3: Calcaneal neck opening wedge osteotomy would exacerbate the patient's deformity

Answer 5: Triple arthrodesis would be not be a viable option in a patient with a preserved tibiotalar and subtalar joint.

 

 

 

 

Which of the following descriptions is true regarding APC-II (anterior-posterior compression) pelvic injuries as classified by Young and Burgess?

Pubic symphysis diastasis, intact anterior sacroiliac ligaments, intact sacrotuberous ligament, intact posterior sacroiliac ligaments

Pubic symphysis diastasis, torn anterior sacroiliac ligaments, intact sacrotuberous ligament intact posterior sacroiliac ligaments

Pubic symphysis diastasis, intact anterior sacroiliac ligaments, torn sacrotuberous ligament, intact posterior sacroiliac ligaments

Pubic symphysis diastasis, torn anterior sacroiliac ligaments, torn sacrotuberous ligament, intact posterior sacroiliac ligaments

Pubic symphysis diastasis, intact anterior sacroiliac ligaments, torn sacrotuberous ligament, torn posterior sacroiliac ligaments

Correct answer: 4

APC II injuries are unstable injuries and occur as a result of high-energy trauma. Anatomic structures which are injured or torn include the pubic symphysis, anterior iliosacral ligaments, and the sacrotuberous ligaments. The posterior sacroiliac ligaments are spared in APC-II injuries, and differentiate an APC-II injury from an APC-III injury, in which the posterior ligaments are also torn.

 

Burgess et al review the classifications of pelvic ring disruptions and their association with mortality. They concluded that APC injuries required more blood replacement and were related to death more often than lateral compression, vertical shear, or combined mechanism pelvic injuries.

 

Tile studied the anatomy of anterior to posterior pelvic ring injuries. Although the anterior structures, the symphysis pubis and the pubic rami, contribute to 40% to the stiffness of the pelvis, clinical and biomechanical studies have shown that the posterior sacroiliac complex is more important to pelvic-ring stability. The posterior sacroiliac ligamentous complex is more important to pelvic-ring stability than the anterior structures and therefore, the classification of pelvic fractures is based on the stability of the posterior lesion.

 

Illustration A shows the APC classification.

 

 

 

 

 

Which of the following deformities is most likely to occur with dynamic hip screw fixation of unstable left sided standard obliquity hip fractures?

Posterior spike displacement of the proximal fragment

Anterior spike displacement of the proximal fragment

Lateral displacement of the proximal fragment relative to the distal fragment

Shortening of the proximal fragment relative to the distal fragment

Medial displacement of the proximal fragment in relation to the distal fragment

Correct answer: 2

Left-sided unstable intertrochanteric hip fractures are at increased risk of malreduction compared to unstable right-sided fractures fixed with dynamic hip screws. In left-sided fractures the rotational torque imparted to the proximal head and neck fragment can cause loss of reduction leading to potential failures of fixation. With these left sided injuries, the rotational torque can cause an anterior spike, whereas with right-sided injuries the rotational torque causes compression and reduction of the fracture. In addition, if a nail is used for these injuries and the proximal fracture fragment is not being held by the nail itself, this phenomenon can be seen as well.

 

Mohan et al conducted a study to assess the effect of clockwise rotational torque onto the fracture configuration in unstable and stable intertrochanteric fractures fixed with a dynamic hip screw construct. They found that 11 out of 30 unstable fractures showed an anterior spike (flexion malreduction) in left-sided fixations due to clockwise torque. This malreduction was not present in right-sided or stable fractures.

 

Illustrations A and B are images from Mohan et al's study showing the rotational affect on the fracture with placement of a dynamic hip screw.

 

 

 

 

 

A 45-year-old male sustains a proximal third tibia fracture as an isolated injury and elects to undergo operative treatment with intramedullary nailing. Post-operative radiographs show excessive procurvatum deformity. Which of the following operative techniques would have helped to best avoid the procurvatum deformity?

Tibial nailing with increased knee flexion

Lateral blocking screw in the proximal fragment

Medial blocking screw in the proximal fragment

Anterior blocking screw in the proximal fragment

Posterior blocking screw in the proximal fragment Correct answer: 5

Posterior blocking screws in the proximal tibial segement help to avoid tibial procurvatum deformity and malunion. Proximal third tibia fractures are often times difficult to reduce anatomically due to the tendency for both valgus and flexion deformity at the fracture site. The posterior blocking screw helps to eliminate the tendency for the nail to be too posterior and cause the fracture to flex. Blocking screws should be placed on the concavity of the deformity to minimize the procurvatum and valgus deformities of this fracture pattern.

 

Krettek et al looked at the importance of using blocking screws during intramedullary nailing of metaphyseal fractures using small diameter nails. They found less procurvatum deformity and malunions associated with using of blocking screws and found no complication with their utilization.

 

Ricci et al reviewed fractures of the proximal third of the tibial shaft treated with intramedullary nails and blocking screws. Their results supported that blocking screws were effective in maintaining alignment of fractures of the proximal third of the tibial shaft treated by intramedullary nailing.

 

Hiesterman et al review different reduction techniques to avoid malalignment, including the use of a proper starting point and insertion angle, blocking screws, unicortical plates, and a universal distractor.

 

Illustration A shows a radiograph utilizing posterior proximal and distal blocking screws to prevent procurvatum deformity.

 

Incorrect Answers:

would exacerbate procurvatum deformity.

would help avoid valgus deformity.

would exacerbate valgus deformity.

would exacerbate procurvatum deformity.

 

 

 

 

 

A 66-year-old male with a remote history of prostate cancer sustains a fall down a hill while gardening. During intramedullary nailing of his fracture, which intraoperative reduction maneuvers should take place to the proximal fragment to properly align the fracture?

 

 

Flexion and internal rotation

Extension and internal rotation

Flexion and external rotation

Extension and external rotation

Abduction and internal rotation Correct answer: 2

Subtrochanteric fractures will cause a proximal fragment to be flexed, abducted, and externally rotated due to the imbalanced proximal muscular attachments. The proximal fragment would likely have to be extended, adducted, and internally rotated to obtain a proper reduction.

 

Lundy did a review on subtrochanteric fractures. He reviews that these fractures can be effectively stabilized with 95 degrees plates, femoral reconstruction nails, or trochanteric femoral nails. Although intramedullary nails produce very stable constructs that are a great treatment option for this

fracture, 135 degrees hip screw-plates are not suitable in the treatment of subtrochanteric femoral fractures due to the high risk of loss of fixation and fracture displacement.

 

Illustration A depicts the applicable deforming muscle forces.

 

 

 

 

 

A 14-year-old presents on the request of her pediatrician for evaluation of her left hip. The patient reports having a recent history of lower abdominal pain, and as part of the work-up a KUB radiograph was obtained. The abdominal work-up was negative, and her pain has since resolved, however, the pediatrician noted an abnormal radiographic finding in the left hip and requested a formal orthopedic evaluation. The patient denies any history of hip trauma or pain. A left hip radiograph is shown in Figure A, and the the abnormality in question is indicated by the white arrow. The radiographic finding is most consistent with which of the following?

 

Os acetabuli marginalis superior

Fovea capitis

Myositis ossficans

Avascular necrosis.

Acetabular fracture

 

Correct answer: 1

 

The patients hip radiograph demonstrates an os acetabuli marginalis superior which is a benign accessory ossification center found in the superior aspect of the acetabulum. This can be commonly confused with an acute fracture or avascular necrosis. Although the os acetabuli marginalis superior occasionally persists into adult life, it usually fuses to the acetabulum by the time an individual reaches age 20.

 

Caudle et al provide a case report of a a patient with a painful os acetabuli marginalis superior. This was successfully treated with resection of the fragment, and bone grafting. This was noted to be a very unusual source of hip pain in adolescents.

 

Incorrect Answers:

The fovea capitis is the depression on the head of the femur where the ligamentum teres inserts. This can appear as a small ossicle on the surface of femoral heads in skeletally immature patients. An example of this is shown in Illustration A.

Myositis ossificans is soft tissue calcification which develops after trauma, or more rarely, surgery. An example of myositis ossificans around the hip is shown in Illustration B.

Avascular necrosis of the femoral head classically occurs in patient with a history of alcoholism, steroid use, or sickle cell disease. Radiographs can demonstrate femoral head sclerosis, and eventually collapse of the articular surface. An example of femoral head AVN in a patient with sickle cell disease is shown in Illustration C.

Acetabular fractures occur in the setting of trauma, and are relatively rare in the pediatric population. An example of a left sided acetabular fracture is shown in Illustration D. Illustration E shows a right sided acetabular fracture through the triradiate cartilage.

 

 

 

 

 

 

 

Which of the following tibial plateau fractures would be most appropriately treated by buttress plating alone?

 

 

 

Figure A

Figure B

Figure C

Figure D

Figure E

 

Correct answer: 4

 

Buttress plating is best indicated for simple partial articular fractures, such as shown in Figure D. Buttress plates can support a metaphyseal fragment and neutralize the shear and compressive forces across the cancellous bone. The actual buttress effect is only on the side of the plate. Pre-shaped plates are frequently used as buttress plates, as they conform to local anatomy, however exact contouring of the plate is necessary. Isolated buttress plating is not appropriate for articular depression fractures, such as those shown in Figures A, C, and E. Furthermore, buttress plating would provide inadequate fixation for a metadiaphyseal fracture, such as that shown in Figure B.

 

Illustration A shows an example of a buttress plate used to treat a tibial plateau fracture.

 

 

 

 

A 73-year-old female presents with persistent right shoulder pain 3 months after undergoing open reduction and internal fixation for a right proximal humerus fracture. Which of the following could have best prevented the complication shown in the current radiograph shown in Figure A?

 

Insertion of both cortical and locking screws into the humeral head

Addition of a 20-gauge intraosseous tension band laterally through the greater tuberosity

Treatment of the fracture with closed reduction and percutaneous k-wire fixation

Addition of an inferomedial locking screw within the calcar

Intramedullary nailing of the fracture Correct answer: 4

Figure A shows varus collapse and intra-articular joint penetration of the the proximal locking screws. This could have potentially been prevented by the addition of an inferomedial calcar screw, which would have provided greater strength to the fixation construct and resistance to fracture collpase.

Illustration A is an immediate post-operative fluoro image of the fracture and shows the proximal humerus to be near anatomically aligned. The illustration also demonstrates simulated overlay of where the recommended inferomedial locking screw would be placed. Illustration B shows ORIF of a different proximal humerus fracture with placement of an inferomedial calcar screw.

 

Konrad et al present a prospective, multicenter, observational study to evaluate the functional outcome and the complication rate after ORIF of proximal humeral fractures with use of a locking proximal humeral plate. At 1 year follow-up, they found an overall complication rate of 34%: most

commonly due to screw perforation through the humeral head. Nineteen percent of the patients required re-operation within 1 year of their index surgery.

 

Gardner et al discuss a technique of using a segment of fibula allograft, placed endosteally and incorporated into the locking construct, to aid in the reduction and restoration of the mechanical integrity of the medial column of the proximal humerus. An example of this fixation construct is shown in Illustration C.

 

Gardner et al, in another study, sought determine what factors influence the maintenance of fracture reduction after locked plating of proximal humerus fractures, and particularly the role of medial column support. They found that achieving an anatomic or slightly impacted stable reduction, as well as meticulously placing a superiorly directed oblique locked screw in the inferomedial region of the proximal fragment, may achieve more stable medial column support and allow for better maintenance of reduction.

 

 

 

 

 

A 54-year-old female sustains a communited tibial shaft fracture from an accident at work. She undergoes simultaneous external fixation and ORIF using minimally invasive plate osteosynthesis. Following surgery, she complains of numbness along the dorsum of her medial and lateral foot. In which location (labeled A - E) on Figure A did percutaneous placement without careful dissection of a pin/screw likely cause her nerve injury?

 

1. A

2. B

3. C

4. D

5. E

 

Correct answer: 5

 

The above clinical scenario describes a post-op superficial peroneal nerve (SPN) deficit following ORIF of a tibial fracture using both external fixation and minimally invasive plate osteosynthesis (MIPO). The less invasive stabilization system (LISS) plate by Synthes is a system which utlizes the MIPO technique. This minimally invasive technique can increase the risk of damage to the SPN without careful identification of the nerve distally due to its close proximity to LISS plate holes 11-13.

 

Deangelis et al studied the anatomy of the superficial peroneal nerve in relation to fixation of tibia fractures with the LISS plate using cadaveric dissections. They found that the superficial peroneal nerve is at significant risk during percutaneous screw placement in holes 11 through 13 of the 13-hole proximal tibia LISS plates. They recommended using an incision and careful dissection down to the plate in this region of distal locking screws to minimize the risk of damage to the nerve.

Roberts et al also studied neurovascular anatomy of the leg in relation to screw placement, but did it in relation to locking screws used in intramedullary nailing. They concluded that intramedullary nail locking screws placed from a lateral-to-medial direction minimized the risk of injuries to the SPN and tibial neurovascular bundle. A disadvantage of lateral-to-medial locking screw placement was less resistance to nail bending forces.

 

 

 

 

What structure is 7cm from the acromion and at greatest risk of injury during a deltoid splitting approach for a proximal humerus fracture?

Radial nerve

Suprascapular nerve

Axillary nerve

Axillary artery

Axillary vein

 

Correct answer: 3

 

The axillary nerve is located approximately 7cm from the tip of the acromion. The axillary nerve comes off the of the brachial plexus (middle trunk, posterior division, posterior cord) carrying fibers from C5 and C6. The axillary nerve travels through the quadrangular space with the posterior circumflex humeral artery and vein to innervate the teres minor and deltoid muscles and supply sensation over the lateral shoulder.

 

Based on the knowledge of the course of the axillary nerve, and potential complications regarding the vascular supply to the humeral head with the delto-pectoral approach, some authors are suggesting deltoid-splitting approach to the proximal humerus for reduction and fixation of proximal humeral fractures.

 

Cetik et al evaluated 24 cadaver shoulders to better identify the course of the axillary nerve and identify the "safe zones" for deltoid-splitting incisions. They found the distance from the tip of the acromion to the axillary nerve varied depending on patient specific factors like arm length and they identified a "safe area" where dissection was free of injury to the axillary nerve (Illustrations A and B).

 

 

 

 

A 48-year-old active female runner underwent percutaneous screw fixation of a minimally displaced femoral neck fracture six months ago. There were no immediate post-operative complications, and she was progressed to full weight bearing three months after surgical fixation. Initial post-operative radiographs, and radiographs taken 3 months post-op revealed anatomic reduction of the fracture with no shortening. At her latest clinic visit she reports severe right groin pain, and difficulty ambulating. A current radiograph is shown in Figure A. What is the most appropriate surgical treatment?

 

 

Hemiarthroplasty

In situ dynamic hip screw revision fixation

Core decompression and bone grafting

Valgus intertrochanteric osteotomy with blade fixation

Open reduction, bone grafting, and revision percutaneous screw fixation Correct answer: 4

The clinical presentation is consistent with a femoral neck nonunion, which is supported by the presence of new varus collapse and shortening which was not noted on prior radiographs. The most appropriate method to treat this complication is valgus intertrochanteric osteotomy of the femur with blade plate fixation.

 

Valgus intertrochanteric osteotomies function by making a vertical fracture more horizontal, converting shear into compressive forces. It also helps correct the varus position of the fracture nonunion.

 

Watson et al performed a retrospective review of the complications associated

with combination femoral neck/shaft fractures and found 13 patients who had healing complications develop after their index surgical procedure. Eight femoral neck nonunions occurred, and these healed after either valgus intertrochanteric osteotomy (seven patients) or compression hip screw fixation (one patient).

 

Jackson et al state that nonunion may occur in up to 43% of displaced intracapsular femoral neck fractures. The authors present an evidence-based algorithm regarding procedures for treatment of femoral neck nonunion and the roles of refixation, osteotomy, grafting, and prosthetic replacement when indicated.

 

Angelini et al provide a review on salvage procedures after failed fixation of hip fractures. The authors state that in the setting of a nonunion in the younger patients with a well-preserved hip joint, treatment should typically involve revision internal fixation with or without osteotomy or bone grafting. They conclude that overall, salvage of nonunions of femoral neck and intertrochanteric hip fractures in properly selected patients can provide patients with good to excellent results.

 

Figure A shows a femoral neck nonunion with varus collapse. Illustration A shows an example of a valgus intertrochanteric osteotomy with blade fixation.

 

 

 

 

 

A 32-year-old male sustains a posterior wall acetabulum fracture as the result of a high-speed motor vehicle collision. Improved patient-reported outcomes after surgical treatment are associated with which of the following variables?

Increased age

Increased hip flexion-extension arc

Immediate weight-bearing

Increased hip muscle strength

Decreased stride length Correct answer: 4

Patient functional outcomes after acetabular fractures have been shown to be related to postoperative hip strength, regardless of surgical approach.

 

The reference by Borrelli et al evaluated muscle strength and outcomes after acetabular surgery via an anterior approach. They report that hip extension strength was affected least (6%), whereas abduction, adduction, and flexion strength was affected to a greater degree. They note that hip muscle strength after operative treatment of a displaced acetabular fracture directly influences patient outcome.

 

The reference by Engsberg et al is a review of patients that underwent ORIF of acetabular fractures through anterior or posterior approaches. They report that maximizing hip muscle strength may improve gait, and improvement in hip muscle strength and gait is likely to improve functional outcome. Worsening functional outcomes were correlated with decreased gait kinematics and stride length.

 

 

 

 

A 22-year-old male undergoes retrograde intramedullary nailing for the injury seen in Figure A. Which of the following would place branches of the femoral nerve and deep femoral artery at greatest risk during placement of the interlocking screw seen in Figure B?

 

 

Anterior to posterior placement above the lesser trochanter

Anterior to posterior placement below the lesser trochanter

Lateral to medial placement above the lesser trochanter

Lateral to medial placement below the lesser trochanter

Open placement with blunt dissection down to bone Correct answer: 2

Branches of the deep femoral artery and femoral nerve are most at risk during placement of anterior to posterior interlocking screws below the level of the lesser trochanter as seen in Figure B.

 

Riina et al performed a cadaveric study examining the neurovascular structures at risk during proximal interlocking of retrograde femoral nails and found that the first division of the femoral nerve crossed the femur on average approximately 4cm distal to the piriformis fossa. In addition, the femoral artery was closest to the medial aspect of the femur 4cm distal to the lesser trochanter. The authors recommend placement of interlocking screws proximal to the lesser trochanter to minimize risk of iatrogenic neurovascular injury.

 

Brown et al attempt to define relative safe zones (RSZs) for placement of both lateral-medial and anterior-posterior interlocking screws of femoral nails in patients with acetabular fractures with reference to the location of the femoral neurvascular structures at the level of the lesser trochanter. The authors found that the available safe zone for placement of anterior-posterior screws decreased more than 50% in the setting of hematoma from acetabular fracture, and they recommend blunt dissection to bone and use of a single incison between the proximal interlocking holes to minimize risk to these structures.

 

Handolin et al present two case reports of injury to the deep femoral artery with proximal interlocking screws during retrograde nailing. Based on their experiences, the authors advocate for blunt dissection to bone to avoid entrapment of critical structures when places these screws.

 

Illustration A shows a cadaveric dissection and illustration of the numerous branches of the deep femoral artery and femoral nerve as they cross the femur from medial to lateral on the anterior surface just below lesser trochanter. The view is of the anterior thigh from the medial aspect. The sartorius has been cut and reflected and the retractors are beneath the rectus femoris.

 

 

 

 

A 23-year-old male is involved in a motor vehicle accident and sustains a left open femur fracture, right open humeral shaft fracture, and an LC-II pelvic ring injury. Which of the following best describes the radiographic findings associated with this pelvic injury pattern using the Young-Burgess Classification system?

Crescent fracture located on the side of impact

Widened anterior SI joint, disrupted sacrotuberous and sacrospinous ligaments with intact posterior SI ligaments

Complete SI disruption with lateral displacement

Sacral compression fracture on side of impact with transverse pubic rami fractures

Open-book injury with contralateral sacral compression fracture Correct answer: 1

Lateral compression type II fractures (as described by the Young-Burgess Classification System) are associated with a crescent fracture of the iliac wing located on the side of impact. A representative CT scan image and illustration of this injury are shown in Illustrations A and B respectively. A table describing each pelvic injury and their associated complications is shown in Illustration C. Illustration D shows each Young-Burgess pelvic injury type.

Burgess et al discuss the effectiveness of a treatment protocol as determined by their pelvic injury classification and hemodynamic status. The injury classification system was based on lateral compression, anteroposterior compression, vertical shear, and combined mechanical injury types. They found that their classification-based treatment protocols reduce the morbidity and mortality related to pelvic ring disruption.

 

Tile discusses acute pelvic trauma and his classification system for pelvic injuries (ie. Types A, B, and C). He states that any classification system must be seen only as a general guide to treatment, and that the management of each patient requires careful, individualized decision making.

 

Incorrect Answers:

Answer 2: This describes an APC-II injury Answer 3: This describes an APC-III injury Answer 4: This describes and LC-I injury

Answer 5: This describes an LC-III injury (ie. "wind-swept pelvis")

 

 

 

 

 

 

A 65-year-old male presents with continued left hip and thigh pain, and inability to bear full weight after undergoing ORIF of a left proximal femur fracture 3 months ago. Current radiographs are shown in Figure A. The patient denies any fevers, or other systemic signs of illness. Which of the following would have potentially decreased the risk of excess fracture collapse and implant failure in this patient?

 

Use of a six-hole 135 degree compression plate

Addition of iliac crest autograft to the fracture site

Application of long strut allografts around the fracture site

Placement of a cephalomedullary nail

Addition of an 7.3mm de-rotation screw in the femoral head Correct answer: 4

The patient is presenting with a reverse obliquity peritrochanteric fracture nonunion, as shown in Figure A. 135-degree compression plate implants (such as the one used in this patient) are designed to stabilize intertrochanteric femoral fractures, and they may be unable to resist the deforming forces inherent in subtrochanteric fractures. When this device is used, the distal fragment often displaces medially and proximally as the fracture settles. The proximal fragment also may rotate on the compression screw because the plate design allows only for one screw in the proximal fragment.

Cephalomedullary nails, such as that shown in Illustration A, have been shown to offer biomechanical superiority and diminished risk of implant failure when compared to plating of these injuries.

 

Lundy provides a review article on the evaluation and treatment of subtrochanteric femur fractures. He states that these fractures can be effectively stabilized with 95° plates, femoral reconstruction nails, or trochanteric femoral nails with interlocking options. With regards to plates, he states that a 135° hip screw-plate is not suitable in the treatment of subtrochanteric femoral fractures, and that use of these implants may result in loss of fixation and fracture displacement.

 

Menezes et al reviewed 155 consecutive patients who were treated with a proximal femoral nail from 1997 to 2001 to determine the rate of implant specific complications. They concluded that low rates of femoral shaft fractures and failure of fixation support the use of the proximal femoral nail for treatment of unstable trochanteric and subtrochanteric fractures.

 

Robinson et al used the long Gamma nail to treat a consecutive series of 302 local patients who had sustained a subtrochanteric fracture during low-energy trauma over an 8 year period. They found that trochanteric-entry cephalomedullary nails are associated with an acceptable rate of perioperative complications and favorable functional outcomes.

 

Incorrect Answers:

Increasing the length of the the 135-degree compression plate to 6-holes will not improve its biomechanical properties in this fracture pattern.

Addition of autograft would not improve the biomechanical stability of the

fracture, and is not appropriate during the index procedure.

Long strut allografts are not indicated in the initial treatment of reverse obliquity subtrochanteric fractures.

5-Addition of a de-rotation screw would not change the stability of the fixation construct.

 

 

 

 

 

A 34-year-old female sustains a proximal third tibia fracture as an isolated injury and elects to undergo operative treatment with intramedullary nailing. Which of the following operative techniques would help to best avoid a procurvatum deformity of the tibia?

Semiextended position during nailing

Lateral blocking screws in proximal tibia fragment

Use of a radiolucent triangle to flex the knee

Anterior blocking screw in the proximal tibia fragment

Medial parapatellar arthrotomy avoiding the patellar tendon Correct answer: 1

The semiextended position for intramedullay nailing of proximal tibia shaft fractures has shown to cause less complications of flexion deformity and malunion. Proximal third tibia fractures are often times difficult to reduce anatomically due to the tendency for both valgus and flexion deformity at the fracture site. The semiextended position helps to eliminate the tendency for

the fracture to flex, due to the avoidance of excessive knee flexion during the reduction. Illustration A is an example of semiextended positioning for tibial nailing. Illustration B shows radiograph utilizing posterior blocking screws to prevent procurvatum deformity.

 

Tornetta et al looked at the semiextended intramedullary nailing of proximal tibia fractures. Of the 24 patients nailed using this technique, 19 had no anterior angulation, while only 5 had less than 5 degrees of flexion deformity.

 

In their study, Kubiak et al also advocate the semiextended position for tibial nailing. They note better control of fracture alignment and stabilization especially in the sagittal plane, and better ability to obtain imaging and maintenance of reduction during nail insertion and locking.

 

Incorrect answers:

Answer 2 would help to avoid valgus deformity. Answer 3 would worsen the flexion deformity.

Answer 4 would worsen the flexion deformity and drive the nail posterior in distal segment.

Answer 5 would not affect flexion deformity.

 

 

 

 

 

Which of the following cannulated screw configurations used in the treatment of subcapital femoral neck fractures is optimal?

Inverted triangle pattern with the inferior screw posterior to midline and adjacent to the calcar

Inverted triangle pattern with the inferior screw anterior to midline and adjacent to the calcar

Triangle pattern with the superior screw posterior to midline and adjacent to the calcar

Inverted triangle pattern with the inferior screw posterior to midline and central in the femoral neck

Inverted triangle pattern with the inferior screw anterior to midline and central in the femoral neck

Correct answer: 1

 

The strongest portion of the femoral neck is the posterior inferior neck in the region of the femoral calcar. The optimal biomechanical configuration includes an inverted triangle pattern with the single screw in the inferior aspect of the femoral neck adjacent to the calcar.

 

Booth et al performed a cadaveric study comparing central versus calcar (cortical-adjacent) fixation. The results demonstrated significant improved stability, load, stiffness, and displacement in all tested parameters for the group with calcar-adjacent screw fixation.

 

Lindequist and Törnkvist performed a Level 4 study of 72 femoral neck fractures. They found that all 5 of their nonunions had screws placed greater than 3mm from the femoral calcar. Additionally, 16 of 18 fractures healed in the group of displaced fractures where both the fixating screws were placed within 3 mm from the femoral neck cortex.

 

Gurusamy et al performed a Level 4 study of 395 patients undergoing femoral neck fixation. They found a reduced spread of the screws on the lateral view was associated with an increased risk of nonunion of the fracture.

 

Illustration A depicts the optimal configuration of an inverted triangle with the single screw being inferior and all of the screws being cortical adjacent.

 

 

 

 

A 63-year-old female sustained a distal radius and associated ulnar styloid fracture 3 months ago after being involved in a motor vehicle collision. Radiographs obtained at the time of injury are shown in Figure A. She underwent open reduction and fixation of the distal radius fracture, and current radiographs are shown in Figure B. At the time of the index operation, there was no distal radioulnar joint instability after plating of the radius. Which of the following is true post-operatively regarding this patient's ulnar styloid fracture?

 

 

 

Worse outcomes on the Mayo wrist score are expected without fixation

Chronic distal radioulnar joint instability can be expected to occur without fixation

Wrist function depends on the level of ulnar styloid fracture and initial displacement

Grip strength and wrist range of motion are improved with fixation

There is no adverse effect on wrist function or stability without fixation Correct answer: 5

An accompanying ulnar styloid fracture in patients with stable fixation of a distal radial fracture has no apparent adverse effect on wrist function or stability of the distal radioulnar joint.

 

Kim et al evaluated 138 patients who underwent surgical treatment of an unstable distal radial fracture, without fixation of an associated ulnar styloid fracture if present. Postoperative evaluation included measurement of grip strength and wrist range of motion; calculation of the modified Mayo wrist score; as well as testing for instability of the distal radioulnar joint at a mean of 19 postoperatively. They did not find a significant relationship between wrist functional outcomes and ulnar styloid fracture level or the amount of displacement.

 

af Ekenstam et al performed prospective and randomized study of two different treatments of extraarticular Colles' fracture with a fractured ulnar styloid. In one group, the ulnar styloid was left alone, and in the other group it was transfixed and/or the triangular ligament was repaired after closed reduction of the fractured radius. They concluded that repair of the ulnar styloid complex in extraarticular fractures of the distal radius is not better than conventional treatment.

 

 

 

 

Each of the following are guidelines for management of a domestic violence victim EXCEPT:

Socioeconomic status should not preclude evaluation for domestic violence

Interview the patient outside the presence of other non-medical personnel

Federal law mandates photographs be taken of injuries

Document your opinion if the patient’s injuries are not consistent with the offered explanation

Physicians should check requirements to see if there is mandatory reporting statute in their state

Correct answer: 3

 

It is important to fully document the abuse as it has been described to you, however there is no federal law mandating photographic documentation of domestic violence injuries. Photographs may be taken but only with the patient's permission. Disclosure of a diagnosis of abuse to any third party and reporting it to the authorities should be done only with the abused patient’s knowledge and consent, unless there is a mandatory reporting statue in the particular state of practice. Being a female, age 19-29, pregnant, or of a low

socioeconomic status (<$10,000 per/yr) increase one's risk for domestic abuse. Pregnancy is the highest risk factor for abuse with 40% to 60% of battered women reporting that they were abused during pregnancy.

Interviewing should be done outside the presence of others to minimize potential interference by the abusive spouse/partner.

 

The review article by Zillmer outlines that as many as 35% of women presenting to ERs for trauma care have injuries that are a result of domestic violence.

 

 

 

 

An otherwise healthy 34-year-old female undergoes an elective L5-S1 microdiscectomy. At her 4 week followup, she is noted to have drainage from her wound. Labs reveal a CRP of 30 mg/L (normal < 3) and an ESR of 88 mm/hr (normal < 20). The patient undergoes irrigation and debridement followed by a 6-week course of IV vancomycin (intra-op cultures positive for MRSA). At 20-week followup, the patient had another set of CRP and ESR drawn. Which set of values would indicate that the patient's infection has been successfully treated?

CRP 20 mg/L, ESR 40 mm/hr

CRP 15 mg/L, ESR 20 mm/hr

CRP 6 mg/L, ESR 10 mm/hr

CRP 1 mg/L, ESR 25 mm/hr

CRP 8 mg/L, ESR 5 mm/hr

 

Correct answer: 4

 

A normalized C-reactive protein (CRP) value (< 3 mg/L) would be the most likely value seen in a patient with a resolution of a post-op wound infection. Erythrocyte sedimentation rate (ESR) has been shown to decrease at a much slower place and its normalization is much less specific for infection resolution.

 

CRP has been found to correlate with clinical response in patients treated with antibiotics for wound infection following spinal surgery. ESR has been found to remain elevated despite the presence of resolution of clinical symptoms and normalized CRP in these patients. It must be recognized that CRP can be elevated for reasons other than infection, including a slight increase with aging, rise during late pregnancy, and abnormal levels with liver failure.

Similarly, ESR can be increased with pregnancy, some renal diseases, cancer, and anemia. Additionally, it can be decreased in sickle cell, leukemia, congestive heart failure, and hyperviscosity. Normal ESR values are between

0-15 mm/hr for men under age 50 and between 0 and 20 mm/hr and for women under age 50.

 

Khan et al. studied 21 patients with postoperative wound infections after spinal surgery. Of 21 patients they found resolution of all postoperative wound infections at the 20-week mark, and found that all patient had normalized CRPs (mean 0.8 mg/dL). However, they found ESR remained elevated (mean 29 mm/hr) despite normalized CRPs and complete resolution of signs and symptoms of infection.

 

Fang et al. reviewed 48 patients with post-operative spinal surgery wound infections and compared to a random control group of 95 uninfected spinal surgery patients. They noted that Age >60 years, smoking, diabetes, previous surgical infection, increased body mass index, and alcohol abuse were all statistically significant preoperative risk factors for developing a post-op wound infection.. The most likely procedure to be complicated by an infection was a combined anterior/posterior spinal fusion performed in a staged manner under separate anesthesia. Infections were primarily monomicrobial and the most common organism cultured from the wounds was Staphylococcus aureus.

 

Illustration A shows the CRP trends in the study by Khan et al. with postoperative wound infections after spinal surgery treated with IV antibiotics. ER refers to early responder while LR refers to late responder. At the 4-week time point, 16 patients have shown clinical improvement with no fevers, wound drainage, erythema, or need for wound packing. The other 5 patients, referred to as late responders, have shown one or more clinical signs of infection in addition to having an elevated CRP. At 20 weeks, both early and late responders both have shown normalized CRP levels and no clinical evidence of infection. Illustration B shows the ESR trends in the those which demonstrated incomplete normalization of ESR values despite resolution of infection.

 

Incorrect Answers:

Answer 1, 2, 3, 5: CRP levels > 3 mg/L are elevated and indicate incomplete response to infection

 

 

 

 

 

A 25-year-old man is struck by car while crossing the street. His injuries include the closed left tibial shaft fracture shown in Figure A. He is a smoker, but is otherwise healthy. Intramedullary nailing is performed without initial complications. Which of the following puts this patient at greatest risk for tibial nonunion?

 

Use of anti-inflammatories post-operatively

Post-operative gapping at the fracture site

Presence of an associated fibular fracture

History of smoking

Mechanism of injury Correct answer: 2

Post-operative gapping at the fracture site significantly increased the risk of reoperation due to nonunion or malunion.

 

Bhandari et al performed a retrospective study to identify which prognostic factors were associated with an increased risk of reoperation for nonunion in surgically treated tibial shaft fractures. They examined over 200 fractures, and found the presence of an open fracture wound (RR 4.32), lack of cortical continuity between the fracture ends following fixation (RR 8.33), and the presence of a transverse fracture (RR 20.0) were the three variables most predicitive of reoperation.

 

Audige et al analyzed 416 patients with operatively treated tibial shaft fractures who were followed for at least 6 months. They found that the greatest risk for delayed healing or nonunion was the presence of an open injury, fractures of the distal 1/3 of the tibia, and postoperative gapping at the

fracture site (The risk of healing problems was doubled for fractures of the distal shaft and for fractures showing a postoperative diastasis).

 

 

 

 

Figure A is a radiograph of a healthy, independent 51-year-old male. He is treated with immediate open reduction internal fixation to prevent which of the following complications?

 

 

Fracture non-union

Avascular necrosis

Skin necrosis

Plantar flexion weakness

Ankle stiffness

 

Correct answer: 3

 

Figure A shows an avulsion fracture of the calcaneal tuberosity. Immediate open reduction and internal fixation is required to prevent wound complications.

 

Displaced avulsion fractures of the calcaneal tuberosity should be managed urgently to prevent necrosis of the soft tissues overlying the heel. In these injuries, the Achilles tendon is securely attached to the fractured tuberosity. Urgent closed reduction and casting is usually not possible due to the power and proximal pull of the triceps surae. Surgical fixation is required. The best treatment modality is open reduction and bone-to-bone fixation with screws. Closed reduction and percutaneous pinning fixation is not strong enough to provide a stable fixation of the tuberosity.

Lui reported on avulsion fractures of the bony insertion of the Achilles tendon at the calcaneus. He stated that screw fixation alone is not sufficient for repair of these injuries. His technique involved two suture anchors used capture the small bone fragment to the calcaneus. This allowed for the pull of the triceps surae to be neutralized and early physical therapy.

 

Hess et al. looked at a case series of calcaneal tuberosity avulsion fractures that were treated in a delayed fashion. All three patients with posterior tuberosity calcaneal avulsion fractures developed skin necrosis because of a delay in treatment.

 

Figure A shows a displaced posterior tuberosity calcaneal avulsion fracture. Illustration A shows skin breakdown overlying the posterior tuberosity calcaneal avulsion fracture.

 

Incorrect Answers:

Answer 1: The amount of displacement is an indication for fixation, however urgent treatment does not improve union rates with these fractures.

Answer 2: Tuberosity calcaneal avulsion fractures rarely disrupt the blood supply to the avulsion fragment and are not associated with avascular necrosis.

Answer 3: Plantar flexion weakness is a known complication of these injuries despite many treatment options.

Answer 4: Ankle stiffness is most commonly related to surgical fixation methods and post-operative immobilization and delayed rehabilitation.

 

 

 

 

A 22-year-old male presents 4 weeks following open reduction and internal fixation of his unstable ankle fracture. He has had three days of increasing pain, swelling and the new onset of purulent drainage from the mid-portion of the lateral incision. Laboratory values, including white blood cell count, sedimentation rate, and C-reactive protein are elevated. Current radiographs are seen in Figures A and B. On examination the wound probes deep and likely involves the lateral plate. What is the best step in management at this time?

 

Suppression with broad spectrum oral antibiotics until fracture healing

Suppression with broad spectrum intravenous antibiotics until fracture healing

Surgical debridement, removal of internal fixation, culture specific antibiotics, casting until fracture healing

Surgical debridement, maintenance of internal fixation, culture specific antibiotics until fracture healing

Wound culture in the office and suppression with culture specific antibiotics until fracture healing

Correct answer: 4

 

The patient is presenting with an acute deep infection following open reduction and internal fixation of an unstable ankle fracture. Recent studies have shown that a protocol of early aggressive surgical debridement, maintenance of internal fixation and culture specific antibiotics can be effective at achieving fracture healing.

 

Management of early postoperative infection following open reduction and internal fixation can be challenging. Effective treatment typically involves a combination of surgical debridement and culture specific antibiotics. Removing internal fixation prior to fracture healing can lead to additional insult to the soft tissue and ongoing inflammation secondary to fracture instability. Recently published protocols have shown effective treatment with maintenance of implants and culture specific antibiotics following early, aggressive surgical debridement.

Berkes et al. performed a multi-center retrospective study of 121 patients with acute postoperative infection (defined as less than 6 weeks from surgery) following internal fracture fixation. The authors demonstrated a 71% rate of success (defined by maintenance of implants until fracture healing) with a protocol of debridement and suppression with culture specific antibiotics. Risk factors for failure of this technique include open fractures and the use of an intramedullary nail for fracture fixation.

 

Figures A and B show an ankle status post open reduction and internal fixation of a lateral malleolus fracture. There are no signs of loosening of fixation or cortical erosions concerning for osteomyelitis.

 

Illustrations A and B are weight bearing X-rays that demonstrate the same fracture, now healed, after debridement and culture specific antibiotics.

Illustrations C and D demonstrate the same fracture after elective removal of implants at 10 months following the index procedure. Intraoperative cultures at the time of hardware removal were negative for recurrent infection.

 

Incorrect answers:

Answers 1 and 2: Broad spectrum antibiotics without surgical debridement would not be effective in dealing with the infection in this clinical scenario with purulence tracking to the level of the plate

Answer 3: Surgical debridement and antibiotics would control the infection adequately, however removal of fracture fixation in an unstable fracture would lead to instability, soft tissue inflammation and likely malunion or nonunion that would require complex revision

Answer 5: Although culture specific antibiotics are an improvement over broad spectrum, again surgical debridement in conjunction with antibiotics is the most appropriate choice in this scenario. Intraoperative deep cultures in a sterile environment are preferred over cultures obtained in the office.

 

 

 

 

 

A 28-year-old male sustained an ankle injury 3 months ago, and was treated with closed management and splinting; a current x-ray is shown in Figure A. Which of the following is the most important factor in deciding between a joint sacrificing and a joint preserving operation for this patient at this time?

 

Workers' Compensation involvement

Gender

Date of injury

Degree of tibiotalar arthritis

Degree of deformity Correct answer: 4

This patient presents with malunion after sustaining a bimalleolar ankle fracture. Surgical treatment options consist of osteotomy for deformity correction with internal fixation (joint preserving) versus fusion (joint sacrificing) with the primary determinant being the amount of ankle arthritis present.

 

When treating ankle malunions, the decision to perform deformity correction and preserve the joint versus fusing the joint is dependent on signs of progressive, advanced tibiotalar arthritis on radiographs.

 

Yablon and Leach followed 26 patients following corrective fibular osteotomy following malunion and noted excellent results at a mean follow-up of 7 years. All but 3 returned to preinjury level of activity and had desired outcomes following deformity correction.

Reidsma et al. prospectively followed 57 patients with a minimum follow-up of 10 years following corrective osteotomy and fixation for ankle fracture malunions. Good to excellent results were obtained for 85% of the cohort. The authors concluded that those ankle fracture malunions with none to minimal arthritic changes should still receive corrective osteotomy to prevent further progression of arthritis.

 

Yablon et al. following 53 patients for 6-9 months, reported on the importance of anatomic restoration of the lateral malleolus when fixing bimalleolar ankle fractures. With anatomic reduction and fixation, no progression of arthritis was noted with return to function.

Figure A is an AP radiograph of a right ankle fracture malunion. Incorrect answers:

Answers 1-3, 5: Factors such as age, gender, and degree of deformity are not

as important as the presence of advanced arthritis that may suggest requiring a joint sacrificing fusion procedure. The date of injury may portend a poor outcome (the longer from the date of injury/subsequent malunion), but is not the most important factor in deciding between corrective osteotomy versus fusion.

 

 

 

 

A 45-year-old male presents after falling off of a ladder. Radiograph is shown in Figure A. Which of the following is the appropriate sequence in management?

 

Closed reduction, splint application, computed tomography (CT) scan, delayed open reduction and internal fixation

Closed reduction, cast application, close observation

Splint application, CT scan, application external fixator, delayed open reduction and internal fixation

Splint application, application external fixator, CT scan, delayed open reduction and internal fixation

Splint application, acute open reduction and internal fixation Correct answer: 4

This patient has sustained a pilon fracture, with severe comminution and impaction at the articular surface.

 

The correct sequence of management includes (1) immediate splinting, (2) application of an external fixator, (3) restoration of length alignment and rotation with temporizing external fixation, (4) computed tomography, followed by (5) definitive fixation once soft tissues are amenable.

 

Tornetta and Gorup analyzed the use of preoperative CT scans in comparison to radiographs in preparation for fixing pilon fractures. The authors noted increased recognition of intra-articular fragments, comminution and noted a high percentage of operative planning changes following CT analysis.

Furthermore, the authors recommended CT scans AFTER external fixation, for even better fragment characterization.

 

Marsh et al. in their instructional course lecture provide tips and tricks in successful management of pilon fractures. One of the highlighted points include staged, delayed treatment of pilon fractures via spanning external fixator as well as highlighting the importance of obtaining the CT after restoring length and alignment.

 

Figure A exhibits a radiograph of a comminuted, impacted, shortened pilon fracture.

 

Incorrect answers:

Answer 1: With such a short, impacted fracture, external fixation as a temporizing measure is recommended to help calm the soft tissue envelope. Answer 2: Closed treatment is not accepted in this type of fracture, due to the high incidence of arthritis and malunion.

Answer 3: CT scan should be obtained AFTER external fixation, not before. Answer 5: This high-energy injury likely has severe soft tissue injury, which is not amenable to acute fixation.

 

 

 

A 45-year-old male with long-standing diabetes sustains the injury shown in Figure A. He has a BMI of 38, established peripheral neuropathy, and his most recent HbA1c is 8.8. What is the most appropriate definitive management option of Figures B through F?

 

 

Figure B

Figure C

Figure D

Figure E

Figure F

 

Correct answer: 4

 

Open reduction and internal fixation (ORIF) remains the mainstay of treatment for ankle fractures in patients with diabetes.

 

ORIF for ankle fractures in diabetics can be augmented with increased density of fixation to account for notable, pathologic bone. Specifically, multiple quadricortical syndesmotic screws, bicortical medial malleolar screws, and stiffer plates are all viable options. Furthermore, due to delayed healing properties, prolonged immobilization may also be required to avoid fixation failure.

 

Guo et al. performed a cohort controlled comparison between diabetics and non-diabetic patients with operative ankle fractures. Although they hypothesized that there would be more complications in the diabetic group, there was no statistical differences in fixation failure or complications when adhering to treatment principles for diabetics (including prolonged non-weight bearing for 10-12 weeks and increased density of fixation).

 

Chaudhary et al. review the notable complications following ankle fracture treatment in patients with diabetes. In reviewing the literature, the authors recommend ORIF with meticulous soft tissue handling, increased density of fixation, and prolonged immobilization as the mainstay of diabetic ankle fracture treatment. External fixation and frames, while treatment options, should be reserved for salvage or infectious clinical scenarios.

 

Figure A depicts a bimalleolar ankle fracture. Figure B depicts a cam walker.

Figure C depicts a cast. Figure D standard fixation for ankle fracture for a patient without diabetes. Figure E exhibits definitive fixation with additional screws to increase construct stability. Figure F depicts a ring fixator.

 

Incorrect Answers:

Answers 1,2: Non-operative treatment is not appropriate for this fracture pattern

Answer 3: Figure D, typically used for those without diabetes is less appropriate for this patient/clinical scenario.

Answer 5: While external fixation and a ring fixator may be utilized, ORIF should be attempted first, however, as it provides stiffer fixation.

 

 

 

 

Distraction bone block arthrodesis alone would most likely help a patient suffering from a painful calcaneus fracture malunion with all of the following except:

Low talar declination angle

Hindfoot varus alignment

Subtalar arthritis

Talonavicular subluxation

Peroneal impingement

 

Correct answer: 5

 

Distraction bone block arthrodesis in isolation would be unlikely to improve pain related to peroneal impingement. Lateral wall exostectomy would likely be needed for this, and should be concomitantly performed in most cases.

 

Calcaneal fracture malunions demonstrate several common patterns. Patients may manifest pain from anterior tibiotalar impingement (a result of a low talar declination angle), difficulty with shoe wear due to shortening and widening of the hindfoot, painful talonavicular subluxation, subfibular impingement, and post-traumatic subtalar osteoarthritis. The distraction bone block arthrodesis procedure was developed to eliminate the pain of subtalar arthritis while simultaneously normalizing hindfoot height by inserting a contoured structural bone graft into the subtalar joint. The procedure can correct pathologic hindfoot varus/valgus and restore a normal talocalcaneal angle, thereby improving symptomatic post-traumatic pes planus. The procedure alone does not address lateral wall blowout causing subfibular or peroneal impingement, which requires lateral wall exostectomy. Lateral wall exostectomy therefore should be done in addition to address this issue.

Carr et al. first reported the use of subtalar distraction bone block arthrodesis for salvage of post-traumatic subtalar arthritis following calcaneus fractures. In their 16 patient series, distraction of the subtalar joint with insertion of the bone block allowed for correction of talocalcaneal angle, restoration of hindfoot height, and improvement in symptoms related to tibiotalar impingement.

 

Clare et al. evaluated a treatment protocol for calcaneal malunions based upon the classification of Stephens and Sanders. Type II and III malunions were treated with subtalar bone-block arthrodesis and other concomitant procedures. They found that their treatment protocol was effective for pain relief, re-establishing a plantigrade foot, and improving function. The most difficult component of the case was restoration of calcaneal height.

 

Braley et al. evaluated isolated lateral decompression in the treatment of symptomatic calcaneal malunions without concomitant subtalar arthrodesis. These 11 patients had persistent lateral sided pain and their malunions did demonstrate subtalar involvement. The authors reported 9 of the 11 patients had a satisfactory outcome with lateral decompression alone. They concluded that a lateral decompression in management of symptomatic malunions with lateral-sided symptoms is an essential consideration.

 

Incorrect Answers:

Answer 1: Talar declination angle describes the relative plantar- or dorsiflexion of the talus relative to the ground.

Answer 2: Hindfoot alignment can be improved with distraction arthrodesis. Answer 3: Arthrodesis addresses painful subtalar arthritis.

Answer 4: Restoration of the normal talocalcaneal angle can be achieved, which can also normalize the talonavicular relationship.

 

 

 

 

A 31-year-old patient has had activity related lateral ankle pain for 4 months. She underwent the procedure shown in Figure A approximately 8 months ago. What surgical technique has most likely resulted in this patients pain?

 

Failure to recognize the most distal screw penetrating the joint surface

Low posterior plating with prominent distal screw

Failure to recognize an associated syndesmosis disruption

Fracture malreduction causing shortening of the fibula

Failure to use a longer plate with the lag screw positioned outside the plate Correct answer: 2

Figure A shows a low posterior plate with a prominent screw head in the most distal hole of the plate. This fixation technique is correlated with peroneal pathology, which usually presents months after fixation when the patient increases their activity level.

 

The two most common fixation techniques of lateral malleolus fractures are (1) lag screw plus lateral neutralizing plating and (2) posterolateral antiglide plating. The disadvantages of the lateral plating includes the risk of intraarticular screws distally, prominent lateral hardware, and poor distal screw fixation. To overcome these complications, posterolateral antiglide plating allows for bicortical distal fixation with no articular perforation. However, low placement of the plate with a prominent screw head in the most distal hole is associated with symptomatic peroneal pathology. If the most distal screw is not prominent, or absent, this is less likely to cause peroneal complications.

 

Weber et al. examined the effect of antiglide plate and screw positioning on peroneal tendon pathology. They showed that low posterior plating and large screw heads caused significant retromalleolar pain in most patients. To decrease peroneal pathology, they state that the distal end of the plate should stay proximal to the osteosynovial peroneal groove. Radiologically this level

corresponds to the junction of the proximal and middle thirds of the lateral malleolus.

 

Figure A shows a posterior positioned 5 hole 1/3 tubular plate. There is no articular screw penetration and the fracture is healed in an anatomical position. The distal aspect of the plate is is the distal third of the lateral malleolus.

 

Incorrect Answers:

Answer 1: The most distal screws rarely penetrates the joint with the use of fibular antiglide plates. In addition, there is no evidence of screw penetration in this patient.

Answer 3: A missed syndesmosis disruption would usually show some radiographic findings. The tibiofibular clear space is usually most sensitive, which is measured radiographically by the distance from the lateral border of the posterior malleolus in the distal tibia to the medial border of the fibula. As a general rule, it is considered normal if the measurement is less than approximately 6 mm on both AP and mortise views.

Answer 4: The fracture reduction looks anatomic. The talocrural angle, 'dime' sign and “Shenton's line” of the ankle all normal.

Answer 5: The construct used to fix this isolated lateral trans-syndesmotic fracture is acceptable. The one-third tubular plate, which is placed posterolaterally on the fibula as an antiglide plate, indirectly reduces the fracture and acts as a buttress to resist the posterior and proximal displacement of the distal fragment. Insertion of a lag screw through the plate is a described technique.

 

 

 

 

A 35-year-old painter falls from a ladder sustaining an isolated fracture of his left calcaneus. Months later at follow-up, he is noted to have pain and a catching sensation in his medial foot with active flexion of the great toe. What is the most likely initial injury leading to this complication?

Displaced calcaneal beak fracture

Displaced fracture of the calcaneal tuberosity

Comminuted posterior facet fracture

Fracture of the sustentaculum tali

Lateral wall blowout fracture Correct answer: 4

A known complication of fractures of the sustentaculum tali is stenosis (delayed) or injury (acute) of the flexor hallucis longus (FHL) tendon. Stenosis

can cause pain and popping with great toe flexion.

 

Fractures of the calcaneus often occur after falls from height, and in addition, may be associated with vertebral fractures due to the high-impact mechanism. The FHL tendon runs directly underneath the sustentaculum tali on the medial calcaneus and can be injured causing frank tears or delayed stenosis. These fractures may be missed on ankle or foot plain films alone, and advanced imaging should be ordered if clinical suspicion for calcaneus fractures exists given mechanism and location of pain/swelling.

 

Komiya et al. present a case report of direct impalement of the FHL tendon in the tunnel under the sustentaculum tali. Though this particular complication is quite rare, such a report highlights the relevant anatomy and structures at risk as well as demonstrates the need for a good clinical exam as the injury was not noted on imaging but the concern was raised on physical exam of FHL involvement.

 

Della Rocca et al. report their 19-patient series on operatively managed isolated sustentaculum tali fractures. They report a high rate of associated ipsilateral foot injuries (14 patients) and describe fixation using a medial approach and retracting the flexor tendons and neurovascular bundle.

 

Illustration A is an axial CT image from Della Rocca (2009) et al. showing a representative sustentaculum tali fracture

 

Incorrect Answers:

Answer 1 - beak fractures are posterior, anatomically related to the Achilles tendon insertion, not the FHL tendon.

Answer 2 - FHL tendon is not associated with the tuberosity.

Answer 3 - FHL tendon runs anterior to the posterior facet and would unlikely be involved in that injury.

Answer 5 - FHL tendon runs medially, not laterally.

 

 

 

 

A 30-year-old male patient involved in a hang-gliding accident sustains a knee dislocation with multiligamentous knee injury and transection of his peroneal nerve. He undergoes multiple reconstructive surgeries. Two years later, he continues to have a foot drop and dynamic tendon transfer is recommended. This treatment most commonly involves transferring a tendon from which native insertion point to which new insertion point?

Plantar distal phalanges to medial navicular

Medial navicular to dorsal lateral cuneiform

Plantar 1st metatarsal to dorsal lateral cuneiform

5th metatarsal base to dorsal medial cuneiform

Plantar distal phalanx of the hallux to dorsal distal phalanx of hallux Correct answer: 2

Dynamic tendon transfer to restore active dorsiflexion of the foot involves transferring the posterior tibial tendon (PTT) insertion on the medial navicular to the dorsal lateral cuneiform.

 

Common peroneal nerve (CPN) injuries following traumatic knee dislocation are common, with an incidence of 25-40%. CPN palsy is characterized by foot drop

due to loss of ankle dorsiflexors with a steppage gait and eventual development of a supinated equinovarus foot secondary to the unopposed pull of the PTT. Nonsurgical management involves use of an ankle-foot orthosis and physical therapy. Surgical options include acute primary repair, nerve grafting with either autologous sural nerve or nerve conduits and dynamic tendon transfer. The PTT is harvested from its insertion at the navicular, passed through the interosseous membrane (IOM) and anchored to the lateral cuneiform (see Illustration A). The classic bridle procedure involves concomitant anastamosis of the PTT to the tibialis anterior (TA) and peroneus longus (PL) tendons.

 

Garozzo et al reported a case series of 62 patients with post-traumatic CPN palsy who underwent a one-stage procedure consisting of nerve repair and PTT transfer. Nerve repair combined with PTT transfer improved postoperative outcomes compared to nerve repair alone. At 2-year follow up, neural regeneration was demonstrated in 90% of patients. The authors hypothesized that poor outcomes following nerve repair alone are due to force imbalance between the functioning flexors and paralyzed extensors, which is somewhat equalized by performing a PTT transfer at time of repair.

 

Niall et al reviewed 55 patients with traumatic knee dislocation and reported a 41% incidence of CPN injury, exclusively associated with dislocations involving disruption of the posterior cruciate ligament (PCL) and posterolateral corner (PLC). Complete neurologic recovery was found in only 21% of patients. The best prognosis was found with lesions in continuity, less than 7cm of nerve involvement, and short conduction block and muscle activity on nerve conduction and EMG studies.

 

Vigasio et al described a dynamic tendon transfer technique for traumatic complete CPN injury, involving transfer of the PTT to the TA rerouted to a new origin at the lateral cuneiform to restore ankle dorsiflexion and flexor digitorum longus (FDL) to the extensor digitorum longus (EDL) and extensor hallucis longus (EHL) to restore digit dorsiflexion. Rerouting the TA towards the transferred PTT ensures the PTT harvest length is sufficient. This avoids excessive tensioning of the PTT, which may limit tendon excursion and result in a static tenodesis rather than dynamic function, as well as the need for PTT lengthening which may decrease strength of the transfer

 

Illustration A is a series of intraoperative photographs demonstrating PTT transfer from Garg et al. An incision is made distal to the medial malleolus and the PTT is harvested subperiosteally (A). The PTT is delivered through a second incision ~15cm proximal to the medial malleolus (B-C). The PTT is then passed through the interosseous membrane and out a third incision over the anterior

fibula (D). Lastly, the PTT is passed through a fourth incision over the dorsal midfoot and anchored to the lateral cuneiform (E).

 

Incorrect Responses:

Answer 1: Transferring the FDL (insertion = plantar distal phalanges) to the medial navicular is used for correction of flexible flatfoot deformity arising from PTT insufficiency. Some surgeons transfer the FDL to the medial navicular at the time of PTT transfer to the dorsum of the foot, to compensate for loss of PTT function and minimize risk of flatfoot development.

Answers 3: Transferring the PL (insertion = plantar 1st metatarsal) is not recommended, as this muscle is innervated by the CPN via the superficial peroneal nerve and therefore would not be functional.

Answer 4: Transferring the peroneus brevis (PB; insertion = 5th metatarsal base) is not recommended, this muscle is innervated by the CPN via the superficial peroneal nerve and therefore would not be functional.

Answer 5: Transferring the flexor hallucis longus (FHL; insertion = plantar distal phalanx of the hallux) to the insertion of the EHL (dorsal distal phalanx of hallux) is recommended for correction of claw toe deformity and would not help restore foot dorsiflexion in this patient.

 

 

 

 

 

Which of the following represents the most common complication following operative treatment of the injury shown in Figure A?

 

 

 

Figure B

Figure C

Figure D

Figure E

Figure F

 

Correct answer: 3

 

This patient has a displaced talar neck fracture. The most common complication is post-traumatic arthritis.

 

Complications after treatment of displaced talar neck fractures are common. Both tibiotalar and subtalar arthritis occur, with subtalar arthritis being the most common. Osteonecrosis is also common, but post-traumatic arthritis is the most common.

 

Lindvall et al. reviewed 26 displaced talar fractures treated with internal fixation and found that post-traumatic arthritis was the most common complication, occurring in 100% of patients. Osteonecrosis was found in 13 of 26 patients (50%).

 

Vallier et al. reviewed patients presenting with talar neck fractures and found post-traumatic arthritis in 21 of 39 patients (54%). Osteonecrosis was found in 19 of 39 patients (49%).

 

Figure A shows a displaced talar neck fracture. Figure B shows talar neck nonunion (arrow) and osteonecrosis of the talar body. Figure C is an AP of the same patient shown in Figure B and again shows osteonecrosis of the talar body. Figure D shows subtalar arthritis after internal fixation of a talar neck fracture via medial malleolar osteotomy. Figure E shows a clinical photo of a patient with a varus malunion after talar neck fracture. Figure F shows a wound dehiscence.

 

Incorrect answers:

Answers 1, 2, 4, 5. These are all known complications of surgical treatment of displaced talar neck fractures but occur less frequently than post-traumatic arthritis.

 

 

 

 

A 25-year-old woman began training for a marathon and she reports a 2-week history of heel pain. She has pain throughout the day that worsens with prolonged weight-bearing. On exam, the location of

maximal tenderness is indicated by the white arrow in Figure A. The patient denies point tenderness at the location of the yellow arrow in Figure A. Which of the following MRI images (Figures B to F) would you expect to find in this patient?

 

 

 

 

 

Figure B

Figure C

Figure D

Figure E

Figure F

 

Correct answer: 2

 

The clinical presentation is consistent with a stress fracture of the calcaneus. The T1 MRI shows a fracture line within the calcaneus which is consistent with

a calcaneal stress fracture.

 

Calcaneal stress fractures most commonly occur in long-distance runners and military recruits. Usually, they are caused by overload and inability of bone formation to match resorption. Patients usually begin a rapid increase of activity level from a prior sedentary lifestyle. Patients complain of heel pain that persists throughout the day. The pain is located along the medial and lateral walls of the calcaneus. Diagnosis can be made with radiographs 2-3 weeks after symptom onset by the appearance of a sclerotic line. If the diagnosis is uncertain, MRI may be obtained.

 

Gehrmann et al. performed a review of stress fractures in the foot. They report that most studies of calcaneal stress fractures are from military recruits.

Primary treatment includes rest, avoidance of weight-bearing, and physical therapy. Patients usually return to full duty between 8 and 10 weeks. Recruits with inadequate treatment had a recurrence of symptoms. They conclude that most studies are from the 1940’s and 1950’s and lack any significant detail into specific treatment protocols.

 

Sormaala et al. performed a retrospective study in which they reviewed MRIs in all military recruits who had exercise-induced heel pain, over an eight-year period. They report that only 15% of injuries were visible on radiographs and a portion of patients had stress fractures of another tarsal bone. They conclude that a majority of calcaneal stress fractures occur in the posterior part of the bone, but some fractures may be found in the middle and anterior parts. They recommend obtaining an MRI if a radiograph is negative in a patient with exercise-induced foot or heel pain.

 

Figure A demonstrates the most common area of tenderness and pain of a calcaneal stress fracture, indicated by the white arrow, while the yellow arrow points to the area of tenderness consistent with plantar fasciitis. Figure B is a sagittal T2-weighted MRI demonstrating plantar fasciitis. Figure C is a sagittal T1-weighted MRI demonstrating a calcaneal stress fracture. Figure D is an axial T1-weighted MRI demonstrating a mass on the medial aspect of the calcaneus consistent with a ganglion cyst. Figure E is a sagittal T1-weighted MRI of the foot demonstrating an intraosseous lipoma within the calcaneus. Figure F is a sagittal T2-weighted MRI demonstrating insertional Achilles tendinopathy.

Illustration A demonstrates the most common etiologies of plantar foot pain.

 

Incorrect Answers:

Answer 1: Figure B, a sagittal T2-weighted MRI, demonstrates plantar fasciitis. Symptoms of plantar fasciitis are sharp heel pain, usually worse in the morning. Tenderness is usually found on the plantar aspect of the foot.

Answer 3: Figure D, an axial T1-weighted MRI, demonstrates a ganglion cyst in the tarsal tunnel. Symptoms of tarsal tunnel syndrome include sharp burning pains in the foot and parasthesias and numbness in the plantar foot.

Answer 4: Figure E, a sagittal T1-weighted MRI, demonstrates an intraosseous lipoma. Intraosseous lipomas are usually incidental findings.

Answer 5: Figure F, a sagittal T2-weighted MRI, demonstrates insertional Achilles tendinitis. This usually presents with pain over the insertion of the Achilles. It may have a relapsing and remitting course, which worsens with activity.

 

 

 

 

 

A 35-year-old male fell and sustained an open talar neck fracture. He underwent operative fixation of his fracture. He presents at 2 months after surgery. He denies any constitutional symptoms and his pain is well controlled. On exam, his wounds are well healed with no erythema. Imaging is shown in Figure A. What can the patient be told about his condition?

 

Hawkins sign is positive. The likelihood of developing osteonecrosis is high

Hawkins sign is positive. The likelihood of developing osteonecrosis is low

Hawkins sign is negative. The likelihood of developing osteonecrosis is high

Hawkins sign is negative. The likelihood of developing osteonecrosis is low

He has developed chondrolysis Correct answer: 2

This patient has a positive Hawkins sign, which signifies that he is unlikely to develop osteonecrosis.

 

A subchondral radiolucency of the talar dome after a talar neck fracture is known as the Hawkins sign. It is an indication that the talar body is viable. It usually appears by 6-8 weeks after injury and is best seen on the mortise view. If the Hawkins sign is present, it is unlikely that the patient will develop osteonecrosis.

 

Early writes a review on talus fracture management. He reports that talar neck fractures occur through the extra-articular portion of the talus and represent nearly half of talus fractures. The mechanism of injury is a combined ankle dorsiflexion followed by axial compression of the tibiotalar joint. He concludes that anatomic reduction gives the patient the best chance of a good outcome.

Leduc et al. wrote a review on posttraumatic avascular necrosis of the talus. They note that AVN is diagnosed on plain radiography by the absence of the Hawkins sign. They report that CT can also reveal characteristic talar AVN patterns and can be used to confirm radiographic findings. CT helps to assess subtle depression, collapse, fragmentation, and arthritic changes. MRI remains the most sensitive technique for detecting osteonecrosis of the talus, especially in the early stages. They conclude that although there are many published treatments of AVN of the talus, outcome studies are still lacking.

 

Tezval et al. performed a retrospective review to determine the prognostic reliability, sensitivity, and specificity of the Hawkins sign. They found that in all patients who developed osteonecrosis, none had the Hawkins sign present. In all patients that exhibited a Hawkins sign, none developed osteonecrosis. They determined the sensitivity of the Hawkins sign to be 100% while the specificity was 57.5%. They conclude that if a full or partial positive Hawkins sign is detected, it is unlikely that AVN will develop.

 

Figure A demonstrates the Hawkins sign with subchondral radiolucency which is noted by arrows in Illustration A.

 

Incorrect Answers:

Answers 1, 3, 4: The patient has a positive Hawkins sign, therefore the chance of developing osteonecrosis is low.

Answer 5: Chondrolysis is characterized by rapid destruction of articular cartilage on both sides of the joint which leads to loss of joint space

 

 

 

 

A 25-year-old male presents to the emergency department after a lawnmower accident with traumatic loss of his great toe. On examination, his wound is grossly contaminated with soil. In addition to a cephalosporin and an aminoglycoside, penicillin is given. Which of the following is true with regards to the organism that penicillin is targeting in this injury?

It is an Aerobic, Gram-positive rod

It is an Anaerobic, Gram-positive coccus

It is an Anaerobic, Gram-negative rod

It is Catalase positive

It may cause botulism Correct answer: 5

The organism being covered with penicillin is Clostridia spp. Clostridium botulinum is a Gram Positive Bacilli that is the cause of botulism.

 

Clostridia spp, is a Gram-positive, obligate anaerobic spore-forming rod that is found in soil and gut flora. It produces gas by the fermentation of glucose and

may cause gas gangrene. Common bacteria of this genus are C. perfringens (most common), C. tetani (causes tetanus), C. difficile, and C. botulinum (causes botulism). If wounds are grossly contaminated with soil, penicillin is given to cover against Clostridia.

 

Decoster et al. performed a review of traumatic foot wounds. They report that lawnmower injuries to the foot are relatively common. IV antibiotic therapy should be initiated with a broad-spectrum cephalosporin and an aminoglycoside to provide coverage against Gram-negative organisms.

Penicillin should be given to protect against Clostridial infection. They conclude that irrigation and debridement is indicated initially followed by packing of open wounds. Repeat debridements are necessary as nonviable tissue demarcates. If major reconstructive procedures are necessary, they should be delayed as they have a high rate of failure if performed too soon.

 

Cross et al. wrote a review on treatment principles in the management of open fractures and they note that in open fractures with soil contamination, additional coverage should be added for anaerobic bacteria, typically Clostridia. Another member of the genus Clostridia is C. tetani, the causative agent of tetanus. Vaccine status for tetanus must also be assessed in these situations as well.

 

Illustration A is a radiograph demonstrating soft tissue swelling and subcutaneous emphysema, consistent with gas gangrene. Illustration B is a clinical photograph of gas gangrene evidenced by edema, discoloration, ecchymosis, and hemorrhagic bullae. Illustration C is a pathology slide of C. perfringens, a Gram-positive rod.

 

Incorrect Answers:

Answers 1, 2, 3: Clostridia are Gram-positive, obligate anaerobes. Answer 4: Clostridia are catalase negative.

 

 

 

 

 

All of the following are ways that a negative pressure dressing is beneficial to wound healing EXCEPT:

Accelerated granulation tissue formation

Removes excess proteins and electrolytes from wound

Reducing anaerobic colonization

Causes cells to release vascular endothelial growth factor by mechanical force

Causes an increase in capillary afterload Correct answer: 5

Negative pressure dressings or vacuum-assisted wound closures (VAC) apply a negative pressure to the wound bed which allows a decrease in capillary afterload which produces a better inflow of blood.

 

VAC dressings exert their positive effects on wound healing in multiple ways. Firstly, they remove interstitial fluids, which have been found to contain inhibitory factors that suppress the formation of fibroblasts, vascular endothelial cells, and keratinocytes. This also eliminates the formation of any superficial purulence or slime which also reduces the potential for anaerobic colonization. Removal of excess fluid also pulls out excess proteins and electrolytes to help maintain and osmotic and oncotic gradient. VACs allow arterioles to dilate which produces a proliferation of granular tissue. Also, there is a decrease in capillary afterload, (the pressure against which the heart must overcome to eject blood), which promotes better inflow of blood. Lastly, applying a mechanical force to the surrounding soft tissues allows the edges of the wound to be drawn towards the center, uniformly. This decreases the size of the wound over time. The micromechanical forces exerted on individual cells causes a release in local growth factors, such as vascular endothelial growth factor (VEGF), which stimulate wound healing.

 

Herscovici et al. applied VAC dressings to 21 consecutive patients with open, high-energy soft tissue injuries. They found that wounds averaged 4.1 sponge changes and the device was used for an average of 19.3 days. 12 wounds avoided the need for further treatment. Only 9 patients required free tissue transfer. They concluded that the VAC is a viable treatment adjunct for the treatment of open, high-energy injuries.

 

Clare et al. reported their experience with the VAC dressing in the treatment of non-healing, diabetic and dysvascular wounds. They retrospectively reviewed 17 patients with non-healing wounds of the lower extremity. 9 had diabetes and 8 had severe peripheral vascular disease. The average length of treatment

was 8.2 weeks and 14/17 wounds successfully healed, and only 3 failed VAC treatment. They concluded that the VAC dressing is an acceptable option for wound care of the lower extremity.

Illustration A is a photo of an open wound being treated with a VAC dressing. Incorrect Answers:

Answers 1, 2, 3, and 4 are all ways that VAC dressings are beneficial to wound

healing.

 

 

 

 

 

Which of the following is true regarding anterior sternoclavicular joint dislocations?

Reduction may result in tracheal injury

They are usually stable following closed reduction

They require fusion to hold the reduction

They are rarely symptomatic when left unreduced

They should be treated acutely with medial clavicle excision Correct answer: 4

From the Bicos article, “Anterior SC joint instability should primarily be treated conservatively. The patients should be informed that there is a high risk of persistent instability with nonoperative or operative care, but that the persistent instability will be well tolerated and have little functional impact in the vast majority. Therefore, operative intervention for anterior SC joint instability is mainly cosmetic in nature."

 

 

 

An otherwise healthy 45-year-old female slips and falls with immediate right ankle pain. Stress examination of the right ankle is shown in Figure A. Which of the following is the most important for achieving a satisfactory outcome following open reduction internal fixation for this injury?

 

 

Weight-bearing before 3 weeks.

Medial clear space >5mm

Talocrural angle of 83 degrees

Tibiofibular clear space of >6mm

Talar tilt of >5 degrees

 

Correct answer: 3

 

Anatomic reduction of a rotational ankle fracture is considered the most important factor in achieving a satisfactory outcome. A talocrural angle of 83 degrees suggests an anatomic reduction has been achieved.

 

Unstable rotational ankle fractures should be treated surgically in order to achieve anatomic reduction (if the fracture is not overly comminuted) as well as restore length, rotation, and alignment. Satisfactory outcomes can be best achieved when these surgical goals are achieved. Postoperative protocols

regarding immobilization and weight bearing may be somewhat variable depending on surgeon preference, fracture pattern, modifiable patient factors, and non-modifiable patient factors.

 

Lin et al. performed a systematic review of randomized studies looking at postoperative immobilization and rehabilitation following ankle fractures. They found that after surgical fixation, starting exercise in a removable brace improved pain, ankle range of motion, and improved activity limitations but led to a higher rate of adverse events. Early weight-bearing improved ankle range of motion as well. They concluded that there is limited evidence to support removable braces, early weight-bearing, and no immobilization following surgical fixation of ankle fractures.

 

Reidsma et al. retrospectively reviewed 57 malunited ankle fractures treated with revision osteotomy with a minimum of 10 years of follow-up. They found that 85% of patients had good or excellent outcomes and that prolonged time to reconstructive surgery led to a worse outcome. They concluded that reconstructive surgery should be performed early in the setting of a malunited ankle fracture even with early arthritic changes.

 

Figure A shows a displaced Weber B fibula fracture with medial joint space widening on stress examination. Illustration A shows intraoperative imaging following open reduction internal fixation of the same patient. Illustration B demonstrates the talocrural angle and medial clear space parameters in an anatomically reduced ankle.

 

Incorrect Answers:

Answer 1: Nonweightbearing for at least 6 weeks is common; however, some studies have shown improved outcomes in range of motion if weight-bearing is started early in the immobilization period.

Answer 2, 4, and 5: Postoperatively the parameters of the medial clear space

<4mm, tibiofibular clear space <6mm, and <5 degrees of talar tilt signifies anatomic correction of length, rotation, and alignment.

 

 

 

 

 

Which of the following arteries supplies the surgical flap in the extensile open treatment of the injury shown in Figure A?

 

Lateral calcaneal branch of the anterior tibial artery

Lateral calcaneal branch of the peroneal artery

Lateral malleolar branch of the peroneal artery

Lateral malleolar branch of the dorsalis pedis artery

Lateral malleolar branch of the anterior tibial artery Correct answer: 2

This patient has a displaced calcaneal fracture that is commonly treated through a lateral extensile approach. The flap of the lateral extensile approach is supplied by the lateral calcaneal branch of the peroneal artery.

 

Intraarticular fractures of the calcaneus represent approximately 2% of all fractures and are commonly the result of high-energy trauma such as motor vehicle accidents and falls. Successful operative treatment of these injuries depends on the anatomic reduction of the articular surface; restoration of the alignment, height, and length of the calcaneus; and avoidance of complications. These fractures are usually treated with a lateral extensile approach. The artery which provides blood supply to this flap is the lateral calcaneal branch of the peroneal artery. It is critical to maintain the integrity of this vessel in order to avoid complications.

Borrelli et al. performed a study to describe the arterial blood supply of the subcutaneous tissues of the lateral hindfoot and to define the relationships between these arteries and the lateral extensile incision. The lateral calcaneal artery appeared to be responsible for the blood supply to the corner of the flap.

 

Figure A demonstrates a displaced fracture of the calcaneus. Illustration A depicts the arteries on the lateral foot (PA: peroneal artery, LCA: lateral calcaneal artery, LMA: lateral malleolar artery, LTA: lateral tarsal artery). Illustration B (Borrelli et al.) is a lateral radiograph of the hindfoot and ankle that demonstrates skin staples placed along the surgical incision and vascular clips placed along the path of each artery. Illustration C is a clinical photograph of the lateral extensile approach.

 

Incorrect Answers:

Answer 1: The lateral calcaneal artery is a branch of the peroneal artery, not the anterior tibial artery.

Answers 3-5: The lateral malleolar artery is a branch of the anterior tibial artery. It does not supply the flap of the lateral extensile approach.

 

 

 

 

 

 

A 65-year-old patient sustains the injury shown in Figure A. His medical history is significant for hypertension, Type 2 diabetes and dialysis dependent chronic kidney failure. A postoperative radiograph is shown in Figure B. Based on his risk factors, what is his most likely post operative mortality at two years after surgery?

 

 

1. 13%

2. 25%

3. 45%

4. 60%

5. 100%

Correct answer: 3

 

Based on the injury sustained and the risk factors (namely chronic renal failure) shown, the patient will likely have a postoperative mortality of 45% at 2-years post operatively.

 

Hip fractures often occur older patients from low energy type mechanisms of injury. In femoral neck fractures, the relative pre-injury mobility is the most significant determining factor for the postoperative survival.

 

Bhattacharya et al. reviewed factors affecting acute mortality in patients undergoing orthopaedic procedures. They found that patients with history of chronic renal failure had an univariate mortality rate of 9%. The most predictive factors of death were: chronic renal failure, CHF, COPD, hip fracture and an age greater than 70 years of age.

 

Karaeminogullari et al. reviewed clinical outcomes of operative treatment of hip fractures in patients on chronic hemodialysis. A total of 29 patients sustained 32 hip fractures. The mortality rate, with an average follow up of 23 months, was found to be 45%. They found a significant association between age and risk of mortality.

 

Figure A shows an AP radiograph demonstrative of a displaced femoral neck fracture. Figure B shows a postoperative radiograph with an appropriately placed hip hemi-arthroplasty. Illustration V is a video that provides a brief overview of the evaluation and management of femoral neck fractures.

 

Incorrect Answers

1, 2, 4, 5: Based on the current literature, the post operative mortality rates at close to 2 years of follow up are 45%

 

 

 

 

A 25-year-old male presents following a motor vehicle collision with a Glasgow Coma Scale of 7. Subsequent imaging in the trauma bay demonstrates a bifrontal cerebral contusion, an L4 burst fracture, multiple rib fractures, an LC-1 type pelvic ring injury, a femoral shaft fracture, and an open ipsilateral tibial shaft fracture. He is intubated and an intracranial pressure monitor is placed which consistently measures 30mm Hg. He is normotensive with a lactate of 1.5 after 2 liters of crystalloid and 1 unit of packed red blood cells. Which of his injuries would most dictate a temporizing approach with external

fixation of his femoral shaft fracture instead of reamed intramedullary nailing?

L4 burst fracture

Bifrontal cerebral contusion

Open ipsilateral tibia fracture

LC1 pelvic ring injury

Rib fractures

 

Correct answer: 2

 

In the setting of a severe closed head injury such as a bifrontal cerebral contusion with elevated intracranial pressures, external fixation of a femoral shaft fracture is indicated to limit the risk of intraoperative hypotension and decreased cerebral perfusion pressure.

 

Immediate reamed nailing remains the standard treatment for the vast majority of femoral shaft fractures, however patients with multiple injuries with incomplete resuscitation and patients with severe intracranial trauma may benefit from a damage control approach with external fixation.

 

Anglen et al retrospectively reviewed the intracranial pressure (ICP) and cerebral perfusion pressure (CPP) in patients undergoing femoral nailing. The authors found a significant decrease in intraoperative CPP, especially in those patient undergoing femoral nailing in the first 24 hours, however they were unable to demonstrate a link between the decreased CPP and poor patient outcomes.

 

Pietropaoli et al examined the effects of intraoperative hypotension on patients with blunt head trauma. The authors found that 32% of patients experienced intraoperative hypotension (systolic blood pressure less than 90mm Hg) and those patients with a hypotensive episode had an 82% mortality and significantly worse outcomes on the Glasgow Outcomes Scale.

 

McKee et al conducted a retrospective cohort study comparing matched groups of patients with femoral shaft fractures with and without a closed head injury. In contrast to previous studies, the authors found no significant difference in outcome between the groups including mortality, hospital length of stay or neuropsychologic testing.

 

Illustration A shows a femoral shaft fracture treated with external fixation. Incorrect Answers:

Answer 1, 3-5: Immediate reamed nailing would not change the outcome of any of these injuries

 

 

 

 

 

A 22-year-old left hand dominant laborer sustains the injury shown in Figures A and B as the result of a fall from a ladder. Which of the following has been shown to be true regarding operative versus nonoperative treatment of this injury?

 

 

Decreased chance of nonunion with nonoperative treatment

Improved Constant and DASH scores with operative treatment at all time points

Increased symptomatic malunion rate with operative treatment

No change in shoulder abduction strength

Increased time to union with operative treatment Correct answer: 2

Surgical management of displaced, shortened clavicle fractures is associated with a decreased rate of nonunion and malunion. General recommendations for surgical treatment include shortening of greater than 2 centimeters.

 

Kim et al. review clavicle fracture treatment history and current indications. They report that although previous thought was that nearly all clavicle fractures should be treated nonoperatively, outcomes can be improved with fixation of certain clavicle fracture patterns.

 

COTS et al. performed a multicenter, randomized controlled trial of 132 patients with a displaced midshaft fracture of the clavicle. They found that Constant and DASH scores were improved in the operative fixation group at all points in time, with union time being 28 weeks in the nonoperative group and

16 weeks in the operative group. Malunion was higher in the nonoperative group as well.

 

McKee et al. reviewed 30 patients who underwent closed treatment of a displaced midshaft clavicle fracture. They found that range of motion of the shoulder was maintained but the strength of the shoulder was decreased to 81% for flexion, 82% for maximum abduction, 81% for maximum external rotation, and 85% for maximum internal rotation. Endurance for these movements was also significantly decreased as compared to the contralateral side.

 

Figure A shows a clinical photo of a patient with a clavicle fracture. Figure B shows a displaced, comminuted clavicle fracture.

 

Incorrect Answers:

Answer 1: Operative treatment increases the union rate.

Answer 3: Operative treatment decreases the rate of symptomatic malunion. Answer 4: Operative treatment increases shoulder abduction strength.

Answer 5: Time to union is decreased with operative treatment.

 

 

 

 

Figures A and B are radiographs of a 20-year old male athlete that sustained a high impact tackle during a football game. What percentage of these injuries will present with an associated vascular injury?

 

 

1. 10%

2. 20%

3. 40%

4. 70%

5. 90%

 

Correct answer: 3

 

Approximately 40% of low-velocity anterior knee dislocations are associated with popliteal vascular injury.

Anterior knee dislocations, which are the most common of all directional dislocations, are produced by a hyperextension mechanism. This causes the tibia to translate anterior to the femur and the popliteal vessels to stretch, causing intimal tears.

 

Wascher et al. reviewed the association of vascular injury with traumatic knee dislocations. They showed that 50% of all knee dislocations spontaneously reduce. However, patients who present with reduced knee dislocations have a similar risk of vascular injury (~ 40%) and other concurrent injuries as those who present with a dislocated knee.

 

Levy et al. reviewed the timing of treatment of multiligament-injured knee injuries arising from acute knee dislocations. They suggest that early operative treatment of the multiligament-injured knee yields improved functional and clinical outcomes compared with nonoperative management or delayed surgery. They noted that repair of the PLC, either acute or delayed, may yield higher revision rates compared with reconstruction options.

 

Figures A and B show AP and lateral radiographs of an anterior knee dislocation. A video is provided that gives a brief overview of knee dislocations.

 

 

 

 

A 45-year-old female presents to the office wearing a right upper arm splint with radiographs shown in Figure A and B. She sustained an isolated closed injury to the right arm 9 days ago. Her soft-tissues and neurological examination are normal. What would be the most appropriate treatment for this injury?

 

 

Continue current splint for 6 weeks

Continue current splint for 3 weeks and transition to hanging arm sling for additional 3 weeks

Transition to functional brace for additional 6-8 weeks

Open reduction internal fixation with compression plating

Staged procedure with humeral external fixator, then open reduction internal fixation with compression plating

Correct answer: 3

 

Figures A and B show radiographs on a minimally displaced humeral shaft fracture. The most appropriate treatment for this injury would be functional bracing (Sarmiento) for an additional 6-8 weeks or until healed.

 

Commonly accepted parameters for closed treatment include less than 30 degrees of varus angulation, 20 degrees of anterior/posterior angulation, and 3 cm of shortening. Functional bracing has become the gold standard for humeral shaft fractures as it consistently shows excellent healing results as well as preventing the complication of shoulder +/- elbow stiffness associated with joint spanning splints or slings.

 

Sarmiento et al. treated 922 patients with humeral diaphysis fractures with a prefabricated brace. They found a 97% rate of union with the use of the brace. In addition, only 2% of the patients had lost more than 25 degrees of shoulder motion at the time of brace removal.

 

Koch et al. reviewed 67 humeral shaft fractures that were treated by Sarmiento bracing in a 15-year period. Fifty-eight cases (87%) had healed clinically at a mean of 10 weeks. Among 9 patients with delayed or nonunion leading to operative intervention, there were 6 cases with transverse fractures

 

Figures A and B show a moderately displaced right humeral shaft fracture with 13 degrees of AP angulation, 10 degrees of varus/valgus angulation and no shortening, treated in a coaptation splint. A nondisplaced proximal humeral fracture is also seen. Illustration A shows an image taken of a patient wearing the sarmiento brace.

 

Incorrect Answers:

Answer 1,2: Joint spanning splints or slings have not shown to be superior to functional bracing. They are associated with joint stiffness post removal.

Answer 4: Operative indications are: associated vascular injuries, bilateral humeral shaft fractures, polytrauma patient (including paraplegia), injury to the brachial plexus, pathological fractures, floating elbow, and floating shoulder.

Answer 5: Staged operative procedure would be indicated in open fractures or significant deformity with soft-tissue swelling.

 

 

 

 

Which of the following is an appropriate initial step in the management of a multiply injured patient with an unstable pelvic ring fracture and hemodynamic instability?

Application of an external fixator

Pelvic angiography

Pelvic packing

Application of a pelvic binder

Percutaneous Iliosacral screws Correct answer: 4

Patients with multiple injuries including a pelvic ring fracture who present with hemodynamic instability should have a pelvic binder or circumferential pelvic sheet placed as part of their initial resuscitation.

 

A systematic approach to search for sources of bleeding and control ongoing hemorrhage is necessary for patients who present with hemodynamic changes in the setting of a pelvic ring fracture. Management of continued hypotension after pelvic binder placement is controversial and varies among trauma centers.

 

Krieg et al. prospectively evaluated 16 patients with unstable pelvic ring injuries initially managed with a novel circumferential compression device. The authors found substantial reduction in pelvic width with the use of this

compressive device in patients with volume expanding pelvic ring fractures.

 

Croce et al. retrospectively compared patients with unstable pelvic ring injuries who were treated with either emergent pelvic fixation (EPF) or a pelvic orthotic device (POD). The authors found that those patients treated with POD had decreased transfusion requirements and shorter length of hospital stay.

 

Routt et al describe their technique for circumferential pelvic antishock sheeting (CPAS). The authors provide an illustrative case and discuss the potential advantages of sheet application versus other techniques of pelvic stabilization.

 

Illustration A is the initial AP radiograph of a patient with a pelvic fracture and hemodynamic instability. The pelvic binder was placed in the field prior to arrival. Illustration B demonstrates the same patient in the angiography suite after removal of the pelvic binder. Note the increased widening of bilateral SI joints, greater on the left than the right.

 

Incorrect Answers:

Answer 1: External fixation of pelvic ring fractures can be used to assist with resuscitation but pelvic binder application should be attempted first

Answer 2: The use of pelvic angiography is controversial and institution specific however some centers utilize pelvic angiography as part of the algorithm for management of ongoing hemorrhage.

Answer 3: Pelvic packing is utilized in some centers to control ongoing pelvic hemorrhage however it is not used as initial management of patients with hemodynamic instability

Answer 5: Percutaneous iliosacral screws can also be utilized as a form of resuscitation however they should not be used as as first line of management

 

 

 

 

 

A 19-year-old female sustains the injury shown in Figures A thru C as the result of a motor vehicle collision. Which of the following is the most common cause of death with this type of pelvic injury pattern?

 

 

Hypovolemic shock

Spinal injury

Solid organ rupture

Acute respiratory distress syndrome

Closed head injury Correct answer: 5

The injury pattern described in the question and images is a lateral compression pelvic ring injury. Of the choices provided, the most common associated cause of death is a closed head injury.

 

Pelvic ring disruptions are the result of high energy blunt trauma and are associated with other significant injuries in greater than 50% of the cases. These injuries may involve neurovascular structures and other organ systems.

 

Burgess et al. retrospectively reviewed their pelvic ring injuries and reported their classification system based upon the vector of force involved and the quantification of disruption from that force, i.e., lateral compression, anteroposterior compression, vertical shear, and combined mechanical injury. Overall blood replacement was highest in anterior-posterior patterns. Mortality was also highest in anteroposterior patterns. The most common identifiable cause of death in patients with lateral compression fractures is closed head injury. In contrast, the identifiable cause of death in patients with anteroposterior compression injuries is combined pelvic and visceral injury.

Watnik et al. reviewed lower urinary tract injuries and noted that they occur in as much as 25% of patients with pelvic ring disruptions. They also report that early repair of bladder injury can facilitate the placement anterior pelvic fixation, in efforts to minimize infection.

 

Smith et al. reviewed hemodynamically unstable pelvic ring fracture patients and found that there is a positive association of blood replacement requirements and mortality. They also reported that death within the first 24 hours after admission was most often a result of acute blood loss while death after the first day was most often caused by multi-organ failure.

 

Figure A shows an AP pelvic radiograph with evident anterior pelvic ring fractures. Figure B and C are axial CT cuts showing the posterior and anterior ring fractures, respectively. This fracture pattern is consistent with a lateracl compression mechanism.

 

Incorrect Answers:

Answer 1-4: These options are less commonly reported as causes of death than closed head injury in a lateral pelvic ring injury mechanism.

 

 

 

 

A 67-year-old male is involved in a motor vehicle accident and presents with the closed orthopedic injuries shown in Figures A and B. He is also noted to have a grade 1 splenic laceration and lung contusion. He is cleared by the trauma team, and undergoes early total care with reamed femoral and tibial nailing. A tourniquet is used for the tibial nailing portion of the case, and the tibial isthmus is over reamed to accept a larger nail. The use of a tourniquet in this case has been most clearly shown to be associated with which of the following?

 

Tibia shaft necrosis post-operatively

Increased pulmonary morbidity post-operatively

Increased cortical bone temperature during reaming

Increased nonunion rates

Decreased pain post-operatively Correct answer: 2

In patients with multitrauma, combining reamed femoral nailing with fracture fixation (ie. tibial shaft) under tourniquet control has been shown to increase pulmonary morbidity.

 

Limb reperfusion after tourniquet ischemia causes pulmonary microvascular injury. Similarly, microembolization, like that associated with reamed femoral nailing, can induce pulmonary microvascular injury. Both processes result in increased pulmonary capillary membrane permeability and edema, and ultimately increased pulmonary morbidity.

 

Karunakar et al showed in a canine model that there is no significant difference in the heat generated during reaming with and without a tourniquet. The factor that made the most difference was related to the size of the reamer used compared with the diameter of the isthmus. They concluded that the risk of

thermal necrosis appears to be related more to the process of intramedullary reaming than to the tourniquet.

 

Giannoudis and associates performed a prospective randomized trial on 34 patients to measure the rise of temperature during reaming of the tibia before intramedullary nailing with and without the use of a tourniquet. The factor that generated the most heat was using large reamers (11 mm to 12 mm) in a patient with a small isthmus (8 mm to 9 mm). Use of a tourniquet, steroid use, and knee flexion during reaming were not shown to be associated with diaphyseal necrosis after reamed tibial nailing.

 

Pollak et al evaluated the association between femoral nailing followed by tourniquet ischemia and clinical lung injury. They reviewed 72 patients with femoral shaft fractures and tibial or ankle fractures requiring internal fixation over a six year period. All femoral shaft fractures were treated with reamed intramedullary nails, and the patients were divided into groups, based on whether the tibial or ankle injury was managed surgically with or without a tourniquet. They noted increased pulmonary morbidity in the group where a tourniquet was used.

 

Figure A shows a femoral shaft fracture at the junction of the middle and proximal one-third of the femoral shaft. Figure B shows a contralateral tibial shaft fracture.

 

Incorrect Answers:

1-Tourniquet use has not been shown to lead to thermal necrosis of the bone during reaming of the tibial shaft.

The cortical temperature does not increase to a greater degree with tourniquet use versus non-tourniquet use during reaming.

There is no evidence that use of a tourniquet affects tibial shaft union rates after reaming and intramedullary nailing.

Tourniquet use has been associated with increased pain post-operatively.

 

 

 

 

 

A 24-year-old male sustains the isolated injuries shown in Figures A and B during a high-speed motor vehicle accident. On physical examination, the overlying skin is intact and there is no evidence of a Morel-Lavallée lesion. Which of the following surgical techniques is considered to have the highest rate of fracture malreduction with this combined injury?

 

 

Antegrade cephalomedullary nail

Retrograde intramedullary nail and 3 cannulated screws

Retrograde intramedullary nail and sliding hip screw

Antegrade intramedullary nail and 3 cannulated screws

Plate fixation of the diaphyseal fracture and 3 cancellous screws Correct answer: 1

Figures A and B show displaced ipsilateral femoral neck and shaft fractures. Fixation with a single implant, such as an antegrade cephalomedullary device, has been shown to have the highest rate of fracture malreduction with displaced fractures.

 

Approximately 5% of femoral shaft fractures are accompanied by ipsilateral neck fractures. Fixation methods to stabilize these fractures may be accomplished by using a single implant or two separate implants. Single implant techniques are thought to reduce operative time and blood loss by simultaneously fixing nondisplaced fractures. With displaced fractures, single implant techniques have been shown to have a higher rate of malreduction of

at least one of the two fractures. Two separate implant devices are recommended in these scenarios.

 

Bedi et al. examined a retrospective cohort of 40 patients with ipsilateral femoral neck and shaft fractures. They showed that using a single cephalomedullary device for fixation of both femoral shaft and neck fractures led to a significantly higher rate of fracture malreduction (3 of 9), in comparison to a staged, two implant strategy (0 of 28) (P = 0.01).

 

Peljovich et al. reviewed the presentation and management of ipsilateral femoral neck and shaft fractures. To reduce complications of AVN, malunion and nonunion, they suggest obtaining anatomic reduction and rigid fixation of the femoral neck fracture first with 3 cannulated screws, blade plate, or sliding hip screw. The shaft fracture can then be reduced and stabilized with either retrograde intramedullary femoral nailing or plating.

 

Incorrect Answers:

Answers 2,3,4,5: Fixation of femoral neck and shaft fractures using two separate implant devices has a lower rate of malreduction. No study to date has conclusively demonstrated superiority of any particular combination of devices in long-term studies.

 

 

 

 

A 35-year-old patient sustains an upper extremity injury after a motor vehicle collision. Radiographs are shown in Figures A and B. Which treatment modality will optimize internal stability of the elbow?

 

 

Open reduction and internal fixation with k-wires

Open reduction and internal fixation with tension band wiring

Open reduction and internal fixation with plate fixation

Open reduction and internal fixation with an intramedullary screw

Cast immobilization

 

Correct answer: 3

 

Based on the radiographs shown, the patient has a comminuted trans-olecranon fracture dislocation of the elbow. Stability will be optimized with internal fixation in the form of a plating construct.

 

Trans-olecranon fracture dislocations often occur in the young secondary to high energy mechanisms. Direct blows often lead to high levels of comminution; the distal humerus is driven into the proximal ulna and olecranon, thereby leading to a concomitant dislocation. In this setting, the optimal treatment is use of a plate and screw fixation construct to provide mechanical stability.

 

Veillette et al. review the epidemiology, pathophysiology and treatment options for olecranon fractures. They highlight that because olecranon injuries involve the articular surface, fixation constructs should allow anatomic restoration of the joint surface. Additionally, fixation constructs should allow for early mobilization in order to prevent stiffness of the elbow.

 

Mortazavi et al. retrospectively reviewed eight patients who sustained anterior trans-olecranon fracture dislocations of the elbow. Based on their experience, they recommend use of plate fixation to optimize stability of the elbow.

Additionally, they indicate that satisfactory results can be obtained if the greater sigmoid notch is appropriately reduced.

 

Figures A and B show the AP and lateral radiographs of a trans-olecranon fracture dislocation of the elbow. Severe comminution of the olecranon can be seen on the lateral radiograph in Figure B.

 

Incorrect Answers:

Answers 1, 2, 4: K-wires alone would not provide optimal stability and allow for early motion. A tension-band construct or IM screw fixation is contraindicated with this degree of comminution.

Answer 5: Use of cast immobilization would lead to high levels of stiffness and is not indicated for this patient.

 

 

 

A 25-year-old male sustained an isolated injury to his right foot after a fall from height. On examination, he has moderate swelling and pain over the dorsum of the foot. The overlying skin is intact. Radiographs of the foot are seen in Figures A and B. A CT scan image is seen Figures C. When consenting the patient for open reduction and internal fixation of this injury, what would you document as the most common complication?

 

 

 

Symptomatic implants

Subtalar arthritis

Tibiotalar arthritis

Malunion

Wound dehiscence

 

Correct answer: 2

 

This patient has sustained a displaced talar neck fracture. The most common complication associated with this injury is post-traumatic arthritis of the subtalar and tibiotalar joint.

 

Talar neck fractures are the most common fracture of the talus. They occur most frequently during forced dorsiflexion of the foot with axial load. Displaced fractures are best treated with anatomic reduction and stable internal fixation.

Complications of surgery include nonunion or malunion, post-traumatic arthritis, infection, osteonecrosis, and secondary surgery. The prevalence of post-traumatic subtalar arthritis has been shown to be approximately 50-100% with these injuries.

 

Lindvall et al. examined a retrospective cohort of twenty-six displaced talar neck fractures that had been treated with open reduction and stable internal fixation. They reported that 26/26 fractures developed post-traumatic arthritis and pain within 48 months of follow-up after fixation. They concluded that post-traumatic arthritis is a more common complication than osteonecrosis following operative treatment.

 

Vallier et al retrospectively reviewed the records of 39 fractures of the talar neck treated with open reduction and internal fixation. Twenty-one (54%) of thirty-nine patients had development of posttraumatic arthritis, which was more common after comminuted fractures (p < 0.07) and open fractures (p = 0.09).

 

Figures A and B show plain radiographs of a Hawkins II talar neck fracture. There is dislocation of the subtalar joint. The tibiotalar and talonavicular joints appear congruent. Figures C is a CT scan imaging showing some comminution at the fracture site.

 

Incorrect Answers:

Answer 1: Symptomatic implants ~ 18%

Answer 3: Tibiotalar arthritis ~ 33%

Answer 4: Malunion ~ 25 %

Answer 5: Wound dehiscence ~ 3 %

 

 

 

 

 

A 23-year-old man presents with the injury seen in Figure A after a motor vehicle collision. He undergoes the treatment seen in Figure B. Which of the following statements is most accurate when comparing his treatment with open reduction and internal fixation?

 

 

Higher rates of radial nerve injury

Higher total complication rate

Lower rates of nonunion

Lower rates of shoulder impingement

Lower rates of malunion Correct answer: 2

Antegrade intramedullary (IM) nailing of humeral shaft fractures has been found to be associated with increased complication rates when compared with open reduction and internal fixation (ORIF).

 

Operative treatment of humeral shaft fractures remains controversial, with prospective randomized studies demonstrating small differences between IM nailing and ORIF.

 

Heineman et al. (2010) conducted a meta-analysis of prospective randomized studies comparing IM nailing with ORIF for humeral shaft fractures. The authors found no significant difference between the two treatment modalities for either their primary outcome (complications) or any of the secondary outcomes (nonunion, infection, nerve palsy, re-operation)

 

Heineman et al. (2012) have recently conducted an update on their meta-analysis to include more recent randomized studies. With the inclusion of these newer studies the author found a statistically significant increase in total complication rate with the use of IM nailing compared with ORIF.

 

Figure A demonstrates a displaced, transverse humeral shaft fracture. Figure B demonstrates antegrade IM nailing of a humeral shaft fracture

 

Incorrect Answers:

Answer 1: Radial nerve injury has not been shown to be different between IM nailing and ORIF

Answer 3: No difference in union rates between the two modalities in prospective studies

Answer 4: Higher rates of shoulder impingement have been seen with IM nailing in some studies

Answer 5: No difference in rates of malunion between IM nailing and ORIF

 

 

 

 

Elevated interleukin 6 (IL-6) is most closely associated to which of the following clinical outcomes in orthopedic trauma patients?

Decreased mortality rates

Increased mortality rates

Decreased osteomyelitis infection rates

Increased rhabdomyolysis rates

Increased compartmental syndrome rates Correct answer: 2

Elevated levels of Interleukin 6 (IL-6) is most closely associated with higher injury severity scores and increased mortality rates in polytrauma orthopaedic patients.

 

Hyperstimulation of the inflammatory system by major trauma is considered to be the key element in the pathogenesis of severe inflammatory response syndrome and multi-organ dysfunction syndrome. IL-6 is a complex acute-reactant cytokinase that is expressed by cells in response to tissue injury. IL-6 levels are associated with injury severity, complications, and mortality.

Patients with the most severe injuries have the highest IL-6 serum levels.

 

Sears et al. reviewed the markers of inflammation in major trauma. They suggest that interleukin-6 and human leukocyte antigen-DR class II molecules appear to have the greatest potential for use in predicting the clinical course and outcome in trauma patients. Early identification of traumatic patients, based on inflammatory markers and genomic predisposition, could help to guide intervention and treatment.

 

Pape et al measured the perioperative concentrations of interleukin-6 in sixty-eight blunt trauma patients with non-life threatening pelvic fractures. Release of proinflammatory cytokines were higher in patients undergoing surgical procedures that cause increased blood loss. The release of markers seems to be related to the type and magnitude of surgery, rather than to the duration of the procedure.

 

Illustration A shows a diagram of the acute inflammatory response after major trauma

 

Incorrect Answers:

Answer 1: Mortality rates have shown to be increased with elevated levels of IL-6 in early trauma.

Answer 3: Osteomyelitis is a complication of open fracture trauma. Elevated levels of IL-6 and CRP are seen with this infection.

Answer 4: IL-6 is not commonly measured in rhabdomyolysis. Creatine kinase is commonly elevated with musculoskeletal injury.

Answer 5: Elevated IL-6 levels are not closely correlated with incidence incidence of compartmental syndrome.

 

 

 

 

A 26-year-old female sustained a closed femoral shaft fracture which was treated as shown in Figure A. She subsequently developed a painful oligotrophic nonunion and an exchange nailing was performed 6 months later. However, she has had persistent pain in the thigh since time of revision surgery seven months ago, but denies any fevers, chills, or night sweats. Current examination reveals a mildly tender right thigh with no obvious deformity. Laboratory studies for infectious markers are normal, but her radiograph in Figure B continues to show an oligotrophic nonunion. Which of the following factors has most likely contributed to her current presentation?

 

One pack of cigarettes per day for last 8 years

Positive intraoperative cultures for coagulase-negative Staphylococcus at the time of nail exchange

Iliac crest bone grafting at the time of nail exchange

Exchange nail was dynamically interlocked

A larger diameter nail was used in the exchange Correct answer: 1

The clinical presentation and radiographs are consisted with persistent oligotrophic non-union of the femur, after reamed nail exchange. Of the following factors, cigarette smoking has been shown to have the biggest effect on non-union in this population.

 

Reamed intramedullary nail exchange has become widely accepted as the treatment of choice for most diaphyseal femoral nonunions. The use of reamed exchange nailing are thought to allow for more biomechanical stability via larger diameter nails and improved biologics with bone reaming. These will help to generate bone healing and bridge fracture gaps. Smoking has been shown clinically and experimentally to affect union rate and fracture callus strength. Patients with femoral shaft non-unions should be forewarned of the its effect on fracture healing in nail exchange.

 

Hak et al retrospectively reviewed a cohort of 23 patients treated with reamed nail exchange for symptomatic femoral shaft non-unions. 15 of the patients were smokers and only 10 (66.7 percent) went on to heal after reamed nail exchange. This was in comparison to all 8 non-smoking patients who healed without complication after exchange reamed nailing. They suggest that smokers should be referred to an appropriate smoking cessation program before revision nail exchange is considered.

Figure A shows a proximal third femoral shaft fracture that was treated with intramedullary nailing. The oblique fracture line remains evident with a small callus response at the fracture site at 8 months post-operative. Figure B shows an radiograph of the exchange femoral nail 7 months after revision sugary.

There is increased fracture callus at fracture site, however the fracture line remains evident.

 

Incorrect Answers:

Answer 2: Intraoperative coagulase-negative staph. cultures are likely contaminant. The patient denies infectious symptoms and inflammatory markers for infection are negative. In addition, the radiographs do not suggest infection as there is no cortex irregularity, bone lysis, or implant loosening.

Answer 3: Bone grafting at the time of nail exchange has shown to increase bone union.

Answer 4: Dynamic interlocked screws were shown to increase bone union greater than static interlocked screws at the time nail exchange.

Answer 5: Bone grafting at the time of nail exchange has shown to increase bone union.

 

 

 

 

The sacrospinous and sacrotuberous ligaments are disrupted in which of the following injury patterns?

Ischial tuberosity avulsion fracture

Type I anterior-posterior compression pelvic ring injury

Type II lateral compression pelvic ring injury

Type II anterior-posterior compression pelvic ring injury

Both column acetabular fracture Correct answer: 4

Type II anterior-posterior compression (APC) pelvic ring injuries have disruption of the symphysis pubis, as well as disruption of the anterior SI ligaments, sacrotuberous ligament, and sacrospinous ligament. The alternative pattern of disruption of the pelvic floor ligaments is sometimes seen as an avulsion injury from the bony attachments of these structures (sacrum or ischium).

 

The progression of this pattern to involve the posterior SI joint ligaments creates a type III pattern, which is vertically and rotationally unstable.

 

Tile published a comprehensive review of pelvic ring injuries, focusing on the

anatomy and pathology of these injuries. He noted that the posterior SI complex is the most important to pelvic ring stability, which is the reason why these are generally classified by the grade of posterior injury. He also reminds the reader that these classification systems are not a substitute for individualized treatment decision making.

 

Illustration A shows an APC III pelvic ring injury, with significant symphyseal diastasis and SI joint widening. Illustration B shows the ligamentous anatomy of the pelvic ring.

 

Incorrect Answers:

Answer 1: These injuries are generally seen in sports-related trauma, with avulsion of the hamstring(s) from the ischium.

Answer 2: This injury pattern does not have any involvement of these two ligaments by definition.

Answer 3: This injury pattern involves inward rotation of the hemipelvis, thus protecting these structures. A type III LC injury may have a contralateral rupture of these ligaments, however.

Answer 5: This injury pattern typically does not involve rupture of these ligaments unless an ipsilateral pelvic ring injury is seen.

 

 

 

 

 

A 34-year-old man is involved in a motorcycle accident and sustains a closed tibia fracture and multiple rib fractures. A radiograph is provided in Figure A. During surgical treatment of this fracture, which of the following techniques will help facilitate a successful reduction and intramedullary fixation?

 

Hyperflexion to help prevent apex anterior angulation

A medial parapatellar incision to help prevent valgus angulation

Starting point just lateral to the medial tibial eminence to help prevent valgus angulation

A medially placed blocking screw to help prevent valgus angulation

Suprapatellar nailing technique to help prevent apex anterior angulation Correct answer: 5

The clinical scenario and radiograph demonstrate a proximal tibia fracture amenable to intramedullary nail fixation. The suprapatellar nailing technique helps prevent apex anterior or procurvatum deformity typical of these injuries.

 

Proximal third tibial shaft fractures are often difficult to reduce anatomically due to the tendency for both valgus and flexion deformity at the fracture site. Many different techniques have been devised to overcome the deforming forces. These include (1) Poller blocking screws posterior and lateral to the intramedullary nail (IMN), (2) utilizing a semi-extended knee position during IMN of proximal tibia fractures, (3) use of a suprapatellar approach for IMN,

usage of a slightly more lateral starting point during conventional IMN, and

application of unicortical plate.

 

Lang et al. review surgical management of proximal tibia fractures. The typical

deformity is flexion and valgus. An appropriate starting point just medial to the lateral tibial eminence will help facilitate the appropriate position of the intramedullary nail.

 

Hiesterman et al. review the management of extraarticular proximal tibia fractures. They elaborate on the anatomic considerations of the classic procurvatum and valgus deformity. The procurvatum deformity results from the extensor mechanism inserting on the proximal fragment, which can be neutralized in the semi-extended suprapatellar nailing technique. The valgus deformity comes from a combination of the intramedullary anatomy of the tibia and the insertion of the pes anserinus muscles on the proximal fragment. They emphasize that attaining and maintaining a proper reduction and utilizing an appropriate starting point is critical for the treatment of these fractures.

 

Figure A demonstrates an extra-articular proximal tibia fracture. Illustration A demonstrates the proper start for a proximal tibia fracture which should be just medial to the lateral tibial eminence on a perfect AP view and just anterior to the articular margin sagittally (as borrow from Hiesterman et al.).

 

Incorrect Answers:

Answer 1: Hyperflexion of the knee can accentuate an apex anterior deformity if measures are not taken to prevent it such as posterior blocking screw placement or a unicortical plate.

Answer 2: The medial parapatellar approach is generally not used for tibial nailing, although a lateral parapatellar approach has been advocated by some to help obtain a more lateral start point.

Answer 3: The proper nail starting point is just medial to the lateral tibial eminence.

Answer 4: A laterally placed blocking screw will help prevent valgus angulation when placed in the proximal segment.

 

 

 

 

A 21-year-old male is brought to the emergency department with multiple gun shot wounds. Initial radiographic evaluation discovers a femoral shaft fracture, distal tibia fracture, and the injury shown in Figure A. Figure B shows a single entry wound located at the left distal humerus. Systemic injuries include multiple abdominal bullet wounds with associated intra-abdominal free fluid. Using the 'damage-control' approach to orthopaedic trauma, what would be the best initial management for the injury seen in Figure A?

 

Closed reduction and splinting

Irrigation and debridement, then splinting

Irrigation and debridement, then spanning external fixation

Open reduction and internal fixation with a compression plate

Irrigation and debridement, then intramedullary nailing of the humerus Correct answer: 1

Figure A shows a distal third humeral fracture caused by a low-velocity bullet. According to concepts of 'damage control' orthopaedics, this fracture would be best managed initially with closed reduction and splinting.

 

Low-velocity gun shot wounds are not usually considered open fractures. Therefore, according to the principles of damage control orthopaedics (DCO), GSW and closed humerus fractures can be treated initially with closed reduction and splinting. The concept of DCO is to limit further systemic inflammatory insult after major trauma. The extent of injury is closely tied to elevated levels of serum inflammatory markers (e.g. IL-6 and IL-8) as well as lactate and acid/base deficits. The "second-hit" hypothesis of DCO is based on the cumulative effect of the systemic inflammatory response on the body, which is increased by the additional release of inflammatory markers with early surgery.

 

Roberts et al. reviewed the concepts of damage control orthopaedics. They suggest that the goal of DCO is to avoid provoking a severe inflammatory response through the stabilization and control of injury rather than repair. To further quantify the impact of the 'second-hit' hypothesis, the authors assessed the role of femoral nailing in early treatment. Specifically, the inflammatory response induced by reamed femoral nailing was shown to increase IL-6 levels within the venous system.

 

Hildebrand et al. discussed the treatment of extremity injury in damage control orthopaedics. They suggest that most upper extremity injuries can be stabilized early with closed reduction and simple splints or slings. For closed below knee fractures with stable soft tissues, splinting is the single best option.

 

Figure A shows a distal third humeral fracture caused by a low-velocity bullet. Figure B shows a small entry wound from a low-velocity bullet. Illustration A, in contrast, shows a humerus fracture caused by a shot-gun injury. Shot-gun injuries are traditionally treated as open fractures, which require irrigation and debridement and spanning external fixation according to the DCO approach.

 

Incorrect Answers:

Answer 2: Low velocity gun shot wounds are not usually considered open

fractures and do not require formal irrigation and debridement.

Answer 3: Humerus fractures stabilized with a spanning external fixator may be considered with open humeral fractures, vascular injury or a floating elbow. Answer 4,5: Invasive procedures to fix the humerus acutely will increase the release of systemic inflammatory markers. For most upper extremity injuries, simple external stabilization with a splint or a sling is recommended initially.

Definitive fixation would be considered at a later date if there is an associated lower extremity fracture, high level athlete, significant displacement, nonunion or patient preference. The timing of surgery would be based on skin and soft tissue status, and their systemic response after injury.

 

 

 

 

 

A 25-year-old is involved in a motor vehicle accident and sustains an isolated upper extremity injury. A representative radiograph is shown in Figure A. What physical exam findings are most predictive of functional outcomes?

 

Presence of open fractures

Asymmetry of pulses

Ecchymosis of the upper shoulder girdle

Swelling of the shoulder

Neurologic compromise of the extremity Correct answer: 5

Based on the radiograph and mechanism of injury, this patient has sustained a scapulothoracic dissociation. In the context of this injury, neurologic status of the extremity is the exam finding most predictive of functional outcome.

 

Scapulothoracic dissociation is a traumatic injury to the scapulothoracic articulation that results from lateral traction to the shoulder girdle. It is often associated with significant chest wall, heart and lung trauma. A hallmark physical exam finding is decreased or absent pulses in the affected extremity. Because of the significant traction placed on the brachial plexus, functional outcome is dependent on the extent of neurologic injury. In patients with a complete brachial plexopathy, forequarter amputation should be considered.

 

Clements et al. review the diagnosis and management of scapulothoracic dissociation. They indicate that this entity is defined by subclavian or axillary vascular disruption, lateralization of the scapula, separation of the clavicular articulations, and nerve root or brachial plexus injury. For patients with flail extremities, an above elbow amputation is recommended

 

Figure A shows an AP radiograph of a patient with scapulothoracic dissociation. Note the significant lateral displacement of the scapula and scapular body comminution

Illustration A shows a coronal 3D reconstruction of a CT scan of the chest. Note the significant lateral displacement of the scapula relative to the chest wall.

There is greater than 1 cm displacement relative to the vertebral spinous processes. A video is provided that reviews diagnosis and management of scapulothoracic dissociation.

 

Incorrect Answers

Answers 1, 2, 3, 4: While these may all be seen in the setting of this injury, neurological status of the extremity is the most predictive of functional outcomes.

 

 

 

 

 

Which of the following figures depicts a lateral compression mechanism of injury?

 

 

 

Figure A

Figure B

Figure C

Figure D

Figure E

 

Correct answer: 2

 

Figure B depicts a left lateral compression mechanism of injury or more specifically, an LC-II injury in the Young-Burgess classification system of pelvic fractures. Figure B depicts superior and inferior pubic rami fractures and an ipsilateral "crescent" fracture. The "crescent" injury involves a combination of ligamentous injury at inferior portion of SI joint, and vertical fracture of the posterior ilium which extends thru iliac crest.

 

Proper diagnosis of pelvic ring fractures including LC-type fractures is made via radiographs including AP, inlet, and outlet views of the pelvis. CT of the pelvis can be an adjunct to the diagnosis of pelvic ring fractures and is commonly

obtained in the general trauma workup, but is not necessary to make the diagnosis.

 

Tile et al. describes his classification system of pelvis ring fractures focusing on the importance of the posterior sacroiliac ligaments for stabilization. This classification system is A-C with a Tile C being a fracture with complete disruption of the posterior sacroiliac ligaments and thus the most instability.

 

Lefaivre et al. retrospectively review CT scans of LC-1 pelvic injuries to more specifically categorize the CT findings of that fracture pattern. A large portion were found to have complete disruption of the sacrum.

 

Koo et al. measured the interobserver reliability of both the Tile and Young-Burgess classification systems. They found that CT scans of the pelvis increased the reliability of the assessment of pelvic stability.

 

Illustration A is pelvic radiograph that is labelled to help distinguish the crescent fracture from the SI joint.

 

Incorrect Answers:

Answer 1: Figure A shows an APC-II (anterior-posterior compression) injury. Answer 3: Figure C shows a vertical shear injury.

Answer 4: Figure D is a CT scan of an APC-III injury.

Answer 5: Figure E is a CT scan of an anterior wall acetabular fracture.

 

 

 

 

A 35-year-old female presents with the orthopaedic injuries shown in Figures A-D following a high-speed motor vehicle collision. She is also found to have a right-sided diaphragmatic hernia (Figure E) and a stable subarachnoid hemorrhage. The femoral and tibial plateau fractures are open with no gross contamination, and there is an ipsilateral Morel-Lavelle lesion of the left thigh. What is the most appropriate initial management of the patient’s injuries in addition to debridement and irrigation of the open injuries?

 

 

 

Application of a knee immobilizer, splinting of the ankle and forearm

External fixation of the femur and tibial plateau, splinting of the ankle and forearm

Retrograde intramedullary nailing of the femur, limited internal fixation of the tibial plateau, splinting of the ankle and forearm

External fixation of the femur, ORIF of the tibial plateau, splinting of the ankle and forearm

Retrograde intramedullary nailing of the femur, ORIF of the tibial plateau, ORIF of the ankle and forearm

Correct answer: 2

 

In the setting of a poly-trauma patient with an open high-energy tibial plateau fracture, the best initial treatment is irrigation and debridement with external fixation of the femur and tibial plateau along with splinting of the ankle and forearm.

 

Early total care of high-energy peri-articular and intra-articular proximal tibia fractures is wrought with an unacceptably high risk of wound complications. Acute internal fixation through compromised soft tissues, whether due to severe swelling or contaminated open wounds, substantially increases the risk of delayed wound healing, dehiscence, and infection. Staged fixation utilizing immediate spanning external fixation is recommended in the setting of these injuries and in a poly-trauma patient, as it often offers at least a provisional reduction, serves to restore the length and alignment of the proximal tibia, and thereby promotes soft tissue healing. The external fixator further enables resuscitation of the patient, access to the soft tissues for wound care, and facilitates pre-operative planning for definitive fixation.

 

Parekh et al. retrospectively evaluated perioperative complications following staged internal fixation of high-energy distal femoral and proximal tibia fractures. At final follow-up, there were 8 (16%) deep infections, all in open fractures, with one that required below-knee amputation. The authors concluded that temporary bridging external fixation offers a decreased risk for wound complications over immediate internal fixation and is therefore suitable for patients who are unfit for initial definitive fixation.

 

Berkson et al. reviewed temporary external fixation of complex high-energy tibial plateau fractures. The authors report that knee-spanning external fixation serves to restore tibial length, obtain provisional reduction via ligamentotaxis, and maintain alignment, thereby supporting more expedient soft tissue recovery and mitigating wound complications. They concluded that, in the setting of open fractures, acute debridement and irrigation, application of

knee-spanning external fixation, and serial debridement is indicated to minimize the risk of infection.

 

Haidukewych et al. reviewed the indications, techniques, results, and complications regarding the management of complex intra-articular and periarticular tibial plateau fractures. They highlight the benefits of and recommend

staged treatment with initial external fixation and delayed definitive fixation to minimize wound complications in high-energy tibial plateau fractures with a compromised soft-tissue envelope.

 

Lichte et al. present a current review of the evidence supporting damage control orthopedics (DCO). The authors conclude that DCO is more appropriate in the critically ill polytrauma patient to minimize the risk of a “second hit” phenomenon with the plan for conversion to definitive fixation within the 7-14 day window.

 

Figures A and B show a displaced distal femoral shaft fracture with an ipsilateral bicondylar tibial plateau fracture. Figure C shows an ipsilateral bimalleolar ankle fracture-dislocation. Figure D shows an ipsilateral displaced distal radius and distal ulna fracture. Figure E shows a right-sided diaphragmatic hernia.

 

Incorrect Answers:

Answer 1: A knee immobilizer would not provide sufficient stability for such a significantly length-unstable injury.

Answer 3: While there are advocates of limited internal fixation in the acute setting, acute retrograde nailing of the femur is not recommended in a polytraumatized patient with a closed head injury and major abdominal trauma.

Answers 4 and 5: Acute open reduction and internal fixation of a high-energy open tibial plateau fracture is not recommended as it is associated with a substantially increased risk of wound complications.

 

 

 

 

A 34-year-old male presents with a closed left leg injury after falling off a 20ft ladder. His injury radiographs are shown in Figure A. He is taken for intramedullary nail (IMN) fixation. Which of the following correctly combines techniques used to decrease the incidence of the most common deformities associated with this fracture pattern?

 

 

 

Starting point in Figure B with blocking screw in Figure D

Starting point in Figure B with blocking screw in Figure E

Starting point in Figure C with blocking screw in Figure D

Starting point in Figure C with blocking screw in Figure E

Starting point in Figure C with blocking screw in Figure F Correct answer: 4

The patient has sustained a proximal third tibial shaft fracture, which has a tendency to fall into valgus and procurvatum during IMN fixation. A more lateralized starting point (Figure C) and use of a posterior blocking screw (Figure E) are techniques which have been shown to minimize these deformities, respectively.

 

Proximal third tibial shaft fractures are subject to the deforming forces exerted by the extensor mechanism and pes anserinus, commonly resulting in apex anterior (procurvatum) and valgus angulation of the fracture, respectively.

During IMN fixation, these deformities may be mitigated by techniques including use of the semi-extended approach, lateralization of the starting point, and the application of blocking screws placed in the posterior half of tibia within the coronal plane and in the lateral concave side of the proximal fragment within the sagittal plane. Unicortical plate fixation may also be used

to maintain fracture alignment.

 

Ricci et al. describes the technique and outcomes for use of blocking screws with tibial IMN fixation in proximal third tibial shaft fractures. The authors found less than 5 degrees of angular deformity in all patients in the planes in which blocking screws had been used to control alignment. Ten of the twelve patients went on to uneventful union, and all patients had maintained their alignment at final follow-up. The authors concluded that blocking screws are effective to help obtain and maintain alignment of proximal third tibial shaft fractures treated with IMN nail fixation.

 

Krettek et al. evaluated the mechanical effects of blocking screws in supplementing IMN fixation of both proximal and distal third tibial shaft fractures. The authors noted that with the addition of blocking screws, the deformation of the bone construct segment decreased by 25% for proximal fractures and by 57% for distal fractures. The authors concluded that blocking screws can increase the primary stability of proximal and distal metaphyseal tibial fractures after IMN fixation.

 

Figure A shows the AP and lateral radiographs of a proximal third tibial shaft fracture with characteristic procurvatum and valgus alignment.

Figure B is an AP radiograph of the knee showing the standard entry IMN entry point, at the medial aspect of the lateral tibial spine.

Figure C is an AP radiograph showing a more lateralized starting point. Figure D is a lateral radiograph demonstrating an incorrect location of the blocking screw within the anterior half of the proximal segment.

Figure E is a lateral radiograph demonstrating an appropriately posterior location of the blocking screw within the proximal segment.

Figure F is an AP radiograph demonstrating an incorrect location of the blocking screw within the medial (instead of lateral) aspect of the proximal tibia in the sagittal plane.

 

Incorrect Answers:

Answer 1: Figure B is the standard starting point of a tibia IMN. For a proximal third tibial fracture, lateralizing the starting point would be recommended to mitigate valgus deformity. Figure D shows the blocking screw incorrectly placed in the anterior half of the tibia, which would drive the IMN posterior and leave the fracture susceptible to procurvatum deformity.

Answer 2: Figure B is the standard starting point of a tibia IMN. For a proximal third tibial fracture, lateralizing the starting point would be recommended to mitigate valgus deformity. Figure E is however the correct location for placement of a blocking screw in the coronal plane.

Answer 3: Figure C is the appropriate lateralized starting point to reduce the

chance of valgus deformity for a proximal third tibial fracture. Figure D shows the blocking screw incorrectly placed in the anterior half of the tibia, which would drive the IMN posterior and leave the fracture susceptible to procurvatum deformity.

Answer 5: Figure C is the appropriate lateralized starting point to reduce the chance of valgus deformity for a proximal third tibial fracture. Figure D shows the blocking screw incorrectly placed in the medial aspect of the proximal fragment, which would drive the IMN lateral and leave the fracture susceptible to valgus deformity.

 

 

 

 

A 25-year-old patient is involved in a motor vehicle accident. An isolated orthopaedic injury is sustained to the upper extremity with no compromise of skin integrity or neurovascular function. A radiograph of the injury is shown in Figure A. The patient is interested in pursuing surgical intervention. What is a reported outcome of surgery when compared to nonoperative management at 1 year postoperatively?

 

 

Increased rates of symptomatic nonunion

Similar rates of symptomatic nonunion

No differences in cosmetic results

Increased functional outcome scores

Improved range of motion of the shoulder Correct answer: 4

Surgical fixation of this significantly displaced clavicle fracture (greater than 100%) has been associated with greater functional outcomes up to 1 year after injury compared to non-operative measures

 

Clavicle fractures are often seen in young active patients as the result of falls or direct compressive trauma. Non-operative management of midshaft clavicle

ORTHOPEDIC MCQS ONLINE OB 20 TRAUMA 2D
ORTHOPEDIC MCQS ONLINE OB 20 TRAUMA 2B
ORTHOPEDIC MCQS ONLINE 20 OB TRAUMA 2A
ORTHOPEDIC MCQS ONLINE 20 OB TRAUMA 1D
ORTHOPEDIC MCQS 20 OB TRAUMA 1C
ORTHOPEDIC MCQS 20 OB TRAUMA 1B
ORTHOPEDIC MCQS 20OB TRAUMA 1A
ORTHOPEDIC MCQS ONLINE 010 PEDIATRIC
ORTHOPEDIC MCQS 010 ONLINE
ORTHOPEDIC MCQS 010 Adult Reconstructive Surgery...
ORTHOPEDIC MCQS ONLINE 011 RECONSTRUCTION
ORTHOPEDIC MCQS ONLINE 011 PATHOLOGY
ORTHOPEDIC MCQS 011 ANATOMY IMAGING
ORTHOPEDIC MCQS O11 UPPER EXTREMITY
ORTHOPEDIC MCQS ONLINE 012 SPINE
ORTHOPEDIC MCQS ONLINE 012 TRAUMA
Orthopedic MCQS online 012 FOOT AND ANKLE
ORTHOPEDIC MCQS ONLINE 013 PEDIATRIC
ORTHOPEDIC MCQS ONLINE 013 SPORT
ORTHOPEDIC MCQS ONLINE 013 BASIC
ORTHOPEDIC MCQS ONLINE 014 ANATOMY IMAGING
ORTHOPEDIC MCQS ONLINE 014 UPPER EXTREMITY
ORTHOPEDIC MCQS ONLINE 014 PATHOLOGY
ORTHOPEDIC MCQS ONLINE 015 TRAUMA
ORTHOPEDIC MCQS ONLINE 015 FOOT AND ANKLE e
ORTHOPEDIC MCQS ONLINE 015Spine
ORTHOPEDIC MCQS ONLINE PEDIATRIC 016
ORTHOPEDIC MCQS ONLINE RECONSTRUCTION 016
ORTHOPEDIC MCQS ONLINE SPORT016
ORTHOPEDIC MCQS ONLINE HAND 017
ORTHOPEDIC MCQS ONLINE PATHOLOGY 017
Orthopedic MCQS online Shoulder and Elbow 017
Orthopedic MCQS online Anatomy 017
Orthopedic MCQS online Basic 018
Orthopedic MCQS online Spine 0018
Orthopedic MCQS Trauma 0018
Orthopedic MCQS RECON0019
Orthopedic Mcqs Sport 0019
Orthopedics Mcqs Hand0019
ORTHO MCQS RECON019
HAND AND WRIST MCQS 019
ORTHO MCQS Shoulder and Elbow 0192
ORTHO MCQS Shoulder and Elbow 019
ORTHO MCQS PEDS 10
ORTHO MCQS SPORT 10
ORTHO MCQS BANK 011 FREE 04
ORTHO MCQS BANK 011 FREE 03
ORTHO MCQS 011 FREE BANK 02
ORTHO MCQS 011 FREE BANK
Orthopedic MCQS online Hip and knee ADULT...
ORTHOPEDIC MCQS OB 20 TRAUMA1
ORTHOPEDIC MCQS OB 20 BASIC5
ORTHOPEDIC MCQS OB 20 BASIC7
ORTHOPEDIC MCQS OB 20 BASIC 6
ORTHOPEDIC MCQS OB 20 BASIC 44
ORTHOPEDIC MCQS OB 20 BASIC 4
ORTHOPEDIC MCQS OB 20 BASIC 3
ORTHOPEDIC MCQS OB 20 BASIC 2
ORTHOPEDIC MCQS OB 20 BASIC 1
ORTHOPEDIC MCQS OB 20 SHOULDER AND ELBOW4
ORTHOPEDIC MCQS OB 20 SHOULDER AND ELBOW3
ORTHOPEDIC MCQS OB 20 SHOULDER AND ELBOW 2
1ORTHOPEDIC MCQS OB 20 SHOULDER AND ELBOW
ORTHOPEDIC MCQS BANK WITH ANSWER HIP 01
ORTHOPEDIC MCQS BANK WITH ANSWER PEDS 01
ORTHOPEDIC MCQS BANK WITH ANSWER SPORT 01
ORTHOPEDIC MCQS BANK WITH ANSWER ANATOMY 02
ORTHOPEDIC MCQS BANK WITH ANSWER PATHOLOGY 02
ORTHOPEDIC MCQS BANK WITH ANSWER SHOULDER 02
ORTHOPEDIC MCQS WITH ANSWER FOOT 03
ORTHOPEDIC MCQS WITH ANSWER SPINE 03
ORTHOPEDIC MCQS WITH ANSWER TRAUMA 03
ORTHOPEDIC MCQS WITH ANSWER HIP 04
ORTHOPEDIC MCQS WITH ANSWER PEDS 04
ORTHOPEDIC MCQS WITH ANSWER SPORT 04
ORTHOPEDIC MCQS WITH ANSWER ANATOMY 05
ORTHOPEDIC MCQS WITH ANSWER TUMOR/ONCOLOGY 05
ORTHOPEDIC MCQS WITH ANSWER UPPER LIMB 05
ORTHOPEDIC MCQS WITH ANSWERS ONLINE SPINE 06
ORTHOPEDIC MCQS WITH ANSWERS ONLINE FOOT AND ANKLE...
ORTHOPEDIC MCQS WITH ANSWERS ONLINE TRAUMA 06
ORTHOPEDIC MCQS with Answers ONLINE BASIC 06
ORTHOPEDIC MCQS ONLINE PEDIATRICS 07
ORTHOPEDIC MCQS ONLINE HIP AND KNEE RECON 07
ONLINE ORTHOPEDIC MCQS SPORT07
ONLINE ORTHOPEDIC MCQS UPPER LIMB08
ONLINE ORTHOPEDIC MCQS ONCOLOGY/TUMOR08
ONLINE ORTHOPEDIC MCQS ANATOMY08
ONLINE ORTHOPEDIC MCQS FOOT0 9
ONLINE ORTHOPEDIC MCQS SPINE0 9
ONLINE ORTHOPEDIC MCQS TRAUMA 9
Orthopedic MCQS online sports Medicine
Orthopedic MCQS online Shoulder and Elbow
Orthopedic MCQS online Hip and knee
online orthopedic mcqs
Shoulder and elbow: Mcqs AND EMQS Answers
Shoulder And Elbow: Questions Mcqs AND EMQS
Hand and wrist: Answers MCQS EMQS
Hand and wrist: MCQ AND EMQ Questions
MIND MAP
action tree
action2
Action
patient
Bop
New
Trial
Pupup
SSEE
SE
2021 SHOULDER AND ELBOW MCQS FREE
Pediatric Orthopaedic MCQS Self-Assessment...
Self-Assessment Examination 2020 Adult Spine MCQS
Foot and Ankle free MCQS2020 Online
UPDATED ORTHOPEDIC MCQS
FREE Orthopedics MCQS 2022 1951.-2000.
FREE Orthopedics MCQS 2022 1901.-1950.
FREE Orthopedics MCQS 2022 1851-1900.
FREE Orthopedics MCQS 2022 1751-1850..
FREE Orthopedics MCQS 2022 1751-1800..
Foot and Ankle FREE ORTHOPEDICS MCQS Question 11
FREE Orthopedics MCQS 2022 1701-1750.
FREE Orthopedics MCQS 2022 1651-1700
FREE Orthopedics MCQS 2022 1601-1650.
ORTHOPEDIC MCQS FREE 2023
FREE Orthopedics MCQS 2022 1551-1600
FREE Orthopedics MCQS 2022 1501-1550
FREE Orthopedics MCQS 2022 1451-1500
FREE Orthopedics MCQS 2022 1401-1450
FREE Orthopedics MCQS 2022 1351 -1400
ORTHOPEDICS HYPERGUIDE 2022 MCQ-1301-1350
ORTHOPEDICS HYPERGUIDE 2022 MCQ-1251-1300
ORTHOPEDICS HYPERGUIDE 2022 MCQ-1151-1200
ORTHOPEDICS HYPERGUIDE 2022 MCQ-1101 1150
ORTHOPEDICS HYPERGUIDE 2022 MCQ1051-1100
ORTHOPEDICS HYPERGUIDE 2022 MCQ1001-1051
ORTHOPEDICS HYPERGUIDE MCQ 951-1000
ORTHOPEDICS HYPERGUIDE MCQ 901-950
ORTHOPEDICS HYPERGUIDE MCQ 851-900
ORTHOPEDICS HYPERGUIDE MCQ 800-850
ORTHOPEDICS HYPERGUIDE MCQ 751-800
ORTHOPEDICS HYPERGUIDE MCQ 701-750
ORTHOPEDICS HYPERGUIDE MCQ 651-700
ORTHOPEDICS HYPERGUIDE MCQ 601-650
ORTHOPEDICS HYPERGUIDE MCQ 551-600
ORTHOPEDICS HYPERGUIDE MCQ 501-550
ORTHOPEDICS HYPERGUIDE MCQ 451-500
ORTHOPEDICS HYPERGUIDE MCQ 401-450
ORTHOPEDICS HYPERGUIDE MCQ 351-400
ORTHOPEDICS HYPERGUIDE MCQ 301-350
ORTHOPEDICS HYPERGUIDE MCQ 251-300
ORTHOPEDICS HYPERGUIDE MCQ 201-250
ORTHOPEDICS HYPERGUIDE MCQ 151-200
ORTHOPEDICS HYPERGUIDE MCQ 101-150
FREE Orthopedics MCQS 2022 51-100
Orthopedics Hyperguide MCQ 1-50
Dr. Mohammed Hutaif

About the Author: Prof. Dr. Mohammed Hutaif

Vice Dean of the Faculty of Medicine at Sana'a University and a leading consultant in orthopedic and spinal surgery. Learn more about my expertise and achievements.

Share this article